Essential Surgical Procedures [1 ed.] 9780323375672

Accessible on a variety of devices, Essential Surgical Procedures is an electronic-only resource that provides general s

1,982 352 270MB

English Pages [1674] Year 2015

Report DMCA / Copyright

DOWNLOAD FILE

Polecaj historie

Essential Surgical Procedures [1 ed.]
 9780323375672

Table of contents :
Essential Surgical Procedures
Copyright Page
Consulting Editors
Preface
Acknowledgements
1 Exploratory Laparotomy – Laparoscopic
Goals/Objectives
1-1 Minimal-Access Surgery
Applications for Minimal-Access Surgery
Diagnostic Laparoscopy
Therapeutic Laparoscopy
General Guidelines for Minimal-Access Surgery
Evaluation and Selection of Patients
Preoperative Management of Patients
Choice of Equipment and Techniques
Procedures Involved in Abdominal Laparoscopic Operations
Creating a CO2 Pneumoperitoneum and Initial Port
Closed Technique
Open Technique
Placing Additional Ports
Recognizing and Managing Complications
Suggested Reading
1-2 Diagnostic Peritoneal Lavage and Laparoscopy in Evaluation of Abdominal Trauma
Laparoscopy in Trauma
1-3 Common Access Techniques
1-4 Laparoscopy-Assisted ERCP
1-5 Self Assessment
References
2 Exploratory Laparotomy – Open
Goals/Objectives
2-1 Acute Abdomen
Evaluation and Diagnosis
Laboratory Studies
Imaging Studies
2-2 Self Assessment
References
3 Peritoneal Dialysis Catheter Insertion
Goals/Objectives
3-1 Peritoneal Dialysis
Peritoneal Physiology and Transport
The Peritoneal Catheter and Access
Catheter-Related Complications
References
3-2 Peritoneal Dialysis Catheter Placement
3-3 Advanced Laparoscopic Techniques Significantly Improve Function of Peritoneal Dialysis Catheters
Methods
Surgical Technique
Selective Omentopexy
Subcutaneous Catheter Tunneling
Statistical Analysis
Results
Discussion
Conclusions
References
3-4 Peritoneal Dialysis Catheter Placement
Further Reading
3-5 Self Assessment
References
4 Peritoneal Lesion – Biopsy
Goals/Objectives
4-1 Peritoneum And Peritoneal Cavity
Peritoneum And Peritoneal Cavity
Anatomy
Physiology
Peritoneal Disorders
Ascites
Pathophysiology and cause.
Clinical Presentation and Diagnosis.
Ascitic Fluid Analysis.
Treatment of Ascites in Cirrhotic Patients.
Chylous Ascites.
Peritonitis.
Spontaneous Bacterial Peritonitis.
Tuberculous Peritonitis.
Peritonitis Associated With Chronic Ambulatory Peritoneal Dialysis.
Malignant Neoplasms of the Peritoneum
Pseudomyxoma Peritonei.
Malignant Peritoneal Mesothelioma.
References
Further Reading
4-2 Self Assessment
References
5 Inguinal and Femoral Hernia – Laparoscopic Repair
Goals/Objectives
5-1 Hernias
Laparoscopic Repair.
Results of Hernia Repair
Femoral Hernias
5-2 Laparoscopic Inguinal Hernia Repair
Further Reading
5-3 Self assessment
References
6 Inguinal and Femoral Hernia – Open Repair
Goals/Objectives
6-1 Hernias
Diagnosis
Classification
Treatment
Nonoperative Management
References
6-2 Open Inguinal Hernia Repair with Plug and Patch Technique
Complications of mesh repair
Mesh Fixation
Mesh Infection and/or Exposure
Enterocutaneous Fistula
Hernia Recurrence
Vas Deferens Obstruction
Nerve Entrapment/Chronic Inguinal Pain
References
6-3 Hernia Repair: General Principles – Tension-Free versus Tension
6-4 Self Assessment
References
7 Ventral Hernia – Laparoscopic Repair
Goals/Objectives
7-1 Ventral Herniation in Adults
Laparoscopic Operative Method
7-2 Laparoscopic Ventral and Incisional Hernia Repair
Overview
Indications
Preoperative Planning
Equipment and Materials
Operative Technique
A New Variation on Standard Technique
Laparoscopic Myofascial Separation of Components and Advancement
Anatomic Variants: Epigastric, Suprapubic, and Flank Hernias
Results of Treatment
Summary
Suggested Reading
7-3 Laparoscopic Ventral Hernia Repair – Standard
7-4 Self Assessment
References
8 Ventral Hernia – Open Repair
Goals/Objectives
8-1 Hernias
Incisional Hernia
Treatment: Operative Repair
Prosthetic Materials for Ventral Hernia Repair
Synthetic Materials.
Biologic Materials.
Operative Technique
Ventral Hernias.
Intraperitoneal Mesh Placement.
Retromuscular Mesh Placement.
Component Separation.
Endoscopic Component Separation.
Results of Incisional Hernia Repairs
References
8-2 Open Retromuscular Ventral Hernia Repair
8-3 Incisional/Ventral Hernia – Mesh and Tissue Flap
8-4 Open Retromuscular Ventral Hernia Repair
8-5 Self Assessment
References
9 Cholecystectomy with or without Cholangiography – Laparoscopic
Goals/Objectives
9-1 Laparoscopic Cholecystectomy
Overview
Indications
Technique
Complications
Summary
Suggested Reading
9-2 Cholecystectomy
Step 1. Surgical Anatomy
Step 2. Preoperative Considerations
Patient Preparation
Equipment and Instrumentation
Anesthesia
Room Setup and Patient Positioning
Step 3. Operative Steps
Access and Port Placement
Description of Procedure
Step 4. Postoperative Care
Step 5. Pearls and Pitfalls
Suggested Reading
9-3 Laparoscopic Cholecystectomy
Operative Indications
Preoperative Evaluation
Patient Positioning and Placement of Trocars
Operative Technique
Postoperative Care
Procedure-Specific Complications
Results and Outcome
Suggested Reading
9-4 Laparoscopic Approach to Common Duct Pathology
Technique
Results
Comments
Conclusions
References
Further Reading
9-5 Self Assessment
References
10 Cholecystectomy with or without Cholangiography – Open
Goals/Objectives
10-1 Technique of Cholecystectomy
Overview
Laparoscopic versus Minilaparotomy Cholecystectomy
Indications for Open Cholecystectomy
Preoperative Assessment
Operation
Anatomy
Technique
Incision
Initial Assessment
Placement of Retractors and Optimizing Exposure
Emptying the Gallbladder
Retrograde Cholecystectomy
Anterograde, or Fundus-Down, Cholecystectomy
Cholecystectomy Through Small Incisions
Partial Cholecystectomy
Intraoperative Problems
References
10-2 Self Assessment
References
11 Hepatic Biopsy – Laparoscopic
Goals/Objectives
11-1 Liver Resections
Further Reading
11-2 Self Assessment
References
12 Hepatic Biopsy – Open
Goals/Objectives
12-1 Normal Liver Anatomy and Biopsy Techniques
Normal Anatomy
Biopsy
References
Normal Anatomy
Biopsy
Further Reading
12-2 Self Assessment
References
13 Splenectomy – Laparoscopic
Goals/Objectives
13-1 The Spleen
13-2 Laparoscopic Splenectomy
13-3 Minimally Invasive Splenectomy
13-4 Self Assessment
References
14 Splenectomy – Open
Goals/Objectives
14-1 The Spleen
Splenectomy
Benign Hematologic Conditions
Immune Thrombocytopenic Purpura
Hereditary Spherocytosis
Hemolytic Anemia Caused by Erythrocyte Enzyme Deficiency
Hemoglobinopathies
Malignancy
Lymphomas
Hodgkin’s Disease.
Non-hodgkin’s Lymphomas.
Leukemia
Hairy Cell Leukemia.
Chronic Lymphocytic Leukemia.
Chronic Myelogenous Leukemia.
Non-Hematologic Tumors of the Spleen
Miscellaneous Benign Conditions
Splenic Cysts
Splenic Abscess
Wandering Spleen
References
14-2 Splenectomy/Splenic Repair
14-3 Open Splenectomy
14-4 Self Assessment
References
15 Antireflux Procedure – Laparoscopic
Goals/Objectives
15-1 Hiatal Hernia and Gastroesophageal Reflux Disease
Gastroesophageal Reflux Disease
Pathophysiology
Clinical Presentation
Physical Examination
Preoperative Evaluation
Endoscopy
Manometry
pH Monitoring
Esophagography
Other Tests
References
15-2 Nissen Fundoplication
15-3 Surgical Management of Esophageal Reflux and Hiatal Hernia
Surgical Management of Esophageal Reflux and Hiatus Hernia: Long-Term Results with 1,030 Patients
References
15-4 Laparoscopic Nissen Fundoplication
Further Reading
15-5 Self Assessment
References
16 Gastrostomy – Open
Goals/Objectives
16-1 Stomach: Anatomy
Anatomy
Gross Anatomy
Divisions
Blood Supply
Lymphatic Drainage
Innervation
Gastric Morphology
Gastric Microscopic Anatomy
16-2 Intubation of the Stomach and Small Intestine
Surgical Placement
Stamm Gastrostomy
Laparoscopic Stamm Gastrostomy
Janeway Gastrostomy
Laparoscopic Janeway Gastrostomy
References
16-3 Stamm Gastrostomy
16-4 Self Assessment
References
17 Gastrostomy – Percutaneous Endoscopic
Goals/Objectives
17-1 Percutaneous Endoscopic Gastrostomy Placement and Replacement
Indications
Placement
Replacement
Contraindications
Placement
Absolute
Relative
Replacement
Absolute
Relative
Equipment and Materials
Placement
Replacement
Preprocedure Patient Preparation
Antibiotic Prophylaxis
Technique
Placement
Patient Positioning
Anesthetic Administration
Endoscope Insertion and Evaluation
Identification of Percutaneous Endoscopic Gastrostomy Abdominal Insertion Site
Percutaneous Endoscopic Gastrostomy Tube Placement
A. “Pull” Technique.
B. “Push” Technique.
Postprocedure Endoscopy
Post-Percutaneous Endoscopic Gastrostomy Placement
Special Considerations
Technique
Replacement
Dislodged Percutaneous Endoscopic Gastrostomy, Closed Mature Tract (≥4 Weeks after Placement)
Dislodged Percutaneous Endoscopic Gastrostomy, Open Mature Tract (≥4 Weeks after Placement)
Dislodged Percutaneous Endoscopic Gastrostomy, Immature Tract (

Citation preview

ESSENTIAL SURGICAL PROCEDURES

ESSENTIAL SURGICAL PROCEDURES

Edinburgh  London  New York  Oxford  Philadelphia  St Louis  Sydney  Toronto  2016

© 2016 Elsevier Inc. All rights reserved. No part of this publication may be reproduced or transmitted in any form or by any means, electronic or mechanical, including photocopying, recording, or any information storage and retrieval system, without permission in writing from the publisher. Details on how to seek permission, further information about the Publisher’s permissions policies and our arrangements with organizations such as the Copyright Clearance Center and the Copyright Licensing Agency, can be found at our website: www.elsevier.com/permissions. This book and the individual contributions contained in it are protected under copyright by the Publisher (other than as may be noted herein). Notices Knowledge and best practice in this field are constantly changing. As new research and experience broaden our understanding, changes in research methods, professional practices, or medical treatment may become necessary. Practitioners and researchers must always rely on their own experience and knowledge in evaluating and using any information, methods, compounds, or experiments described herein. In using such information or methods they should be mindful of their own safety and the safety of others, including parties for whom they have a professional responsibility. With respect to any drug or pharmaceutical products identified, readers are advised to check the most current information provided (i) on procedures featured or (ii) by the manufacturer of each product to be administered, to verify the recommended dose or formula, the method and duration of administration, and contraindications. It is the responsibility of practitioners, relying on their own experience and knowledge of their patients, to make diagnoses, to determine dosages and the best treatment for each individual patient, and to take all appropriate safety precautions. To the fullest extent of the law, neither the Publisher nor the authors, contributors, or editors, assume any liability for any injury and/or damage to persons or property as a matter of products liability, negligence or otherwise, or from any use or operation of any methods, products, instructions, or ideas contained in the material herein.

Inkling ISBN: 978-0-323-37567-2 Executive Content Strategist: Michael Houston Content Development Specialist: Fiona Conn Project Manager: Jo Souch Marketing Manager: Kristin Koehler

CONSULTING EDITORS

Senior Consulting Editor

Jose M Velasco MD, FACS, FCCM Director Surgical Innovation Professor of Surgery Department of General Surgery Rush University Medical Center Chicago, Illinois Consulting Editors

Rana Ballo MD

Resident Physician Department of General Surgery Rush University Medical Center Chicago, Illinois

Keith Hood MS MD Resident Physician Department of General Surgery Rush University Medical Center Chicago, Illinois

Jennifer Jolley MD Resident Physician Department of General Surgery Rush University Medical Center Chicago, Illinois

Daniel Rinewalt MD Resident Physician Department of General Surgery Rush University Medical Center Chicago, Illinois

Benjamin Veenstra MD Assistant Professor of Surgery Department of General Surgery Rush University Medical Center Chicago, Illinois

ev

PREFACE

With the reduction in the number of working hours, it has become increasingly difficult for a general surgery trainee to gain the proper amount of experience needed to learn, competently, and autonomously perform the finite number of surgical procedures that are taught in the course of their five year residency program. This eBook is aligned with the Essential-Common surgical procedures listed in the Surgical Council on Resident Education (SCORE) curriculum for general surgery. It is hoped that this eBook will enable time-pressed general surgery residents to gain access to relevant knowledge in a user-friendly, readily available format so that they learn the necessary competencies to successfully perform any given Essential-Common operative technique. Essential Surgical Procedures is accessible on a variety of devices and provides general surgery residents/trainees with a single, portable resource containing procedural related text, images and video, selected from 52 leading surgical reference works and 13 journals published by Elsevier. For each of the 89 procedures, Essential Surgical Procedures delivers clear, definitive guidance on clinical anatomy, perioperative considerations, operative technique as well as highlighting clinical pearls and pitfalls. This content is supplemented with copious line drawings, clinical photographs and, wherever possible, real time video clips capturing key moments of a surgical procedure. Each procedure is accompanied by self-assessment questions and answers together with a brief rationale behind the answers to each question. The Consulting Editors have designed and organized the content of this eBook based on their past experience and needs accumulated during their surgical training and academic careers, thus, amplifying the content and structure found in the SCORE curriculum. Elsevier

Acknowledgements Dr Jose Velasco would like to acknowledge the invaluable contribution of Kathy Martin, who coordinated the work of the consulting editors, collated and prepared the sessions, and kept the team on track.

evi

1 

Exploratory Laparotomy – Laparoscopic

GOALS/OBJECTIVES • • •

INDICATIONS ANATOMIC CONSIDERATIONS TECHNICAL CONSIDERATIONS

MINIMAL-ACCESS SURGERY Nathaniel

J. Soper

/ Valerie

J.

Halpin

From Becker JM, Stucchi AF: Essentials of Surgery, I st edition (Saunders 2006)

APPLICATIONS FOR MINIMAL-ACCESS SURGERY The terms minimal-access surgery and minimally invasive surgery refer to operations in which the inci­ sions are much smaller than those involved in traditional ("open'') surgery and in which video endo­ scopic imaging techniques are commonly used. Laparoscopic techniques were first described as early as 1901. The modern era of minimally invasive surgery began in the 1980s, with the introduction of the miniature video camera and transmitting equipment that allowed an entire operating team to view surgical procedures on a video screen. Since the introduction of laparoscopic cholecystectomy in the 1980s, this technique as become the gold standard for treating symptomatic cholelithiasis. Other laparoscopic procedures are now being per­ formed with increasing frequency, and many minimal-access techniques are undergoing scrutiny in prospective randomized trials. As new technology has continued to evolve nd find additional applica­ tions, surgeons and patients alike have rapidly embraced it because its us is generally associated with shorter hospitalizations, less postoperative pain, faster recuReration, and decreased costs. Diagnostic Laparoscopy

Laparoscopy is becoming an increasingly valuable tool for evaluating a variety of intra-abdominal and other disorders. Elective applications include the assessment 0£ chronic abdominal pain, abdominal masses, liver disease, ascites, inguinal hernias, and ventral he nias. Laparoscopy is frequently useful in the diagnosis and staging of malignant neoplasms. The magnified view can often identify small metastatic peritoneal implants that cannot be detected by com uted tomography, magnetic resonance imaging, or ultra­ sonography. It also can help assess tumor response to neoadjuvant chemotherapy or radiation therapy. In some cases, laparoscopy can replace "second-look'' laparotomy. Emergency applications of laparoscopy include the evaluation of acute abdominal pain and peri­ tonitis. Diagnostic laparoscopy is especially useful in assessing acute abdominal pain in young women, whose gynecologic problems are frequently confused with acute appendicitis. In these women, the procedure has been estimated to reduce the rate of unnecessary laparotomies by one third. In medically compromised patients in intensive care settings, laparoscopy can be used to exclude acute biliary tract disease or ischemic bowel disease and thereby avoid nontherapeutic laparotomies. In some medical centers, laparoscopy has occasionally been used to evaluate tangential gunshot wounds and stab wounds to the abdomen; however, this use remains controversial, because the introduction of gas into the peritoneal cavity may lead to severe hypotension in the presence of hyp ovolemia. Therapeutic Laparoscopy

As mentioned earlier, laparoscopic cholecystectomy is now considered the gold standard for removing a diseased gallbladder. While the patient is undergoing this procedure, the common bile duct also can be explored and assessed. Laparoscopic tubal ligation is another well-accepted minimally invasive procedure. Laparoscopic appendectomy is less well accepted than laparoscopic cholecystectomy, because it may not be more cost-effective or less painful than a standard appendectomy. However, laparoscopic appendectomy does have a place in the armamentarium of the general surgeon and is particularly eJ

e4   SECTION 1  ■  ABDOMEN – GENERAL appropriate for patients who are obese, women of childbearing age, and patients whose diagnosis is unclear. Laparoscopic inguinal herniorrhaphy, which can be performed transabdominally or by using totally extra-peritoneal access to place prosthetic mesh in the preperitoneal space, has ardent enthusiasts and opponents. The procedure seems ideally suited for patients with bilateral or recurrent hernias. In patients with gastroesophageal reflux disease (GERD), studies have shown that open Nissen fundoplication is more effective than medical management. Although few patients are willing to undergo a major open abdominal or thoracic operation for the treatment of GERD, many more have expressed their willingness to undergo laparoscopic fundoplication or other laparoscopic antireflux procedures. Initial reports indicate that laparoscopic fundoplication yields results similar to those of the open procedure, yet is associated with decreased postoperative pain and a shorter hospital stay. At this time, longterm studies are ongoing to verify the effectiveness of minimally invasive surgery for patients with GERD. Laparoscopic colon resection in patients with benign colon disease has been reported to decrease the length of the hospital stay and shorten the duration of postoperative ileus, but to cost about the same amount as open colectomy. Randomized trials are currently under way to assess the role of laparoscopic colectomy for the treatment of malignant disease. The largest U.S. trial recently showed laparoscopic-assisted colectomy to be as oncologically successful as open colectomy. The surgical management of morbid obesity has rapidly expanded in the last decade with the introduction of laparoscopic gastric bypass. This procedure has been shown to reduce wound complications, decrease postoperative pain, and improve respiratory function, all resulting in shorter hospital stays and improved recovery. Weight loss is equivalent to that with the open procedure. Other restrictive bariatric procedures include laparoscopic vertical banded gastroplasty and laparoscopic adjustable gastric banding. These procedures are less technically challenging; however, they have not been shown to reliably produce the same degree of weight loss in the U.S. population. The surgical removal of solid organs, including laparoscopic splenectomy, laparoscopic adrenalectomy, and laparoscopic nephrectomy, is becoming the standard of care at institutions where the expertise is available. A variety of other minimal-access procedures have been adopted after initial investigation. These include minimally invasive types of vagotomy, esophagocardiomyotomy, biliary bypass, saphenous vein harvest, pelvic lymph node dissection, salpingo-oophorectomy, hysterectomy, and bladder-neck suspension. Minimally invasive parathyroidectomy with directed single-gland exploration is now possible with the use of preoperative sestamibi scanning and intraoperative rapid parathyroid hormone assay. Anecdotal reports have described pancreatic pseudocyst-gastrostomy, pancreatic resection, gastrectomy, rectal prolapse repair, thyroidectomy, hepatic resection, coronary artery bypass graft, and cardiac valve repair.

GENERAL GUIDELINES FOR MINIMAL-ACCESS SURGERY Evaluation and Selection of Patients Candidates for minimal-access surgery should undergo a thorough and careful preoperative history and evaluation, because they must be able to tolerate not only a laparoscopic procedure but also an open procedure if conversion to one becomes necessary. As surgeons gain more experience, the contraindications to laparoscopic surgery are decreasing. The following remain absolute contraindications to laparoscopic abdominal procedures: advanced generalized peritonitis, hypovolemic shock, massive abdominal distention with clinical evidence of bowel obstruction, uncorrected coagulopathy, and inability of the patient to tolerate a formal laparotomy. Relative contraindications include prior abdominal or pelvic surgery, previous generalized peritonitis, obesity, advanced cardiopulmonary disease, and pregnancy.

Preoperative Management of Patients In most patients undergoing elective abdominal surgery, general anesthesia is the anesthesia of choice, because it allows for the greatest control of ventilation and abdominal muscle relaxation. Patients should be instructed to take nothing by mouth at least 8 hours before the procedure begins. Antibiotics and a histamine H2-receptor antagonist are given by the intravenous route preoperatively. Compression stockings and sequential compression devices are applied to the lower extremities to

CHAPTER 1-1  ■  Minimal-Access Surgery  

prevent deep venous thrombosis. Consideration should be given to tube decompression of the stomach and bladder. A Foley catheter in the bladder may be appropriate for some procedures.

Choice of Equipment and Techniques Laparoscopic operations require the presence of gas in the peritoneal cavity (pneumoperitoneum). Various gases can be used, and either a closed technique or an open technique can be used to create the pneumoperitoneum, as described later. Currently, the vast majority of laparoscopic operations involve the insufflation of carbon dioxide. However, because of the potential for hypercapnia, acidosis, cardiac arrhythmias, and other detrimental effects caused by transperitoneal absorption of CO2, other insufflating gases have been used experimentally for creating the pneumoperitoneum. These include nitrous oxide, helium, and argon, each of which has other potential drawbacks, such as combustibility, insolubility, or expense. Any gas insufflated into the peritoneal cavity can affect the normal physiology by elevating the diaphragm or reducing venous return via compression of the vena cava. Thus some surgeons use miniaturized retracting devices to elevate the abdominal wall manually to create a working space. However, the devices are generally cumbersome, and the exposure is usually not as good as that afforded by creating a pneumoperitoneum. A spring-loaded Veress needle (Figure 1-1-1) and trocar are used in the closed technique for creating the pneumoperitoneum. Many instruments commonly used in open surgery have been modified to facilitate laparoscopic procedures. These include clip appliers, linear cutting staplers, argonbeam coagulators, and various monopolar and bipolar cautery devices. For coagulation and division of vascular pedicles and small vessels (vessels smaller than 4 mm), ultrasound energy may be used in the form of harmonic shears (LaparoSonic Coagulating Shears, Ethicon-Endosurgery, Inc., Cincinnati, OH). With any device transmitting potentially harmful energy into the closed space of a pneumoperitoneum, great care must be taken to prevent danger to surrounding tissues.

PROCEDURES INVOLVED IN ABDOMINAL LAPAROSCOPIC OPERATIONS Creating a CO2 Pneumoperitoneum and Initial Port The initial step in laparoscopy is creation of a pneumoperitoneum. This can be done with a closed or open technique.

Closed Technique The steps in the closed technique are as follows: 1. The Veress needle is carefully examined to ensure that the spring mechanism is intact (see Figure 1-1-1) and that the lumen flushes easily.

FIGURE 1-1-1  Testing the retractable tip of a disposable Veress needle. A, The blunt tip retracts as it contacts resistance (e.g., a knife handle or abdominal fascia). B, When the needle is pulled away from the point of resistance, the blunt tip springs forward and protrudes in front of the sharp edge of the needle. (Modified from Soper NJ, Odem RR, Clayman RV, McDougall EM (eds): Essentials of Laparoscopy. St. Louis, Quality Medical Publishing, 1994, with permission.)

e5

e6   SECTION 1  ■  ABDOMEN – GENERAL 2. The lower abdominal fascia is grasped and elevated to protect the intra-abdominal organs. The Veress needle is inserted at a right angle to the abdominal wall, toward the pelvis, but angled away from the aortic bifurcation and the iliac vessels. The surgeon should hear two or three clicks as the needle passes through the fascia and the peritoneum. 3. The needle position is checked by aspiration with a syringe that is partially filled with saline solution. From 3 to 5 mL of the solution is injected into the needle. If blood, urine, or intestinal contents are aspirated, the needle should be removed and reinserted. If resistance is met, the syringe is most likely located in the abdominal muscle or the omentum and should be repositioned. If no resistance is met, the syringe is aspirated again. A drop test is performed by removing the plunger from the syringe and observing the saline meniscus. The saline solution should flow rapidly by gravity into the peritoneal cavity. Elevating the abdominal wall will create negative intra-abdominal pressure and enhance the flow of the solution. 4. Once the surgeon is satisfied that the needle is in the proper location, the CO2 line from the insufflator is attached to the needle for insufflation. The initial abdominal pressure should be less than 10 mmHg, and the abdomen should be slowly insufflated to a pressure between 10 and 15 mmHg. If the initial abdominal pressure is high, the Veress needle should be rotated to ensure that it is not abutting omentum, bowel, or abdominal wall. If the pressure remains high, the needle should be removed and reinserted. On insufflation, the abdomen should expand symmetrically and be tympanitic to percussion. 5. When the abdomen is distended with a pressure of 12 to 15 mmHg, the Veress needle is removed, and a trocar is inserted. To allow passage of the full circumference of the trocar sheath, a skin incision is made at an appropriate site (either at the umbilical crease or at another site, depending on the procedure). 6. While the abdominal wall is stabilized manually or by use of towel clips, the trocar is passed blindly into the abdomen, initially in the direction perpendicular to the skin and then in the direction of the operating field (Figure 1-1-2). 7. After the trocar enters the peritoneum, the trocar is removed, and the outer sheath is secured in place. This forms the first port.

FIGURE 1-1-2  Trocar insertion. A, Towel clips placed near the edges of the umbilical incision are used to stabilize the abdominal wall during trocar insertion. B, Upward traction on the towel clips maintains the distance between the abdominal wall and the underlying structures during trocar insertion. (Modified from Jones DB, Wu JS, Soper NJ (eds): Laparoscopic Surgery: Principles and Procedures. St. Louis, Quality Medical Publishing, 1997, with permission.)

CHAPTER 1-1  ■  Minimal-Access Surgery  

Open Technique In the open technique, the first port is placed into the peritoneal cavity under direct vision. This reduces the risks of blood vessel, bowel, and bladder injuries that are associated with the blind placement of the Veress needle and first trocar described in step 6. Many surgeons prefer to use the open technique routinely when the umbilicus is used for insertion of the initial port. It is particularly useful in patients with previous abdominal surgery, pregnancy, or evidence of bowel distention. The steps in the open technique are as follows: 1. The appropriate location for the incision is determined, and a 1.5- to 2.0-cm incision is made. The location depends on the procedure being performed. For laparoscopic cholecystectomy, the initial trocar is placed at the umbilical ring with either a vertical or a semicircular incision. In laparoscopic procedures at the gastroesophageal junction, the trocar is placed left and superior to the umbilicus. 2. Blunt dissection of the subcutaneous tissue is performed to expose the fascia. A Kocher clamp is placed on the linea alba, and traction is exerted upward. A 1-cm vertical incision is made in the linea alba. The two lateral edges are grasped with Kocher clamps and elevated. 3. A Kelly clamp is inserted into the incision and pushed through the peritoneum. As the clamp is withdrawn from the peritoneum, its jaws are spread to enlarge the peritoneal opening. A finger is inserted into the opening to confirm that it is intraperitoneal and to sweep away any adhesions. 4. Stay sutures are placed into the superior and inferior aspects of the fascial incision. A blunt-tipped (Hasson) trocar is placed under direct vision through the opening into the peritoneum. The stay sutures are pulled up tightly around the suture wings of the cannula to ensure an airtight seal and prevent the escape of CO2 around the trocar (Figure 1-1-3). 5. The CO2 insufflator is attached to the trocar, and the abdomen is insufflated to a pressure of 15 mmHg.

Placing Additional Ports One trocar is commonly used for diagnostic laparoscopy, but additional trocars are required for therapeutic laparoscopy. The appropriate location of additional ports must be determined. All additional

FIGURE 1-1-3  Securing the Hasson cannula to the abdominal fascia. The fascial stay sutures are tightly wound around the suture wings on the outer sheath. This secures the sheath in place and seals the fasciotomy and peritoneotomy. (Modified from Jones DB, Wu JS, Soper NJ (eds): Laparoscopic Surgery: Principles and Procedures. St. Louis, Quality Medical Publishing, 1997, with permission.)

e7

e8   SECTION 1  ■  ABDOMEN – GENERAL ports should be placed under direct video monitoring. The location of each port depends on the procedure being performed. Ideally, operating trocars should be placed at a 30- to 60-degree angle with the axis of the videoscopic line of vision and operative site to form an equilateral triangle or a diamond. The angle between two operating ports should be 60 to 120 degrees. The distance from the port to the operative site should be about half the total length of the instrument being used for dissection. Because most instruments are 30 to 40 cm in length, this translates into 15 cm from port to operating site. This gives the least distortion at the tip while allowing maximal movement at the fulcrum (the port site). The site of the port is checked before the trocar is inserted. The abdominal wall is indented manually and the site is identified with the video camera. The abdominal wall is transilluminated to locate any superficial vessels to be avoided. If the surgeon is unsure about the appropriate location for port placement, a Veress needle may be passed, and its location and angle of approach to the operative field viewed. The skin and peritoneum are infiltrated with a local anesthetic, and a small stab incision is made with a size 11 blade. The trocar is grasped in the palm of the surgeon’s hand, with the middle finger extending down the trocar sheath to act as a brake against the abdominal wall. The trocar is introduced in a direct line with the planned surgical field to point naturally in the operative direction. This minimizes the pressure placed on the sheath during the procedure and maximizes the surgeon’s touch and feel for dissection and tissue palpation. Under direct video monitoring, the trocar is inserted with slow, steady pressure through the abdominal wall. Care is taken to avoid injuring the abdominal viscera as the obturator tip and sheath are guided into the peritoneal cavity. If difficulty is found in passing the trocar, the abdominal wall can be grasped with towel clips on either side of the trocar and elevated to increase the distance between the wall and the abdominal viscera. PEARLS FOR ROUNDS In comparison with open surgery, laparoscopic surgery is usually associated with smaller incisions, shorter hospitalizations, less postoperative pain, faster recuperation, and decreased costs. Diagnostic laparoscopy is especially useful for the evaluation of abdominal pain in women of childbearing age or in patients whose diagnosis is unclear. Laparoscopic techniques have reduced the threshold for surgical referral for several common diseases, including symptomatic cholelithiasis and gastroesophageal reflux disease. Laparoscopic cholecystectomy is now the gold standard for removing a diseased gallbladder. Laparoscopic inguinal herniorrhaphy may be performed transabdominally or completely extraperitoneally. Contraindications to laparoscopic abdominal procedures include advanced generalized peritonitis, hypovolemic shock, massive abdominal distention with clinical evidence of bowel obstruction, uncorrected coagulopathy, and inability of the patient to tolerate a formal laparotomy. The CO2 pneumoperitoneum may cause hypercapnia, acidosis, and cardiac arrhythmias, especially in patients with cardio­ pulmonary diseases. Alternatives to the CO2 pneumoperitoneum should be considered in high-risk patients. Improper trocar placement during laparoscopic surgery can result in major vascular injury, intestinal injury, or air embolism.

Recognizing and Managing Complications In minimal-access surgery, the three most significant complications are major vascular injury, intestinal injury, and air embolism. The first two injuries are usually associated with puncture by the Veress needle or the first trocar. Air embolism occurs with inadvertent placement of a Veress needle into a major vessel and insufflation with CO2. A major vascular injury should be suspected if sudden hemodynamic compromise develops during a laparoscopic procedure. If a major vessel is injured, the abdomen should be opened immediately and the injury repaired. The magnitude of intestinal injuries varies. A visceral injury that is small and sealed may require only observation. If a trocar lacerates the bowel, suture repair by laparoscopy or laparotomy is required. If air embolism occurs, the abdomen should be desufflated and the patient placed in Trendelenburg position with the left side down. Placement of a central venous catheter allows aspiration of the gas from the right side of the heart, where it can block blood flow through the pulmonary valve.

CHAPTER 1-1  ■  Minimal-Access Surgery  

Suggested Reading Apelgren KN, Cowan BD, Metcalf AM, Scott-Conner CE: Laparoscopic appendectomy and the management of gynecologic pathologic conditions found at laparoscopy for presumed appendicitis. Surg Clin North Am 76:469–482, 1996. Callery MP, Strasberg SM, Soper NJ: Complications of laparoscopic general surgery. Gastrointest Endosc Clin North Am 6:423–444, 1996. Conlon KC, Dougherty E, Klimstra DS, et al: The value of minimal access surgery in the staging of patients with potentially resectable peripancreatic malignancy. Ann Surg 223:134–140, 1996. Gadacz TR: Update on laparoscopic cholecystectomy, including a clinical pathway. Surg Clin North Am 80:1127–1143, 2000. Hartley JE, Monson JRT: The role of laparoscopy in the multimodality treatment of colorectal cancer. Surg Clin North Am 82:1019–1033, 2002. Jones DB, Wu JS, Soper NJ, eds: Laparoscopic Surgery: Principles and Procedures. St. Louis, Quality Medical Publishing, 1997. Nguyen NT, Wolfe BM: Laparoscopic Bariatric Surgery. Adv Surg 36:39–63, 2002. Soper NJ: Laparoscopic management of hiatal hernia and gastroesophageal reflux. Curr Prob Surg 36:765–838, 1999.

e9

1-2 

DIAGNOSTIC PERITONEAL LAVAGE AND LAPAROSCOPY IN EVALUATION OF ABDOMINAL TRAUMA R. Stephen Smith  /  John A. Aucar  /  William R. Fry From Asensio JA, Trunkey DD: Current Therapy of Trauma and Surgical Critical Care, 1st edition (Mosby 2008)

LAPAROSCOPY IN TRAUMA Evaluation of the abdomen in potentially injured patients remains one of the greatest challenges faced by surgeons. None of the current diagnostic modalities available to the trauma surgeon are completely accurate. All of the available techniques, including DPL, sonography, CT, and laparoscopy, have advantages and disadvantages. At present, laparoscopy is not considered a frontline method for evaluation of the abdomen, but it is an important adjunct. Laparoscopy has been used sporadically over the past four decades in the evaluation of patients at risk for abdominal injury. Utilization of laparoscopy in the trauma setting has increased dramatically over the past 15 years. This increase in utilization corresponds to the greater availability of high-quality laparoscopic equipment and the greater penetration of laparoscopy into general surgery training programs. At present, laparoscopy holds the greatest promise in evaluating a select group of patients with penetrating injury. A number of recent series have documented the utility of laparoscopy in the evaluation of the diaphragm in hemodynamically stable patients with a history of thoracoabdominal wounds. The use of laparoscopy in the blunt trauma setting is much less frequent and its indications remain controversial in this group of patients. Sporadic reports of novel uses of laparoscopy, for example, laparoscopically guided blood salvage and laparoscopic decompression of abdominal compartment syndrome, have appeared more recently. In some centers, laparoscopy is used for the assessment of the hollow viscera in patients who are suspected of having a seat-belt injury that cannot be ruled out with other diagnostic modalities. Therapeutic laparoscopy for a select group of isolated patients, that is, those with small diaphragmatic lacerations, is used more frequently and may be applicable for a small subset of patients. In addition, some centers have reported repair of small enterotomies with laparoscopic techniques, if these injuries are isolated. Sound surgical judgment must be used in choosing patients for laparoscopic evaluation after injury. Any injured patient with hemodynamic instability or obvious complex intra-abdominal injury, is not a candidate for laparoscopy, but instead requires immediate laparotomy. Several large series performed by experienced groups have demonstrated that only about 15% of patients with suspected intraabdominal injury are reasonable candidates for adjunctive laparoscopic evaluation or treatment. For patients with gunshot wounds to the abdomen, laparoscopy has proved most useful for evaluation of the diaphragm in patients with thoracoabdominal wounds (Figure 1-2-1). In addition, laparoscopy has proven useful to determine if peritoneal penetration has occurred from tangential gunshot wounds or stab wounds. The use of laparoscopy is not without risk in these patients. In patients with a diaphragmatic injury, production of tension pneumothorax upon insufflation of CO2 for creation of pneumoperitoneum has been reported. Although this does not occur in every patient with diaphragmatic laceration, it is estimated that this will occur in 10% of patients with diaphragmatic injury. The surgeon who uses laparoscopy for evaluation of potential diaphragmatic wounds must be prepared to immediately decompress the pneumoperitoneum and place a tube thoracostomy if signs and symptoms of tension pneumothorax develop. Patients with gunshot wounds of the abdomen are not candidates for laparoscopy, as this group of patients has a very high incidence of significant intra-abdominal injury that must be treated at laparotomy. e10

CHAPTER 1-2  ■  Diagnostic Peritoneal Lavage and Laparoscopy in Evaluation of Abdominal Trauma  

Gunshot wound (stable patient)

Tangential

Thoraco-abdominal

Laparoscopy

Laparoscopy

Peritoneal penetration

No penetration

Exploratory laparotomy or laparoscopic repair1

Observation

Diaphragm injury

Exploratory laparotomy

Diaphragm intact

Mid-abdominal

Formal exploration3

Observation

Laparoscopic repair2

FIGURE 1-2-1  Algorithm for management of a gunshot wound in a stable patient. (1) Laparoscopic repair may be performed for limited injuries, depending on the capabilities of the surgeon. (2) Posterior wounds may be more easily identified and repaired with a thoracoscopic approach. Identification of injuries of associated abdominal organs may necessitate laparotomy. (3) Gunshot wounds in this location have a greater than 90% probability of producing injuries that require definitive surgical repair.

Victims of stab wounds are somewhat more likely to benefit from laparoscopic examination (Figure 1-2-2). In this group of patients, as many as 50% will not have significant intra-abdominal injury. Laparoscopy can be used to reduce the number of negative and nontherapeutic laparotomies in patients with minimal injuries. Laparoscopic repair of small diaphragmatic injures and limited hollow viscus injuries secondary to stab wounds has been reported in the literature and appears to be a viable technique in the hands of a skilled laparoscopic surgeon. The use of laparoscopy in most trauma centers has been associated with a small decrease in the incidence of negative and nontherapeutic laparotomy. A very small percentage of patients with blunt abdominal trauma may benefit from laparoscopic evaluation. Previous reports using laparoscopy for the examination of hepatic and splenic lacerations are not pertinent at this time. The increasing trend toward nonoperative treatment of these injuries and the emergence of arteriography and embolization has made this approach to solid organ injury less viable. An area of potential benefit in the blunt trauma setting is examination of the entire small bowel and colon in a patient at risk for the so-called “seat-belt” syndrome. With moderately developed laparoscopic skills, the vast majority of the small bowel and colon can be examined with laparoscopic techniques. Large injuries to hollow viscera are easily detected; however, small enterotomies may still be missed. Therefore, a very low threshold for conversion to laparotomy must be maintained in the patient at risk for hollow viscus injury. Unfortunately, the literature does not clearly delineate or document the efficacy of laparoscopy in the blunt trauma setting. Standard laparoscopic equipment is used for examination of the trauma patient. Laparoscopic exploration of the abdomen in the trauma setting is similar to patients who present with an acute abdomen. In most cases, the laparoscope is inserted through a periumbilical incision to provide optimal visualization of all quadrants of the abdomen. Additional small operating ports are located to permit the use of other laparoscopic instruments. Location of these ports should allow the greatest manipulation of abdominal contents in the area of interest. A nasogastric tube and urinary catheter should be inserted before the laparoscopic examination for trauma. Most trauma surgeons who use laparoscopy prefer a 30-degree angled lens, as this provides enhanced visualization of areas that are difficult to examine, such as posterior aspects of the diaphragm. Carbon dioxide pneumoperitoneum is used in the trauma setting just as for elective laparoscopy. Initial insufflation may be conducted with either a Veress needle or a Hasson cannula. In the trauma setting, initial insufflation pressures should be limited to 8–0 mm Hg. This threshold is maintained to minimize the risk of the development of tension pneumothorax in patients with diaphragmatic defects, and to minimize the onset of hypotension in a patient with less than optimal intravascular volume. If insufflation to this level is tolerated, then the limit may be increased to 15 mm Hg to

e11

e12   SECTION 1  ■  ABDOMEN – GENERAL Abdominal stab wound Stable

Unstable

Local wound exploration1

Exploratory laparotomy

No penetration of anterior fascia

Penetrates anterior fascia

Observation

Laparoscopy

Peritoneal penetration

No peritoneal penetration

Extensive laparoscopic exam and minilap2

Observation

No injury

Observation

Injury identified

Minimally invasive repair3

Exploratory laparotomy

FIGURE 1-2-2  Algorithm for management of abdominal stab wounds. (1) Local wound exploration performed in the emergency room. (2) Majority of examination is performed by laparoscopy; examination of the small bowel is performed via a 4-cm minilaparotomy incision. (3) Limited injuries may be repaired laparoscopically depending on the capability of the surgeon.

enhance exposure. Gas embolism is also a theoretical risk in patients with intra-abdominal venous injury, but is rarely encountered. If significant enteric spillage or hemorrhage is found on the initial laparoscopic assessment, then laparoscopy should be halted and conversion to a laparotomy should be performed without delay. In most trauma centers that use laparoscopy, significant penetration of the peritoneum secondary to a gunshot wound is an indication for conversion to exploratory laparotomy. Peritoneal penetration secondary to a stab wound allows a more selective approach to further exploration. Certainly, not all injuries identified at the time of laparoscopy require conversion to laparotomy. For example, if an isolated injury to the liver without significant bleeding is encountered, conversion to laparotomy is contraindicated. If the trauma surgeon possesses laparoscopic suturing skills, isolated small diaphragmatic lacerations may be repaired readily with laparoscopic techniques. Posterior areas of the diaphragm are somewhat more difficult to visualize and injuries in this area are difficult to repair with laparoscopic techniques. The liver and spleen are relatively easy to visualize during laparoscopic techniques, but this may require rotating the operating table to the right or left and placing the patient in reverse Trendelenburg position for full visualization of the spleen. Visualization of the pancreas is accomplished by dividing the gastrocolic ligament with laparoscopic vascular clips, staplers, or other devices, such as the harmonic scalpel, which are used for vessel ligation. Approximately 5% of all patients who undergo laparoscopic examination are candidates for therapeutic intervention. These patients most commonly have small diaphragmatic lacerations or a very limited, isolated enterotomy. Sporadic reports of laparoscopic repair of the colon, as well as hepatorrhaphy and splenorrhaphy, have appeared in the literature, but cannot be advocated based on the present evidence. In summary, laparoscopy is a useful adjunct for the evaluation of a select group of injured patients. Laparoscopic techniques are limited to hemodynamically stable patients without clear indication for laparotomy. At present, the best indication for laparoscopy is in patients with a penetrating mechanism of injury who have either tangential injuries of the abdominal wall or injuries to the thoracoabdominal region in which the diaphragm is at risk.

COMMON ACCESS TECHNIQUES Mark A. Carlson From Complications of first entry into the peritoneal cavity. In: Frantzides CT, Carlson MA: Video Atlas of Advanced Minimally Invasive Surgery, 1st edition (Saunders 2012)

1-3 

FIGURE 1-3-1  Pneumoperitoneum (Veress) needle, with tip shown in the inset. Needle is 14 gauge (outer diameter about 2.1 mm).

e13

e14   SECTION 1  ■  ABDOMEN – GENERAL

FIGURE 1-3-2  Radially expanding trocar system (VersaStep, Covidien). A, Pneumoperitoneum needle and expandable sleeve, separated. B, Needle inserted into sleeve. C, Dilator and cannula assembly. D, Dilator and cannula inserted into sleeve. E, Cannula within sleeve (final in vivo configuration).

CHAPTER 1-3  ■  Common Access Techniques  

FIGURE 1-3-3  Blunt trocar (Hasson type) for open laparoscopy. Cannula diameter = 12 mm.

FIGURE 1-3-4  Optical trocar, with tip shown in the inset (Endopath Xcel, Ethicon Endosurgery). Cannula diameter = 12 mm.

FIGURE 1-3-5  Optical trocar system with actuated blade (Visiport Plus RPF system, Covidien). A, Assembled system. B, Blade-containing optical obturator (“gun”) and 12-mm cannula, separated. C, Close-up of clear window tip, showing recessed blade. Inset: lateral view.

e15

e16   SECTION 1  ■  ABDOMEN – GENERAL

FIGURE 1-3-6  Optical trocar with insufflation tip (Fios First Entry, Applied Medical). A, Close-up of trocar tip. B, Intraoperative view of tip entering the peritoneal cavity; distance indicated by double-headed white arrow = 3 mm. C, Assembled trocar. (Images Courtesy of Applied Medical Resources Corporation. All rights reserved.)

FIGURE 1-3-7  Genicon Bladeless Tip Trocar System. (Image courtesy of Genicon Corporation.)

FIGURE 1-3-8  Ternamian EndoTIP (Endoscopic Threaded Imaging Port, Karl Storz). Inset: Close-up of threaded cannula. (© 2012 Photo Courtesy of Karl Storz Endoscopy-America, Sugar Land, Tex.)

CHAPTER 1-3  ■  Common Access Techniques  

FIGURE 1-3-9  A, Insertion locations for the Veress needle, optical trocar, and blunt (Hasson) trocar. Approximate location of epigastric vessels shown. B, Angle of insertion for lateral entry points; transverse plane. a, Appropriate angle for device inserted at Palmer’s point. b, Inappropriate and (c) appropriate angles, respectively, for a device inserted lateral to Palmer’s point.

e17

e18   SECTION 1  ■  ABDOMEN – GENERAL

FIGURE 1-3-10  Open laparoscopy technique. A, Curvilinear infraumbilical skin incision has been made, and the midline raphe can be seen running caudally from the invaginated periumbilical skin; the raphe is placed on stretch between the Kocher clamp and the Adson forceps. B, Exposure of the infraumbilical linea alba by incising the midline raphe with an electrocautery blade. C, Tips of the DeBakey forceps demonstrate the site of the transverse incision through the linea alba for the blunt cannula. D and E, Transverse 1-cm incision has been made through the linea alba, and traction sutures of 1-0 polyglactin have been placed in the incision corners. Continued

CHAPTER 1-3  ■  Common Access Techniques  

e19

FIGURE 1-3-10, cont’d F, Peritoneal cavity is entered with the closed end of a hemostat.

FIGURE 1-3-11  Optical trocar technique. A, Trocar has been inserted through the skin and sits within the subcutaneous fat. B, Trocar has penetrated through the anterior fascial layer, under which the abdominal wall musculature is visible. C, Trocar has penetrated down to but not through the posterior fascial layer and peritoneum. The center of the trocar is shown just about to breach the peritoneum. D, Tip of trocar has entered the peritoneal cavity; omental fat is present directly beneath. E, CO2 has been insufflated through the trocar with the camera still inserted, developing an adequate intraabdominal space into which the trocar may safely be advanced.

1-4 

LAPAROSCOPY-ASSISTED ERCP L. Tercio  /  R. Clements  /  C. Wilcox From Laparoscopy-assisted ERCP: experience of a high-volume bariatric surgery center. Gastrointest Endosc 2009;70(6):1254–1259

The video for this procedure can be accessed here

e20

SELF ASSESSMENT Kyle A. Perry  /  Jonathan A. Myers From Velasco J: Rush University Medical Center Review of Surgery, 5th edition (Saunders 2011)

1-5 

1. Which of the following hemodynamic parameters decreases during laparoscopy with CO2 pneumoperitoneum? A. Mean arterial pressure B. Pulmonary vascular resistance C. Systemic vascular resistance D. Heart rate E. Peripheral blood flow Ref.: 1 COMMENTS: CO2 pneumoperitoneum produces consistent cardiopulmonary effects, but their magnitude depends on several factors, including the anesthetic agent, patient cardiopulmonary status, and metabolic factors. Cardiac effects include decreased venous return, which produces decreased preload, stroke volume, and cardiac output. Direct myocardial depression from CO2 pneumoperitoneum also reduces stroke volume and cardiac output. These changes produce a compensatory increase in heart rate and systemic and pulmonary vascular resistance. Increased intra-abdominal pressure on the aorta, vena cava, and splanchnic vasculature along with the compensatory release of renin and vasopressin produces increased systemic vascular resistance and decreased peripheral blood flow.

ANSWER: E 2. Which of the following conditions represents a contraindication to advanced laparoscopic operations? A. Pregnancy B. Morbid obesity C. Contraindication to general anesthesia D. Previous laparotomy E. Cirrhosis Ref.: 1 COMMENTS: The indications for laparoscopic surgery are generally the same as those for open surgery, but several factors may complicate or increase the difficulty associated with laparoscopic surgery. High-risk (American Society of Anesthesiologists [ASA] class IV) patients are not ideal candidates for laparoscopic surgery because of the prerequisite for establishing pneumoperitoneum. Gasless laparoscopy with regional anesthesia has been used in the past to avoid the use of gas insufflation. Yet high-risk patients may not tolerate the increased intra-abdominal pressure required for adequate visualization. Morbid obesity, previous abdominal surgery, and pregnancy are no longer considered contraindications to laparoscopy; however, these patients are at high risk for the development of complications of surgery and anesthesia, so adequate preoperative preparation is essential. e21

e22   SECTION 1  ■  ABDOMEN – GENERAL Morbidly obese patients may be difficult to intubate and might require large doses of muscle relaxants. Proper positioning is paramount to prevent nerve injuries. Selecting an appropriate access point for establishing pneumoperitoneum is important in those who have previously undergone laparotomy. Ultrasound imaging of the abdominal wall (visceral sliding technique) and selection of the left upper quadrant as a point of entry (Palmer’s point) are useful techniques. These considerations are important in pregnant patients to maintain adequate fetal blood flow and prevent uterine injury. Although it increases the likelihood of complications, early cirrhosis with preserved hepatic synthetic function is no longer considered an absolute contraindication to laparoscopy.

ANSWER: C 3. Which of the following is not an appropriate technique for initial trocar placement during laparoscopic surgery? A. Veress needle insufflation followed by blind trocar placement B. Open placement of a Hasson cannula without pneumoperitoneum C. Optical trocar placement D. Blind trocar placement without pneumoperitoneum E. All of the above are acceptable Ref.: 2 COMMENTS: Initial intraperitoneal access for laparoscopic surgery may be obtained with a number of open or closed approaches. One standard closed method involves inserting a Veress needle to achieve insufflation, most commonly at the umbilicus or in the left upper quadrant, followed by blind trocar placement. Another closed technique involves using an optical trocar to visualize the abdominal wall layers during insertion. Open access techniques use a cutdown and open the fascia and peritoneum under direct vision. A blunt Hasson trocar is then placed and secured with a conical sleeve before peritoneal insufflation. Blind trocar placement alone is not a recommended technique for abdominal access. Aside from this, with appropriate training and good surgical judgment, all of the remaining methods listed can be used safely.

ANSWER: D

References 1. Jamal MK, Scott-Connor CH: Patient selection and practical considerations in laparoscopic surgery. In Soper NJ, Swanstrom LL, Eubanks WS, editors: Mastery of endoscopic and laparoscopic surgery, ed 3, Philadelphia, 2009, Lippincott Williams & Wilkins. 2. Fingerhut A, Millat B, Borie F: Prevention of complications in laparoscopic surgery. In Soper NJ, Swanstrom LL, Eubanks WS, editors: Mastery of endoscopic and laparoscopic surgery, ed 2, Philadelphia, 2005, Lippincott Williams & Wilkins.

Exploratory Laparotomy – Open

GOALS/OBJECTIVES • • •

REVIEW PERTINENT ANATOMY INDICATIONS TECHNICAL CONSIDERATION

2 

2-1 

ACUTE ABDOMEN Ronald A. Squires  /  Russell G. Postier From Algorithms in the acute abdomen. In: Townsend CM: Sabiston Textbook of Surgery, 19th edition (Saunders 2012)

Box 2-1-1  NONSURGICAL CAUSES OF THE ACUTE ABDOMEN Endocrine and Metabolic Causes

Toxins and Drugs

Uremia Diabetic crisis Addisonian crisis Acute intermittent porphyria Hereditary Mediterranean fever

Lead poisoning Other heavy metal poisoning Narcotic withdrawal Black widow spider poisoning

Hematologic Causes Sickle cell crisis Acute leukemia Other blood dyscrasias

Box 2-1-2  SURGICAL ACUTE ABDOMINAL CONDITIONS Hemorrhage

Perforation

Solid organ trauma Leaking or ruptured arterial aneurysm Ruptured ectopic pregnancy Bleeding gastrointestinal diverticulum Arteriovenous malformation of gastrointestinal tract Intestinal ulceration Aortoduodenal fistula after aortic vascular graft Hemorrhagic pancreatitis Mallory-Weiss syndrome Spontaneous rupture of spleen

Perforated gastrointestinal ulcer Perforated gastrointestinal cancer Boerhaave’s syndrome Perforated diverticulum

Infection Appendicitis Cholecystitis Meckel’s diverticulitis Hepatic abscess Diverticular abscess Psoas abscess

e24

Blockage Adhesion induction small/large bowel obstruction Sigmoid volvulus Cecal volvulus Incarcerated hernias Inflammatory bowel disease Gastrointestinal malignancy Intussusception Ischemia Buerger’s disease Mesenteric thrombosis/embolism Ovarian torsion Ischemic colitis Testicular torsion Strangulated hernias

CHAPTER 2-1  ■  Acute Abdomen  

Box 2-1-3  LOCATIONS AND CAUSES OF REFERRED PAIN Right Shoulder

Left Shoulder

Liver Gallbladder Right hemidiaphragm

Heart Tail of pancreas Spleen Left hemidiaphragm Scrotum and Testicles Ureter

TABLE 2-1-1  Abdominal Examination Signs Sign

Description

Diagnosis or Condition

Aaron

Pain or pressure in epigastrium or anterior chest with persistent firm pressure applied to McBurney’s point Sharp pain created by compressing appendix between abdominal wall and iliacus Transient abdominal wall rebound tenderness Loss of abdominal tenderness when abdominal wall muscles are contracted Extreme lower abdominal and pelvic pain with movement of cervix Intermittent right upper abdominal pain, jaundice, and fever Accentuation of breath and cardiac sounds through abdominal wall Palpable gallbladder in presence of jaundice Varicose veins at umbilicus (caput medusa) Periumbilical bruising Shoulder pain on inspiration Abdominal wall mass that does not cross midline and remains palpable when rectus contracted Local areas of discoloration around umbilicus and flanks Elevation and extension of leg against resistance creates pain

Acute appendicitis

Bassler Blumberg Carnett Chandelier Charcot Claybrook Courvoisier Cruveihier Cullen Danforth Fothergill Grey Turner Iliopsoas Kehr Mannkopf Murphy Obturator Ransohoff Rovsing Ten Horn

Left shoulder pain when supine and pressure placed on left upper abdomen Increased pulse when painful abdomen palpated Pain caused by inspiration while applying pressure to right upper abdomen Flexion and external rotation of right thigh while supine creates hypogastric pain Yellow discoloration of umbilical region Pain at McBurney’s point when compressing the left lower abdomen Pain caused by gentle traction of right testicle

Chronic appendicitis Peritoneal inflammation Intra-abdominal source of abdominal pain Pelvic inflammatory disease Choledocholithiasis Ruptured abdominal viscus Periampullary tumor Portal hypertension Hemoperitoneum Hemoperitoneum Rectus muscle hematomas Acute hemorrhagic pancreatitis Apppendicitis with retrocecal abscess Hemoperitoneum (especially from splenic origin) Absent if malingering Acute cholecystitis Pelvic abscess or inflammatory mass in pelvis Ruptured common bile duct Acute appendicitis Acute appendicitis

EVALUATION AND DIAGNOSIS Laboratory Studies A number of laboratory studies are considered routine in the evaluation of a patient with an acute abdomen (Box 2-1-4). They help confirm that inflammation or infection is present and also aid in the elimination of some of the most common nonsurgical conditions. A complete blood count with differential is valuable because most patients with an acute abdomen will have a leukocytosis or bandemia. Measurement of serum electrolyte, blood urea nitrogen, and creatinine levels will assist in evaluating the effect of factors such as vomiting or third space fluid losses. In addition, they may suggest an endocrine or metabolic diagnosis as the cause of the patient’s problem. Serum amylase and lipase level determinations may suggest pancreatitis as the cause of the abdominal pain but can also be elevated in other disorders, such as small bowel infarction or duodenal ulcer perforation. Normal serum amylase and lipase levels do not exclude pancreatitis as a possible diagnosis caused by the effects of chronic inflammation on enzyme production and timing factors. Liver function tests, including determination

e25

e26   SECTION 1  ■  ABDOMEN – GENERAL Box 2-1-4  LABORATORY STUDIES FOR THE ACUTE ABDOMEN Hemoglobin level White blood cell count with differential Electrolyte, blood urea nitrogen, creatinine levels Urinalysis Urine human chorionic gonadotropin level Amylase, lipase levels

Total and direct bilirubin levels Alkaline phosphatase level Serum aminotransferase Serum lactate levels Stool for ova and parasites C. difficile culture and toxin assay

FIGURE 2-1-1  Appendicitis. A, CT scan of uncomplicated appendicitis. A thick-walled, distended, retrocecal appendix (arrow) is seen with inflammatory change in the surrounding fat. B, CT scan of complicated appendicitis – a retrocecal appendiceal abscess (A) with an associated phlegmon posteriorly found in a 3-week postpartum, obese woman. Inflammatory change extends through the flank musculature into the subcutaneous fat (arrow).

of total and direct bilirubin, serum aminotransferase, and alkaline phosphatase levels are helpful in evaluating potential biliary tract causes of acute abdominal pain. Lactate levels and arterial blood gas determinations can be helpful in diagnosing intestinal ischemia or infarction. Urine testing, such as urinalysis, is helpful in the diagnosis of bacterial cystitis, pyelonephritis, and certain endocrine abnormalities, such as diabetes or renal parenchymal disease. Urine culture can confirm a suspected urinary tract infection and direct antibiotic therapy but cannot be done in time to be helpful in the evaluation of an acute abdomen. Urinary measurements of human chorionic gonadotropin level can suggest pregnancy as a confounding factor in the patient’s presentation or aid in decision-making regarding therapy. The fetus of a pregnant patient with an acute abdomen is best protected by providing the best care to the mother, including surgery, if indicated. Stool testing for occult blood can be helpful in the evaluation of these patients but is nonspecific. Testing stool for ova and parasite evaluation, as well as culture and toxin assay for Clostridium difficile, can be helpful if diarrhea is a component of the patient’s presentation.

Imaging Studies Improvements in imaging techniques, especially multidetector CT, have revolutionized diagnosis of the acute abdomen. The most difficult diagnostic dilemmas of the past – appendicitis in young women and ischemic bowel in older adults – can now be diagnosed with greater certainty and speed (Figures 2-1-1 and 2-1-2). This has resulted in more rapid operative correction of the problem, with less morbidity and mortality. Despite its usefulness, CT is not the only imaging technique available and is also not the first step in imaging for most patients. In addition, no imaging technique can replace a careful history and physical examination. Plain radiographs continue to play a role in imaging for patients with acute abdominal pain. Upright chest radiographs can detect as little as 1 mL of air injected into the peritoneal cavity. Lateral decubitus abdominal radiographs can also detect pneumoperitoneum effectively in patients who cannot stand; as little as 5 to 10 mL of gas may be detected with this technique. These studies are particularly helpful for patients suspected of having a perforated duodenal ulcer, because approximately 75% of these patients will have a large enough pneumoperitoneum to be visible (Figure 2-1-3). This obviates the need for further evaluation in most patients, allowing for laparotomy with little delay.

CHAPTER 2-1  ■  Acute Abdomen  

e27

FIGURE 2-1-2  Small bowel infarction associated with mesenteric venous thrombosis. A, Note the low-density thrombosed superior mesenteric vein (solid arrow) and incidental gallstones (open arrow). B, Thickening of proximal small bowel wall (arrow) coincided with several feet of infarcting small bowel at time of operation.

FIGURE 2-1-3  Upright chest radiograph depicting moderate-sized pneumoperitoneum consistent with perforation of abdominal viscus.

Plain films also show abnormal calcifications. Approximately 5% of appendicoliths, 10% of gallstones, and 90% of renal stones contain sufficient amounts of calcium to be radiopaque. Pancreatic calcifications seen in many patients with chronic pancreatitis are visible on plain films, as are the calcifications in abdominal aortic aneurysms, visceral artery aneurysm, and atherosclerosis in visceral vessels. Upright and supine abdominal radiographs are helpful in identifying gastric outlet obstruction, and obstruction of the proximal, mid, or distal small bowel. They can also aid in determining whether a small bowel obstruction is complete or partial by the presence or absence of gas in the colon. Colonic gas can be differentiated from small intestinal gas by the presence of haustral markings caused by the taenia coli present in the colonic wall. An obstructed colon appears as distended bowel with haustral markings (Figure 2-1-4). Associated distention of small bowel may also be present, especially if the ileocecal valve is incompetent. Plain films can also suggest volvulus of the cecum or sigmoid colon. Cecal volvulus is identified by a distended loop of colon in a comma shape, with the concavity facing

e28   SECTION 1  ■  ABDOMEN – GENERAL

FIGURE 2-1-4  Upright abdominal x-ray in a patient with an obstructing sigmoid adenocarcinoma. Note the haustral markings on the dilated transverse colon that distinguished this from small intestine.

FIGURE 2-1-5  Upright abdominal x-ray in a patient with a sigmoid colon volvulus. Note the characteristic appearance of a bent inner tube, with its apex in the right upper quadrant.

inferiorly and to the right. Sigmoid volvulus characteristically has the appearance of a bent inner tube, with its apex in the right upper quadrant (Figure 2-1-5). Abdominal ultrasonography is extremely accurate for detecting gallstones and assessing gallbladder wall thickness and presence of fluid around the gallbladder. It is also helpful for determining the diameter of the extrahepatic and intrahepatic bile ducts. Its usefulness in detecting common bile duct stones is limited. Abdominal and transvaginal ultrasonography can aid in the detection of abnormalities of the ovaries, adnexa, and uterus. Ultrasound can also detect intraperitoneal fluid. The presence of abnormal amounts of intestinal air in most patients with an acute abdomen limits the ability of ultrasonography to evaluate the pancreas or other abdominal organs. There are important limits to the value of ultrasonography in the diagnosis of diseases that present as an acute abdomen. Ultrasound

CHAPTER 2-1  ■  Acute Abdomen  

images are more difficult for most surgeons to interpret than plain radiographs and CT scans. Many hospitals have radiologic technologists available at all times to perform CT but this is often not the case with ultrasonography. As CT has become more widely available and less likely to be hindered by abdominal air, it is becoming the secondary imaging modality of choice in the patient with an acute abdomen, following plain abdominal radiography. A number of studies have demonstrated the accuracy and usefulness of CT of the abdomen and pelvis in the evaluation of acute abdominal pain. Many of the most common causes of the acute abdomen are readily identified by CT scanning, as are their complications. A notable example is appendicitis. Plain films and even barium enemas add little to the diagnosis of appendicitis; however, a well-performed CT using oral, rectal, and IV contrast is highly accurate for evaluating this disease. It is equally important that an experienced radiologist, accustomed to reading abdominal CT scans, interprets the study to maximize the sensitivity and specificity of the exam. A prospective study from the Netherlands has illustrated the variability of CT interpretation in the diagnosis of appendicitis. Three blinded groups of radiologists read CT scans of patients suspected of having appendicitis. All patients then underwent exploratory laparoscopy and 83% of patients were found to have appendicitis at surgery. Radiology group A was made up of radiology residents on call and trained in CT interpretation. Group B consisted of call staff radiologists; group C was composed of expert abdominal radiologists. For groups A, B, and C radiologists, the sensitivities of CT scanning for the diagnosis of acute appendicitis were 81%, 88%, and 95%, the specificities were 94%, 94%, and 100%, and the negative predictive values were 50%, 68%, and 81%, respectively. Differences between groups A and C were statistically significant. CT is also excellent for differentiating mechanical small bowel obstruction from paralytic ileus and can usually identify the transition point in mechanical obstruction (Figure 2-1-6). Some of the most difficult diagnostic dilemmas, including acute intestinal ischemia and bowel injury following blunt abdominal trauma, can often be identified by this method. Traumatic small bowel injuries can be a clinical diagnosis challenge. Associated abdominal wall, pelvic, or spinal injuries can be significant distracters that could compromise an otherwise careful history and physical examination. In addition, many patients suffering a blunt abdominal trauma will have altered mental states from coexisting closed head injuries or from intoxicating substances. When a bowel injury is suspected, optimal CT scanning uses oral and IV contrast agents. Zissin and colleagues have reported an overall sensitivity of 64%, specificity of 97%, and accuracy of 82% when diagnosing small bowel injury following blunt trauma using dual-contrast CT scanning. Diagnostic clues include recognition of bowel wall thickening, identification of any gas outside the lumen of the intestine, and a moderate to large amount of intraperitoneal fluid without visible solid abdominal organ injury.

FIGURE 2-1-6  CT scan of a patient with a partial small bowel obstruction. Note the presence of dilated small bowel and decompressed small bowel. The decompressed bowel contains air, indicating a partial obstruction.

e29

e30   SECTION 1  ■  ABDOMEN – GENERAL

FIGURE 2-1-7  Algorithm for the treatment of acute-onset, severe, generalized abdominal pain. NG, nasogastric tube; NL, normal study; OR, operation.

FIGURE 2-1-8  Algorithm for the treatment of gradual-onset, severe, generalized abdominal pain. ERCP, endoscopic retrograde cholangiopancreatography; LFTs, liver function tests.

SELF ASSESSMENT Nadine D. Floyd  /  Theodore J. Saclarides From Velasco J: Rush University Medical Center Review of Surgery, 5th edition (Saunders 2011)

2-2 

1. A 35-year-old woman experiences an acute onset of epigastric and right upper quadrant pain several hours after a large dinner. She has had similar episodes in the past that resolved after a few hours. This episode persists, and she has fever and nonbilious vomiting. What is the most likely source of the abdominal pain? A. Perforated ulcer B. Acute appendicitis C. Perforation following bowel obstruction D. Cholecystitis E. Diverticulitis Ref.: 2–4 COMMENTS: See Question 5.

ANSWER: D 2. A 60-year-old man with chronic alcoholism awakens at 3:00 am with severe, sharp epigastric pain that 3 hours later becomes diffuse abdominal pain. What is the most likely source of the abdominal pain? A. Perforated ulcer B. Acute appendicitis C. Perforation following bowel obstruction D. Cholecystitis E. Diverticulitis Ref.: 1–3 COMMENTS: See Question 5.

ANSWER: A 3. A 55-year-old man with a 2-day history of abdominal distention, vomiting, crampy abdominal pain, and obstipation is experiencing severe, diffuse abdominal pain. What is the most likely source of the abdominal pain? A. Perforated ulcer B. Acute appendicitis C. Perforation following bowel obstruction D. Cholecystitis E. Diverticulitis Ref.: 1–3 COMMENTS: See Question 5.

ANSWER: C 4. A 22-year-old man awakens with periumbilical abdominal pain followed by nonbilious vomiting. What is the most likely source of the abdominal pain? A. Perforated ulcer B. Acute appendicitis e31

e32

SECTION I



ABDOMEN - GENERAL

C. Perforation following bowel obstruction D. Cholecystitis E. Diverticulitis

Ref.: 2-4

COMMENTS: See Qtestion 5. ANSWER: B

5. A 65-year-old man with a history of chronic constipation has a 3-day history of abdominal disten­ tion without a bowel movement. He has fever and abdominal rigidity. What is the most likely source of the abdominal pain? A. Perforated ulcer B. Acute appendicitis C. Perforation following bowel obstruction D. Cholecystitis E. Diverticulitis

Ref.: 2-4 COMMENTS: The examples in Qtestions 1 to 5 demonstrate the importance of a thorough history

in determining a patient's diagnosis and tailoring the initial work-up in the management of an acute abdomen. Differentiating between patients who require immediate interv:en.tion and those who can undergo a more gradual work-up is also essential to avoid unnecessarx delays in treatment. Biliary pain is typically midepigastric, with radiation to the right upper uadrant and right subscapular area. It often occurs after the intake of fatty food. It may be intermittent, associated with nausea and vomiting. Patients with a perforated ulcer will classically remember the exact moment when the perforation occurred. There may be an initial period of diminished pain followed by severe pain when diffuse chemical peritonitis sets in. Risk factors include a previous history o£p€ptic ulcer disease, untreated Helicobacterpylori infection, use of medications such as steroids and nonsteraiclal antiinflammatory drugs, and alcohol abuse. When vomiting is part of the history, it is im ortant to differentiate between patients with mechani­ cal obstruction of the bowel, bile duct, Oli pancreatic duct and patients who have ileus in response to problems from a nonintestinal source. A atient with acute appendicitis and periumbilical pain may have one or two episodes of nonbilious emesis before localization of pain in the lower right quadrant. The early abdominal pain and vomiting associated with appendicitis may resemble gastroenteritis. However, in appendicitis, pain is the predominant clinical feature and precedes diarrhea and vomiting in most instances. With gastroenter"tis, the vomiting is typically more profuse and frequent and may be accompanied by rofuse iarrhea as well. A history of weight loss or new-onset obstipation and changes in stool patterns may suggest a colorectal malignancy:. The duration of time over which these symptoms have developed and progressed may give insignt regaraing the urgency of the problem. A patient with a 3-day history of progressively obstructive symptoms (i.e., distention, crampy pain, and vomiting) and who has peritonitis and fever is more likely to have a complicated obstruction (e.g., ischemic, gangrenous, or perforated bowel) requiring immediate intervention. If the pain is diffuse, it may herald a free perforation causing diffuse contamination of the peritoneal cavity. If the pain is localized, it may represent a contained perforation, as can occur with diverticulitis. This type of pain typically occurs in the lower left quadrant. In contrast, weight loss, cachexia, a slow decrease in stool caliber, and mild cramping reflect a more gradual process that permits elective work-up and treatment. ANSWER: E References 1. Postier RG, Squires RA: Acute abdomen. In Townsend CM, Beauchamp RD, Evers BM et al, editors: Sabiston textbook of surgery: the biological basis of modern surgical practice, ed 18, Philadelphia, 2008, WB Saunders. 2. Silen W, editor: Cape's early diagnosis of the acute abdomen, ed 19, New York, 1996, Oxford University Press. 3. Martin RF, Rossi RL: The acute abdomen: an overview and algorithms, Surg Clin North Am 77:1227-1243, 1997. 4. Jaffe BM, Berger DH: The appendix. In Brunicardi FC, Andersen DK, Billiar TR, et al, editors: Schwartz's principles of surgery, ed 9, New York, 2010, McGraw-Hill.

Peritoneal Dialysis Catheter Insertion

GOALS/OBJECTIVES • • • •

BASIC PRINCIPLES ANATOMY TECHNICAL CONSIDERATIONS MANAGEMENT OF COMPLICATIONS

3 

3-1 

PERITONEAL DIALYSIS Ricardo Correa-Rotter  /  Alfonso Cueto-Manzano  /  Ramesh Khanna From Taal MW, Chertow GM, et al: Brenner and Rector’s The Kidney, 9th edition (Saunders 2011)

PERITONEAL PHYSIOLOGY AND TRANSPORT During PD, both diffusion and convection are responsible for solute transport. Diffusion results from a difference in solute concentrations across a membrane, which in turn is governed by Fick’s first law of diffusion (the rate of transfer of a solute is determined by the diffusive permeability of the membrane to that solute, the surface area available for transport, and the concentration). Convective transport, or solute drag, occurs with water transport during ultrafiltration. Determinants of convective transport include the water flux, the mean solute concentration, and the solute reflection coefficient.1 The reflection coefficient in semipermeable membranes is related to how such a membrane can prevent solute particles from passing through. When the value is 0, all particles pass through. When the value is 1, no particle can pass through. The effective surface area of the peritoneal membrane and its intrinsic permeability determine the ability of solutes to be transported. The effective surface area is determined by the number of capillaries perfused and the rate of flow within these capillaries.2,3 There are several barriers to the transport of solutes across the peritoneum.4,5 The peritoneal capillary represents the major barrier for peritoneal transport. According to the two-pore theory of capillary transport, small pores of 40 to 50 Å are abundant, and large pores larger than 150 Å are sparse.6,7 After the demonstration of aquaporin-1–mediated water transport through red blood cells,8,9 these even smaller pores were also described in endothelial cells of peritoneal capillaries and venules.10,11 To explain the phenomenon of sieving observed in PD, a three-pore model was proposed.12,13 According to this model, about half of the transcapillary ultrafiltration occurs through ultra-small pores 3 to 5 Å in size, and the other half occurs through the small pores.14,15 The mesothelium has been shown not to be a significant barrier to small solute transport.5 However, the interstitium, despite its meshlike architecture,16 could be such a barrier.17–19 Changes in solute transport are currently assumed to result from the ultrastructural alteration of perfused capillaries. The mass transfer of solutes of low and medium molecular weight is dependent primarily on their size and not on the intrinsic permeability of the peritoneum.20,21 The stagnant blood in the capillaries and peritoneal cavity could offer some resistance to solute transport.22,23 Macromolecular transport is size selective; therefore, diffusion and convection are limited.12,24 Hence, transport is dependent primarily both on the effective surface area and permeability of the membrane and on the molecular size of the solute. In animal models, the negative electric charges at various levels of peritoneum and microvessels25,54 appear to restrict clearance of macromolecules, but this is not so in humans.24,26,27 In summary, the transport of solutes of low molecular weight is affected by changes in effective surface area of the peritoneal membrane, and macromolecule transport is determined mainly by the structural alteration of the capillary wall or changes in the interstitium, or both. Several distributed models and solute transport parameters have been proposed18,28–31 to describe kinetics during a PD exchange. The mass transfer area coefficient (MTAC) is the theoretical instantaneous maximal clearance at time 0 without ultrafiltration. Several simple and other more complicated models have been developed to calculate MTAC.32 In contrast, in clinical practice, relatively simple measures – such as 24-hour clearance or 4-hour dialysate/plasma (D/P) ratios of low-molecular-weight solutes – are used to assess the efficacy and adequacy of PD. Studies have shown good correlation e34

CHAPTER 3-1  ■  Peritoneal Dialysis  

between D/P ratios and MTAC for all ranges except in the low and high extremes of MTAC.33 During a dialysis exchange, diffusion accounts for the majority of the mass removal of solutes of lower molecular weight (e.g., urea or creatinine)34; convective transport is a small fraction of total mass removal.35 Water transport during PD is driven through an osmotic gradient, generated by agents such as glucose. Under physiologic conditions, the differences among hydrostatic, crystalloid, and colloid osmotic pressures in the peritoneal capillaries and the peritoneal cavity allow for a small amount of transcapillary ultrafiltration into the peritoneum to occur continuously. These pressures are exerted over small pores and through water (aquaporin-1) channels in the endothelium of peritoneal capillaries, which results in transcapillary ultrafiltration. There is a constant backward absorption of water from the peritoneal cavity through transcapillary backwards filtration and fluid uptake through peritoneal lymphatic vessels.36 Therefore, the net amount of ultrafiltration (water removal from the body) during a PD exchange is a balance between the transcapillary ultrafiltration from the peritoneal capillaries into the peritoneum and the backwards absorption of fluid from the cavity through capillaries and lymphatic vessels. Because peritoneal dialysate is devoid of proteins, it exerts only crystalloid osmotic pressure and hydrostatic pressure in the peritoneal cavity. The effectiveness of dialysate in inducing ultrafiltration is expressed by the osmotic reflection coefficient. Impermeable solutes such as macromolecular protein exert a reflection coefficient of 1, and freely permeable solutes exert a reflection coefficient of 0. Therefore, during a PD exchange, the reflection coefficient for glucose transport through small pores is very low, at 0.02 to 0.05,37–39 and for glucose transport through aquaporin-1, it is 1. The events of water transport that take place during a PD exchange with glucose-based dialysate can be summarized as follows36: At the beginning of an exchange (time 0), the glucose concentration of the dialysate is highest, and therefore, crystalloid osmotic pressure and ultrafiltration rate are both also highest. As glucose is absorbed from the dialysate (approximately 61% of the total glucose content of a solution over a 4 hour period),40 the crystalloid pressure and ultrafiltration diminish. Ultrafiltration volume accumulates progressively, and the ultrafiltration volume peaks before osmotic equilibrium between serum and dialysate is reached; this equilibrium occurs when the net transcapillary ultrafiltration rate progressively diminishes to equal rates of backwards absorption, plus lymphatic absorption. Thereafter, when the back absorption rate exceeds the net transcapillary ultrafiltration rate, the intraperitoneal volume slowly diminishes. With further extension of dwell time, additional fluid would be absorbed. Patients deemed “high transporters” experience rapid transport of small solutes; thus, these patients experience more rapid (and often more extensive) dialysate glucose absorption and less cumulative transcapillary ultrafiltration. Ultrafiltration fails when daily reabsorption equals or exceeds daily transcapillary ultrafiltration.

THE PERITONEAL CATHETER AND ACCESS Key to successful PD therapy is permanent and safe access to the peritoneal cavity. A good catheter provides obstruction-free access to the peritoneum. In addition, it should not be a source of peritoneal infection. Catheter-related problems and infections are responsible for approximately 20% of technique failure.41 Peritoneal catheters in current use have intraperitoneal and extraperitoneal segments.

TABLE 3-1-1  Considerations for Adequate Dialysis Clinical manifestations Fluid balance, systemic blood pressure control, and cardiovascular risk Renal residual function Acid-base homeostasis Nutritional status Calcium-phosphorous metabolism homeostasis Inflammation Small solute clearance Middle molecule clearance Psychologic and quality-of-life indicators

e35

e36   SECTION 1  ■  ABDOMEN – GENERAL

FIGURE 3-1-1  Intraperitoneal and extraperitoneal designs of currently available peritoneal catheters.

The extraperitoneal segment passes through a tunnel within the abdominal wall (intramural), exits through the skin, and has an external (outside the exit site) segment. Figure 3-1-1 shows different intraperitoneal and extraperitoneal designs of currently available peritoneal catheters. Globally, the catheter most widely used is the Tenckhoff catheter, followed by the swan-neck catheter.42 More than 90% of the catheters used have two cuffs, and the majority incorporate a coiled intraperitoneal segment. Tenckhoff catheters are made of silicone rubber tubing with a 2.6-mm internal diameter and a 5-mm external diameter. The catheter may contain one or two polyester cuffs, 1 cm long. The straight double-cuff catheter is about 40 cm long with an intraperitoneal segment about 15 cm long, an intramural segment about 5 to 7 cm long, and an external segment about 16 cm long. The open-ended intraperitoneal segment has multiple 0.5-mm side openings in the terminal 11-cm segment. The coiled Tenckhoff catheter has a coiled, perforated intraperitoneal end that is 18.5 cm long. Most Tenckhoff catheters have a barium-impregnated radiopaque stripe throughout the catheter length to assist in radiologic visualization. The swan-neck catheter, a modified Tenckhoff catheter, features a molded bend between cuffs.42,43 These catheters can be placed in an arcuate tunnel with both external and internal segments of the tunnel directed downwards. A long tunnel, downward-directed exit, two intramural cuffs, and an optimal sinus length tend to reduce exit and tunnel infection rates. The molded bend between cuffs eliminates the rubber “shape memory” from causing the external cuff extrusion. A downward-directed peritoneal entrance, aided by a slanted polyester disc, a feature similar to one in the Toronto Western catheter (described later), tends to keep the internal segment in the true pelvis, reducing its migration. Insertion of catheters through the rectus muscle decreases pericatheter leaks by promoting fibrous ingrowth onto the polyester cuff. Finally, swan-neck catheters with a coiled intraperitoneal segment minimize infusion and pressure pain. The intraperitoneal segment of the swan-neck catheter is identical to that of the Tenckhoff catheter in that its terminal segment is either straight or coiled. Presternal catheters were designed to allow for an exit site above the abdomen. The chest is a rather rigid structure with minimal wall motion; the catheter exit located on the chest wall is subjected to

CHAPTER 3-1  ■  Peritoneal Dialysis  

minimal trauma; therefore, chances of contamination are decreased. Also, in patients with abdominal ostomies and in children with diapers, a chest exit location reduces the chances of cross contamination. Implantation directly over the sternum should be avoided, so as to prevent catheter damage during any cardiac surgery that necessitates sternotomy. A long catheter tunnel, combined with three cuffs, may reduce pericatheter bacterial contamination of the peritoneal cavity and hence reduce the incidence of peritonitis.44 The presternal catheter is composed of two silicone rubber tubes, cut to an appropriate length and connected end to end at the time of implantation. The lower tube, including the internal segment, is identical to that of the swan-neck abdominal catheter. A titanium connector is used to connect the two components at the time of implantation. The Moncrief-Popovich catheter is a modified swan-neck coiled catheter with a longer subcutaneous cuff (2.5 cm instead of 1 cm). This catheter is inserted with the Moncrief–Popovich implantation technique.45 The other catheters in use are the T-fluted catheter; the self-locating catheter; the Cruz catheter; the Toronto Western Hospital catheter; the Ash (Life) catheter; the column disc catheter; and the Gore-Tex peritoneal catheter.46 Rigid catheters for acute dialysis, rarely used in developed nations, are still used in some countries. Complications of rigid catheter insertion include minor bleeding; leakage of dialysis solution; extravasation of fluid into the abdominal wall, particularly in patients who have had a previous abdominal operation or multiple catheter insertions; and inadequate drainage as a result of omental wrapping, loculation, or misplaced catheter in the upper abdomen. Loss of a part or the entire rigid catheter after manipulation of a poorly functioning rigid catheter has been reported. The incidence of peritonitis varies widely with rigid catheters; the rate may be dependent on the duration of dialysis and catheter manipulation, among other factors.47,48 For long-term use, PD catheters such as the Tenckhoff or swan-neck can be inserted surgically42 at the patient’s bedside by an experienced nephrologist or by a surgeon or through a laparoscopic insertion, a procedure that has gained favor.46

CATHETER-RELATED COMPLICATIONS The most common complications of PD catheters include exit and tunnel infection, external cuff extrusion, poor function, dialysate leaks, peritonitis, and infusion or pressure pain. Several factors adversely influence the normal healing process and lead to early infections: foreign body-induced tissue reaction, poor tissue perfusion, mechanical factors, sinus bacterial colonization, delayed epithelialization, local cleansing agents, exit direction, and several other systemic problems. After the exit site is well healed, a factor that predisposes to infection is bacterial colonization of the sinus tract in association with local trauma.46,49–52 The catheter tip, as it rests against the pelvic wall or intra-abdominal organs,51 may cause localized pain from irritation. The jet effect of rapidly flowing dialysis solution may also cause abdominal pain. In some rare instances, compartmentalization from adhesion formation around the catheter may cause severe abdominal pain.53 Coiled catheters are less likely to induce abdominal pain than are straight catheters. The extrusion of the external synthetic cuff can be prevented by creating the tunnel in a shape similar to the shape of the catheter and placing this cuff approximately 2 to 3 cm beneath the skin. In the absence of catheter infection, shaving off the extruded external cuff may help prolong the life of the catheter.50 Entrapment, or “capture,” of the catheter by the active omentum may cause outflow obstruction in the postimplantation period. Omental “capture” as a late event is rare. From time to time in some patients, drainage slows as a result of catheter translocation, obstruction by omentum, or fibrin clot formation. Laxatives or addition of heparin, 500 U per liter of dialysis solution, or both may be successful in restoring good dialysate flow. In some patients, catheters have migrated out of the true pelvis. If the catheter continues to function appropriately, it is not necessary to reposition it. If the catheter fails to function after simple maneuvers are implemented, more aggressive measures (e.g., laxatives, forced flushing) may be tried. When these measures fail, laparoscopic repositioning of the catheter tip back to the true pelvis and anchoring may be necessary. The Toronto Western catheter has two silicone discs in the intraperitoneal segment of the catheter that hinder the free movement of catheter tip out of the pelvis after placement.46

e37

e38   SECTION 1  ■  ABDOMEN – GENERAL Insertion of the deep cuff into the center of the rectus muscle, as opposed to midline placement, has significantly reduced the incidence of early leakage of pericatheter dialysis solution.46,51 Pericatheter leaks are rare with catheters that have a bead and polyester flange at the deep cuff (Toronto Western Hospital catheter, swan neck Missouri catheter, swan neck presternal peritoneal catheter). In contrast to the early leaks, which are usually external, the late leaks infiltrate the abdominal wall through prior healed incisions. PD catheters may cause hemoperitoneum by causing minor tears of small vessels. On occasion, a peritoneal catheter erodes into the mesenteric vessels, leading to hemoperitoneum. In rare cases, a peritoneal catheter damages the internal organs, which leads to intra-abdominal bleeding. Transvaginal leakage of peritoneal fluid is rare, but the possibility should be considered in an appropriate clinical setting.46,51

References 1. Krediet R. The physiology of peritoneal solute, water, and lymphatic transport. In: Khanna R, Krediet R, eds. Nolph and Gokal’s textbook of peritoneal dialysis. 3rd ed. New York: Springer; 2009:137–172. 2. Miller FN, Nolph KD, Harris PD, et al. Microvascular and clinical effects of altered peritoneal dialysis solutions. Kidney Int. 1979;15:630–639. 3. Hirszel P, Shea-Donohue T, Chakrabarti E, et al. The role of the capillary wall in restricting diffusion of macromolecules. Nephron. 1988;49:58–61. 4. Rippe B, Stelin S. How does peritoneal dialysis remove small and large molecular weight solutes? Transport pathways: fact and myth. Adv Perit Dial. 1991;7:13–18. 5. Flessner MF, Henegar J, Bigler S, et al. Is the peritoneum a significant transport barrier in peritoneal dialysis? Perit Dial Int. 2003;23:542–549. 6. Bundgaard M. Transport pathways in capillaries; in search of pores. Annu Rev Physiol. 1980;42:325–336. 7. Rippe B, Haraldsson B. Transport of macromolecules across microvascular walls: the two-pore theory. Physiol Rev. 1994;74:163–219. 8. Preston GM, Carroll TP, Guggino WB, et al. Appearance of water channels in Xenopus oocytes expressing red cell CHIP28 protein. Science. 1992;256:385–387. 9. Dempster JA, van Hoek AN, van Os CH. The quest for water channels. News Physiol Sci. 1992;7:172–176. 10. Pannekeet MM, Mulder JB, Weening JJ, et al. Demonstration of aquaporin-chip in peritoneal tissue of uremic and CAPD patients. Perit Dial Int. 1996;16(suppl 1):S54–S57. 11. Devuyst O, Nielsen S, Cosyns J-P, et al. Aquaporin-1 and endothelial nitric oxide synthase expression in capillary endothelia of human peritoneum. Am J Physiol. 1998;275:H234–H242. 12. Rippe B, Stelin G. Simulations of peritoneal solute transport during CAPD. Application of two-pore formalism. Kidney Int. 1989;35:1234–1244. 13. Stelin G, Rippe B. A phenomenological interpretation of the variation in dialysate volume with dwell time in CAPD. Kidney Int. 1990;38:465–472. 14. Ho-dac-Pannekeet MM, Schouten N, Langedijk MJ, et al. Peritoneal transport characteristics with glucose polymer based dialysate. Kidney Int. 1996;50:979–986. 15. Smit W, Struijk DG, Ho-dac-Pannekeet MM, Krediet RT. Quantification of free water transport in peritoneal dialysis. Kidney Int. 2004;66:849–854. 16. Levick JR. Flow through interstitium and fibrous matrices. Q J Exp Physiol. 1987;72:409–438. 17. Fox JR, Wayland H. Interstitial diffusion of macromolecules in the rat mesentery. Microvasc Res. 1979;18:255–276. 18. Flessner MF, Fenstermacher JD, Dedrick RL, et al. A distributed model of peritoneal-plasma transport: tissue concentration gradients. Am J Physiol. 1985;248:F425–F435. 19. Wiig H, DeCarlo M, Sibley L, et al. Interstitial exclusion of albumin in rat tissues measured by a continuous infusion method. Am J Physiol. 1992;263:H1222–H1233. 20. Leypoldt JK, Parker HR, Frigon RP, et al. Molecular size dependence of peritoneal transport. J Lab Clin Med. 1987;110:207–216. 21. Krediet RT, Zuyderhoudt FMJ, Boeschoten EW, et al. Alterations in the peritoneal transport of water and solutes during peritonitis in continuous ambulatory peritoneal dialysis patients. Eur J Clin Invest. 1987;17:43–52. 22. McGary TJ, Nolph KD, Rubin J. In vitro simulations of peritoneal dialysis. J Lab Clin Med. 1980;96:148–157. 23. Levitt MD, Kneip JM, Overdahl MC. Influence of shaking on peritoneal transport. Kidney Int. 1989;35:1145–1150. 24. Krediet RT, Koomen GCM, Koopman MG, et al. The peritoneal transport of serum proteins and neutral dextran in CAPD patients. Kidney Int. 1989;35:1064–1072. 25. Galdi P, Shostak A, Jaichenko J, et al. Protamine sulfate induces enhanced peritoneal permeability to proteins. Nephron. 1991;57:45–51. 26. Krediet RT, Struijk DG, Koomen GCM, et al. The peritoneal transport of macromolecules in CAPD patients. Contrib Nephrol. 1991;89:161–174. 27. Krediet RT, Zemel D, Struijk DG, et al. Individual characterization of the peritoneal restriction barrier to macromolecules. Adv Perit Dial. 1991;7:16–20. 28. Flessner MF, Dedrick RL, Schultz JS. A distributed model of peritoneal plasma transport: theoretical considerations. Am J Physiol. 1984;246:R597–R607. 29. Seasmes El, Moncrief JW, Popovich RP. A distributed model of fluid and mass transfer in peritoneal dialysis. Am J Physiol. 1990;258:958–972.

CHAPTER 3-1  ■  Peritoneal Dialysis   30. Smeby LC, Wideroe T-E, Jorstad S. Individual differences in water transport during continuous peritoneal dialysis. ASAIO J. 1981;4:17–27. 31. Waniewski J, Werynski A, Heimbürger O, et al. Simple membrane models for peritoneal dialysis: evaluation of diffusive and convective solute transport. ASAIO J. 1992;38:788–796. 32. Lysaght MJ, Farrell PC. Membrane phenomena and mass transfer kinetics in peritoneal dialysis. J Membrane Sci. 1984;44:5–53. 33. Heimbürger O, Waniewski J, Werynski A, Park MS, et al. Dialysate to plasma solute concentration (D/P) versus peritoneal transport parameters in CAPD. Nephrol Dial Transplant. 1994;9:47–59. 34. Twardowski ZJ, Nolph KD, Khanna R, et al. Peritoneal equilibration test. Perit Dial Bull. 1987;7:138–147. 35. Smit W, Langeijk M, Schouten N, et al. Peritoneal function and assessment of reference values using a 3.86% glucose solution. Perit Dial Int. 2003;23:440–449. 36. Mactier RA, Khanna R, Twardowski ZJ, et al. Contribution of lymphatic absorption to loss of ultrafiltration and solute clearances in continuous ambulatory peritoneal dialysis. J Clin Invest. 1987;80:1311–1316. 37. Krediet RT, Imholz ALT, Struijk DG, et al. Ultrafiltration failure in continuous ambulatory peritoneal dialysis. Perit Dial Int. 1993;13(suppl 2):S59–S66. 38. Imholz ALT, Koomen GCM, Struijk DG, et al. Fluid and solute transport in CAPD patients using ultralow sodium dialysate. Kidney Int. 1994;46:333–340. 39. Leypoldt JK. Interpreting peritoneal osmotic reflection coefficients using a distributed model of peritoneal transport. Adv Perit Dial. 1993;9:3–7. 40. Pannekeet MM, Imholz ALT, Struijk DG, et al. The standard peritoneal permeability analysis: a tool for the assessment of peritoneal permeability characteristics in CAPD patients. Kidney Int. 1995;48:866–875. 41. Flanigan M, Gokal R. Peritoneal catheters and exit-site practices toward optimum peritoneal access: A review of current developments. Perit Dial Int. 2005;25:132–139. 42. Negoi D, Prowant BF, Twardowski ZJ. Current trends in the use of peritoneal dialysis catheters. Adv Perit Dial. 2006;22:147–152. 43. Twardowski ZJ, Nolph KD, Khanna R, et al. The need for a “Swan Neck” permanently bent, arcuate peritoneal dialysis catheter. Perit Dial Bull. 1985;5:219–223. 44. Twardowski ZJ, Prowant BF, Pickett B, et al. Four-year experience with swan neck presternal peritoneal dialysis catheter. Am J Kidney Dis. 1996;27:99–105. 45. Dasgupta MK. Moncrief-Popovich catheter and implantation technique: the AV fistula of peritoneal dialysis. Adv Ren Replace Ther. 2002;9:116–124. 46. Kathuria P, Twardowski ZJ, Nichols WK. Peritoneal dialysis access and exitsite care including surgical aspects. In: Khanna R, Krediet R, eds. Nolph and Gokal’s textbook of peritoneal dialysis. 3rd ed. New York: Springer; 2009:371–446. 47. Chitalia VC, Almeida AF, Rai H, et al. Is peritoneal dialysis adequate for hypercatabolic acute renal failure in developing countries? Kidney Int. 2002;61:747–757. 48. Phu NH, Hein TT, Mai NT, et al. Hemofiltration and peritoneal dialysis in infection associated acute renal failure in Vietnam. N Engl J Med. 2002;347:895–902. 49. Werner S, Grose R. Regulation of wound healing by growth factors and cytokines. Physiol Rev. 2003;83:835–870. 50. Khanna R, Twardowski ZJ. Peritoneal catheter exit site. Perit Dial Int. 1988;8:119–123. 51. Diaz-Buxo JA. Complications of peritoneal dialysis catheters: early and late. Int J Artif Organs. 2006;29(1):50–58. 52. Crabtree JH, Fishman A, Siddiqi RA, et al. The risk of infection and peritoneal catheter loss from implant procedure exitsite trauma. Perit Dial Int. 1999;19:366–371. 53. Diaz-Buxo JA. Peritoneal dialysis catheter malfunction due to compartmentalization. Perit Dial Int. 1997;17:209–210. 54. Gotloib L, Bar-Sella P, Jaichenko J, et al. Ruthenium-red-stained polyanionic fixed charges in peritoneal microvessels. Nephron. 1987;47:22–28.

e39

3-2 

PERITONEAL DIALYSIS CATHETER PLACEMENT William P. Robinson III  /  Matthew T. Menard From Vernon AH, Ashley SW: Atlas of Minimally Invasive Surgical Techniques, 1st edition (Saunders 2012)

FIGURE 3-2-1  Trocar insertion sites and placement of catheter.

e40

CHAPTER 3-2  ■  Peritoneal Dialysis Catheter Placement  

FIGURE 3-2-2  Note the equipment used for the insertion of a coiled peritoneal dialysis catheter through a laparoscopically guided percutaneous insertion technique.

FIGURE 3-2-3  Note position of catheter cuffs.

FIGURE 3-2-4  Positioning of the catheter at Douglas Pouch is confirmed with the camera before desufflation of the abdomen.

e41

3-3 

ADVANCED LAPAROSCOPIC TECHNIQUES SIGNIFICANTLY IMPROVE FUNCTION OF PERITONEAL DIALYSIS CATHETERS Vikram Attaluri  /  Christopher Lebeis  /  Stacy Brethauer  /  Steven Rosenblatt From Advanced laparoscopic techniques significantly improve function of peritoneal dialysis catheters. JACS 2010;211:699–704

BACKGROUND: Continuous ambulatory peritoneal dialysis (CAPD) catheters provide

a preferred alternative to hemodialysis in a growing population with chronic kidney disease. However, CAPD catheters traditionally have been associated with a high rate of nonfunction with both open and laparoscopic procedures. New advanced laparoscopic techniques using rectus sheath tunneling and omentopexy have been reported to improve catheter function. STUDY DESIGN: This study retrospectively reports the Cleveland Clinic experience during the transition from basic to advanced laparoscopic techniques from June 2002 to July 2008. A total of 197 patients were identified: 68 who underwent insertion with basic techniques and 129 who received catheters with advanced techniques. Primary nonfunction, procedural complications, and overall nonfunction rate were analyzed using the most recent follow-up to June 2008. RESULTS: Primary nonfunction occurred in 25 of 68 (36.7%) patients in the basic group; this occurred in only 6 of 129 patients (4.6%) in the advanced group (p < 0.0001). The overall rate of complications including nonfunction from primary and secondary sources, peritoneal leak, peritonitis, port-site hernia, and bleeding occurred in 31 of 68 (45.6%) patients in the basic group and 21 of 129 (16.28%) patients in the advanced group (p < 0.0001). CONCLUSIONS: These data clearly show a significant improvement in CAPD catheter function using omentopexy and rectus sheath tunneling. These advanced laparoscopic techniques should become the preferred method of CAPD catheter insertion. ( J Am Coll Surg 2010;211:699– 704. © 2010 by the American College of Surgeons)

Dialysis is becoming more common as the number of patients with end-stage renal disease (ESRD) increases and the number of available donor kidneys fails to keep up with the demand. There are approximately half a million patients in the United States with ESRD, and the incidence of this disease is around 330 per million people per year. The need for dialysis has increased as the waiting list for kidney transplantation has doubled, even with a rise in the number of kidneys available for transplantation.1,2 The two main modalities for dialysis are hemodialysis (HD) and peritoneal dialysis (PD), with the majority of patients on HD.3,4 Continuous ambulatory peritoneal dialysis (CAPD) has been used as a major renal replacement therapy since the early 1980s.5 Although the results comparing PD and e42

CHAPTER 3-3  ■  Peritoneal Dialysis Catheters  

HD have been conflicting, several studies have shown short-term benefit in survival with PD.6 Compared with HD, PD has shown improved preservation of residual kidney function, improving initial survival.7 Additional advantages of PD include increased patient mobility and independence, fewer dietary restrictions, and no required systemic anticoagulation. However, CAPD catheters have historically been associated with high complication rates. Traditionally, CAPD catheters have been inserted using a small laparotomy and blind placement of the catheter in the pelvis. This technique has been associated with catheter obstruction rates as high as 36%. Other techniques have since been described that use fluoroscopy, percutaneous puncture using laparoscopy, and straight laparoscopy. However, there is a lack of consensus in the literature as to the preferred operation. Most importantly, the rate of catheter obstruction has been reported to vary between 2% and 36%.8–11 This malfunction is often the result of outflow obstruction, usually related to a mechanical problem, such as catheter tip migration and/or omental wrapping.12 Additionally, other complications also vary widely, including pericatheter leaks (1% to 27%) and superficial cuff extrusion (4% to 10%). The absence of a standard procedure using best surgical practice may help to explain this wide variation. Rectus sheath/extraperitoneal tunneling and omentopexy in CAPD catheter placement have shown a vast decrease in catheter malfunction.8,13 We review our experience as we transitioned between laparoscopic CAPD catheters without rectus sheath tunneling to the implementation of rectus sheath tunneling and selective omentopexy.

METHODS A retrospective review of all patients undergoing CAPD catheter placement at the Cleveland Clinic from June 2002 through July 2008 was completed. This time period was divided into 2 groups encompassing patients who underwent CAPD catheter placement without rectus sheath tunneling ( June 2002 to December 2004) and patients who underwent CAPD catheter placement using rectus sheath tunneling and selective omentopexy (December 2004 through July 2008). All patients were evaluated by the Cleveland Clinic Nephrology Department and deemed eligible for PD. The nonrectus tunneling group consisted of 68 patients with laparoscopic placement. The rectus tunneling group contained 129 patients who had laparoscopic implantation using advanced methods that included rectus sheath tunneling and selective prophylactic omentopexy. Both groups also had selective adhesiolysis. All patients had a traditional pelvic catheter placement with a Tenckhoff style catheter. The primary endpoint was nonfunction of the catheter. Secondary endpoints included pericannicular leak, peritonitis, port-site hernia, and bleeding.

Surgical Technique All patients underwent consultation before surgery, and the exit site was planned with the patient in a seated position to ensure the catheter would not fall below the belt line, below a skin crease, or into a skin fold and was easily visible for the patient. We used a standard Tenckhoff style catheter with a deep cuff to be placed into the rectus sheath and a superficial cuff meant to be placed in the subcutaneous tissue. All catheters were placed under general endotracheal anesthesia, and patients were given a dose of preoperative prophylactic antibiotics before incision in the operating room.14,15 For placement of the deep cuff, we begin by planning out the course that the catheter will run by positioning the coiled tip of the catheter on the patient’s pubic symphysis and marking the location where the deep cuff will sit in the rectus abdominis muscle that will maintain the position of the catheter. The incision is made at the paramedian location and is about 3 cm from the midline; this location allows the catheter to travel through the rectus sheath and avoid the epigastric vasculature. A 2-port technique is used for laparoscopic CAPD catheter placement, with an additional port as needed for selective omentopexy. Pneumoperitoneum and laparoscopic access is gained via a 5-mm paramedian incision far enough away so as not to interfere with the catheter insertion site. We insert a 0-degree scope and an Endopath (Ethicon Endosurgical) trocar under direct visualization, and we then insufflate with CO2. A 30-degree scope is then inserted and the abdomen is visualized for adhesions that could cause compartmentalization of the abdomen, undiagnosed abdominal wall hernias, and a redundant omentum. The patient is put into steep reverse

e43

e44   SECTION 1  ■  ABDOMEN – GENERAL Trendelenburg position to see if the omentum falls into the pelvis and therefore requires an omentopexy. A bladeless trocar port system is used to perform rectus sheath tunneling (7/8-mm Auto Suture Mini Step, Covidien AG). The rectus sheath tunnel is started by a small 3-cm paramedian incision over the rectus muscle and dissected down to the level of the anterior fascia, which is then incised using electrocautery. A Veress needle with an expandable plastic sheath is then used to create the rectus sheath tunnel under direct laparoscopic visualization. A 4- to 6-cm intramuscular tract is created before entering the peritoneal cavity. After retracting the Veress needle, a 7/8-mmdilator-cannula assembly is used to expand the plastic sleeve to a size large enough for catheter insertion. This allows for catheter placement within the rectus sheath through a smaller hole than a cutting trocar would allow, which may reduce the incidence of pericannular hernias, leaks, and bleeding of the rectus muscle. The CAPD catheter is soaked in saline before implantation and is placed over a straight stylet to assist in its passage into the abdomen. The catheter is then inserted through the port into the abdomen until the deep cuff is visualized, ensuring its passage through the anterior sheath, and the stylet is withdrawn while the catheter is being inserted. The coiled tip is placed in the retrovesical space. Under laparoscopic visualization, the catheter is withdrawn until the distal cuff is placed within the rectus muscle. The pneumoperitoneum is released so that the subcutaneous tract can be created with the abdomen in normal contour without the distortion that occurs with insufflation. After placing the catheter in the abdominal wall and the coiled tip sits in the retrovesical space, the remaining catheter is tunneled subcutaneously using a Faller stylet (Covidien AG) to the planned exit site. This stylet is smaller in diameter than the catheter so that the tunnel and exit site are snug around it and it can be advanced through the skin without an incision. The patient is placed in steep reverse Trendelenburg position, and the catheter is then tested for flow obstruction by infusion of 500 mL of isotonic saline through the catheter and return of this fluid by gravity. With completion of this test we again insufflate the patient with CO2 to confirm placement of the catheter tip and check for bleeding and then we remove the trocars. The exit site is not sutured and is covered with a dressing to prevent infection as it heals around the catheter. The catheter is flushed with 50 mL of heparinized saline to prevent clot formation and closed with a Betadine (Purdue Pharma) cap. We recommend that our patients wait at least 2 weeks before using their catheter for PD to allow their wounds to heal and avoid the risk of early leaks.8,16

Selective Omentopexy Because the omentum is a common cause of flow obstruction, its placement and size should always be observed when placing a CAPD catheter. Crabtree and Fishman17 described a similar technique for performing the omentopexy. If the omentum is redundant and falls into the pelvis, an omentopexy is performed. The omentopexy is carried out using a suture passer with #0 Prolene (Ethicon) suture through a stab incision located in the upper abdominal quadrant lateral to the rectus sheath and a 5-mm laparoscopic grasper inserted through an additional port. The grasper directs the multiple folds of the omentum through the Prolene suture with care to avoid the omental vessels. After securing the omentum to the abdominal wall, the patient is put into steep reverse Trendelenburg position to ensure that the omentum does not fall into the pelvis.

Subcutaneous Catheter Tunneling An algorithm described by Crabtree18 assists in the location of the exit site for the catheter and location of the deep cuff. Briefly, it is done by keeping pressure on the deep cuff location and using the catheter like a compass from a point 2 cm beyond the superficial cuff and swinging it around, creating a 90-degree arc. A point is selected on the arc approximately 30 degrees above the horizontal plane to roughly indicate the exit site location. A point on the catheter 4 cm external to the superficial cuff is then bent over to the selected exit site to produce a gentle tubing arc that is directed both laterally and slightly downward. This process helps to avoid excessive bending of the catheter in its subcutaneous tunnel that could lead to kinking or shape memory stress, producing tube straightening and superficial cuff extrusion through the exit site. At the time of catheter insertion, the superficial cuff will rest 4 cm from the exit wound. In the event that some tube straightening occurs over time, the

CHAPTER 3-3  ■  Peritoneal Dialysis Catheters  

described algorithm prevents the superficial cuff from coming closer than 2 cm to the exit site. This site will vary from patient to patient depending on body habitus and previous abdominal surgical scars. The subcutaneous tunnel configuration produced by the algorithm avoids pooling of water and/or sweat in the skin exit sinus.

Statistical Analysis Chi-square and Fischer’s exact t-test were used for nominal variables. A Student t-test was used for continuous variables.

RESULTS During our implantation of all 197 catheters, there were no deaths, and no operations in either group were converted to open procedures. Demographics for both groups can be seen in Table 3-3-1. There was no statistical difference between the age, sex, or previous abdominal surgery of the patients in either group. There was a statistical difference in length of follow-up, which was expected because the groups were defined by chronologic order. Patients in the rectus sheath tunneling group underwent a prophylactic omentopexy when the omentum was found to lie within the pelvis/retrovesical space. Sixty-nine (53.5%) patients had an omentopexy performed during the catheter implantation procedure. Adhesiolysis was performed in 21 patients (16.3%), 12 of whom also had an omentopexy, to prevent compartmentalization or to clear the area in the abdominal wall for catheter implantation. Five patients (3.9%) had intraoperative umbilical herniorrhaphy to prevent worsening of the hernia from PD. Mechanical flow obstruction occurred in 6 of the 129 CAPD catheter implantations that involved the rectus sheath tunneling (Table 3-3-2). The low rate (4.6%) of catheter dysfunction was significantly better (p < 0.0001) than the rate in the non-rectus sheath tunneling group (36.8%). Three of the patients with flow obstruction had blockage due to a large amount of sigmoid epiploic fat. Two patients had dysfunction caused by postoperative adhesions, and all had previous abdominal surgery and underwent adhesiolysis. One patient’s catheter tip became entangled in a rent of the median umbilical ligament. Of these 6 patients who had mechanical obstruction, only 2 were not amendable to revision. The overall rate of complications, including nonfunction, peritoneal leak, peritonitis, port-site hernia, and bleeding, occurred in 31 of 68 (45.6%) patients in the basic group and 21 of 129 (16.28%) patients in the advanced group (p < 0.001) (Table 3-3-2). All identifiable umbilical, ventral, and inguinal hernias were repaired at the time of catheter implantation, and this occurred in 5 (3.9%) patients in the advanced group and only 1 (1.5%) in the basic group. There was only 1 case (0.51% overall) of a pericannular leak in all of our patients and we believe this low rate of occurrence is attributable to our paramedian implantation and rectus sheath tunneling. Port-site hernias did not occur in

TABLE 3-3-1  Patient Characteristics Between the Non-Rectus Sheath Tunneling and Rectus Sheath Tunneling Groups

Parameter n Age, y, mean ± SD Male, n (%) Postoperative follow-up, mo* Previous abdominal surgery, n (%) Selective prophylactic omentopexy, n (%)* Selective prophylactic omentectomy, n (%) Selective prophylactic adhesiolysis, n (%) Intraoperative umbilical hernia repair, n (%) *p < 0.05.

Non-Rectus Sheath Tunneling, June 2002–December 2004 68 48.6 ± 15.7 38 (55.9) 19.9 45 (66.2) 3 (4.4) 1 (1.5) 9 (13.2) 1 (1.5)

Rectus Sheath Tunneling, December 2004–June 2008 129 51.4 ± 14.8 72 (55.8) 15.1 79 (61.2) 69 (53.5) 0 21 (16.3) 5 (3.9)

e45

e46   SECTION 1  ■  ABDOMEN – GENERAL TABLE 3-3-2  Breakdown of Specific Complications Between the Non-Rectus Sheath Tunneling and Rectus Sheath Tunneling Groups Non-Rectus Sheath Tunneling

Rectus Sheath Tunneling

Procedural Complication

n

%

n

%

Catheter flow obstruction Omentum Adhesions Catheter migration Cuff erosion Sigmoid epiploics Subcutaneous kink Caught in median umbilical ligament Unknown (drainage issues)

25 11 3 3 2 0 1 0 5

36.8 44 12 12 08 0 4 0 20

6 0 2 0 0 3 0 1 0

4.6 0 33 0 0 50 0 17 0

either group, possibly due to the use of 5-mm ports. In addition, there was 1 instance of postoperative hemorrhage in the basic group and none in the advanced group. There were no cases of superficial cuff extrusion in the advanced group and 4 (5.9%) in the basic group, the absences of extrusions being attributed to the use of a subcutaneous tunneling algorithm that minimized tubing shape memory forces and assured proper positioning of the subcutaneous cuff.

DISCUSSION PD has continued to undergo technical improvement to become a common, and increasingly preferred, treatment for patients with ESRD. One of the groundbreaking advancements in PD was the invention of the catheter cuff by Tenckhoff in 1968. The resulting fibrosis around the cuff helps create a seal against leaks and a barrier against infection.19 This seal allows PD to be used for long-term cases and for acute kidney failure. Intermittent PD was used for ESRD until 1977, when Popovich and colleagues20 developed CAPD. Others further perfected CAPD and it became very popular due to its low cost, simplicity of technique, and ease of use for the patient.21,22 The addition of laparoscopic surgery and best surgical practice has only continued to improve these results. Crabtree and colleagues8 have built upon the early success of surgeons such as McIntosh and associates23 in 1985, who performed omentopexy by using an open incision through which the omentum was gathered, pleated, and then secured to the abdominal wall: the “omental hitch,” which improved their catheter dysfunction rate from 67% to 17%. Gajjar and coworkers11 showed improvement in functional success and a lower incidence of PD catheter revision in the group that underwent laparoscopic-assisted CAPD catheter placement compared with the traditional “blind” technique through a small lower abdominal incision using a malleable catheter guide. Additionally, Crabtree and colleagues24,25 showed that laparoscopic procedures are more cost-effective. Our data with the addition of rectus sheath tunneling, selective omentopexy, and proper subcutaneous catheter tunneling have greatly reduced the most common complications of CAPD catheters. Looking at the causes of nonfunction between the 2 groups, it is evident there are entirely different causes. Omentum used to be the single largest cause of our catheter nonfunction, but it was nonexistent with the new techniques. Similarly, we no longer had a problem with cuff extrusion. These data support the view that it is not necessary to insert an additional trocar and complete an omentopexy in all patients. Rather, clinical judgment at the time of operation saved almost half the patients from undergoing this additional technique. The effect of selective omentopexy could be greater than directly noted by the data. A significant group of patients used to have unknown drainage issues causing obstruction (20% in the non-rectus sheath tunneling group). It is possible that temporary blockage by the omentum could have caused these drainage issues. However, we are unable to provide a certain answer for the lack of these problems in the rectus sheath tunneling group. Alternative methods have been described to laparoscopically guide placement of a catheter into a rectus sheath tunnel; however, these techniques either use more than 1 port in the tunneling process13,26

CHAPTER 3-3  ■  Peritoneal Dialysis Catheters  

or subject the peritoneal catheter to potential damage by forcefully pulling it through the abdominal wall, the latter resulting in imprecise positioning of the catheter cuffs.27 Another approach has been to anchor the catheter in the pelvis with a stitch. The problem with this approach is that extra laparoscopic ports are required to place the stitch, and the suture sometimes fails by pulling out of the tissues, but at other times, its secure hold complicates catheter removal.28 Rectus sheath tunneling, as described in this report, using an expandable sleeve as a conduit for placement of a nonbladed laparoscopic port, is a simple, safe, accurate, and reproducible method for immobilization of the catheter tip in the pelvis, and it prevents migration. It is important to note that the improved results in this study are not due to a single new intervention but rather a collection of improvements. An advancement that is particularly important in the growing obese patient population is the use of an extended catheter, allowing placement of exit sites in a more manageable position above abdominal skin folds. This allows better care of catheters, leads to decreased complication rates, and extends CAPD to those previously troubled by body habitus, presence of stomas, or urinary-fecal incontinence.24

CONCLUSIONS Laparoscopic placement of CAPD catheters using rectus sheath tunneling, selective omentopexy, and proper subcutaneous catheter tunneling is clearly advantageous to other techniques reported in the literature and should be strongly encouraged.

References 1. Liu KD, Chertow GM. Dialysis in the treatment of renal failure. In: Kasper DL, Fauci AS, Longo DL, et al. Harrison’s Principles of Internal Medicine. 17th ed New York, NY: McGraw-Hill;2008. 2. Carpenter CB, Milford EL, Sayegh MH. Transplantation in the treatment of renal failure. In: Kasper DL, Fauci AS, Longo DL, et al. Harrison’s Principles of Internal Medicine. 17th ed New York, NY: McGraw-Hill;2008. 3. Grassmann A, Gioberge S, Moeller S, Brown G. ESRD patients in 2004: global overview of patient numbers, treatment modalities and associated trends. Nephrol Dial Transplant 2005;20:2587–2593. 4. Miskulin DC, Athienites NV, Yan G, et al. Comorbidity assessment using the Index of Coexistent Diseases in a multicenter clinical trial. Kidney Int 2001;60:1498–1510. 5. Sharma A, Blake PG. Peritoneal dialysis. In: Brenner B, ed. Brenner and Rector’s The Kidney. 8th ed. Philadelphia, PA: Saunders Elsevier;2007. 6. Schaubel DE, Morrison HI, Fenton SS. Comparing mortality rates on CAPD/CCPD and hemodialysis. The Canadian experience: fact or fiction? Perit Dial Int 1998;18:478–484. 7. Heaf JG, Løkkegaard H, Madsen M. Initial survival advantage of peritoneal dialysis relative to haemodialysis. Nephrol Dial Transplant 2002;17:112–117. 8. Crabtree J, Fishman A. A laparoscopic method for optimal peritoneal dialysis access. Am Surg 2005;71:135–143. 9. Draganic B, James A, Booth M, Gani JS. Comparative experience of a simple technique for laparoscopic chronic ambulatory peritoneal dialysis catheter placement. ANZ J Surg 1998;68:735–739. 10. Ogünç G. Videolaparoscopy with omentopexy: a new technique to allow placement of a catheter for continuous ambulatory peritoneal dialysis. Surg Today 2001;31:942–944. 11. Gajjar A, Rhoden D, Kathuria P, et al. Peritoneal dialysis catheters: laparoscopic versus traditional placement techniques and outcomes. Am J Surg 2007;194:872–876. 12. Yilmazlar T, Yavuz M, Ceylan H. Laparoscopic management of malfunctioning peritoneal dialysis catheters. Surg Endosc 2001; 15:820–822. 13. Ogünç G. Minilaparoscopic extraperitoneal tunneling with omentopexy: a new technique for CAPD catheter placement. Peritoneal Dialysis International: J Int Soc Peritoneal Dialysis 2005;25:551–555. 14. Strippoli GF, Tong A, Johnson D, et al. Antimicrobial agents to prevent peritonitis in peritoneal dialysis: a systematic review of randomized controlled trials. Am J Kidney Dis 2004;44:591–603. 15. Gadallah MF, Ramdeen G, Mignone J, et al. Role of preoperative antibiotic prophylaxis in preventing postoperative peritonitis in newly placed peritoneal dialysis catheters. Am J Kidney Dis 2000;36:1014–1019. 16. Del Peso G, Bajo MA, Costero O, et al. Risk factors for abdominal wall complications in peritoneal dialysis patients. Peritoneal Dialysis International: J Int Soc Peritoneal Dialysis 2003;23:249–254. 17. Crabtree J, Fishman A. Selective performance of prophylactic omentopexy during laparoscopic implantation of peritoneal dialysis catheters. Surg Laparosc Endosc Percutaneous Tech 2003;13:180–184. 18. Crabtree J. Construction and use of stencils in planning for peritoneal dialysis catheter implantation. Peritoneal Dialysis International: J Int Soc Peritoneal Dialysis 2003;23:395–398. 19. Tenckhoff H, Schechter H. A bacteriologically safe peritoneal access device. Trans Am Soc Artif Intern Organs 1968;14:181–187. 20. Popovich RP, Moncrief JW, Decherd JF. The definition of a novel portable-wearable equilibrium peritoneal technique. Am Soc Artif Intern Org 1976;5:64 (abstract).

e47

e48   SECTION 1  ■  ABDOMEN – GENERAL 21. Popovich RP, Moncrief JW, Nolph KD, et al. Continuous ambulatory peritoneal dialysis. Ann Intern Med 1978;88:449–456. 22. Oreopoulos DG, Robson M, Izatt S, et al. A simple and safe technique for continuous ambulatory peritoneal dialysis (CAPD). Trans Am Soc Artif Intern Organs 1978;24:484–489. 23. McIntosh G, Hurst PA, Young AE. The ‘omental hitch’ for the prevention of obstruction to peritoneal dialysis catheters. Br J Surg 1985;72:880. 24. Crabtree J, Fishman A. Laparoscopic implantation of swan neck presternal peritoneal dialysis catheters. J Laparoendoscop Advanced Surg Tech Part A 2003;13:131–137. 25. Crabtree JH, Kaiser KE, Huen IT, Fishman A. Costeffectiveness of peritoneal dialysis catheter implantation by laparoscopy versus by open dissection. Adv Perit Dial 2001; 17:88–92. 26. Comert M, Borazan A, Kulah E, Uçan BH. A new laparoscopic technique for the placement of a permanent peritoneal dialysis catheter: the preperitoneal tunneling method. Surg Endosc 2005;19:245–248. 27. Schmidt S, Pohle C, Langrehr J, et al. Laparoscopic-assisted placement of peritoneal dialysis catheters: implantation technique and results. J Laparoendoscop Advanced Surg Tech Part A 2007;17:596–599. 28. Lu CT, Watson DI, Elias TJ, et al. Laparoscopic placement of peritoneal dialysis catheters: 7 years experience. ANZ J Surg 2003;73:109–111.

PERITONEAL DIALYSIS CATHETER PLACEMENT William P. Robinson III  /  Matthew T. Menard From Vernon AH, Ashley SW: Atlas of Minimally Invasive Surgical Techniques, 1st edition (Saunders 2012)

3-4 

The video for this procedure can be accessed here

Further Reading Hagen SM, Lafranca JA, Steyerberg EW, IJzermans JN, Dor FJ. Laparoscopic versus open peritoneal dialysis catheter insertion: a meta-analysis. PLoS ONE. 2013; 8(2); e56351. Gajjar AH, Rhoden DH, Kathuria P, Kaul R, Udupa AD, Jennings WC. Peritoneal dialysis catheters: laparoscopic versus traditional placement techniques and outcomes. Am J Surg. 2007; 194: 872–876.

e49

3-5 

SELF ASSESSMENT Michelle A. Kominiarek  /  Edward F. Hollinger From Velasco J: Rush University Medical Center Review of Surgery, 5th edition (Saunders 2011)

1. A 62-year-old woman with long-standing end-stage renal disease who is being maintained on peritoneal dialysis (PD) has had several months of intermittent abdominal pain and difficulty obtaining normal dwell volumes for her peritoneal catheter. She goes to the emergency department because she was unable to adequately drain her peritoneal fluid. Review of her medical records reveals that her creatinine level has slowly been increasing for the last several months without any change in her dialysis regimen. Abdominal CT shows ascites; shortened, thickened small bowel mesentery; and diffusely thickened small bowel with areas of luminal narrowing. There are punctuate calcifications throughout the peritoneum. Which of the following is least appropriate? A. Trial of tamoxifen therapy B. Oral steroid pulse C. Replacement of the PD catheter D. Immunosuppressive therapy with azathioprine E. Exploratory laparoscopy and enterolysis Ref.: 1, 2 COMMENTS: Encapsulating peritoneal sclerosis (EPS, sclerosing peritonitis) is one of the most feared complications of peritoneal dialysis. EPS is characterized by a decrease in the efficacy of PD and the development of extensive intraperitoneal fibrosis, mesenteric shortening, and encasement of the bowel. It can progress to bowel obstruction. Radiologic features include mesenteric, bowel, and peritoneal thickening, often with calcifications. Loculated ascites, adherent bowel loops, and luminal narrowing of the bowel may also be visualized. The etiology of EPS is not well understood. Risk factors include the duration of PD therapy, episodes of peritonitis, and acetate dialysis. Treatment is often unsuccessful. Most patients with EPS are switched to hemodialysis (although such a switch can sometimes precipitate EPS). Steroid therapy, tamoxifen, and immunosuppressive regimens, including azathioprine or cyclosporine, have all been used to treat EPS. When bowel obstruction is present, total parenteral nutrition may be required. The role of surgical therapy for EPS remains controversial. Early results with enterectomy and anastomosis have shown high mortality rates, but more recent studies suggest a role for early enterolysis.

ANSWER: C 2. A 54-year-old woman with end-stage renal disease treated by PD complains of abdominal pain and fever. When performing her exchanges she has noted turbid fluid for the last several days. She has been undergoing PD for 3 years and has never had any complications. Which of the following statements is correct? A. She should undergo immediate peritoneal exploration with removal of the dialysis catheter. B. Fungal peritonitis requires long-term antifungal therapy through the PD catheter. e50

CHAPTER 3-5  ■  Self Assessment  

C. PD-associated peritonitis from coagulase-negative staphylococci can be cured with antibiotics alone in more than 80% of cases. D. She will need to resume hemodialysis while the infection is treated. E. Broad-spectrum empirical antibiotic therapy is required because peritoneal fluid cultures have little value. Ref.: 3 COMMENTS: Peritonitis is a common complication of peritoneal dialysis and occurs about 1.4 times per patient-year of PD. It is one of the most important reasons for failure of PD and accounts for nearly one half of all technical failures. Typically, patients have abdominal pain and tenderness (75%), fever (33%), and cloudy dialysate. The diagnosis is confirmed by a fluid leukocyte count of greater than 100/mL with more than one half of the cells being neutrophils. Most infections are caused by gram-positive organisms, but gram-negative bacilli and fungi can also be responsible. Initial treatment should consist of intraperitoneal antibiotics, most commonly vancomycin or a first-generation cephalosporin. About 75% of infections are cured with culture-directed antibiotic therapy without discontinuation of PD. Persistent or recurrent infection may require removal of the PD catheter and a switch to hemodialysis. Cure rates with antibiotics alone are best for coagulase-negative staphylococci (90%) and less for Staphylococcus aureus (66%) or gram-negative bacilli (56%). Fungal infections require prompt removal of the catheter. Prompt treatment of peritoneal infections is important to reduce the formation of adhesions and the loss of peritoneal area, which can limit the patient’s ability to continue with PD.

ANSWER: C

References 1. Blake PG, Sharma A: Peritoneal dialysis. In Brenner BM, editor: Brenner & Rector’s the kidney, ed 8, Philadelphia, 2008, WB Saunders. 2. Kawaguchi Y, Tranaeus A: A historical review of encapsulating peritoneal sclerosis, Perit Dial Int 25(S4):S7–13, 2005. 3. Li BD, McDonald JC, Richardson KA, et al: Abdominal wall, umbilicus, peritoneum, mesenteries, omentum, and retroperitoneum. In Townsend CM, Beauchamp RD, Evers BM, et al, editor: Sabiston textbook of surgery: the biological basis of modern surgical practice, ed 18, Philadelphia, 2008, WB Saunders.

e51

4 

Peritoneal Lesion – Biopsy

GOALS/OBJECTIVES • • •

ANATOMIC CONSIDERATIONS INDICATIONS TECHNIQUE (OPEN/LAPAROSCOPIC)

PERITONEUM AND PERITONEAL CAVITY Richard H. Turnage  /  Brian Badgwell

4-1 

From Abdominal wall, umbilicus, peritoneum, mesenteries, omentum and retroperitoneum. In: Townsend CM: Sabiston Textbook of Surgery, 19th edition (Saunders 2012)

PERITONEUM AND PERITONEAL CAVITY Anatomy The peritoneum consists of a single sheet of simple squamous epithelium of mesodermal origin, termed mesothelium, lying on a thin connective tissue stroma. The surface area is 1.0 to 1.7 m2, approximately that of the total body surface area. In males, the peritoneal cavity is sealed, whereas in females it is open to the exterior through the ostia of the fallopian tubes. The peritoneal membrane is divided into parietal and visceral components. The parietal peritoneum covers the anterior, lateral, and posterior abdominal wall surfaces and the inferior surface of the diaphragm and the pelvis. The visceral peritoneum covers most of the surface of the intraperitoneal organs (i.e., stomach, jejunum, ileum, transverse colon, liver, spleen) and the anterior aspect of the retroperitoneal organs (i.e., duodenum, left and right colon, pancreas, kidneys, adrenal glands). The peritoneal cavity is subdivided into interconnected compartments or spaces by 11 ligaments and mesenteries. The peritoneal ligaments or mesenteries include the coronary, gastrohepatic, hepatoduodenal, falciform, gastrocolic, duodenocolic, gastrosplenic, splenorenal, and phrenicocolic ligaments and the transverse mesocolon and small bowel mesentery (Figure 4-1-1). These structures partition the abdomen into nine potential spaces – right and left subphrenic, subhepatic, supramesenteric and inframesenteric, right and left paracolic gutters, pelvis, and lesser space. These ligaments, mesenteries, and peritoneal spaces direct the circulation of fluid in the peritoneal cavity and thus may be useful in predicting the route of spread of infectious and malignant diseases. For example, perforation of the duodenum from peptic ulcer disease may result in the movement of fluid (and the development of abscesses) in the subhepatic space, right paracolic gutter, and pelvis. The blood supply to the visceral peritoneum is derived from the splanchnic blood vessels, whereas the parietal peritoneum is supplied by branches of the intercostals, subcostal, lumbar, and iliac vessels.

Physiology The peritoneum is a bidirectional, semipermeable membrane that controls the amount of fluid in the peritoneal cavity, promotes the sequestration and removal of bacteria from the peritoneal cavity, and facilitates the migration of inflammatory cells from the microvasculature into the peritoneal cavity. Normally, the peritoneal cavity contains less than 100 mL of sterile serous fluid. Microvilli on the apical surface of the peritoneal mesothelium markedly increase the surface area and promote the rapid absorption of fluid from the peritoneal cavity into the lymphatics and portal and systemic circulations. The amount of fluid in the peritoneal cavity may increase to many liters in some diseases, such as cirrhosis, nephrotic syndrome, and peritoneal carcinomatosis. The circulation of fluid in the peritoneal cavity is driven in part by the movement of the diaphragm. Intercellular pores in the peritoneum covering the inferior surface of the diaphragm (termed stomata) communicate with lymphatic pools in the diaphragm. Lymph flows from these diaphragmatic lymphatic channels through subpleural lymphatics to the regional lymph nodes and, ultimately, the thoracic duct. Relaxation of the diaphragm during exhalation opens the stomata and the negative intrathoracic pressure draws fluid and particles, including bacteria, into the stomata. Contraction of e53

e54   SECTION 1  ■  ABDOMEN – GENERAL

FIGURE 4-1-1  Peritoneal ligaments and mesenteric reflections in the adult. These attachments partition the abdomen into nine potential spaces – right and left subphrenic, subhepatic, supramesenteric and inframesenteric spaces, right and left paracolic gutters, pelvis, and omental bursa (inset, right). (From McVay C: Anson and McVay’s surgical anatomy, ed 6, Philadelphia, 1984, WB Saunders, p 589.)

the diaphragm during inhalation propels the lymph through the mediastinal lymphatic channels into the thoracic duct. It is postulated that this so-called diaphragmatic pump drives the movement of peritoneal fluid in a cephalad direction toward the diaphragm and into the thoracic lymphatic vessels. This circulatory pattern of peritoneal fluid toward the diaphragm and into the central lymphatic channels is consistent with the rapid appearance of sepsis in patients with generalized intra-abdominal infections, as well as the perihepatitis of Fitz-Hugh Curtis syndrome in patients with acute salpingitis. The peritoneum and peritoneal cavity respond to infection in five ways: 1. Bacteria are rapidly removed from the peritoneal cavity through the diaphragmatic stomata and lymphatics. 2. Peritoneal macrophages release proinflammatory mediators that promote the migration of leukocytes into the peritoneal cavity from the surrounding microvasculature. 3. Degranulation of peritoneal mast cells releases histamine and other vasoactive products, causing local vasodilation and the extravasation of protein-rich fluid containing complement and immunoglobulins into the peritoneal space. 4. Protein in the peritoneal fluid opsonizes bacteria, which, along with activation of the complement cascade, promotes neutrophil- and macrophage-mediated bacterial phagocytosis and destruction. 5. Bacteria become sequestered within fibrin matrices, thereby promoting abscess formation and limiting the generalized spread of the infection.

Peritoneal Disorders Ascites Pathophysiology and cause.  Ascites is the pathologic accumulation of fluid in the peritoneal cavity. The principal causes of ascites formation and their pathophysiologic bases are listed in Box 4-1-1. Cirrhosis is the most common cause of ascites in the United States, accounting for approximately 85% of cases. Ascites is the most common complication of cirrhosis, with approximately 50% of compensated cirrhotic patients developing ascites within 10 years of diagnosis. The onset of ascites is an important prognostic factor for poor outcome in patients with cirrhosis because of its association with the occurrence of spontaneous bacterial peritonitis, renal failure, a worsened quality of life, and an increased likelihood of death within 2 to 5 years.

CHAPTER 4-1  ■  Peritoneum And Peritoneal Cavity  

Box 4-1-1  PRINCIPAL CAUSES OF ASCITES FORMATION CATEGORIZED ACCORDING TO UNDERLYING PATHOPHYSIOLOGY Portal Hypertension Cirrhosis Noncirrhotic • Prehepatic portal venous obstruction • Chronic mesenteric venous thrombosis • Multiple hepatic metastases • Posthepatic venous obstruction: Budd-Chiari syndrome Cardiac Congestive heart failure Chronic pericardial tamponade Constrictive pericarditis Malignancy Peritoneal carcinomatosis • Primary peritoneal malignancies • Primary peritoneal mesothelioma • Serous carcinoma • Metastatic carcinoma • Gastrointestinal carcinomas (e.g., gastric, colonic, pancreatic cancer) • Genitourinary carcinomas (e.g., ovarian cancer) Retroperitoneal obstruction of lymphatic channels • Lymphoma • Lymph node metastases (e.g., testicular cancer, melanoma) Obstruction of the lymphatic channels at the base of the mesentery • Gastrointestinal carcinoid tumors Miscellaneous Bile ascites • Iatrogenic after operations of the liver or biliary tract • Traumatic after injuries to the liver or biliary tract Pancreatic ascites • Acute pancreatitis • Pancreatic pseudocyst Chylous ascites • Disruptions of retroperitoneal lymphatic channels • Iatrogenic during retroperitoneal dissections: Retroperitoneal lymphadenectomy, abdominal aortic aneurysmorrhaphy • Blunt or penetrating trauma • Malignancy • Obstruction of retroperitoneal lymphatic channels • Obstruction of lymphatic channels at the base of the mesentery • Congenital lymphatic abnormalities Primary lymphatic hypoplasia Peritoneal infections • Tuberculous peritonitis • Myxedema • Nephrotic syndrome • Serositis in connective tissue disease

The two principal factors underlying the formation of ascites in cirrhotic patients are renal sodium and water retention and portal hypertension. Renal sodium retention is driven by activation of the renin-angiotensin-aldosterone and sympathetic nervous systems, which cause proximal and distal renal tubule sodium reabsorption. It is postulated that the abnormal release of nitric oxide within the splanchnic circulation causes vasodilation and a decrease in the effective circulating blood volume. Renin, aldosterone, and other hormones are generated as a counterregulatory mechanism to restore the effective circulating blood volume to normal. Portal hypertension is produced by postsinusoidal vascular obstruction from the deposition of collagen in the cirrhotic liver. Increased hydrostatic pressure within the hepatic sinusoids and splanchnic vasculature drives the extravasation of fluid from the microvasculature into the extracellular compartment. Ascites results when the capacity of the lymphatic

e55

e56   SECTION 1  ■  ABDOMEN – GENERAL system to return this fluid to the systemic circulation is overwhelmed. Some recent studies have reviewed the pathophysiology underlying fluid retention, hyponatremia, and ascites formation that characterizes patients with cirrhosis.1,2 Obstruction of the portal or hepatic venous blood flow in the absence of cirrhosis (e.g., portal vein thrombosis or Budd-Chiari syndrome, respectively) also causes ascites formation by increasing hydrostatic pressure within the splanchnic microvasculature. A similar pressure-based mechanism contributes to ascites formation in patients with heart failure, although the release of vasopressin and renin-angiotensin-aldosterone also promote sodium and water retention in these patients. Patients with malignancies develop ascites by one of three mechanisms: 1. Multiple hepatic metastases cause portal hypertension by narrowing or occluding branches of the portal venous system. 2. Malignant cells scattered throughout the peritoneal cavity release protein-rich fluid into the peritoneal cavity, as in peritoneal carcinomatosis. 3. Obstruction of retroperitoneal lymphatics by a tumor, such as lymphoma, causes rupture of major lymphatic channels and the leakage of chyle into the peritoneal cavity. Finally, ascites may result from the leakage of pancreatic juice, bile, or lymph into the peritoneal cavity after an iatrogenic or inflammatory disruption of a major pancreatic, bile, or lymphatic duct. Clinical Presentation and Diagnosis.  The diagnosis of ascites is made on the basis of the medical history and appearance of the abdomen. Obviously, risk factors for hepatitis or cirrhosis are sought, as is evidence of cardiac or renal disease or malignancy. A full bulging abdomen with dullness of the flanks on percussion is suggestive of the presence of ascites. Approximately 1.5 liters of fluid must be present before dullness can be detected by percussion. Physical evidence of cirrhosis is also sought, such as palmar erythema, dilated abdominal wall collateral veins, and multiple spider angiomas. Patients with cardiac ascites have impressive jugular venous distention and other evidence of congestive heart failure. Ascitic Fluid Analysis.  Paracentesis with ascitic fluid analysis is the most rapid and cost-effective method of determining the cause of ascites and should be performed on patients with new-onset ascites. Another important indication for early paracentesis in a patient with ascites is the occurrence of signs and symptoms of infection, such as abdominal pain or tenderness, fever, encephalopathy, hypotension, renal failure, acidosis, and/or leukocytosis. Paracentesis can be performed safely in most patients, including those with cirrhosis and mild coagulopathy. It is usually performed in the lower abdomen, with the left lower quadrant preferred over the right. Ultrasound guidance may be useful in obese patients and in those with a history of laparotomy. Runyon3 has suggested that only ongoing disseminated intravascular coagulation or clinically evident fibrinolysis is a contraindication to paracentesis in patients with ascites. In this study, no cases of hemoperitoneum, death, or infection after more than 229 paracenteses performed in 125 cirrhotic patients were reported; abdominal hematomas occurred in 2% of cases, with only 50% of these requiring blood transfusion. Examination of the ascitic fluid begins with its gross appearance. Normal ascitic fluid is slightly yellow and transparent. The presence of more than 5000 leukocytes/mm3 will cause the fluid to be cloudy, whereas ascitic fluid specimens with fewer than 1000 cells/mm3 are almost clear. Blood in the ascitic fluid may be caused by a traumatic tap, in which case the fluid may be blood-streaked and will often clot unless immediately transferred to a tube containing an anticoagulant. Nontraumatic bloodtinged ascitic fluid does not clot because the required factors have been depleted by previous clotting in the peritoneal cavity. Lipid in the ascitic fluid, such as that which accompanies chylous ascites, causes the fluid to appear opalescent, ranging from cloudy to completely opaque. If placed in the refrigerator for 48 to 72 hours, the lipids usually layer out. The most valuable laboratory tests on ascitic fluid are the cell count, differential, and determination of ascitic fluid albumin and total protein concentrations. The leukocyte count in uncomplicated cirrhotic ascites is usually less than 500 cells/mm3, and approximately 50% of these cells are neutrophils. More than 250 neutrophils/mm3 of ascitic fluid suggests an acute inflammatory process, the most common of which is spontaneous bacterial peritonitis. In this case, the total white blood cell and absolute neutrophil counts are elevated, with neutrophils accounting for more than 70% of the total cell count. The serum-ascites albumin gradient (SAAG) is the most reliable method to categorize the various causes of ascites. The SAAG is calculated by measuring the albumin concentration of serum and ascitic

CHAPTER 4-1  ■  Peritoneum And Peritoneal Cavity  

TABLE 4-1-1  Classification of Ascites by Serum-Ascites Albumin Gradient High Gradient (≥1.1 g/dl)

Low Gradient (3 cm) gallstones. Most gallbladder polyps are simply cholesterol depositions in the gallbladder wall, and are therefore treated based on symptoms rather than out of concern for neoplastic risk. Enlarging polyps or those greater than 1 cm in size may rarely be adenomatous and premalignant, and in such settings cholecystectomy is advised regardless of symptoms. Because of the prevalence of cholelithiasis on imaging studies, a final question on indications for cholecystectomy relates to the issue of asymptomatic gallstones. Most patients with asymptomatic stones do not develop subsequent symptomatic disease, and 60% to 70% or more in some series remain asymptomatic long term. Exceptions where “prophylactic” cholecystectomy should be considered would include patients who are immunocompromised, awaiting organ transplantation, and those with sickle cell disease. Patients with diabetes do not have an increased risk of fulminant acute disease, and other than their comorbid status, they are not at risk for advanced initial presentation; accordingly, prophylactic cholecystectomy is not recommended in the setting of diabetes. Contraindications to laparoscopic cholecystectomy are limited to patients in whom general anesthesia and pneumoperitoneum are precluded, such as with acute cardiopulmonary disease. In these settings, nonoperative strategies may serve the patient best. Advanced cirrhosis, suspicion of gallbladder carcinoma, necrotic gallbladder with extensive surrounding inflammation, Mirizzi’s syndrome, concomitant

CHAPTER 9-1  ■  Laparoscopic Cholecystectomy  

acute cholangitis with sepsis unamenable to endoscopic or other nonoperative therapy, and cholecystoenteric fistulous disease are other settings in which an open approach may be advantageous. The surgeon’s experience and ability to safely delineate the anatomy, determine the appropriateness of the laparoscopic approach in more complicated settings, and attend to patient safety is always the most important consideration. Pregnancy is not a contraindication to laparoscopic cholecystectomy, but the operation should be deferred until after delivery when possible. If operation during gestation is necessary, the second trimester is the most advantageous time because of the heightened risk of spontaneous abortion in the first trimester and limited peritoneal access and risk of premature labor in the third trimester.

TECHNIQUE The patient is positioned with arms abducted on arm boards and dual monitors off the patient’s shoulders, so they are respectively in a direct visual line for the surgeon standing on the patient’s left and the first assistant opposite. If there is a high likelihood of cholangiographic guided intervention such as laparoscopic CBD duct exploration, it is helpful to tuck the arms to facilitate positioning of C-arm fluoroscopy and additional equipment that may be used. A foot board is useful to prevent the patient sliding on the table when in reverse Trendelenburg position. Preparation is done from the nipples to the groins and should always include areas needed for laparotomy, should it become necessary. Antibiotic prophylaxis is controversial in elective settings but is still our routine practice: a single dose, given at the time of anesthetic induction. Bacterobilia is present in up to 25% of elderly patients and approximately 10% of the overall population, although the clinical significance is rare in uncomplicated settings. General anesthesia is required, and initial carbon dioxide pneumoperitoneum may be achieved with an open technique or with a Veress needle. Because it facilitates safety in reoperative settings and creates a port site suitable for subsequent organ extraction, we prefer to use an open approach. A paraumbilical entry is made, as dictated by prior surgical scars and the position of the umbilicus in relation to the gallbladder fossa; stay sutures are placed, and a Hasson-type trocar is secured. Initial insufflation should be done at low-flow settings to avoid vasovagal responses and to ensure safe positioning until visually confirmed with laparoscope introduction. A 15 mm Hg pressure limit is typically utilized, and lower pressure settings may be appropriate with pregnancy or cirrhosis. We prefer to use a 5 or 10 mm, 30-degree angled laparoscope for its visual versatility. Additional trocars are placed under laparoscopic visualization in the epigastrium, just to the patient’s right of the falciform ligament (typically 10 mm) and the right subcostal area in the midclavicular and anterior axillary lines (typically 5 mm). Sizes may be decreased to 5 mm at all sites if a 5 mm laparoscope and clip applier are employed. Smaller (2 mm) and so-called needlescopic trocars have been employed but are without added significant cosmetic or recovery benefit, and these may be of limited efficacy in settings of obesity or advanced gallbladder pathology. Various “single site” techniques have also been more recently described and typically involve placement of multiple trocars through a single paraumbilical incision and juxtaposed fascial trocar insertion sites. These techniques provide a single incision cosmetically but may limit the surgeon’s retraction and exposure options compared with standard technique; their ultimate utility and efficacy remain a subject of study, especially in more complicated cases. The patient is placed in reverse Trendelenburg position after initial trocar placement, and it is sometimes helpful, particularly if there is an enlarged left hepatic lobe or pregnancy, to rotate patients slightly to their left to facilitate exposure and venous return respectively. The gallbladder is elevated in a cephalad direction by grasping the fundus from the right lateral trocar, and the infundibulum is grasped via the midclavicular trocar and retracted laterally, toward the patient’s right, to open the hepatocystic triangle. Surgeons may have assistants provide this retraction, or they may control the infundibulum themselves, which facilitates bimanual manipulation and dissection. The visceral peritoneum is then opened over the area of the gallbladder/cystic duct junction by grasping and pulling in the opposite direction of the infundibular retraction, namely medially and inferiorly. Subsequent grasping and tearing of the investing peritoneum of the triangle itself, alternating anteriorly and posteriorly as the assistant provides opposing traction and exposure, allows the entire triangle to be divested of its peritoneal covering before any structures within it are developed. Cautery is avoided until all structures have been so exposed, and the anatomy clearly delineated, to avoid injury to biliary or vascular structures that may variably, but not uncommonly, be present and require preservation.

e159

e160   SECTION 3  ■  ABDOMEN – BILIARY The cystic duct and artery are then developed by gentle dissection from the investing areolar tissues using sweeping, spreading, and gentle teasing motions of the dissecting forceps. The cystic duct and artery in particular are carefully traced to their junctions with the gallbladder proper and are developed to allow control with clips. Care should be taken to note the position of the CBD prior to clip application and to use cholangiography liberally if the anatomy is uncertain or appears atypical. It is very helpful to keep in mind the common variations of cystic duct and cystic arterial anatomy as the dissection is accomplished. A so-called critical view is a useful concept at this phase, meaning that before any structures are clipped or divided, the cystic duct and artery are developed adequately to see their course to the gallbladder, with the infundibulum retracted laterally and hepatic parenchyma visible posteriorly through the window developed between those structures. Although cholangiography has not been shown to clearly prevent bile duct injury, complete and properly interpreted cholangiograms do help to minimize the risk of major ductal injury. The cystic duct and artery are then clipped and divided. If the cystic duct is larger or more edematous than what may be suitable for clipping, absorbable suture with intracorporeal or extracorporeal knot technique is a suitable alternative. It is our practice to gently tease the areolar tissues that stretch between the gallbladder infundibulum and the gallbladder fossa with a dissecting forceps before using cautery, even after division of the cystic duct and artery; this is because it is not uncommon to encounter an accessory cystic artery or, more rarely, a right hepatic biliary ductal or arterial branch. If identified, such structures are carefully developed and controlled or spared as appropriate. Once this is done, the gallbladder is then mobilized off the hepatic fossa, with cautery applied with either a hook instrument or laparoscopic scissors, as the surgeon’s preference dictates. The hook is useful for allowing the operator to create and direct added tension in addition to that provided by the assistant. It is important for the surgeon to keep the dissection on the plane between the gallbladder and liver capsule to minimize the risk of bile spillage, bleeding, and postoperative bile leaks from terminal ductules in the hepatic parenchyma. Cautery application is limited during the dissection to 2 or 3 second applications to minimize the risk of establishing capacitance circuits in the abdomen that may then lead to bowel or other visceral injury. It is also important to avoid cautery application adjacent to any clips staying in the patient, as conduction of the current to these can lead to delayed thermal injuries, which can include strictures and leaks. The camera operator should carefully work during this phase, as throughout the procedure, to ensure that no instrument introduction or cautery application takes place outside the visual field, which may dynamically alter as tension and cautery are applied. Prior to final delivery of the gallbladder, the operative area is reinspected for any evidence of bleeding, bile leakage, or insecure clips, after which the gallbladder is fully released. Reducing the insufflation pressure settings to watch for venous bleeding in the liver bed that may otherwise be offset by the pressure of the pneumoperitoneum may be wise, particularly in patients with portal hypertension. The gallbladder is then delivered via the umbilical incision after transferring the laparoscope to the epigastric port. A bag may be utilized but is not necessary if the gallbladder was not entered during dissection. If the gallbladder was entered, any spilled stones should be retrieved if possible. Scooping forceps are useful for this purpose. Although delayed sequelae of spilled stones are relatively rare compared with the number of cases in which some degree of bile spill occurs (up to 30% in some series), a variety of delayed inflammatory and infectious complications have been described, so such a spill is clearly not innocuous. The trocars are removed under laparoscopic guidance to observe for bleeding. The fascia at the umbilical site is closed with absorbable suture, and stay sutures are placed during Hasson insertion. Fascia is typically not closed at the remaining sites, although closure of the 10 mm epigastric site may be done at the surgeon’s discretion using a suture-passing device under laparoscopic guidance. Skin closure is accomplished with absorbable subcuticular suture, and dressings are applied. In complicated settings such as acute cholecystitis, several adjunctive or alternative techniques may be useful. Perhaps the most useful technique with a tensely distended, acutely inflamed, or gangrenous gallbladder is to aspirate the organ before grasping. This can be done with a laparoscopic aspiration needle introduced through a port or a large length and caliber (14 to 18 gauge) spinal or intravenous access needle introduced directly through the abdominal wall. The gallbladder may also be opened with scissors and suctioned, although this may lead to a higher risk of stone spillage, if this entry site is not readily and persistently controlled with a grasper or suture closure through the subsequent

CHAPTER 9-1  ■  Laparoscopic Cholecystectomy  

dissection. Toothed graspers may be helpful in controlling the gallbladder once decompressed but may also increase the risk of further tearing and spill during dissection. Adhesions are typically present in such advanced inflammatory settings but are often acute and edematous, and blunt sweeping on the plane directly on the gallbladder wall will typically allow freeing of the structure to allow subsequent grasping and retraction. As the infundibulum is exposed, continued dissection of the inflamed tissue planes is safest if it proceeds from right lateral to medial, as this achieves added retraction freedom and often allows the critical structure dissection to be commenced only after optimizing retraction and exposure. Use of a suction/irrigation device is critical in preserving the field of view in such cases where bleeding from the inflamed surfaces can otherwise complicate visualization, and such a device often proves to be a very useful blunt dissecting instrument as well in such settings. If appropriate visualization and exposure cannot be achieved with this approach, conversion to an open procedure should be done in the interest of patient safety. A dome or fundus-down dissection technique is also potentially useful in cases of advanced inflammation or fibrosis that obscures visualization of the hepatocystic triangle via standard techniques. In this setting, the liver may be elevated by a variety of retractors, either table or handheld, and the gallbladder is dissected off the hepatic bed so that infundibular dissection is circumferential down to the level of the cystic duct and artery. The surgeon should also remember that cholangiography can be obtained by injection into the gallbladder if needed to delineate anatomy and guide safe operative decisions, provided no impacted stone obstructs the gallbladder/cystic duct junction. Another technique sometimes required in the setting of advanced inflammatory disease is that of leaving a portion of the posterior wall of the gallbladder on the liver. This may be done if the dissection off the liver bed is not possible in an area of advanced fibrotic change. In such settings, the surgeon should remain cognizant of the possibility of neoplasia, and if suspected, conversion to an open procedure with oncologically appropriate wide excision of the gallbladder bed and regional lymphadenectomy would be advised. If a portion of the posterior wall is left on the hepatic bed because of advanced fibrosis and nonneoplastic disease that renders removal of that portion of the gallbladder unduly hazardous, the residual mucosa is cauterized, and a temporary drain is often left in place. Drain placement is sometimes helpful in settings of advanced inflammation in general, as a means to evacuate fluid and monitor for low-grade bile leaks from the liver bed area following such difficult dissections, and this may be readily done laparoscopically by advancing a grasper from the lateral retracting port through the epigastric trocar with retrograde drain positioning. According to national databases, conversion to an open procedure is required in 5% to 10% of cases, and in up to 25% of cases with severe inflammation. The surgeon should be aware that common duct injuries have been shown to be more likely very early in a surgeon’s learning curve with the procedure but also later in their application of the technique, likely representing a willingness to employ it with more advanced disease as experience and confidence have grown. A paramount commitment to patient safety, steady progress of the dissection, adequate visualization and exposure, and clear delineation of all anatomy before structure division will appropriately serve the surgeon in the decision about whether to convert to an open approach. In any event, such a decision should not be seen as a failure but rather an appropriate judgment in the interest of patient safety. As a final comment on the technique of laparoscopic cholecystectomy, there is growing interest in the concept of natural orifice transluminal endoscopic surgery (NOTES) approaches to cholecystectomy. Clinical experience has been described with both transgastric and transvaginal NOTES approaches to cholecystectomy. Many of the initial reports involve hybrid techniques with laparoscopic or transabdominal access along with the transluminal approach. Meaningful benefits of this technique, as well as the risks and technical limits of such approaches, if and when they are applied on a widespread scale, remain to be determined. Expanded endoscopic and laparoscopic instrumentation capabilities will be a likely beneficial outcome of this area of endeavor, regardless of its eventual broader clinical applications or lack thereof.

COMPLICATIONS The most common intraoperative complications encountered are bleeding and stone spillage. The latter is mentioned above and is estimated in the literature to occur in up to 30% of cases. Spillage is more

e161

e162   SECTION 3  ■  ABDOMEN – BILIARY likely to occur in settings of advanced or acute inflammation and should be addressed with a judicious effort to retrieve all spilled stones owing to the relatively rare, but not insignificant, well-described risk of delayed infectious and other complications of retained spilled stones. When encountered, bleeding is often seen during the dissection of the structures in the hepatocystic triangle, especially in the setting of advanced acute inflammation or chronic fibrosis. In such cases bleeding typically arises from an inadvertent trauma to a cystic artery or other communicating branch of the right hepatic artery. The surgeon should resist the temptation to blindly use cautery or clips without first clearly delineating the anatomy. A useful technique when simple suction, irrigation, or gentle grasping and continued observation fail to allow efficient and appropriately directed control is for the assistant to use the infundibulum as a tamponade agent and push it via the grasper retracting that portion of the gallbladder into the area of bleeding. The surgeon may then place a fifth trocar into the abdomen, typically between the umbilicus and epigastric trocar sites, and work bimanually with suction and dissection to delineate the anatomy and allow safe control of the clearly delineated source. Inability to rapidly control bleeding should prompt conversion to an open approach. Bleeding from the hepatic bed is usually readily controlled with limited cautery application or with topical prothrombotic agents or argon plasma coagulation if necessary. Trocar-site bleeding is often seen during inspected trocar withdrawal if it does occur, and it is most efficiently dealt with by placement of transfascial sutures around the site using suture placement tools such as the Carter–Thompson device, which was designed for such placement. Bile duct injury is the most feared complication of laparoscopic cholecystectomy, and although rare in experienced hands, it continues to be reported more frequently (0.3% to 1.0% incidence) than in series done via an open approach. Avoidance is best achieved by the measures outlined above, including avoidance of cautery early in the dissection or in proximity to major structures or clips that can act as conductors; clear visualization and demonstration of the anatomy, including liberal use of complete and properly interpreted cholangiography in unclear settings; optimization of visualization through careful clearance of any bleeding prior to control of structures during dissection; and exercising particular care in settings of advanced inflammatory disease. Unfortunately, a high percentage of bile duct injuries are not recognized when they occur. If recognized, small tangential injuries of the common duct, such as a partial tear due to avulsion/retraction at the cystic duct insertion site, may be dealt with by T tube placement. More advanced injuries include full-thickness transections of the CBD, injuries associated with excision of a portion of the common duct or right hepatic duct mistaken for the cystic duct, or injuries associated with thermal energy; these should be dealt with via Roux-en-Y hepaticojejunostomy. Injuries recognized in the early postoperative period, manifested by jaundice or biloma formation, should be investigated with high-resolution cholangiography such as ERCP or percutaneous cholangiography. For partial-thickness injuries and low-volume leaks, sphincterotomy and/or stenting via one of these approaches may achieve control of the leak and allow healing. Endoscopic stenting in prolonged fashion may also allow successful nonoperative management of some partial-thickness injuries and partial strictures of the bile duct following operative injury. For more advanced injuries such as complete ductal division, obstruction, or excision, unless recognized very early in the course, the best option is a delayed hepaticojejunostomy reconstruction by a highly experienced biliary surgeon after thorough control of the leak and decompression of the biliary system with detailed cholangiographic delineation of the injury via percutaneous drains and catheters. Other rare complications include trocar site hernias, which are typically at the umbilicus and most commonly occur when a preexisting umbilical hernia was not addressed at the time of surgery, or when inadequate fascial closure is accomplished at this site. If an umbilical hernia is present preoperatively, a useful strategy is to use the hernia as the initial access site and repair it at the end of the procedure. Changes in bowel habits with increased stool frequency is a relatively frequent complaint after cholecystectomy, occurring in up to 25% of patients; but in the vast majority, this resolves over a period of weeks to months after the surgery. If persistent, such complaints should be evaluated to exclude other sources of the change in bowel habits, and if none are identified, symptoms may be ameliorated with medical measures, such as fiber supplementation or cholestyramine administration.

SUMMARY Laparoscopic cholecystectomy offers a well-attested means of control for the symptoms of gallbladder disease, by far the most common intra-abdominal pathology encountered in the typical gastrointestinal

CHAPTER 9-1  ■  Laparoscopic Cholecystectomy  

surgical practice. As such, it is clearly established at this point as the procedure of choice in dealing with symptomatic disease of the gallbladder. The principles learned and repetitively applied in performing this operation form the foundation of most surgeons’ skill set in minimally invasive surgical technique, both before and, by volume criteria if not otherwise, after residency training. A thorough understanding of the indications, anatomical variations, technique, and potential complications of this procedure – with attention to their prevention, recognition, and management – is thus of paramount significance in surgical education and practice.

Suggested Reading MacFadyen BV, Vecchio R, Ricardo AE, et al: Bile duct injury after laparoscopic cholecystectomy: the United States experience, Surg Endosc 12:315, 1998. Mellinger JD: Cholecystectomy in chronic cholecystitis. In MacFadyen BV, Ponsky JL, editors: Operative laparosocopy and thoracoscopy, Philadelphia, 1996, Lippincott-Raven. Mellinger JD, Eldridge TJ, Eddelmon ED, et al: Delayed gallstone abscess following laparoscopic cholecystectomy, Surg Endosc 8:1332, 1994. Olsen DO, Wolfe RS: Laparoscopic cholecystectomy: the technique. In MacFadyen BV, editor: Laparoscopic surgery of the abdomen, New York, 2004, Springer-Verlag. Shamiyeh A, Wayand W: Laparoscopic cholecystectomy: early and late complications and their treatment, Langenbecks Arch Surg 389:164, 2004.

e163

9-2 

CHOLECYSTECTOMY Ashley H. Vernon From Vernon AH, Ashley SW: Atlas of Minimally Invasive Surgical Techniques, 1st edition (Saunders 2012)

STEP 1. SURGICAL ANATOMY The triangle of Calot is the most important anatomic boundary that needs to be defined when performing cholecystectomy. It is formed by the boundaries of the cystic duct, common hepatic duct, and cystic artery. Roughly parallel to this triangle is the hepatocystic triangle in which the cystic artery boundary is replaced with the liver edge. The only structure that should be found within this triangle is the cystic artery.

STEP 2. PREOPERATIVE CONSIDERATIONS Patient Preparation Straightforward symptoms of biliary colic along with objective evidence of gallstones by any imaging modality (ultrasound most sensitive) constitute adequate information to recommend cholecystectomy. In the absence of stones or sludge, studies of biliary function may help to diagnose pathology and need for surgery. Asymptomatic patients with stones do not generally require surgery unless immunocompromised. Obviously, the risks and benefits need to be weighed for each patient in making a recommendation for surgery. In cases of acute cholecystitis, inflammation of the gallbladder and bile ducts makes both open and laparoscopic cholecystectomy more difficult. In early or mild cases, the procedure can usually be managed laparoscopically. However, in severe cases it is often best to manage conservatively with intravenous antibiotics and delay surgery until the inflammation has subsided. Occasionally, in very sick patients, a cholecystostomy tube for drainage is also needed.

Equipment and Instrumentation A 30-degree scope is helpful for providing additional views, especially of the hepatocystic triangle and surrounding structures. Frequently the common bile duct can be seen without any dissection using this type of endoscope. Standard laparoscopic equipment is used, including the following: ○ Locking grasper to secure the fundus of the gallbladder ○ Atraumatic bowel grasper to maneuver the gallbladder infundibulum ○ Hand-operated L hook (attached to Bovie pencil) to perform dissection of the hepatocystic triangle Maryland dissector or a laparoscopic peanut to isolate the cystic duct and artery ○ Scissors, either Metzenbaum or guillotine, to transect the duct after clipping ○ A clip applier is necessary to control the cystic duct. If the cystic duct is too large for a clip, then a pretied loop can be used to occlude the cystic duct stump. A specimen retrieval bag is recommended to prevent spillage of bile and stones into the abdomen during extraction of the gallbladder from the abdomen. e164

CHAPTER 9-2  ■  Cholecystectomy  

Anesthesia Prophylaxis for DVT is important in all patients undergoing laparoscopic procedures. We use sequential compression devices and subcutaneous heparin is started before induction with anesthesia. An orogastric tube is placed to decompress the stomach and duodenum to facilitate exposure during the procedure. Patients are asked to void before being brought to the operating room for their comfort after surgery. A Foley catheter is not usually necessary because ports are not placed in the lower abdomen. If a longer procedure is anticipated, then a catheter is placed at the beginning of the procedure.

Room Setup and Patient Positioning The patient is placed in the supine position. The surgeon operates from patient’s left side. The arms may remain out on armboards unless a cholangiogram is anticipated, in which case the right arm should be tucked to make room for the C-arm. Although not usually an issue with thin patients, footplates are used to secure the patient on the table and prevent sliding, as most of the procedure is performed with the patient in the reverse Trendelenburg position. The table is rolled toward the left so that the abdominal contents fall into the left lower quadrant by gravity. This position is also more comfortable because the patient is brought closer to the surgeon and the left-handed instrument can reach the working port more easily.

STEP 3. OPERATIVE STEPS Access and Port Placement The abdomen is insufflated using a Veress needle at the umbilicus, and a 10-mm port is placed through a vertical incision through the umbilicus. This allows the largest incision to be “hidden” entirely, as the scar will be buried in the base of the umbilicus (Figure 9-2-1). A 5-mm port is placed in the right flank, near the costal margin and lateral, just anterior to the right colon, so that it does not interfere with the operating surgeon’s left hand. This is at the anterior axillary line. Through this port, the locking grasper is placed for retraction of the gallbladder fundus up and over the liver edge. It remains relatively stationary throughout the procedure. The upper epigastric port is used for introduction of the clip applier and must be at least 10 mm in size. The surgeon may choose to orient this in line with a subcostal incision in the event of a conversion to an open procedure. The surgeon’s left-hand instruments are placed through a 5-mm port in the patient’s right upper abdomen. It is optimal if the camera and two operating ports form a diamond shape with the gallbladder at the top corner. The assistant will hold the camera and direct the retraction of the gallbladder fundus.

FIGURE 9-2-1  Access and port placement.

e165

e166   SECTION 3  ■  ABDOMEN – BILIARY The procedure is performed through four ports. To prevent port site hernias, try to use the smallest ports possible for the procedure. For most patients, opt to use two 10-mm ports – one for the 10-mm scope and the other for the 10-mm clip applier. By using a 5-mm scope and a 5-mm clip applier, the procedure can be accomplished using all small instruments, although one port, usually the umbilical port, will need to be enlarged to remove the gallbladder.

Description of Procedure The operation begins by exposing the gallbladder infundibulum. The assistant gently retracts the fundus in a cephalad direction, bringing the entire length of the gallbladder into view. Often, adhesions to the upper part of the gallbladder prevent this maneuver and must be divided using the hook electrocautery or stripped away from the gallbladder bluntly. Once retraction is secured, the patient is moved into a 30-degree reverse Trendelenburg position, with the table rolled with the left side down to improve visualization of the critical structures and to bring the patient closer to the surgeon. The infundibulum of the gallbladder is pulled laterally by the surgeon’s left hand when starting on the medial surface of the gallbladder. This distracts the cystic duct away from the common bile duct so that they are at right angles to each other. This will make identification of the cystic duct easier and helps to prevent inadvertent injury. The peritoneum overlying the lower edge of the gallbladder is opened using the hook electrocautery. The cystic node can be used to judge where the lower edge of the gallbladder is. The peritoneum overlying the node should be divided on the side abutting the gallbladder in order to avoid dividing the vessel coming up to the node. The node is then swept down, exposing the lower edge of the gallbladder (Figure 9-2-2). Once the peritoneum on the medial surface is divided, the gallbladder infundibulum is pulled medially, exposing the right side or “back side.” Occasionally, a posterior branch of the artery is encountered as it branches off of the cystic artery below the gallbladder wall. It can be doubly clipped, or cauterized with the hook, and divided. Division of the peritoneum is continued all the way to the liver edge (Figure 9-2-3).

FIGURE 9-2-2  Dissection of hepatocystic triangle.

CHAPTER 9-2  ■  Cholecystectomy  

FIGURE 9-2-3  Clipping of cystic artery and duct.

The dissection should continue on the lower margin of the gallbladder. Identification of the wall and structures may require going back and forth, changing the position of the gallbladder and the angle of the 30-degree laparoscope, until the structures are defined. Every time the hook electrocautery is used, care should be taken to identify the tissue being divided. The inclination is to divide the deeper tissues, but this is not necessary until the peritoneum is divided on both sides. Being mindful to divide only “see-through” tissues will prevent the inadvertent division of vessels or ducts. The peanut or Maryland dissector is used for dissection of the hepatocystic triangle and stripping the structures to accommodate the clip applier. At this point, the gallbladder is removed from the liver bed using the “heel” of the hook instrument. Maintaining constant tension is critical to avoid entering the gallbladder or liver bed (Figure 9-2-4A and Figure 9-2-4B). If bleeding is encountered, control can be obtained by increasing the setting on the electrocautery. The instrument should be held above the surface of the liver to prevent the instrument from sticking to the liver and then dislodging the eschar. Begin cauterizing the most superior portions of the bleeding to avoid blood rolling down. Every effort should be made to extract stones when they spill in the abdomen. The stones can be picked up individually using stone forceps. To expedite this tedious task, the stones can be placed directly into a specimen retrieval bag within the abdomen. If there are many tiny fragments, then irrigant can be removed with a 10-mm suction device. Any bile spillage should be irrigated. The instruments and ports are removed from the abdomen as the CO2 pneumoperitoneum is evacuated, and the skin is reapproximated.

STEP 4. POSTOPERATIVE CARE Once liquids are tolerated and pain is controlled with oral medications, patients are able to leave the hospital. Most patients go home the same day the operation is performed. Patients can remove the dressings and shower the day after the operation.

e167

e168   SECTION 3  ■  ABDOMEN – BILIARY

FIGURE 9-2-4  A, B, Dissection of liver bed.

Activity depends on how the patient feels. Most patients will return to normal activities within a week. Patients are usually seen in the office 2 to 3 weeks after surgery.

STEP 5. PEARLS AND PITFALLS For patients who have had a prior midline incision or who have an umbilical defect, access is obtained using a Veress needle in the left upper quadrant, and the initial trocar is then placed in the lateral position. In obese patients whose umbilicus is displaced caudally, a port placed through the umbilicus will be too low for the surgeon to see the gallbladder clearly. Instead, the port is placed well above the umbilicus, usually around 20 cm below the xiphoid process. Very difficult procedures in which the patient is super morbidly obese can be made easier by performing the procedure using a split leg table with the surgeon operating from between the legs. The table is placed in a steep reverse Trendelenburg position to bring the operative field close to the surgeon.

CHAPTER 9-2  ■  Cholecystectomy  

A very distended gallbladder can be decompressed with a laparoscopic needle aspirator so that the grasper used for retraction of the fundus is able to hold the tissue. When that instrument is unavailable, a Veress needle placed through the right upper quadrant abdominal wall can be introduced directly into the fundus and used to aspirate bile. Retrograde or “fundus-down technique” can be useful when a very inflamed gallbladder is encountered and the anatomy cannot be determined. The procedure is much like the standard open technique with dissection proceeding down along the gallbladder wall. The cystic duct can be transected at any safe level. If the liver is extremely large, there is excessive inflammation, or the gallbladder wall is not sturdy enough to be retracted, a liver retractor introduced through the right lateral port can hold the liver up and facilitate the dissection.

Suggested Reading Hunter JG: Avoidance of bile duct injury during laparoscopic cholecystectomy. Am J Surgery 162(1):71–76, 1991. Way LW, Stewart LS, et al: Causes and prevention of laparoscopic bile duct injuries: analysis of 252 cases from a human factors and cognitive psychology perspective. Annals of Surg 237(4):460–9, 2003.

e169

9-3 

LAPAROSCOPIC CHOLECYSTECTOMY Constantine T. Frantzides  /  Mark A. Carlson  /  Minh Luu From Frantzides CT, Carlson MA: Atlas of Advanced Minimally Invasive Surgery, 1st edition (Saunders 2009)

In terms of raw numbers, there probably are more experts on the performance of laparoscopic cholecystectomy than on any other minimally invasive procedure. Just about every general surgeon does this procedure, does it a lot, and most do it very well. So we will not make an obnoxious claim that we are the übermensch of laparoscopic cholecystectomy. What we can provide in this chapter is our perspective, which has been generated after performing many of these procedures ourselves, and by reading and listening to what others have said and written about the technical aspects of laparoscopic cholecystectomy. The transition from open to laparoscopic cholecystectomy that occurred during the early 1990s stands as a glaring example of how not to introduce a new technology. The proliferation of ductal injuries that ensued from this transition was overshadowed only by the eagerness of the media and medicolegal elements to publicize and prosecute these events. We in the surgical community have no one to blame but ourselves for this unfortunate chapter in the history of minimally invasive surgery. The rush to embrace the new technology simply was too hasty and ill conceived. Be that as it may, the transition to laparoscopic cholecystectomy was completed quickly, and the issue of common bile duct injury now is under somewhat better control, although not eliminated. We believe that a safe dissection is paramount in the performance of laparoscopic cholecystectomy, and we will describe in detail the technical maneuvers we perform to accomplish such a dissection. Other surgeons may emphasize reliance on intraoperative cholangiography to attain the same information that we like to obtain with dissection. This issue of routine versus selective intraoperative cholangiography has been debated over and over again in the literature. We will not regurgitate this debate in this chapter, but simply acknowledge that it exists and that there are multiple approaches to the performance of a safe laparoscopic cholecystectomy.

OPERATIVE INDICATIONS The indication for cholecystectomy in the vast majority of patients is symptomatic gallstones. This indication covers a spectrum of clinical manifestations, from occasional attacks of biliary colic to gangrenous cholecystitis to gallstone pancreatitis. If the patient has symptoms from gallstones and has no absolute contraindications to an operation under general anesthesia, then laparoscopic cholecystectomy generally is indicated. Nonoperative treatment of gallstones (e.g., oral bile acids or shock wave lithotripsy) has limited efficacy. Routine removal of a gallbladder for asymptomatic cholelithiasis is more controversial, but may be a reasonable treatment alternative in a patient with one or more of the following characteristics: “porcelain” (i.e., calcified) gallbladder; gallbladder mass or polyp; young age; diabetes mellitus; organ transplant; sickle cell anemia; and others (the list is growing). Incidental cholecystectomy for asymptomatic cholelithiasis also may be a reasonable option during a bariatric or other gastrointestinal procedure. The timing of cholecystectomy to treat acute cholecystitis is controversial; there is a wealth of retrospective data to support immediate cholecystectomy, delayed operation months later, and everything in between. There is no doubt that removal of an acutely inflamed gallbladder is more difficult than removal of a noninflamed gallbladder, but as many authors have demonstrated, this can be accomplished without an increased incidence of complications. Our own preference is to treat a patient e170

CHAPTER 9-3  ■  Laparoscopic Cholecystectomy  

with acute cholecystitis medically (IV fluids, antibiotics, bowel rest, pain relief ) for up to 1 week, and then remove the gallbladder under “subacute” conditions. If the patient worsens during the first 24 to 48 hours of this treatment, then an emergency cholecystectomy would be indicated. If a patient has gangrenous cholecystitis with severe inflammation that has made dissection in the region of the porta hepatis particularly hazardous, then laparoscopic partial cholecystectomy with drainage is a treatment option. The timing of cholecystectomy in association with gallstone pancreatitis also is somewhat controversial. For the critically ill patient who has extensive pancreatic necrosis, early laparoscopic cholecystectomy generally is dangerous and not useful; the damage already has been done. For the patient with mild, mostly laboratory-based pancreatitis, early cholecystectomy is more reasonable. The difficulty in timing the gallbladder removal is with the patients who fall in between these two extremes. In general, we guide our decision based on the clinical stability of the patient; we prefer to perform laparoscopic cholecystectomy for gallstone pancreatitis electively on a patient whose disease status is quiescent or markedly improved. Occasionally a patient is referred for treatment of biliary colic, but the patient has no gallstones. In this situation a careful evaluation for other sources of the patient’s symptoms should be carried out (see “Preoperative Evaluation”); if the results of this evaluation rule out other causative diagnoses (e.g., peptic ulcer disease, sphincter of Oddi dysfunction), or if a decreased gallbladder ejection fraction without sphincter dysfunction is noted, then cholecystectomy may relieve the patient’s symptoms. The diagnosis in this situation might be “acalculous cholecystitis.” This particular entity classically has been described in critically ill patients in whom the clinical manifestations typically are much more severe.

PREOPERATIVE EVALUATION The goals of the preoperative evaluation for laparoscopic cholecystectomy may be organized as follows: (1) confirm the clinical diagnosis with objective data; (2) evaluate the status of the common bile duct; (3) determine whether preoperative endoscopic retrograde cholangiopancreatography (ERCP) will be necessary; and (4) determine the appropriateness and urgency for the procedure in relation to the patient’s health and history. Routine tests should include a broad chemistry panel (including liver function tests, amylase, lipase), a complete blood count, and an abdominal ultrasound. If the patient has evidence of biliary obstruction, then one option would be to perform a preoperative ERCP, which generally is what we prefer. With this strategy the common duct can be cleared prior to laparoscopic cholecystectomy; if clearance is not possible, then the surgeon should prepare a strategy to deal with this scenario. For the patient with acute cholecystitis, an abdominal computed tomography (CT) scan can help delineate the severity of the process. If the diagnosis of cholecystitis is not clear, then a nuclear medicine gallbladder emptying study (cholescintigraphy or hepatobiliary iminodiacetic acid [HIDA] scan) may help solve the problem by demonstrating nonfilling of the gallbladder. If gallstones are not present in a patient who is having biliary colic, then an extensive evaluation using the preceding tests and possibly including an upper gastrointestinal (GI) series, magnetic resonance imaging (MRI) scan, esophagogastroduodenoscopy (EGD), and ERCP with sphincter of Oddi manometry may determine the cause of the patient’s pain. Cholecystectomy often is performed for vague symptoms in patients who have no gallstones; in order to prevent unnecessary gallbladder removal, the surgeon should ensure that there are no remedial causes for the patient’s symptoms.

PATIENT POSITIONING AND PLACEMENT OF TROCARS The patient is placed supine on the operating room table with the right arm tucked. The operating table should be compatible with a C-arm apparatus if intraoperative cholangiography/fluoroscopy is necessary. If there is a likelihood of open conversion, then it may be helpful to have the patient positioned over a flex point on the operating room table, so that “cracking” the operating room table will help splay open the subcostal region. The surgeon stands on the patient’s left and faces the monitor; and the assistant stands on the right.

e171

e172   SECTION 3  ■  ABDOMEN – BILIARY

FIGURE 9-3-1  Trocar positions for a laparoscopic cholecystectomy. Positions 1 and 2 represent 10-mm trocars; 3 and 4 represent 5-mm trocars.

We prefer to establish pneumoperitoneum by inserting a Hasson cannula at the umbilicus (Figure 9-3-1). If there is an old incision in this location, then intra-abdominal access may be obtained with a Veress needle or optical bladeless trocar elsewhere. After insufflation, the gallbladder region is inspected, and a 5-mm trocar is inserted subcostally in the right midclavicular line; approximately over the gallbladder. A 10-mm port then is inserted in the subxiphoid region just to the right of the falciform ligament. The table is rotated to elevate the right side, and reverse Trendelenburg (head up) position is applied. The gallbladder is grasped and elevated, and the second 5-mm trocar is inserted in a position such that it can grasp the fundus without causing interference with the other trocars (typically lower than the other 5-mm trocar and in the right anterior axillary line).

OPERATIVE TECHNIQUE Frequently adhesions from the gallbladder to the omentum, transverse colon, or duodenum need to be lysed so that the gallbladder can be visualized. In the presence of acute inflammation, these adhesions sometimes can be peeled away bluntly; if the adhesions have been present chronically, then it may be best to take them down sharply to avoid tissue tearing and bleeding. We prefer to perform careful dissection with the hook cautery for most of this procedure. Some authors have warned against the use of electrocautery, particularly in the triangle of Calot region, out of concern for electrothermal proximity injury to biliary or vascular structures. If the hook cautery is used improperly, then these injuries certainly can happen. Our definition of “proper use” of the hook electrocautery is, when applying energy, always to have the hook’s tip in sight, always to have the hook pulled away from the tissue, never to push the hook into the tissue, and always to take tissue in thin layers in order to avoid hooking into a vital structure. Energy should be applied in short bursts of less than 1 second, particularly inside the triangle of Calot; a tissue strand requiring a longer application for hemostasis likely should not have been handled with the hook but rather clipped. Electrocautery scissors also is an option for this procedure, but we prefer the hook because it also has excellent function as a right-angle dissector. After the omental adhesion is taken down, the surgeon may find that the gallbladder is too tense with fluid to allow a grasper to obtain purchase on the organ. In this case we prefer to puncture the dome of the gallbladder with a laparoscopic needle aspirator, and decompress the organ. The needle entry site then should be closed either with clips or a suture. If the gallbladder still cannot be grasped with a 5-mm instrument, a right lateral 5-mm port can be upsized to a 10-mm port to allow use of

CHAPTER 9-3  ■  Laparoscopic Cholecystectomy  

a 10-mm atraumatic grasper or Babcock forceps. At this point the fundus of the gallbladder is elevated over the liver by the first assistant working through the right lateral port. We prefer to begin the gallbladder dissection by the infundibulum; we have not been impressed with the “top-down” approach, even in the presence of severe acute inflammation. A V-shaped incision is made in the peritoneum (only) overlying the infundibulum, using the hook electrocautery. Each arm of the V extends up on its respective side of the gallbladder (Figure 9-3-2). It is important to follow “proper use” rules of the electrocautery during this and subsequent dissection to avoid injury to underlying structures. Some surgeons prefer to grab the peritoneum overlying the infundibulum and simply rip it downward to expose the underlying vital structures. We find this technique to be esthetically displeasing and potentially rough and dangerous to the underlying ducts and vessels. After the V has been made, the peritoneum is swept downward with gentle blunt dissection (Figure 9-3-3A). The goal of the dissection at this point is to identify the junction of the gallbladder neck (infundibulum) with the cystic duct. We accomplish this with a wide and thorough dissection of the triangle of Calot; we do not employ routine cholangiography. Our indication for cholangiography primarily has been for suspected choledocholithiasis, which typically represents only 10% to 20% of our cases. Occasionally the infundibulum is somewhat redundant, and overrides the junction of the cystic duct with the common bile duct. If this region is not dissected out, then the surgeon may miss a short cystic duct and clip/transect the common bile duct instead. The junction of the gallbladder infundibulum with the cystic duct should be dissected clean circumferentially (Figure 9-3-3B). This means that the space between the posterior gallbladder and the liver bed needs be opened up. We will routinely mobilize the lower one third to one half of the

FIGURE 9-3-2  A, Lateral incision of the peritoneum. B, Medial incision of the peritoneum. Arrows indicate direction of peritoneal incision. HE, hook electrocautery.

FIGURE 9-3-3  A, Peritoneum overlying the junction of the infundibulum with the cystic duct is gently peeled down with a grasper. B, A right-angle dissector (RA) creates a window posterior to the cystic duct (CD). TC, region of the triangle of Calot.

e173

e174   SECTION 3  ■  ABDOMEN – BILIARY gallbladder off the liver bed prior to clipping and cutting any tubular structures in the triangle of Calot. If the gallbladder is completely free of the liver bed in this location and the posterior wall of the gallbladder is plainly visible, then the surgeon can be certain that the common bile duct is not adherent to the gallbladder’s back wall (a common scenario in catastrophic ductal injuries in which a segment of the common bile duct gets resected). The other advantage to this early mobilization of the gallbladder off the liver bed is to identify any ductal or vascular aberrancy. During the dissection in the triangle of Calot, the surgeon should not be fixated on a single view of the dissection. The surgeon should frequently alter the angle of the view, move the gallbladder around, and approach from the infundibulum from both a lateral and medial direction. Lateral stretch on the gallbladder is particularly important, as this tends to splay out the gallbladder, infundibulum, and cystic duct away from the liver bed and common duct. There should be frequent reassessments of the anatomic arrangements as the dissection proceeds, and the surgeon should constantly question any assumptions that she or he has made about the anatomy and rely on a healthy skepticism of the structures as they unfold into view. Most commonly, the cystic artery is positioned medially and slightly posterior to the cystic duct (Figure 9-3-4A). The artery’s identity can be verified by tracing it superiorly into the body of the gallbladder, where it will form an arcade. In addition, if traction is reduced on the gallbladder, the surgeon may be able to see the artery pulsate. Another landmark of cystic artery location is the presence of a lymph node anterior to the artery along its midpoint. Once the junction of the infundibulum and the cystic duct has been delineated, the surgeon may clip and transect the cystic duct and cystic artery with confidence (Figure 9-3-4B). Typically, two clips are placed on the patient side of both the duct and artery, and one clip is placed on the specimen side. A common error that a surgeon-in-training will make is to leave inadequate space between the clips to cut the structure with a scissors. The clip on the specimen side can be buried with impunity into the specimen to allow space for the scissors. The surgeon should not cut a vessel or duct flush with the patient-side clip, because this clip then will be at risk to slide off. Following the preceding technique of complete dissection of the triangle of Calot prior to clipping and cutting, the authors have yet to incur any common bile duct injury in their combined practice. Once the dissection has been accomplished and the cystic duct and artery have been controlled, the superior portion of the gallbladder is removed from the liver bed using hook electrocautery (Figure 9-3-5). The surgeon should dissect close to the gallbladder wall by keeping it on stretch with respect to the liver. We suspect that some of the high ductal injuries (at the confluence or above) that have been inflicted, but not necessarily recognized, during laparoscopic cholecystectomy have been secondary to electrical energy injudiciously applied to the liver bed during the latter part of the procedure. After removal from the liver bed, the gallbladder then is placed into a polyethylene specimen bag and removed from the abdomen through either the subxiphoid or umbilical port. We like to inspect the gallbladder specimen on the back table to verify the correctness of the anatomy and to evaluate for any unsuspected masses. The liver bed is inspected for hemostasis, and the right upper quadrant is washed with saline. If the dissection has been particularly difficult or bloody, then the surgeon may

FIGURE 9-3-4  A, Completed dissection of the triangle of Calot region, demonstrating the cystic duct (CD) and cystic artery (CA). B, The cystic duct has been doubly clipped and now is cut with the hook scissors.

CHAPTER 9-3  ■  Laparoscopic Cholecystectomy  

FIGURE 9-3-5  A, The gallbladder (G) is retracted laterally, allowing division of the medial attachments to the liver with hook electrocautery (HE). B, The gallbladder is retracted medially, allowing similar division of the lateral attachments with hook electrocautery.

leave a closed suction drain in the gallbladder fossa and exiting out one of the 5-mm trocar sites. The pneumoperitoneum is evacuated, the fascia of the 10-mm ports is closed, and the procedure is terminated.

POSTOPERATIVE CARE The typical cholecystectomy can be done as an outpatient procedure or a 23-hour stay; older or more infirm patients may require a longer period of observation. If a patient in good health is not ready for discharge on postoperative day 1, then this should raise the suspicion that the patient may have a complication. A clear liquid diet is given the evening of surgery, and a regular diet may be given the next day. Routine lifting restrictions are given upon discharge, and the patient may be seen in follow-up at 1 week and 1 month postoperatively. The patient also is instructed to report any occurrence of fever, abdominal distention, obstipation, diffuse abdominal pain, jaundice, or incisional drainage.

PROCEDURE-SPECIFIC COMPLICATIONS Intraoperative complications include gallbladder perforation with stone spillage, hemorrhage, enterotomy, and common bile duct injury. A perforation of the gallbladder is a relatively benign complication; the most expedient solution is simply to aspirate the gallbladder empty with the pool sucker, and remove any spilled stones. At the end of the procedure, the right upper quadrant should be irrigated with a large volume of saline until there is a clear return. Hemorrhage from the liver bed usually can be controlled with electrocautery. More severe hemorrhage can occur from injuries to porta hepatis structures such as the portal vein or, more commonly, the cystic artery. These injuries can be avoided by staying close to the gallbladder infundibulum rather than close to the porta hepatis. It should be acknowledged that an infected and inflamed gallbladder with vascular adhesions in the porta hepatis region will increase the risk of hemorrhage. It is not clear that open conversion in the presence of a severe inflammation will decrease the risk of a hemorrhagic complication. Common bile duct injury can occur secondary to multiple mechanisms, and a full discussion of this complication is beyond the scope of this chapter. The best way to avoid a ductal injury is to perform a careful dissection supplemented by cholangiography as needed (see earlier discussion). Despite the best application of preventive measures, a common bile duct injury still may occur and does not necessarily indicate a deviation from the standard of care. A partial transection of the common duct that is recognized intraoperatively may be managed laparoscopically by placement of a T-tube, depending on the experience of the surgeon. A complete transection (or, more commonly, a segmental resection) of the common duct may be managed by a Rouxen-Y choledochojejunostomy, performed by a surgeon who is experienced with this procedure. Intraoperative recognition of an enterotomy (either in the

e175

e176   SECTION 3  ■  ABDOMEN – BILIARY duodenum or secondary to trocar injury to other portions of the bowel) should be managed by primary closure. Postoperative complications of laparoscopic cholecystectomy include cystic stump leak, retained stone, unrecognized common bile duct injury, and unrecognized intestinal perforation. A cystic stump leak typically presents as a fluid collection (biloma) in the right upper quadrant. The patient may have isolated hyperbilirubinemia (conjugated). An investigation including a CT scan (to diagnose the biloma) and an ERCP (to determine the biloma’s source) should be performed. A biloma resulting from a liver bed leak can be percutaneously drained and is self-limiting. A cystic stump leak will require a percutaneous drain and an endoscopically placed common bile duct stent. If the bile leak is from an unrecognized bile duct injury, then this should be managed as described earlier. Unrecognized intestinal perforation most commonly occurs secondary to trocar placement and less commonly with duodenal injuries; this complication may go undetected for several days and has a high morbidity risk if it remains unrecognized beyond the first 3 to 4 days postoperatively. In the present era of outpatient surgery, it is extremely important for the patient to be able to recognize symptoms that may be associated with an intestinal perforation; thus, patient education is critical.

RESULTS AND OUTCOME When performed for symptomatic cholelithiasis, patient satisfaction after a laparoscopic cholecystectomy is remarkably high (95% or greater). If the gallbladder is removed for other diagnoses (e.g., biliary dyskinesia), then overall patient satisfaction drops into the 70% to 80% range. With regard to common bile duct injury, the true incidence in the general community is difficult to know; recent large metaanalyses of published retrospective data place this incidence at less than 0.5%. This injury rate represents an improvement over the rates reported in the early 1990s, but the nature of this data makes it subject to publication bias. More important, however, laparoscopic cholecystectomy is widely regarded as the procedure that heralded a major change in the approach to abdominal surgery. This change continues to the present day, as more and more procedures are being performed with minimally invasive technique. Furthermore, the morbidity associated with a major incision is receiving (appropriately so) an increased amount of attention. Laparoscopic cholecystectomy was the core component of this systemic change.

Suggested Reading Adamsen S, Hansen OH, Funch-Jensen P, et al: Bile duct injury during laparoscopic cholecystectomy: A prospective nationwide series. J Am Coll Surg 1997;184:571–578. Carlson MA, Frantzides CT, Ludwig KA, et al: Routine of selective use of intraoperative cholangiography in laparoscopic cholecystectomy. J Laparoendosc Surg 1993;3:31–37. Davidoff AM, Pappas TN, Murray EA, et al: Mechanisms of major biliary injury during laparoscopic cholecystectomy. Ann Surg 1992;215:196–202. Frantzides CT, Sykes A: A re-evaluation of antibiotic prophylaxis in laparoscopic cholecystectomy. J Laparoendosc Surg 1994;4:375–378. Ludwig K, Bernhardt J, Steffen H, Lorenz D: Contribution of intraoperative cholangiography to incidence and outcome of common bile duct injuries during laparoscopic cholecystectomy. Surg Endosc 2002;16:1098–1104. Shea JA, Healey MJ, Berlin JA, et al: Mortality and complications associated with laparoscopic cholecystectomy: A metaanalysis. Ann Surg 1996;224:609–620. Strasberg SM, Hertl M, Soper NJ: An analysis of the problem of biliary injury during laparoscopic cholecystectomy. J Am Coll Surg 1995;180:101–125.

LAPAROSCOPIC APPROACH TO COMMON DUCT PATHOLOGY Joseph B. Petelin

9-4 

From Petelin J: Laparoscopic approach to common duct pathology. Am J Surg 1993;165:487–491

The author reviews his experience with the laparoscopic management of common duct pathology and compares it with the experience of others as reported in the literature. Routine intraoperative cholangiography is advocated. A variety of methods of managing common duct stones laparoscopically is presented. These include balloon-catheter manipulation, fluoroscopically guided basket extraction, and choledochoscopic evaluation and removal of stones. The accumulated experience indicates that more than 90% of common duct stones can be removed laparoscopically via the cystic duct. This approach significantly reduces the need for either preoperative or postoperative endoscopic retrograde cholangiopancreatography. Although laparoscopic choledochotomy has been employed in a number of cases and can be performed with a high degree of safety and efficacy, it is needed only infrequently. This form of management results in decreased dependence upon T-tubes, thereby reducing postoperative morbidity and the length of hospitalization. A rational protocol for the management of common duct pathology is presented. Choledocholithiasis is present in 9% to 16% of patients with cholelithiasis. Therefore, the evaluation and treatment of common duct pathology is an essential component in the surgical management of biliary tract disease. Preoperative methods of determining the status of the common bile duct have been employed since the introduction of surgical treatment of biliary tract problems. In the history of open surgery, the first cholecystectomy was performed in 1882 and the first common duct exploration in 1890. Intraoperative cholangiography, first popularized by Mirizzi1 in 1932, is usually used to confirm the presence of stones in the common ductal system at the time of open surgery. However, the application of this intraoperative intervention has been controversial for years, extending into the laparoscopic era. Although some authors recommend routine cholangiography, others advocate a selective approach. Laparoscopic cholecystectomy (LC) was first performed in 1987 in France and in 1988 in the United States. Cholangiography is believed to have been first performed during LC in 1989. Those practitioners who recommend routine cholangiography during LC cite three reasons: development of proficiency, delineation of the anatomy, and the evaluation of the ductal system for stones or other pathology. Those who favor a selective approach cite the relative infrequency of positive cholangiograms, the false-positive and false-negative rates, the cost, and the potential for misadventure as the rationale for their stance. Nevertheless, it seems almost axiomatic that intraoperative cholangiographies be performed prior to the initiation of laparoscopic common duct exploration (LCDE). Although LCDE was initially thought to be impossible for the intraoperative treatment of these identified abnormalities, it has been performed successfully and routinely since April 1990. The role of LCDE in LC, however, has only recently assumed a serious place in the algorithm for the treatment of common duct pathology by most laparoscopic surgeons2. Controversies also exist over the relationship of endoscopic retrograde cholangiopancreatography (ERCP), either in the preoperative or postoperative setting, with LC and LCDE. The purpose of this paper is to review my experience with LC and LCDE and to offer some suggestions for a rational protocol for the evaluation and treatment of common duct pathology. e177

e178   SECTION 3  ■  ABDOMEN – BILIARY

TECHNIQUE A number of techniques useful in a laparoscopic approach to common duct exploration have been developed. All of these presuppose that intraoperative cholangiography was performed prior to the initiation of the ductal exploration. These methods of LCDE and/or manipulation will be discussed in their relative order of increasing invasiveness, which is the order in which I use them. That is to say, less invasive methods of common duct manipulation and exploration are usually attempted before more invasive ones. Cholangiography is essential here. Two methods of intraoperative cholangiography have been established. In the portal method, developed by Reddick and Olsen (personal communication), a catheter is inserted into a specialized cholangioclamp, and the entire unit is placed through one of the right upper quadrant portals. The catheter is advanced into the cystic duct, where it is fixed into position via the clamp. The percutaneous method, developed and preferred by the author, employs a 14-gauge polyethylene intravenous catheter, as a fifth port, to provide access to the peritoneal cavity for the cholangiocatheter3. The cholangiocatheter is guided into the cystic duct with forceps inserted through the medial epigastric port, and it is temporarily secured to the cystic duct with a Hemoclip (US Surgical, Norwalk, CT). Both methods have been shown to be effective, safe, and efficient in the performance of laparoscopic cholangiography and in the detection of common duct pathology. The first and easiest maneuver I use during LCDE involves placement of a 4F Fogarty catheter through the 14-gauge sleeve that was used for cholangiography. This catheter is advanced through the cystic duct into the common duct and, if possible, into the duodenum. Occasionally, the catheter will push stones into the duodenum, resulting in a cleared duct. If spasm is suspected to be the cause of the abnormal cholangiogram and if pharmacologic treatment (e.g., glucagon, 1 mg intravenous) to relieve it is unsuccessful, then the balloon may be inflated at the sphincter of Oddi to gently dilate it. The location of the sphincter may be confirmed by gentle retraction of the catheter from the duodenum with the balloon inflated. The resistance encountered and the concomitant movement of the duodenum indicate the location of the papilla of Vater. The balloon is then temporarily deflated, the catheter is withdrawn approximately 1 cm (measured easily by the catheter marks at the cystic duct entrance), and the balloon is gently re-inflated. The catheter may also be withdrawn through the entire common bile duct with the balloon inflated in an attempt to remove any debris present in the common duct. Although this latter procedure usually requires a modest amount of luck, occasionally stones and/or debris are removed from the common duct through the cystic duct with this technique. More commonly, however, these elements are not directed into the cystic duct. When these techniques are unsuccessful, fluoroscopically guided basket extraction, initially promoted by Hunter4, may be employed. The cystic duct is cannulated with a Dormia-type basket, which is advanced through the abdominal wall through the 14-gauge sleeve used for cholangiography. A C-arm fluoroscope is then positioned over the patient, so that the basket may be advanced into the common duct safely and accurately. Stones may then be identified and captured in the contrast-filled common duct. They are then extracted through the cystic duct. If these stones are too large to allow removal through the cystic duct, they may be gently crushed with external pressure applied by an atraumatic forceps through the cystic duct wall. Alternatively, if the stones appear larger than the diameter of the cystic duct, as viewed on the cholangiogram, the cystic duct may be dilated with mechanical or pneumatic devices prior to the insertion of the basket. In patients in whom extended manipulation of the basket is required, the contrast material will leak out of the duct. If this occurs, the duct is easily re-filled by removing the basket and reinserting the cholangiocatheter. The basket is then reinserted for further attempts at stone capture and removal. When the common duct cannot be cleared by either of these methods, choledochoscopic intervention usually becomes necessary. A flexible small-diameter (less than 11F) scope may be inserted through the cystic duct, or choledochotomy may be required. Both of the techniques of insertion are performed with laparoscopic guidance. If the cystic duct approach is used, it will usually require dilatation to at least a 12F size if it is not already sufficiently patulous. The scope is inserted through the midclavicular port and guided into the cystic duct and then the common duct with forceps inserted through the medial epigastric port. The scope is manipulated through the common duct until the stones are identified. A wire basket is then advanced through the working channel of the scope, the stone is trapped, and then the entire ensemble is withdrawn through the cystic duct. Stones thus

CHAPTER 9-4  ■  Laparoscopic Approach to Common Duct Pathology  

retrieved are temporarily stored on the omentum, where they are immediately grasped and removed with forceps inserted through the medial epigastric port. This process may be repeated as necessary until the duct is cleared. If the cystic duct cannot be sufficiently dilated to accept the choledochoscope, then a choledochotomy, approximately 1 cm in length, is made in a standard fashion, and the scope is easily advanced through it into either the common hepatic duct or common bile duct. Stones are then removed with the basket as described above. A T-tube is placed into the abdomen through the medial epigastric port and gently guided into the common duct. The choledochotomy is closed over the T-tube with interrupted or continuous suture of 4-0 Vicryl (Ethicon, Somerville, NJ). The T-tube exits the abdomen through one of the lateral portals. No T-tube is placed when the common duct is explored through the cystic duct. The use of the choledochoscope is facilitated by the application of a second camera system to the head of the scope. This picture may be projected on a second monitor or may be incorporated into the same video image as the laparoscopic image on the same monitor using a video mixer. This option allows a picture-in-picture effect, wherein the choledochoscopic image is pasted onto the laparoscopic image so that both images may be viewed simultaneously. Most mixers are designed so that the relative positions of the two images may be exchanged to allow enhancement of the image of prime interest. This arrangement also provides for capture of the cholechoscopic video on the same tape as the laparoscopic video if the procedure is being recorded. This video mixer may also be used in conjunction with the fluoroscopically guided basket extraction technique described above.

RESULTS From September 21, 1989, through July 10, 1992, 1,025 patients with symptomatic gallbladder disease presented for surgical treatment; 1,000 patients underwent LC performed by the author, and 25 patients underwent open cholecystectomy. Cholangiograms were undertaken in 874 patients in the LC group (87%). Abnormal films were obtained in 95 (11%) of these 874 patients (10% of the entire LC group). Twenty-nine of the 1,000 patients underwent preoperative ERCP (3%). Four of the 1,025 patients who were believed to have common duct stones preoperatively underwent open cholecystectomy and open common duct exploration, without attempted LC or LCDE, during the early experience with LC (0.4% of the total group of 1,025 patients). One patient required conversion from LCDE to open common duct exploration for removal of a 1-cm stone after three other stones had been successfully removed laparoscopically with the choledochoscope. One patient’s procedure was converted to open common duct exploration (CDE) without attempted LCDE because of an impacted distal stone. The procedure in one patient was converted without attempted LCDE because of intrahepatic stones. These three patients represent the only conversions to open surgery from LC for the reason of common duct stones (0.3% of the total group of 1,000 LC patients). Only one of these three had an attempted LCDE. Eighteen patients underwent postoperative ERCP (2% of the total number of patients). Five of the patients in this postoperative ERCP group were found to have common duct stones or debris (28%). Eight of the patients in the postoperative ERCP group had undergone attempted LCDE, and in this group, two patients were known to have retained common duct stones at the time of LC. One patient who had a positive postoperative ERCP (i.e., with a retained stone) was thought to have a normal intraoperative cholangiogram and did not undergo LCDE but was found approximately 2½ months later to have a common duct stone, which was retrieved via ERCP and sphincterotomy. No patient who underwent LCDE was found to have an unknown common duct stone. The fourth patient with a positive postoperative ERCP had also undergone intraoperative cholangiography that was thought to be normal. These two patients represent the only false-negative cholangiograms in the series (0.2%). One other patient who did not undergo successful intraoperative cholangiograms was later found to have ductal stones that were removed via ERCP. In total, and as stated above, 5 of the 18 postoperative ERCP interventions revealed retained common duct stones (28%), and all of these patients were successfully treated with endoscopic sphincterotomy and stone extraction. Seventy-seven patients underwent LCDE (7.7%). Successful clearance of the ductal system was accomplished laparoscopically in 74 of 77 patients who underwent LCDE (96.1%). Choledochoscopic clearance of the common duct was accomplished in 48 of these patients (62.3%). Fogarty balloon

e179

e180   SECTION 3  ■  ABDOMEN – BILIARY extraction of debris and stones and/or dilatation of the distal duct was successful in 27 of these patients (35%). Two patients (2%) underwent successful clearance of the common ductal system by the use of fluoroscopically guided basket extraction of stones and/or debris. Some patients underwent more than one of the above procedures; the most useful method responsible for clearance of the duct in each case is listed. In three of the patients who underwent LCDE, successful clearance of the ductal system was not achieved laparoscopically. One patient required conversion to open CDE for removal of a 1-cm stone, after three other stones had been successfully removed with the choledochoscope. Two patients with known common duct stones had successful postoperative ERCP, instead of conversion to open CDE. The addition of LCDE to an LC procedure resulted in significantly longer operative time, but in no instance was this increased time directly related to any morbidity (Table 9-4-1). Length of stay for patients undergoing LCDE was also longer than that of patients undergoing LC only but was still considerably shorter than that expected for an open CDE (Table 9-4-2). The only 8 patients with failures to achieve clearance of the ductal system laparoscopically in this series of 1,000 patients (0.8%) included these 3 failed LCDE patients, the 2 patients whose procedures were converted to open CDE without attempted LCDE during the early experience with LC, the 2 patients with the false-negative cholangiograms, and the patient who did not undergo intraoperative cholangiography but who was later found to have a common duct stone. All were ultimately cleared with either open CDE or ERCP. Major complications occurred in eight patients in the LCDE group. Two patients required postoperative ERCP for known retained stones. The procedure in one patient required conversion to open CDE after attempted LCDE. One 89-year-old patient who underwent successful removal of two unsuspected common duct stones died of a massive anterior myocardial infarction on the first postoperative day. One 96-year-old patient had a stroke during the first postoperative week; she recovered without a residual neurologic deficit. One patient with transient postoperative right upper quadrant

TABLE 9-4-1  Mean Operative Times for Various Laparoscopic Biliary Tract Procedures Technique Laparoscopic common duct exploration Choledochoscopic technique Fogarty/basket/flush technique Laparoscopic cholecystectomy

Mean Operative Time (min) 146 168 113 83

TABLE 9-4-2  Mean Length of Stays for Various Biliary Tract Procedures Technique Laparoscopic common duct exploration Choledochoscopic technique Fogarty/basket/flush technique Laparoscopic cholecystectomy

Mean Length of Stay (h) 46 51 38 25

CHAPTER 9-4  ■  Laparoscopic Approach to Common Duct Pathology  

e181

pain was evaluated with ERCP and was thought to have a distal ductal leak at the ampulla, but her symptoms resolved without other intervention and without sequelae. Two patients required extended hospitalization of a few days for treatment of urinary retention. These problems represent an overall morbidity of 10.4% for the LCDE group.

COMMENTS LC has become the new gold standard for the treatment of symptomatic gallbladder disease. The documented advantages of decreased pain, limited hospitalization, decreased cost, improved cosmesis, and minimal lifestyle disruption are all responsible for the widespread acceptance of this procedure. As with open surgery, however, questions have arisen and controversies have developed regarding the extent to which the laparoscopic biliary tract surgeon should go to evaluate and/or treat the entire ductal system. The routine or selective use of intraoperative cholangiograms continues to generate vigorous debate5–9, as does the role of preoperative and postoperative ERCP10–17. Although Arregui et al16, Cotton18, Carr-Locke19, and others have reported great success with either preoperative or postoperative ERCP for the treatment of common duct stones, many of these accomplished endoscopists agree that a single-stage treatment of biliary lithiasis, i.e., LC and LCDE, seems to be the ultimate goal of an interventional approach to this disease process. The development of techniques that can be reliably employed to accomplish LCDE has added a new dimension to the surgeon’s approach to more complex biliary tract disease. Numerous reports of initial successful attempts to perform LCDE20–27 have given way to a more critical inspection of larger series such as the current ones (Table 9-4-3). LCDE is no longer just a topic for anecdotal reporting. It is, in fact, an integral part of the treatment algorithm for biliary tract disease for many surgeons in the United States and abroad2.

CONCLUSIONS The laparoscopic evaluation and treatment of common duct pathology present an exciting challenge to laparoscopic biliary tract surgeons. This report demonstrates the applicability of a variety of techniques designed to meet this challenge. Laparoscopic cholangiography, choledochoscopy, choledochotomy, extraction of common duct stones, and placement of T-tubes are all feasible. Their role in the treatment of common duct pathology will depend on the suitability of the patient to undergo a more prolonged procedure, the skill and training of the surgeon, the availability of more sophisticated equipment, and the availability of local expertise in ERCP. A suggested protocol for the management of patients with suspected common duct pathology is presented (Figure 9-4-1).

TABLE 9-4-3  Experience with Laparoscopic Common Duct Exploration (LCDE)

Reference 26 4 2 27 6 21 Current series

No. of Cholecystectomies >200 252 342 500 516 555 1,000

Abnormal LCDE Cholangiograms (%) Clearance 8 (4) 20 (8) 26 (8) 29 (6) 35 (7) 57 (10) 95 (9)

7 17 19 8 21 40 74

ERCP = endoscopic retrograde cholangiopancreatography; NA = not available.

Clearance Clearance as % as % of of Abnormal Attempted Converted to Cholangiograms LCDEs Laparotomy 88 85 73 28 60 70 78

88 85 86 57 NA 98 96

0 3 5 3 8 1 5

Postoperative ERCP or Radiologic Clearance 1 0 5 16 5 NA 18

e182   SECTION 3  ■  ABDOMEN – BILIARY

FIGURE 9-4-1  Protocol for the management of choledocholithiasis. Pre-op, preoperative; ERC, endoscopic retrograde cholangiography; CDE, common duct exploration; post-op, postoperative.

References 1. Mirizzi PL. Operative cholangiography. Surg Gynecol Obstet 1937; 65: 702–10. 2. Petelin J. Laparoscopic approach to common duct pathology. Surg Laparosc Endosc 1991; 1: 33–41. 3. Petelin J. The argument for contact laser laparoscopic cholecystectomy. Clin Laser Monthly May 1990; 71–4. 4. Hunter JG. Laparoscopic transcystic common bile duct exploration. Am J Surg 1992; 163: 53–8. 5. Flowers JL, Zucker KA, Scovill WA, Imbembo AL, Bailey RW. Laparoscopic cholangiography: results and indications. Ann Surg 1992; 215: 209–16. 6. Sackier JM, Berci G, Phillips E, Carroll B, Shapiro S, Paz-Partlow M. The role of cholangiography in laparoscopic cholecystectomy. Arch Surg 1991; 126: 1021–6. 7. Bruhn EW, Miler FJ, Hunter JG. Routine fluoroscopic cholangiography during laparoscopic cholecystectomy: an argument. Surg Endosc 1991; 5: 111–5. 8. Berci G, Sackier JM, Paz-Partlow M. Routine or selected intraoperative cholangiography during laparoscopic cholecystectomy? Am J Surg 1991; 161: 355–60. 9. Scott-Coombes D, Thompson JN. Bile duct stones and laparoscopic cholecystectomy. BMJ 1991; 303: 1281–2. 10. Reddick EJ, Olsen D, Alexander W, et al. Laparoscopic laser cholecystectomy and choledocholithiasis. Surg Endosc 1990; 4: 133–4. 11. Aliperti G, Edmundowicz SA, Soper NJ, Ashley SW. Combined endoscopic sphincterotomy and laparoscopic cholecystectomy in patients with choledocholithiasis and cholecystolithiasis. Ann Intern Med 1991; 115: 783–5. 12. Cooperman AM, Siegel J, Neff R, Reddy S, Hamerman H. Gallstone pancreatitis: combined endoscopic and laparoscopic approaches. J Laparoendosc Surg 1991; 1: 115–7. 13. Cronin KJ, Kerin MJ, Williams NN, et al. Endoscopic management of common duct stones with laparoscopic cholecystectomy. Ir J Med Sci 1991; 160: 265–7. 14. Fletcher DR. Percutaneous (laparoscopic) cholecystectomy: its potential impact on the practice of ERCP. J Gastroenterol Hepatol 1992; 7: 105–6. 15. Fletcher DR. Percutaneous (laparoscopic) cholecystectomy and exploration of the common bile duct: the common bile duct stone reclaimed for the surgeon. Aust N Z J Surg 1991; 61: 814–5. 16. Arregui ME, Davis CJ, Arkush AM, Nagan RF. Laparoscopic cholecystectomy combined with endoscopic sphincterotomy and stone extraction or laparoscopic choledocholscopy and electrohydraulic lithotripsy for management of cholelithiasis with choledocholithiasis. Surg Endosc 1992; 6: 10–5. 17. Dion YM, Morin J, Dionne G, Dejoie C. Laparoscopic cholecystectomy and choledocholithiasis. Can J Surg 1992; 35: 67–74. 18. Cotton PB. Endoscopic management of bile duct stones (apples and oranges). Gut 1984; 25: 587–97. 19. Carr-Locke DL. Acute gallstone pancreatitis and endoscopic therapy. Endoscopy 1990; 22: 180–3. 20. Shapiro SJ, Grundest W. Laparoscopic exploration of the common bile duct: experience in 16 selected patients. J Laparoendosc Surg 1991; 1: 333–41. 21. Carroll BJ, Phillips EH, Daykhovsky L, et al. Laparoscopic choledochoscopy: an effective approach to the common duct. J Laparoendosc Surg 1992; 2: 15–21.

CHAPTER 9-4  ■  Laparoscopic Approach to Common Duct Pathology   22. McAlhany JC, Sim R. Laparoscopic exploration of the common duct with stone extraction. J S C Med Assoc July 1991; 375–7. 23. Smith P, Clayman RV, Soper NJ. Laparoscopic cholecystectomy and choledochoscopy for the treatment of cholelithiasis and choledocholithiasis. Surgery 1992; 111: 2: 230–3. 24. Stoker ME, Leveillee RJ, McCann JC, Maini BS. Laparoscopic common bile duct exploration. J Laparoendosc Surg 1991; 1; 5:287–93. 25. Sackier JM, Berci G, Paz-Partlow M. Laparoscopic transcystic choledochotomy as an adjunct to laparoscopic cholecystectomy. Am Surg 1991; 57: 323–6. 26. Jacobs M, Verdeja JC, Goldstein HS. Laparoscopic choledocholithotomy. J Laparoendosc Surg 1991; 1: 79–82. 27. Spaw AT, Reddick EJ, Olsen DO. Laparoscopic laser cholecystectomy: analysis of 500 procedures. Surg Laparosc Endosc 1991; l; 1: 2–7.

Further Reading Shamiyeh A, Wayand W: Laparoscopic cholecystectomy: early and late complications and their treatment. Langenbecks Arch Surg 389: 164, 2004.

e183

9-5 

SELF ASSESSMENT Daniel J. Deziel From Velasco J: Rush University Medical Center Review of Surgery, 5th edition (Saunders 2011)

1. Laparoscopic cholecystectomy is most strongly contraindicated in which of the following situations? A. Pregnancy B. Previous upper abdominal surgery C. Known common bile duct stones D. Chronic obstructive pulmonary disease E. Gallbladder cancer Ref.: 1, 2 COMMENTS: When laparoscopic cholecystectomy was first introduced worldwide during the late 1980s, there were a number of circumstances in which it was more or less strongly contraindicated. Today, most contraindications are relative, and in fact the laparoscopic approach is preferred when possible in certain situations that were initially considered contraindications (e.g., acute cholecystitis, choledocholithiasis, and obesity). Basically, the surgeon must be adequately trained and the patient reasonably fit for an operation and give informed consent that includes the possibility of laparotomy. It must be recognized that there are patients for whom the potential physiologic consequences of CO2 pneumoperitoneum are more important, but the presence of underlying disease itself does not prohibit a laparoscopic approach. In fact, laparoscopic cholecystectomy may be more beneficial to the postoperative course of a compromised patient. Pregnancy is not a contraindication with appropriate precautions, although the physiologic effects on the fetus are not completely known. Perhaps the strongest contraindication currently involves patients with suspected or known gallbladder cancer because of the risk for dissemination.

ANSWER: E 2. Most major bile duct injuries during laparoscopic cholecystectomy occur in patients under which of the following circumstances? A. Acute cholecystitis B. Gallstone pancreatitis C. Choledocholithiasis D. Elective cholecystectomy E. Conversion of a laparoscopic procedure to an open procedure Ref.: 3 COMMENTS: There are several risk factors for bile duct injury during laparoscopic cholecystectomy. Pathologic risk factors include severe acute or chronic inflammation. Several studies have found a statistical correlation between the rate of duct injury and the presence of acute cholecystitis. Bleeding has long been implicated as a factor predisposing to duct injury during open or laparoscopic cholecystectomy. Injuries are sometimes attributed to the “anomalous” anatomy of the bile ducts. More often than not, however, such “anomalies” are simply common anatomic variations that the surgeon must recognize to prevent injury. The surgeon’s experience, or the “learning curve,” is clearly a risk factor, e184

CHAPTER 9-5  ■  Self Assessment  

because higher rates of duct injury have been well documented in less experienced surgeons. It is interesting to note that there is no convincing evidence that duct injury is more frequent during cases involving laparoscopic management of common bile duct stones, possibly because these procedures are performed by more experienced surgeons. Unfortunately, most major bile duct injuries during laparoscopic cholecystectomy have occurred in elective and otherwise uncomplicated cases. Despite the presence or absence of risk factors, the primary problem resulting in duct injury is misidentification of the anatomy. The most frequent mechanism of injury is mistaking a major bile duct for the cystic duct and clipping and cutting it. This pitfall is best avoided by correct operative strategy, which means appropriate retraction and adequate dissection to obtain the “critical view of safety.” The critical view is achieved by dissecting the base of the gallbladder off the liver for an adequate distance to visualize the cystic plate and to verify that the only structures entering the gallbladder are the true cystic duct and the cystic artery. Intraoperative bile duct imaging with cholangiography or laparoscopic ultrasonography can also aid in discerning the anatomy. If the cystic duct cannot be conclusively identified, the surgeon must resort to alternative approaches such as laparoscopic subtotal cholecystectomy, conversion to an open operation, or termination of the procedure.

ANSWER: D 3. A patient has undergone subtotal cholecystectomy with a portion of the gallbladder infundibulum left in situ. On the second postoperative day, bile is coming from a subhepatic drain placed at the time of surgery. Which of the following is the most appropriate step? A. Endoscopic retrograde cholangiography B. PTC C. Removal of the drain D. Leaving the drain in place and monitoring E. Returning to the operating room for completion of the cholecystectomy Ref.: 4 COMMENTS: Safe management of a patient with a difficult laparoscopic cholecystectomy requires technical skill, considerable judgment, and familiarity with a spectrum of operative options. Such options include open cholecystectomy, “fundus first” cholecystectomy, laparoscopic or open cholecystostomy, and laparoscopic or open subtotal cholecystectomy. Alternatives to total cholecystectomy can help avoid major bile duct or vascular injury under difficult circumstances. Cholecystostomy tube placement can be lifesaving; potential disadvantages include tube complications, the possible need for a reoperation later, and possible inability to place a tube if the gallbladder is necrotic or gangrenous. Subtotal cholecystectomy can help avoid injury and bleeding and reduce the need for cholecystostomy and reoperation. The ability to safely perform subtotal excision of the gallbladder laparoscopically can decrease the rate of conversion to an open operation and potential morbidity in critically ill patients. There are several variations of subtotal cholecystectomy that can be appropriate: leaving portions of the gallbladder infundibulum, posterior wall, or both, depending on the situation. Bile leakage is not uncommon following subtotal cholecystectomy, but most bile leaks are self-limited. Those that persist have often been associated with retained common bile duct stones and have been successfully treated endoscopically. Problems with retained stones in the gallbladder remnant have not been common in reports of subtotal cholecystectomy.

ANSWER: D

References 1. Mullholland MW, Lillemoe KD, Doherty GM, et al, editors: Greenfield’s surgery: scientific principles and practice, Hepatobiliary and Portal Venous System Section, ed 4, Philadelphia, 2006, Lippincott Williams & Wilkins. 2. Fong Y, Brennan MF, Turnbulla A, et al: Gallbladder cancer discovered during laparoscopic surgery, Arch Surg 128:1050– 1054, 1993. 3. Strasberg SM, Hertle M, Soper NJ: An analysis of the problem of biliary injury during laparoscopic cholecystectomy, J Am Coll Surg 180:101–125, 1995. 4. Palanivelu C, Rajan PS, Jani K, et al: Laparoscopic cholecystectomy in cirrhotic patients: the role of subtotal cholecystectomy and its variants, J Am Coll Surg 203:145–151, 2006.

e185

10 

Cholecystectomy with or without Cholangiography – Open

GOALS/OBJECTIVES • • • •

INDICATIONS VS LAPAROSCOPIC RELEVANT ANATOMY TECHNIQUE (TOP DOWN APPROACH) COMPLICATIONS

TECHNIQUE OF CHOLECYSTECTOMY Shishir K. Maithel

10-1 

From Jarnagin WR: Blumgart’s Surgery of the Liver, Biliary Tract, and Pancreas, 5th edition (Saunders, 2012)

OVERVIEW The indications for performing open cholecystectomy have evolved since the advent and widespread use of laparoscopic cholecystectomy. Surgeons are now trained in laparoscopy at the initiation and for the duration of their training. In fact, as the pendulum has swung so far toward performing laparoscopic cholecystectomy, it may be argued that surgeons completing their training today are not appropriately versed in the techniques of the open operation, which ironically are most useful and best applied to the difficult gallbladder that is not amenable to laparoscopic excision. Along with discussing important anatomic and clinical considerations, this chapter focuses on three main techniques for performing open cholecystectomy: 1) retrograde, 2) anterograde (fundus-down), and 3) partial (subtotal) cholecyst­ ectomy for difficult situations.

LAPAROSCOPIC VERSUS MINILAPAROTOMY CHOLECYSTECTOMY Any discussion of open cholecystectomy would be incomplete without briefly reviewing the major comparative studies that have addressed the different approaches mentioned above for removing the gallbladder. A small incision, or minilaparotomy, has been defined by most authors as an incision less than 8 cm in length. Many prospective randomized trials have compared the two techniques. Two such early studies were published in The Lancet in 1992 and 1994, and both reported advantages of the laparoscopic approach over a minilaparotomy, including shorter postoperative hospital stay and a decreased postoperative convalescence (Barkun et al, 1992; McMahon et al, 1994). However, a different conclusion was reported by Majeed and colleagues in 1996; they found no difference in hospital stay or time to return to work and full activity among patients who underwent cholecystectomy by the two techniques. Subsequent comparative analysis of the costs of laparoscopic versus small-incision cholecystectomy, based on the same population included in the Majeed study, suggested that smallincision cholecystectomy was less expensive than laparoscopic cholecystectomy (Calvert et al, 2000). This pattern of conflicting results has persisted in recent studies as well, as some trials report a benefit to the laparoscopic approach, but others report no difference, depending on the outcomes measured (Ros et al, 2001; Keus et al, 2008). Nevertheless, a market demand of sorts has popularized laparoscopic cholecystectomy as the technique of choice today.

INDICATIONS FOR OPEN CHOLECYSTECTOMY The introduction of laparoscopic cholecystectomy has significantly influenced the treatment of patients with gallstones, as evidenced by an increase of 20% to 30% in the total number of cholecystectomies performed (Schwesinger & Diehl, 1996). This increase was predominantly in patients with uncomplicated cholelithiasis and in those undergoing elective surgery (Shea et al, 1998). Although it is likely e187

e188   SECTION 3  ■  ABDOMEN – BILIARY that considerations such as reduced postoperative discomfort and smaller scars have made the procedure more acceptable to patients with minor symptoms, it is worth raising caution that the indications for cholecystectomy should not be extended simply because a minimally invasive approach is available. Cholecystectomy usually is performed for symptomatic cholelithiasis and for its related complications, such as obstructive jaundice or biliary pancreatitis. Cholecystectomy for acalculous cholecystitis, gallbladder adenoma, or suspected gallbladder carcinoma is less frequent, with the open technique more appropriate for suspected carcinoma. Asymptomatic cholelithiasis has been suggested as an indication for cholecystectomy in diabetic or immunocompromised patients, but this indication remains controversial (Schwesinger & Diehl, 1996). Postcholecystectomy pain, which has been observed in as many as 30% of cases, may be the consequence of an operation performed for symptoms unrelated to the presence of gallstones (Bodvall & Overgaard, 1967). In a multivariate comparison of complications after laparoscopic and open cholecystectomy, the overall complication rate was found to be distinctly lower after the laparoscopic approach ( Jatzko et al, 1995). This finding was confirmed, and a lower mortality rate was observed, in a population-based cohort study comparing both surgical approaches (Zacks et al, 2002). However, in the laparoscopic era, it is exceedingly important to consider patient selection and preoperative factors when comparing the two techniques in a retrospective fashion. Wolf and colleagues (2009) recently reported on a single surgeon’s experience with open cholecystectomy over a 9-year period from 1997 to 2006. In this study, more than half (56%) of patients who underwent an open cholecystectomy were American Society of Anesthesiologists class III or IV compared with only 10% of those who underwent a laparoscopic procedure. Furthermore, nearly 10% of patients in the open cholecystectomy group had prior upper abdominal surgery, compared with only 1% of those in the laparoscopic group. These factors clearly predispose to the reported increase in morbidity and mortality rates associated with open cholecystectomy today. Open cholecystectomy increasingly is performed only in cases in which laparoscopic techniques do not allow for a safe procedure or as part of a larger procedure, such as pancreaticoduodenectomy or partial hepatectomy. The presence of severe inflammation, such as in xanthogranulomatous cholecystitis (Guzman-Valdivia, 2005), or a concern for excessive bleeding in patients with cirrhotic portal hypertension are two examples of situations in which surgeons are much more likely to use the open technique. Although two prospective studies from centers where surgeons have considerable experience performing cholecystectomy in patients with cirrhotic portal hypertension have reported the feasibility and superiority of the laparoscopic approach in these patients ( Ji et al, 2005; El-Awadi et al, 2009), it is the author’s opinion that most surgeons still opt for an open procedure. These difficult cases may be recognized preoperatively or during laparoscopy. It has been suggested, however, that attempts at performing a laparoscopic operation prior to converting to an open technique might increase the incidence of major complications (Wolf et al, 2009). Finally, as mentioned above, a gallbladder mass that is concerning for malignancy is usually better suited to an open procedure. Concern over intraoperative gallbladder perforation (Z’Graggen et al, 1998; Weiland et al, 2002), inadequate staging, and an incomplete resection with a laparoscopic procedure are some of the reasons to choose an open technique when carcinoma is suspected.

PREOPERATIVE ASSESSMENT Some clinical features should alert the surgeon to possible operative difficulties. A history of repeated and prolonged attacks of right upper quadrant pain might be associated with chronic inflammation and dense adhesions or fibrous obliteration of the triangle of Calot. Liver function tests should be performed routinely before cholecystectomy. Any abnormality (i.e., elevation) of the serum bilirubin or alkaline phosphatase requires serious attention, because it may not be caused by the presence of stones in the bile duct but may be a sign of other extrahepatic biliary tract disease. Possibilities of such alternate diagnoses include Mirizzi syndrome; tumors of the gallbladder, bile duct, or pancreas; a choledochal cyst; and sclerosing cholangitis. Although patients who are submitted to cholecystectomy usually will have undergone ultrasonography, more detailed investigations should be performed if any doubt exists as to the integrity of the bile duct. Computed tomography (CT) and magnetic resonance imaging cholangiography are the initial studies of choice. Should further diagnostic evaluation be necessary, endoscopic retrograde cholangiography usually helps identify stones or other ductal

CHAPTER 10-1  ■  Technique of Cholecystectomy  

abnormalities. Attention should be directed to any anatomic variants in the biliary system or extrahepatic vasculature that may be visualized on the above imaging modalities. A mild elevation of the aspartate aminotransferase (AST) and alanine aminotransferase (ALT) may also signify considerable pericholecystic inflammation causing localized hepatocellular necrosis. This scenario is particularly relevant in the elderly population, in diabetic patients, and in patients who are relatively immuno­ compromised with a history and physical exam that is not indicative of the underlying severe inflammation. Preoperative assessment may orient the operation toward either a laparoscopic or open technique. The results of these studies should not be used to condemn laparoscopic cholecystectomy for appropriate indications, but rather to argue for more critical preoperative evaluation of imaging studies that may alter the operative approach.

OPERATION Two operative approaches are used to perform open cholecystectomy: 1) the retrograde technique, which entails initial dissection of the hilar structures of the gallbladder in the triangle of Calot, and 2) the anterograde or fundus-down technique, in which the gallbladder is first separated from the liver along the cystic plate, before the cystic duct and artery are ligated and divided. The retrograde approach was traditionally used in most cases, until the advent of laparoscopic cholecystectomy greatly altered the patient population undergoing open cholecystectomy. Now, given that open cholecystectomy is more frequently reserved for surgically difficult situations, the anterograde approach has gained popularity to avoid initial dissection in a hostile triangle of Calot. In addition, the anterograde technique has facilitated the use of small incisions.

Anatomy A commanding knowledge of the anatomy of the bile ducts and of the possible variations is necessary to perform safe cholecystectomy, whether by a laparoscopic or open technique. Unidentified anatomic anomalies during operation may result in iatrogenic injuries to the biliary tree, so precise intraoperative identification of the anatomy is necessary before ligating or dividing any structure. The triangle of Calot, first described in 1891, forms the basis of the anatomic dissection for performing safe cholecystectomy. As originally described, this triangle is formed by the cystic duct, common hepatic duct, and the cystic artery. A common misperception is that the superior border of this triangle is the inferior border of the liver. This, rather, is termed the triangle of cholecystectomy, and it has for its upper limit not the cystic artery but the inferior surface of the liver (Figure 10-1-1; Rocko & Di Gioia, 1981). Routine meticulous dissection of this area will minimize iatrogenic injury and allow for safe cholecystectomy.

FIGURE 10-1-1  A, Triangle of cholecystectomy limited by the common hepatic duct, right hepatic duct, cystic duct, and inferior liver edge. B, The triangle of Calot limited by the common hepatic duct, cystic duct, and cystic artery.

e189

e190   SECTION 3  ■  ABDOMEN – BILIARY The normal location of the gallbladder neck and cystic duct junction is between the peritoneal surfaces within the right superior portion of the hepatoduodenal ligament. A variety of abnormalities can alter the standard anatomy or appearance of the gallbladder. A bilobar gallbladder or presence of septa usually is not relevant, as the gallbladder most often retains its normal location with respect to the liver and vital portal structures. Duplication of the gallbladder is rare and might be associated with one or two cystic ducts. Complete gallbladder agenesis is extremely rare, and the perceived absence of a gallbladder is most often associated with an intrahepatic location. In such cases, the gallbladder infundibulum is usually visualized extrahepatically. The gallbladder may be lying on the left side of a right-sided round ligament, still attached to the right side of the liver. In even rarer instances, the gallbladder has been observed attached to the left lobe of the liver, situated to the left of the round ligament (Figure 10-1-2) (Fujita et al, 1998). Variations in the junction between the cystic duct and common bile duct should be considered the rule rather than the exception in order to maintain a healthy respect for the anatomy and minimize iatrogenic injury during cholecystectomy (Figures 10-1-3 and 10-1-4). The cystic duct may join the right side of the common bile duct after a long parallel course, or it may be very short and almost nonexistent. For the latter, it is imperative not to mistake the common bile duct for the cystic duct, which can lead to inadvertent ligation and division of the common bile duct. On the contrary, a perceived short cystic duct might actually be a long structure that is fused and running parallel to the common hepatic duct, or it may be draining directly into the right hepatic duct. The cystic duct also may empty into the left side of the hepatic duct, having crossed it anteriorly or posteriorly. The cystic duct occasionally may be contracted as a result of a chronic inflammatory process. An unrecognized abnormal confluence of the hepatic ducts probably represents the most important source of error leading to damage to the biliary tract during cholecystectomy. A common anomaly that is worth noting is an early takeoff of the right posterior sectoral bile duct from the common duct either just above or below the cystic duct insertion. This sectoral duct may be confused for the cystic duct as it travels just inferior to or through the triangle of Calot. Alternatively, the cystic duct may actually arise from the right posterior sectoral duct. An abnormal confluence of the hepatic ducts has been reported in 43% of cases, and a low-lying right posterior sectoral duct has been reported in up to 20% of cases (Couinaud, 1957; Puente & Bannura, 1983; Champetier et al, 1989; see Figure 10-1-4).

FIGURE 10-1-2  Gallbladder located on the left side of the umbilical fissure and the ligamentum teres, attached to segment III of the liver.

CHAPTER 10-1  ■  Technique of Cholecystectomy  

FIGURE 10-1-3  Variations in the confluence of the cystic duct and common hepatic duct.

FIGURE 10-1-4  Variations in the confluence of the extrahepatic bile ducts and cystic duct.

Although the necessity to systematically perform intraoperative cholangiography has been controversial for decades (Talamini, 2003), some surgeons have advocated the routine use of this procedure for many years. Besides showing unidentified stones or pathology in the intrahepatic or extrahepatic bile ducts, intraoperative cholangiography provides a precise view of the anatomy of the biliary ductal system. This view may help to avoid errors that result in severe biliary injury, or at least may facilitate early intraoperative detection of an injury. In a review of 78 postcholecystectomy biliary strictures, intraoperative cholangiography was performed in only 29% (Kelley & Blumgart, 1985). Although some investigators advocate routinely performing intraoperative cholangiography in an effort to lower

e191

e192   SECTION 3  ■  ABDOMEN – BILIARY the incidence of iatrogenic bile duct injuries (Flum et al, 2003), its selective application still remains justified (Livingston et al, 2007). Unrecognized abnormalities of the arterial anatomy may result in inadvertent ligation and division of the right branch of the hepatic artery. The cystic artery normally runs transversely toward the gallbladder as it branches off the right hepatic artery. The right hepatic artery travels posterior to the common hepatic duct in 80% of cases, thus an anterior location may be unexpected and predisposes it to injury, as it may be confused for the cystic artery. An accessory or replaced right hepatic artery that originates from the superior mesenteric artery is located posterolateral to the common bile duct and behind the cystic duct, where it also may be vulnerable to injury if unrecognized. Under normal circumstances, the cystic artery is a small vessel; if it appears to be unusually large, dissection must be continued to delineate the anatomy prior to dividing any structures to avoid injury to the right hepatic artery.

Technique The retrograde technique, which involves initial dissection of the hilar structures of the gallbladder and of the cholecystectomy triangle, can be chosen when there is clear visualization of its anatomic limits. Whenever the features in this region are not clear because of acute or chronic inflammation, the anterograde or fundus-down technique is generally considered safer, because initial dissection of the gallbladder from the fundus allows progressive demonstration of the anatomy down to the infundibulocystic junction. The cystic duct may be safely ligated only when its relation with the gallbladder has been clearly delineated. Obtaining the “critical view” prior to cystic duct division as advocated by Strasberg (2002) for laparoscopic cholecystectomy is not only applicable but necessary for open cholecystectomy and to minimize iatrogenic biliary injury. The basic tenet of dissecting close to the gallbladder and clearly demonstrating every structure prior to ligation and transection should be adhered to for every procedure.

Incision The operating surgeon usually stands to the right of the patient and has a choice of various types of incisions to gain access to the gallbladder. Three common incisions used include 1) an upper midline incision, 2) a right subcostal incision, and 3) an upper midline with right lateral extension. Depending on the clinical situation and the patient’s body habitus, one incision may be preferred over the others; a right subcostal incision is most commonly performed, as it provides good, direct access to the gallbladder. The increased emphasis on minimally invasive surgery across all surgical disciplines has resulted in a trend toward making shorter incisions. The safety of the procedure should not be compromised because of lack of exposure, especially because most open cholecystectomies are now performed for technically difficult situations. Ultrasound and axial images (computed tomography or magnetic resonance imaging) obtained preoperatively may help to localize the gallbladder and select the best incision. The minilaparotomy incisions have been described as the “minimum necessary” and “tailored to the individual patient,” and their length may range from 2.5 to 10 cm (Majeed et al, 1996). When utilizing small incisions, it is important to maintain adequate exposure of the triangle of Calot in the right paramedian region, at the level of the twelfth thoracic vertebra. Exposure of the fundus of the gallbladder is less important, as it can be mobilized by traction into the operative field. To reduce abdominal wall trauma and minimize postoperative pain, small incisions should be optimized by using musclesplitting techniques. The minimal-stress triangle is located in the subxiphoid area and has for its base a horizontal line, joining the bilateral eighth costochondral cartilages, and for its vertex, the xiphoid process; the triangle of Calot lies within the boundaries of the minimal-stress triangle. Because the abdominal wall is less subject to tension and movement at this level during ventilatory and other movements, an incision in the minimal-stress triangle is reported to result in less operative pain (Tyagi et al, 1994). The technique of microceliotomy described by Tyagi and colleagues (1994) uses a 3-cm transverse skin incision to the right of midline at the level of the base of the minimal stress triangle with a corresponding vertical incision of the anterior and posterior rectus sheath 1 cm lateral to the linea alba for approximately 5 cm in length extending inferiorly from the xiphoid process. This incision involves a muscle-splitting technique with lateral retraction of the rectus muscle and incision of the peritoneum through the falciform ligament. Clezy (1996) described a transverse incision situated 8 cm below the

CHAPTER 10-1  ■  Technique of Cholecystectomy  

xiphoid process and to the right of midline followed by an incision of the anterior rectus sheath, medial retraction of the rectus muscle, and incision of the posterior sheath to allow access to the triangle of Calot. The advantages of performing a cholecystectomy through a minimal incision versus a conventional one are controversial. Some prospective randomized studies have not demonstrated a clear advantage of small-incision cholecystectomy over conventional cholecystectomy after elective operations (Schmitz et al, 1997), but others have found the small incision to be associated with less postoperative pain, a shorter hospital stay, and an earlier return to full activities after emergency cholecystectomy (Assalia et al, 1997).

Initial Assessment Adhesions to the gallbladder may be present, especially in cases of severe cholecystitis. Such adhesions may be dense, vascular, and inflammatory, thus obscuring the anatomy. Dissection should be performed close to the gallbladder, keeping in mind that a cholecystocolic or cholecystoduodenal fistula might be present. In this case, the fistula must be divided in order to expose the gallbladder, with the opening in the colon or duodenum subsequently sutured closed or attended to in the most appropriate manner. If the gallbladder cannot be identified, one should suspect that it is scarred and contracted or that it is located within the liver parenchyma. In this instance, an intraoperative ultrasound examination may be useful. It may be feasible and safe to try to identify first the distal bile duct and trace it superiorly toward the infundibulum of the gallbladder and cystic duct junction, but this maneuver is not routinely recommended. An abdominal exploration and manual palpation of other organs should be performed whenever possible, with special emphasis on the liver, hepatoduodenal ligament, and pancreas. The gallbladder must be palpated gently and not emptied by compression to prevent distal migration of small stones into the common bile duct. Placement of Retractors and Optimizing Exposure A self-retaining retractor that is fixed to the operating table is best used to provide adequate and constant body-wall retraction and spare the hand of the assistant. A fixed retractor should be placed in the right upper quadrant to provide upward and lateral retraction on the costal margin, regardless of which incision is used. For minilaparotomy incisions, however, retraction may need to be frequently altered and thus may be best suited for manual retraction. If feasible, upward retraction on segment VI and the left lateral segments of the liver facilitates exposure of the triangle of cholecystectomy (see Figure 10-1-1). Care must always be taken not to compromise blood flow in the left portal pedicle with excessive or improperly placed retraction and not to tear the liver capsule. Moist laparotomy pads help to expose and isolate the operative field; pads placed behind and posterior to the right hepatic lobe serve to push the gallbladder into the wound. Perhaps the most crucial maneuver to optimize exposure is caudal retraction of the duodenum, gastric antrum, and colon. This is best accomplished with the assistant’s hand or a hand-held, wide, malleable retractor or carefully applied fixed retraction to provide gentle traction on the hepatoduodenal ligament. Emptying the Gallbladder The dissection usually is facilitated by slight distension of the gallbladder, and thus routine aspiration of the gallbladder contents should not be performed; however, gross distension may obscure adequate exposure and inhibit the surgeon’s ability to handle the gallbladder. In this circumstance, one may choose to puncture the fundus and aspirate bile, and bile culture is indicated for cholecystitis or cholangitis. Opening the gallbladder or spillage of bile should be avoided at all costs when there is a suspicion of gallbladder cancer. Spilled bile from a gallbladder containing malignant cells can result in peritoneal cavity seeding and tumor implantation, thus converting a potentially curable situation into an incurable disease.

Retrograde Cholecystectomy A large Kelly clamp or similar instrument is placed to grasp the fundus of the gallbladder in the region of the Hartmann pouch (Figure 10-1-5A), and dissection of the cholecystectomy triangle is started. The peritoneum covering the hepatoduodenal ligament is incised anteriorly across the region of the Hartmann pouch; this incision is continued posteriorly in the same way, giving complete access to the

e193

e194   SECTION 3  ■  ABDOMEN – BILIARY

FIGURE 10-1-5  A, The gallbladder is grasped with a clamp, and dissection of the cholecystectomy triangle is started. B, The cystic artery is in its normal position, above the cystic duct.

infundibulum of the gallbladder. It is important to keep the dissection close to the gallbladder and to delineate the junction between the gallbladder and the cystic duct. The lower limit of the triangle is the cystic duct, and a suture ligature is passed around it but is not tied; the slight tension produced by a clamp hanging on this ligature helps prevent migration of stones from the gallbladder into the cystic duct. The cystic artery is normally found just above the cystic duct, although a posterior location is also possible. It is important to dissect the artery toward the gallbladder to see its final distribution into the gallbladder wall (Figure 10-1-5B) to prevent inadvertent ligature of an aberrant or anterior right hepatic artery. At this stage, the junction of the gallbladder infundibulum with the cystic duct and the distribution of the cystic artery into the gallbladder wall should be clearly visible. The cystic duct is palpated to detect stones, which if present should be “milked” back into the gallbladder (Figure 10-1-6), and the cystic artery is ligated and transected (Figure 10-1-7). At this juncture, if a cholangiogram will be performed, a ligature or a clip is placed at the junction of the gallbladder and the cystic duct, and the cystic duct is partially opened through a small transverse incision approximately 2 to 3 mm distal. A 5-Fr catheter for cholangiography is inserted gently into the cystic duct, taking care not to tear the duct (Figure 10-1-8). If easy passage is inhibited, and if the presence of a stone has been excluded by palpation, a valve or tortuous cystic duct may be the cause. It is helpful in this event to insert the end of a fine clamp into the cystic duct to dilate it gently; this often facilitates passage of the catheter. The tip of the catheter should remain in the cystic duct. The catheter is fixed by tying the previously passed ligature around the cystic duct or by the loose application of a clip. Some catheters are equipped with an inflatable balloon that prevents the catheter from dislodging. Prior to performing cholangiography, the biliary system should be flushed with 10 to 20 mL of saline. This often allows for passage of small stones measuring less than 4 mm in diameter. All instruments and retractors are removed, and the patient is slightly rotated (20 degrees) to the right before contrast medium is injected. It is important to initially inject a small quantity (1 to 2 mL) of the contrast agent to be able to identify small stones in the bile duct. The ductal system of the biliary tract should be fully displayed, including intrahepatic ducts and free passage into the duodenum on the fluoroscopic images. Intravenous administration of 1 to 2 mg of glucagon may help to relax the

CHAPTER 10-1  ■  Technique of Cholecystectomy  

FIGURE 10-1-6  Palpation reveals a stone in the cystic duct, which may be “milked” back into the gallbladder.

FIGURE 10-1-7  The cystic artery is ligated close to the gallbladder wall.

e195

e196   SECTION 3  ■  ABDOMEN – BILIARY

FIGURE 10-1-8  The cystic duct is ligated at its junction with the gallbladder, and a catheter has been inserted for intraoperative cholangiography.

FIGURE 10-1-9  The catheter is removed, and the cystic duct is suture ligated.

sphincter of Oddi to faciliate passage of small stones. Once the biliary system is clear, the cholangio­ gram catheter is removed, and the cystic duct is ligated with a resorbable suture ligature; it is my preference to use a 3-0 Vicryl suture (Figure 10-1-9) and to place a clip to ensure biliary stasis from the cystic duct stump. The gallbladder is dissected from its fossa with diathermy, gentle traction, or occasionally suction dissection. The dissection should be kept close to the gallbladder and within the cystic plate to avoid damage to the liver parenchyma, which nevertheless may occur in the presence of severe inflammation. In cases of acute cholecystitis with considerable edema, this plane may be best found by sharp dissection.

CHAPTER 10-1  ■  Technique of Cholecystectomy  

Occasionally, small bile ducts may connect the gallbladder to the intrahepatic bile ducts. Transection of these so-called ducts of Luschka is usually without consequence, when the biliary tree is not obstructed distally. Hemostasis of the gallbladder fossa is obtained and, if available, the argon beam coagulator may be valuable here. If the liver parenchyma has been lacerated, however, a gauze pack should be placed in the gallbladder bed and held in place with constant pressure for at least 5 to 10 minutes. If the hemorrhage is not controlled with simple pressure, deep hemostatic sutures can be placed. Care should be taken to ensure that such sutures do not entrap major branches of the right portal pedicle, which can lie in close proximity. Formal closure of the gallbladder bed is not necessary and may even predispose to formation of a postoperative local fluid collection. Once removed, the gallbladder should be routinely opened and assessed for the presence of tumors. The insertion of a drain before closing the abdominal wall is controversial and in most cases unnecessary. If a small bile leak is suspected or identified, insertion of a drain positioned close to the gallbladder bed usually prevents collection of bile. A small leak is without consequence and closes spontaneously within a few days if the distal bile duct is not obstructed. If only serosanguinous fluid drains during the first 48 hours, the drain can be safely removed; however, the drain should be retained if excessive oozing or leakage of bile occurs.

Anterograde, or Fundus-Down, Cholecystectomy A large Kelly clamp or similar instrument is placed to grasp the fundus of the gallbladder, and an Adson (tonsil) clamp is used to grasp the peritoneum at the interface of the gallbladder and liver edge. An incision of the gallbladder serosa is performed 0.5 cm from the liver edge using diathermy, and a plane is developed between the serosa and the gallbladder wall to allow entry to the cystic plate. This plane is developed medially and laterally to dissect the gallbladder away from the liver. It is important to complete this dissection posterolaterally to facilitate lateral retraction of the gallbladder to best expose the cystic duct and artery. The gallbladder is still vascularized via the cystic artery (Figure 10-1-10), and this structure is encountered as the medial dissection is continued toward the triangle of Calot. In the region of the infundibulum, the cystic artery is seen to enter the gallbladder

FIGURE 10-1-10  A, Anterograde, or fundus-down, cholecystectomy; the serosa of the gallbladder is incised 5 mm from the liver edge around the fundus. B, A plane is developed between the serosa of the gallbladder and the gallbladder wall, then between the liver and the gallbladder within the cystic plate.

e197

e198   SECTION 3  ■  ABDOMEN – BILIARY

FIGURE 10-1-11  As the anterograde dissection progresses toward the gallbladder neck medially, the cystic artery is identified, ligated, and divided.

wall (Figure 10-1-11). After ligature and division of the cystic artery close to the gallbladder wall, thus protecting the right hepatic artery, the infundibulum is dissected free down toward its junction with the cystic duct. This technique may cause migration of stones from the gallbladder into the cystic duct, but careful palpation should identify these stones after the cystic duct has been isolated. If detected, the stones should be milked back into the gallbladder. Not more than 0.5 to 1 cm of cystic duct should be dissected to prevent injury to the common bile duct. Cholangiography and cystic duct ligature are performed in the same way as described for the retrograde technique.

Cholecystectomy Through Small Incisions There is no fundamental difference between small-incision cholecystectomy and the techniques described above. Some authors prefer operating from the left side of the patient when a small incision is used; smaller retractors are adequate, and an additional light source (head lamp, light retractor, light suction) may be useful. The anterograde technique is the preferred approach through small incisions. The use of clips for controlling the cystic artery and the cystic duct may be safer than awkwardly tying through a small incision.

Partial Cholecystectomy Cholecystectomy may be hazardous when only the fundus of the gallbladder can be recognized, and when the region of the infundibulum cannot be delineated because of fibrosis and inflammation obscuring the triangle of Calot (Figure 10-1-12). In this case, it is often judicious to open the fundus and introduce a finger into the gallbladder to guide the dissection (Figure 10-1-13A). Once the gallbladder is open, if no bile appears, the cystic duct is probably occluded by fibrosis and inflammation. If visualized, impacted stones should be removed, and care should be taken not to push any stones further into the cystic duct. If a gush of bile appears when a big impacted stone is removed from the infundibulum, a cholecysto­ choledochal fistula may be present (Mirizzi syndrome type II). In this circumstance, continued dissection to identify the cystic duct–bile duct junction risks significant biliary injury. In cases of severe inflammation, a partial cholecystectomy may be the safest procedure (Di Carlo et al, 2009; Sharp et al, 2009). The anterior visible wall of the gallbladder is excised, but the posterior wall that contacts the liver is left in place down to the region of the infundibulum. The posterior wall mucosa may be removed by curettage, or it may be fulgurated with electrocoagulation or argon beam coagulation, although this last step is not mandatory. A closed-suction drain is placed in the region of the infundibulum (Figure 10-1-13B). When the regional anatomy is severely altered by inflammation, any attempt at ligating the cystic duct may predispose to injury and can usually be safely avoided, particularly when no bile leakage is visualized. Should the anatomy favor an attempt at cystic duct closure, or when bile leakage is observed, the orifice of the cystic duct is best closed from within the gallbladder, using a purse-string or oversew technique. When a cholecystocholedochal fistula is suspected, it is

CHAPTER 10-1  ■  Technique of Cholecystectomy  

FIGURE 10-1-12  The common hepatic duct can be mistaken for the cystic duct when the region of the infundibulum cannot be delineated because of fibrosis and inflammation.

FIGURE 10-1-13  A, The fundus of the gallbladder has been opened, and a finger is introduced into the gallbladder for palpation. B, Partial cholecystectomy. The superficial part of the fundus and body of the gallbladder has been excised, leaving in place its attachment to the liver. The remaining mucosa is removed by curettage and/or fulgurated with electrocoagulation, and a closed-suction drain is placed near the infundibulum.

advisable to keep a rim of gallbladder wall intact to allow for a subsequent cholecystoduodenostomy or a cholecystojejunostomy. Adequate biliary drainage with closed-suction drains obviates the need for immediate definitive repair and/or reconstruction; it can be delayed, possibly even avoided, with postoperative endoscopic stenting procedures. Attempts at immediate, direct repair of the fistula are unnecessary, difficult, and potentially hazardous (Baer et al, 1990). Partial cholecystectomy has also been advocated for patients with severe portal hypertension to prevent significant hemorrhage during dissection of the triangle of Calot (Bornman & Terblanche, 1985; Cottier et al, 1991). Bleeding from the transected wall of the remaining gallbladder is controlled with a running absorbable suture.

e199

e200   SECTION 3  ■  ABDOMEN – BILIARY

INTRAOPERATIVE PROBLEMS Intraoperative problems have been related to three main causes: 1) dangerous surgical technique, 2) dangerous anatomy, and 3) dangerous pathology ( Johnston, 1986). Insufficient preoperative assessment of a complicated situation is another avoidable reason for intraoperative difficulties. Dangerous technique arises from inadequate or imprecise application of the principles of cholecystectomy, insufficient experience, inadequate incision and exposure, or inadequate assistance (AndrenSandberg et al, 1985). Some of the anatomic variations that have been mentioned previously are particularly dangerous, especially a narrow common bile duct, which can be mistaken for the cystic duct. Dangerous pathology includes chronic or acute inflammation that results in obscured anatomy and increased vascularity in the region of the cholecystectomy triangle (see Figure 10-1-12). Portal hypertension is associated with increased venous collateralization, which makes the dissection hemorrhagic and dangerous. Partial cholecystectomy has been advocated in both situations (Bornman & Terblanche, 1985; Cottier et al, 1991). Hemorrhage in the cholecystectomy triangle represents a potential danger, because attempts at hemostasis by placing clamps with obstructed and insufficient view may result in inadvertent clamping of the right or common hepatic artery or of the bile duct (Figure 10-1-14). In this situation, the surgeon should first attempt to control the hemorrhage by digital compression or by clamping the hepatoduodenal ligament (Figure 10-1-15) to localize its precise origin. Grasping the bleeding vessel should be done with precision so as to limit the risk of including another structure in the ligature. Cholangiography, even if already performed, may be repeated and analyzed carefully after hemostasis is achieved, because it may reveal an iatrogenic injury to the bile duct, such as a leak or an incomplete or complete occlusion.

FIGURE 10-1-14  Blind placement of clamps for hemostasis can result in injury to the hepatic artery or bile duct.

CHAPTER 10-1  ■  Technique of Cholecystectomy  

FIGURE 10-1-15  Hemorrhage should be controlled first by manual clamping of the hepatoduodenal ligament until exposure is optimized, thus facilitating precise hemostasis.

References Andren-Sandberg A, et al, 1985: Accidental lesions of the common bile duct at cholecystectomy. Pre- and perioperative factors of importance. Ann Surg 201(3):328–332. Assalia A, et al, 1997: Emergency minilaparotomy cholecystectomy for acute cholecystitis: prospective randomized trial – implications for the laparoscopic era. World J Surg 21(5):534–539. Baer HU, et al, 1990: Management of the Mirizzi syndrome and the surgical implications of cholecystcholedochal fistula. Br J Surg 77(7):743–745. Barkun JS, et al, 1992: Randomised controlled trial of laparoscopic versus mini cholecystectomy. The McGill Gallstone Treatment Group. Lancet 340(8828):1116–1119. Bodvall B, Overgaard B, 1967: Computer analysis of postcholecystectomy biliary tract symptoms. Surg Gynecol Obstet 124(4):723–732. Bornman PC, Terblanche J, 1985: Subtotal cholecystectomy: for the difficult gallbladder in portal hypertension and cholecystitis. Surgery 98(1):1–6. Calvert NW, et al, 2000: Laparoscopic cholecystectomy: a good buy? A cost comparison with small-incision (mini) cholecystectomy. Eur J Surg 166(10):782–786. Champetier J, et al, 1989: [Variations of division of the extrahepatic bile ducts: significance and origin, surgical implications]. J Chir (Paris) 126(3):147–154. Clezy JK, 1996: Randomized trial of laparoscopic cholecystectomy and mini-cholecystectomy. Br J Surg 83(2):279. Cottier DJ, et al, 1991: Subtotal cholecystectomy. Br J Surg 78(11):1326–1328. Couinaud C, 1957: Le Foie: Études Anatomiques et Chirurgicales. Paris, Masson. Di Carlo I, et al, 2009: Modified subtotal cholecystectomy: results of a laparotomy procedure during the laparoscopic era. World J Surg 33(3):520–525. El-Awadi S, et al, 2009: Laparoscopic versus open cholecystectomy in cirrhotic patients: a prospective randomized study. Int J Surg 7(1):66–69. Flum DR, et al, 2003: Intraoperative cholangiography and risk of common bile duct injury during cholecystectomy. JAMA 289(13):1639–1644. Fujita N, et al, 1998: Left-sided gallbladder on the basis of a right-sided round ligament. Hepatogastroenterology 45(23):1482–1484. Guzman-Valdivia G, 2005: Xanthogranulomatous cholecystitis in laparoscopic surgery. J Gastrointest Surg 9(4):494–497. Jatzko GR, et al, 1995: Multivariate comparison of complications after laparoscopic cholecystectomy and open cholecystectomy. Ann Surg 221(4):381–386. Ji W, et al, 2005: A randomized controlled trial of laparoscopic versus open cholecystectomy in patients with cirrhotic portal hypertension. World J Gastroenterol 11(16):2513–2517. Johnston GW, 1986: Iatrogenic bile duct stricture: an avoidable surgical hazard? Br J Surg 73(4):245–247. Kelley CJ, Blumgart LH, 1985: Per-operative cholangiography and post-cholecystectomy biliary strictures. Ann R Coll Surg Engl 67(2):93–95.

e201

e202   SECTION 3  ■  ABDOMEN – BILIARY Keus F, et al, 2008: Laparoscopic versus small-incision cholecystectomy: health status in a blind randomised trial. Surg Endosc 22(7):1649–1659. Livingston EH, et al, 2007: Costs and utilization of intraoperative cholangiography. J Gastrointest Surg 11(9):1162–1167. Majeed AW, et al, 1996: Randomised, prospective, single-blind comparison of laparoscopic versus small-incision cholecyst­ ectomy. Lancet 347(9007):989–994. McMahon AJ, et al, 1994: Laparoscopic versus minilaparotomy cholecystectomy: a randomised trial. Lancet 343(8890):135–138. Puente SG, Bannura GC, 1983: Radiological anatomy of the biliary tract: variations and congenital abnormalities. World J Surg 7(2):271–276. Rocko JM, Di Gioia JM, 1981: Calot’s triangle revisited. Surg Gynecol Obstet 153(3):410–414. Ros A, et al, 2001: Laparoscopic cholecystectomy versus mini-laparotomy cholecystectomy: a prospective, randomized, singleblind study. Ann Surg 234(6):741–749. Schmitz R, et al, 1997: Randomized clinical trial of conventional cholecystectomy versus minicholecystectomy. Br J Surg 84(12):1683–1686. Schwesinger WH, Diehl AK, 1996: Changing indications for laparoscopic cholecystectomy. Stones without symptoms and symptoms without stones. Surg Clin North Am 76(3):493–504. Sharp CF, et al, 2009: Partial cholecystectomy in the setting of severe inflammation is an acceptable consideration with few long-term sequelae. Am Surg 75(3):249–252. Shea JA, et al, 1998: Indications for and outcomes of cholecystectomy: a comparison of the pre and postlaparoscopic eras. Ann Surg 227(3):343–350. Strasberg SM, 2002: Avoidance of biliary injury during laparoscopic cholecystectomy. J Hepatobiliary Pancreat Surg 9(5):543–547. Talamini MA, 2003: Routine vs selective intraoperative cholangiography during cholecystectomy. JAMA 289(13):1691–1692. Tyagi NS, et al, 1994: A new minimally invasive technique for cholecystectomy: subxiphoid “minimal stress triangle” micro­ celiotomy. Ann Surg 220(5):617–625. Weiland ST, et al, 2002: Should suspected early gallbladder cancer be treated laparoscopically? J Gastrointest Surg 6(1):50–56. Wolf AS, et al, 2009: Surgical outcomes of open cholecystectomy in the laparoscopic era. Am J Surg 197(6):781–784. Z’Graggen K, et al, 1998: Incidence of port site recurrence after laparoscopic cholecystectomy for preoperatively unsuspected gallbladder carcinoma. Surgery 124(5):831–838. Zacks SL, et al, 2002: A population-based cohort study comparing laparoscopic cholecystectomy and open cholecystectomy. Am J Gastroenterol 97(2):334–340.

SELF ASSESSMENT Kiranjeet Gill  /  Daniel J. Deziel From Velasco J: Rush University Medical Center Review of Surgery, 5th edition (Saunders 2011)

10-2 

1. Which of the following statements is true about the hepatic arterial supply? A. Aberrant hepatic arterial anatomy is present in less than 5% of all patients. B. The cystic artery is usually a branch off the proper hepatic artery. C. A “replaced” right hepatic artery arises from the superior mesenteric artery. D. The hepatic artery provides 75% of blood flow to the liver. E. The hepatic artery lies dorsal to the portal vein within the hepatic hilum. Ref.: 1–3 COMMENTS: The hepatic arterial supply is normally derived from the celiac axis by way of the common hepatic artery, which becomes the proper hepatic artery after giving off the gastroduodenal branch and subsequently bifurcates into right and left hepatic branches. The hepatic artery lies ventral to the portal vein. The middle hepatic artery is usually a branch off the left hepatic artery, and the cystic artery is generally a branch off the right hepatic artery. There is, however, significant variability in hepatic arterial anatomy in up to 50% of patients. In approximately 15% of individuals, the right hepatic artery arises from the superior mesenteric artery (replaced right hepatic artery) and is found in the right dorsal border of the hepatoduodenal ligament. In roughly 10% of individuals, the left hepatic artery originates from the left gastric artery and is located in the gastrohepatic ligament. These commonly encountered variants can have important surgical implications during upper abdominal operations. The arterial blood supply accounts for only 25% of hepatic blood flow, with the remainder being supplied by the portal vein.

ANSWER: C 2. Which surgeon performed the world’s first known cholecystectomy? A. Karl Langenbuch B. Justus Ohage C. Hans Kehr D. Lawson Tait E. Eric Mühe Ref.: 4 COMMENTS: Karl Langenbuch performed the first operation to remove the gallbladder on July 15, 1882. Before that and, in fact, even for years afterward, patients with symptomatic gallstone disease were treated only with ineffective medical remedies or, occasionally, by cholecystostomy to drain the gallbladder. The first cholecystectomy in the Western Hemisphere was performed 4 years later by Justus Ohage in St. Paul, Minnesota. Hans Kehr of Halberstadt and Berlin was an early pioneer in biliary surgery. In 1901, he published a remarkable book describing more than 500 operations for gallstones, including 96 common bile duct operations. Kehr died of sepsis caused by a hand infection incurred after digital exploration of the common bile duct. Lawson Tait was a famed nineteenthcentury English surgeon who advocated cholecystostomy rather than cholecystectomy. Eric Mühe e203

e204   SECTION 3  ■  ABDOMEN – BILIARY performed the first “laparoscopic” cholecystectomy in Germany in 1985. Although technically different from modern laparoscopic cholecystectomy, it was a landmark contribution. Mühe was severely criticized and, in fact, vilified by the surgical community at the time. Only years later was the significance of his accomplishment recognized.

ANSWER: A 3. During palpation of the hepatoduodenal ligament, a pulsation is felt dorsal and slightly to the right of the common bile duct. Which of the following does this pulsation most likely represent? A. A normal common hepatic artery B. A normal right hepatic artery C. A replaced right hepatic artery D. A gastroduodenal artery E. A right renal artery Ref.: 5, 6 COMMENTS: The most common variation in hepatic arterial anatomy is origination of the right hepatic artery from the superior mesenteric artery. This is a replaced hepatic artery and not simply an accessory vessel that can be sacrificed with impunity. When an operation is performed in the right upper part of the abdomen, the pulsations encountered in the porta hepatis and gastrohepatic ligaments should be assessed. If the hepatic artery is absent or small, the surgeon must be alert to the possibility of a replaced hepatic vessel. When the right hepatic artery originates from the superior mesenteric artery, it courses dorsal to the head of the pancreas and the portal vein and is usually identified dorsolateral to the common bile duct. This vessel and its origin can readily be identified with intraoperative ultrasonography. Only rarely does a replaced right hepatic artery course through the pancreas. A replaced left hepatic artery originates from the left gastric artery and is located in the gastrohepatic ligament, where it is frequently encountered during operations on the stomach and gastroesophageal junction.

ANSWER: C 4. Which of the following is decreased after cholecystectomy? A. Size of the bile acid pool B. Rate of enterohepatic recycling C. Rate of bile acid secretion D. Cholesterol solubility in bile E. Rate of bilirubin conjugation Ref.: 5 COMMENTS: The total size of the bile acid pool is diminished after cholecystectomy as a result of loss of the gallbladder reservoir. However, cholecystectomy produces a more continuous flow of bile into the intestine, which increases the frequency of enterohepatic cycling and stimulates bile acid secretion. For these reasons, even though the size of the bile acid pool is diminished, cholecystectomy improves cholesterol solubility in bile. The solubility of cholesterol in bile depends on the relative molar concentration of cholesterol in relation to the concentration of bile acids and the phospholipid lecithin. This relationship, described by W. Admirand and D. M. Small in 1969, is graphically depicted by the following familiar diagram:

CHAPTER 10-2  ■  Self Assessment  

FIGURE 10-3-1 

ANSWER: A 5. A surgeon encounters difficulty during an elective laparoscopic cholecystectomy in a healthy 25-yearold woman and converts to an open procedure. The 4-mm common hepatic duct has been transected 1 cm below the bifurcation. Which of the following procedures is the most appropriate? A. Duct-to-duct repair over a T tube B. Duct-to-duct repair without a stent C. Roux-en-Y hepaticojejunostomy D. Hepaticoduodenostomy E. Ligation of the duct and placement of a drain Ref.: 8 COMMENTS: When a transection or resection injury of the extrahepatic biliary tree is discovered at the time of cholecystectomy, the surgeon must make some careful decisions. Repair at the time is preferable, provided that the surgeon is adequately experienced in performing such a repair so that a successful outcome is likely. Unfortunately, the weight of evidence indicates that most primary repairs by the initial operating surgeon have failed, thus necessitating repeated operations and other interventions. The initial repair of a major duct injury has the best chance for long-term success. A less experienced surgeon should not attempt anastomosis of a small bile duct but seek the help of an experienced colleague if available. Otherwise, drains should be placed and transfer to an experienced hepatobiliary surgeon arranged. If repair at the time is appropriate, the standard reconstruction for this type of injury is a Roux-en-Y hepaticojejunostomy. Duct-to-duct repairs usually fail in this situation. Hepaticoduodenostomy is not recommended for an injury at this level.

ANSWER: C 6. How would the bile duct injury described in Question 5 be classified? A. Bismuth type 1 B. Bismuth type 2 C. Bismuth type 3 D. Bismuth type 4 E. Bismuth type 5 Ref.: 7

e205

e206   SECTION 3  ■  ABDOMEN – BILIARY COMMENTS: The Bismuth classification of bile duct injuries and strictures describes the level of injury in relation to the bifurcation of the main right and left hepatic ducts. Higher injuries are more difficult. They require a greater degree of technical skill and expertise to reconstruct, and reconstructions may have a lower long-term success rate. Many of the injuries resulting from laparoscopic cholecystectomy have been higher than those seen with open cholecystectomy. Moreover, many injuries, initially lower, end up being higher when repaired because of the need to debride unhealthy ductal tissue as a result of ischemia or inflammation and infection caused by bile leakage. With a type 1 injury, 2 cm or more of the common hepatic duct is preserved below the bifurcation. With a type 2 injury, less than 2 cm remains. A type 3 injury reaches the bifurcation with preservation of continuity between the right and left ducts. A type 4 injury involves destruction of the hepatic duct confluence with separation of the right and left hepatic ducts. A type 5 injury involves a separate inserting right sectoral duct with or without injury to the common duct.

Bismuth classification of bile duct injury.

ANSWER: B

References 1. D’Angelica M, Fong Y: The liver. In Townsend CM, Beauchamp RD, Evers BM, et al, editors: Sabiston textbook of surgery: the biological basis of modern surgical practice, ed 18, Philadelphia, 2008, WB Saunders. 2. Geller DA, Goss JA, Tsung A: Liver. In Brunicardi FC, Anderson DK, Billar TR, et al, editors: Schwartz’s principles of surgery, ed 9, New York, 2010, McGraw-Hill. 3. Hepatobiliary and Portal Venous System Section. In Mulholland MW, Lillemoe KD, Doherty GM, et al, editors: Greenfield’s surgery: scientific principles and practice, ed 4, Philadelphia, 2006, Lippincott Williams & Wilkins. 4. Deziel DJ: The journey of the surgeon-hero, Surg Endosc 22:1–7, 2008. 5. Chari RS, Shah SA: Biliary system. In Townsend CM, Beauchamp RD, Evers BM, et al, editors: Sabiston textbook of surgery: the biological basis of modern surgical practice, ed 18, Philadelphia, 2008, WB Saunders. 6. Mullholland MW, Lillemoe KD, Doherty GM, et al, editors: Greenfield’s surgery: scientific principles and practice, Hepatobiliary and Portal Venous System Section, ed 4, Philadelphia, 2006, Lippincott Williams & Wilkins. 7. Blumgart LH: Surgery of the liver, biliary tract and pancreas, ed 4, Edinburgh, 2006, Churchill-Livingstone. 8. Lillemoe KD: Current management of bile duct injury, Br J Surg 95:403–405, 2008.

Hepatic Biopsy – Laparoscopic

GOALS/OBJECTIVES • • • •

ANATOMY INDICATIONS TECHNIQUES COMPLICATIONS

11 

11-1 

LIVER RESECTIONS Nicholas O’Rourke From Vernon AH, Ashley SW: Atlas of Minimally Invasive Surgical Techniques, 1st edition (Saunders 2012)

The video for this procedure can be accessed here

Further Reading Steele, K et al. Video 4 – Flexible Transgastric Peritoneoscopy and liver biopsy: a feasibility study in human beings. Steele, Kimberley, MD, Schweitzer, Michael A., MD, Lyn-Sue, Jerome, MD, Kantsevoy, Sergey V., MD, PhD – Gastrointestinal Endoscopy. Copyright © 2008 American Society for Gastrointestinal Endoscopy Liver Biopsy Guideline. Rockey DC. American Association for the Study of Liver Disease, Hepatology 2009; 49(3): 1017–44.

e210

SELF ASSESSMENT Kiranjeet Gill  /  Daniel J. Deziel From Velasco J: Rush University Medical Center Review of Surgery, 5th edition (Saunders 2011)

11-2 

1. Which of the following statements about the anatomy of the liver is true? A. The right lobe extends to the umbilical fissure and falciform ligament. B. The left lobe ends at the falciform ligament. C. The quadrate lobe is a portion of the medial segment of the right lobe. D. The left lobe contains the anterior and lateral segments. E. The lateral segment of the left lobe in the American system consists of segments II and III. Ref.: 1–3 COMMENTS: The surgical anatomy of the liver is based on the distribution of the hepatic veins and portal structures and has been modified several times. There are two main anatomic classification systems for the liver, the American system and the French system. In both these systems, the liver is divided into right and left lobes by the Cantilie line, a longitudinal plane that extends from the gallbladder fossa to the inferior vena cava. This plane, also called the portal fissure, contains the middle hepatic vein and the bifurcation of the portal vein. In the American system, the liver is further broken down into four segments, with each lobe containing two segments. The right lobe of the liver consists of posterior and anterior segments. The left lobe consists of a medial segment (quadrate lobe) and a lateral segment divided by the falciform ligament. The caudate lobe can be considered anatomically independent of the right and left lobes because it receives portal and arterial blood supply from both sides and has venous drainage directly into the inferior vena cava. In the French system, developed by C. Couinaud, the two lobes of the liver are broken down into eight segments. These eight segments are formed by three vertical planes (scissurae) created by the right, middle, and left hepatic veins, which results in four sectors. These four sectors are further divided by a plane created by the branching portal system. Therefore, the left lobe, according to the French system, is divided into medial and lateral segments by the left hepatic vein. The lateral sector of the left lobe consists of a superior segment (II) and an inferior segment (III). The medial sector of the left lobe is segment IV. The right lobe consists of anteromedial and posterolateral sectors divided by a vertical plane containing the right hepatic vein. The anteromedial sector is made up of segment V (inferior) and segment VIII (superior), and the posterolateral sector is made up of segment VI (inferior) and segment VII (superior).

ANSWER: E 2. Resection of hepatic metastases has most clearly benefited patients with which of the following cancers? A. Colon B. Breast C. Stomach D. Pancreas E. Lung Ref.: 1–3 e211

e212   SECTION 4  ■  ABDOMEN – LIVER COMMENTS: Resection of hepatic metastases from colorectal cancer provides a clear survival advantage over any other treatment and should be performed whenever possible. The 5-year survival rate is approximately 25% and is as high as 40% in favorable subgroups. Resection of metastatic neuroendocrine tumors (e.g., carcinoid, insulinoma, and gastrinoma) can be valuable for controlling the symptoms of excessive endocrine secretion. Experience with hepatic resection for metastases from other portal sites (e.g., stomach, pancreas, and biliary) or nonportal sites (e.g., lung, breast, melanoma, gynecologic, head and neck, and renal) has been more limited, and the results have not generally been as encouraging. Occasionally, a patient with a noncolorectal primary malignancy is cured when the isolated hepatic metastasis is resected. However, the natural history of noncolorectal primary malignancies is such that metastases isolated to the liver rarely develop. Hepatic resection for direct, contiguous growth of the primary tumor (e.g., stomach and biliary) into the liver sometimes produces long-term survivors.

ANSWER: A 3. Which of the following is the most accurate method for identifying hepatic metastases? A. Transabdominal ultrasound B. CT C. Laparoscopy D. Intraoperative palpation E. Intraoperative ultrasound imaging Ref.: 4 COMMENTS: Transabdominal ultrasound is as accurate as CT for detecting liver tumors that are 2 cm in size or larger. For smaller lesions, computed tomography is more accurate, although it can miss the smallest lesions ( 15 years): TC every 2 (pancolitis) to 3 years (left-side colitis) and biopsies every 10 cm. Recent studies recommend routine use of indigo carmine or methylene blue chromoendoscopy with fewer, targeted biopsies of subtle abnormalities of colonic crypts or vessel pattern.

Surveillance of Patients After Resection of One or More Colonic Polyps Hyperplastic polyps (size ≥1 cm, ≥5 in number, location in the proximal colon with a family history of hyperplastic polyposis): TC at 5, and 15 years Low-risk adenomas (V3) or advanced adenomas (size ≥1 cm, ≥25% villous component, high-grade dysplasia (HGD) or in situ carcinoma) or V4.1/V4.2 adenomas: ○ Incomplete resection: TC at 3 months ○ Complete resection: advanced adenoma or ≥3 in number, or a family history of CRC; TC at 3, 8, 13, and 23 years Complete resection: non-advanced adenoma, 10 Tubulous Tubulovillous Villous Low High

1 2 3 1 2 3 1 2 3 1 3

point points points point points points point points points point points

TABLE 40-1-2  Spigelman Score Spigelman Stage

Management

Endoscopic Surveillance

0 (0 points) I (1–4 points) II (5–6 points) III (7–8 points) IV (9–12 points)

Endoscopic surveillance Endoscopic surveillance Endoscopic surveillance Surgery or endoscopic management Consider referral for surgery

4 years 2–3 years 2–3 years 6–12 months 3–6 months

Duodenal adenomas occur in 90% of patients (Figure 40-1-14B) Jejunal and ileal polyps are present in 50–90% of patients Patients should undergo screening with both forward and side-viewing endoscopes between the ages of 25 to 30 Biopsies should be taken from the largest duodenal polyps and from the ampulla Subsequent follow-up should be determined based on the Spigelman score (Tables 40-1-1, 40-1-2).

e719

e720   SECTION 11  ■  ENDOSCOPY WARNING! Biopsying the ampulla. Biopsies should be taken AWAY from the pancreatic orifice to avoid pancreatitis. A safe area to biopsy is the upper left quadrant (see Figure 40-1-6B).

Hereditary Non-Polyposis Colorectal Cancer (HNPCC) Patients with HNPCC are at increased risk of gastric and small bowel cancer Endoscopic surveillance should be considered commencing at age 30.

CONTRAINDICATIONS There are no absolute contraindications to EGD. The examination, however, may be dangerous in the following cases: ○ Known or suspected perforation. EGD should not be performed unless to insert a covered stent to treat the perforation ○ Massive gastrointestinal hemorrhage suggesting an aortoduodenal fistula ○ Acute cardiorespiratory failure not responding to medical therapy ○ Hypovolemic shock not responding to aggressive resuscitation EGD should be performed with caution in the following situations: ○ Large Zenker’s diverticulum ○ Severe respiratory failure ○ Thoracic aortic aneurysm ○ Strictures of the cervical esophagus.

DIAGNOSTIC AND THERAPEUTIC ENDOSCOPY OF THE STOMACH AND SMALL BOWEL

40-2 

Jeffrey L. Ponsky  /  Chike V. Chukwumah From Yeo CJ, et al: Shackelford’s Surgery of the Alimentary Tract, 7th edition (Saunders 2012)

FIGURE 40-2-1  Initial view of the epiglottis before passage of the endoscope into the posterior part of the pharynx.

FIGURE 40-2-2  Visualization of normal vocal cords during EGD is a vital part of a complete EGD.

e721

e722   SECTION 11  ■  ENDOSCOPY

FIGURE 40-2-3  Appearance of the proximal part of the stomach with the presence of numerous rugal folds. Several small fundic gland polyps are also seen.

FIGURE 40-2-4  As the endoscope is advanced into the distal end of the stomach at the juncture of the body and antrum, the rugal folds become less pronounced.

FIGURE 40-2-5  Normal-appearing distal antrum and pylorus.

CHAPTER 40-2  ■  Diagnostic and Therapeutic Endoscopy of the Stomach and Small Bowel  

FIGURE 40-2-6  Notched pylorus secondary to a previous inflammatory process.

FIGURE 40-2-7  Retroflex view in the stomach showing the antrum and fundus simultaneously, separated by the angularis.

e723

e724   SECTION 11  ■  ENDOSCOPY

FIGURE 40-2-8  Full retroflex view showing the endoscope as it traverses the esophagogastric junction.

FIGURE 40-2-9  The smooth surfaces of the duodenal bulb are visualized after advancing through the pylorus. No valvulae conniventes (folds) are present in the bulb.

FIGURE 40-2-10  Normal-appearing view of the second portion of the duodenum. This view is obtained by trolling back (pulling out) the endoscope while looking up and to the right; the endoscope is left in a “short” or lesser curve position.

DIAGNOSTIC UPPER ENDOSCOPY Jean Marc Canard  /  Jean-Christophe Létard  /  Anne Marie Lennon From Canard JM, et al: Gastrointestinal Endoscopy in Practice, 1st edition (Churchill Livingstone 2011)

40-3 

FIGURE 40-3-1  A, Retroflexed view of the fundus and cardia in patient with a hiatus hernia (arrow), B, post-Nissen’s fundoplication.

FIGURE 40-3-2  Familial adenomatous polyposis (FAP). A, Image of multiple fundic gland polyps in a patient with FAP. B, Duodenal adenoma in patient with FAP.

FIGURE 40-3-3  How to handle an upper endoscope. A, The left hand controls up/down (large wheel), B, right/left angulation (small wheel), C, insufflation, water, and suction buttons.

e725

e726   SECTION 11  ■  ENDOSCOPY

FIGURE 40-3-4  A, Right and B, left hand positioning for upper endoscopy. Hold the endoscope approximately 30 cm from its distal end with your right hand. Your left hand should control the up/down angulation.

FIGURE 40-3-5  Zenker’s diverticulum (white arrow). The true esophagus is highlighted with the black arrow.

CHAPTER 40-3  ■  Diagnostic Upper Endoscopy  

FIGURE 40-3-6  A-G, Steps to perform a complete examination of the upper GIT.

FIGURE 40-3-7  Esophagitis. A, LA grade B esophagitis. B, LA grade D esophagitis.

e727

e728   SECTION 11  ■  ENDOSCOPY

FIGURE 40-3-8  Peptic esophageal stricture associated with GERD.

FIGURE 40-3-9  Barrett’s esophagus. A, Long segment Barrett’s esophagus. B, C, Barrett’s esophagus seen under white light and following acetic acid application using narrow band imaging (NBI) to separate the red, blue and green light spectrum. (Courtesy of Dr Marcia Canto, Johns Hopkins Hospital, Baltimore.)

FIGURE 40-3-10  How to subdivide a Type 0 lesion. Examine the lesion and decide if it is protruding or non-protruding. Protruding lesions are either pedunculated (0–Ip) or sessile (0–Is) as documented on the right. If the lesion is non-polypoid, decide if it is slightly elevated (0–IIa), flat (0–IIb) or depressed (0–IIc). An excavated, or ulcerated lesion is classified as 0–III. (Modified from The Paris endoscopic classification of superficial neoplastic lesions. Paris Workshop Participants. Gastrointest Endosc 2003: 58(6); S5.)

CHAPTER 40-3  ■  Diagnostic Upper Endoscopy  

FIGURE 40-3-11  Classification of lesions with combined features. Lesions sometimes combine more than one endoscopic feature. These lesions are classified by whatever features are present. A, B, The lesion protrudes less than the closed biopsy forceps (Type 0–II), and has a slight depression (0–IIc) as well as an elevation (0–IIa). C, The lesion is depressed (Type 0–IIc) but also has an elevation within the depression (Type 0–IIa), the reverse is found in D, where an elevated lesion (Type 0–IIa) has a central depression (Type 0–IIc). E, A large excavated lesion (0–III) contains a central depressed zone (0–IIc), while in F, there is a central section which protrudes above the closed biopsy forceps (type 0–Is), associated with a depression on either side (Type 0–IIc) (m, mucosa; mm, muscularis mucosae; sm, submucosa). (Modified from the Paris Workshop Participants. The Paris Endoscopic classification of superficial neoplastic lesions. Gastrointest Endosc 58(6):S6 and S8, 2003.)

FIGURE 40-3-12  How to differentiate between Type 0–I and Type 0–II. Protruding lesions (Type 0–I), seen on the right, protrude higher than the cups of a closed biopsy forceps (2.5 mm). Non-protruding lesions (Type 0–II) can be differentiated from protruding (Type 0–I) as they protrude less than the height of a closed biopsy forceps (2.5 mm). The endoscopic appearance of the lesion is associated with the depth of penetration into the bowel wall (m, mucosa; mm, muscularis mucosae; sm, submucosa). (Modified from The Paris endoscopic classification of superficial neoplastic lesions. Paris Workshop Participants. Gastrointest Endosc 2003: 58(6); S6.)

e729

e730   SECTION 11  ■  ENDOSCOPY

FIGURE 40-3-13  Submucosal lesions. A, Carcinoid tumor. B, Granular cell tumor of the esophagus (Abrikossoff tumor). C, Lipoma. D, Ectopic pancreas (arrow). E, GIST.

SMALL CALIBER ENDOSCOPY From Ginsberg GG, et al: Clinical Gastrointestinal Endoscopy 2nd edition (Saunders 2011)

40-4 

The video for this procedure can be accessed here

e731

40-5 

NONVARICEAL UPPER GASTROINTESTINAL BLEEDING From Ginsberg GG, et al: Clinical Gastrointestinal Endoscopy 2nd edition (Saunders 2011)

The videos for this procedure can be accessed here

e732

PORTAL HYPERTENSIVE BLEEDING From Ginsberg GG, et al: Clinical Gastrointestinal Endoscopy 2nd edition (Saunders 2011)

40-6 

The videos for this procedure can be accessed here

e733

40-7 

SELF ASSESSMENT Kamran Idrees  /  John D. Christein  /  Kyle A. Perry  /  Jonathan A. Myers From Velasco J: Rush University Medical Center Review of Surgery, 5th edition (Saunders 2011)

1. Which of the following endoscopic ulcer characteristics has the highest risk for recurrent bleeding? A. Oozing ulcer B. Clean based ulcer C. Nonbleeding “visible vessel” D. Nonbleeding ulcer with an overlying clot E. Dieulafoy ulcer Ref.: 1 COMMENTS: Esophagogastroduodenoscopy is not only the diagnostic test of choice but can also be therapeutic in patients with upper gastrointestinal bleeding. EGD can localize the bleeding site and determine the risk for rebleeding based on the appearance of the ulcer bed. The endoscopic features of ulcers with a risk for rebleeding in decreasing order of frequency are active arterial bleeding (approaches 100%), nonbleeding “visible vessel” (≈50%), nonbleeding ulcer with an overlying clot (≈30% to 35%), oozing ulcer (≈10% to 27%), and a clean based ulcer (1.5 cm) of skin is removed from the medial edge, leaving a block of bare fatty subcutaneous tissue.3 This excess block of subcutaneous tissue is folded into the defect. The medial flap edge is sutured to the contralateral cavity edge by absorbable sutures (no. 2/0 polyglactin 910 [Ethicon, Edinburgh, UK]) placed in the dermis (Fig. 54-2-1). The lateral transverse defect that is formed when

FIGURE 54-2-1  Diagram of the V–Y advancement flap method.

e1019

e1020   SECTION 14  ■  SKIN AND SOFT TISSUE

FIGURE 54-2-2  Polypropylene all layers sutures are used for group AL (all layers).

FIGURE 54-2-3  In group SS (subcutaneous sutures), polyglactin 910 sutures were used to approximate the edges of the wound.

the island flap was moved medially is closed horizontally, thus giving the final scar a horizontal “Y” shape. The skin was closed by interrupted polypropylene sutures. In the second group (group AL), all layers, including the skin, of the open wound were closed by no. 1 polypropylene (Doğsan, Istanbul, Turkey) sutures (Fig. 54-2-2). In the third group (group SS), subcutaneous deep sutures (no. 2/0 polyglactin 910) were placed and the skin was closed separately (Fig. 54-2-3). No drains were placed in any of the 3 groups.

Follow-Up Patients were invited for observation on the 4th and 7th days. Sutures were removed at the 7th postoperative day and the surgical site was inspected for infection. Surgical site infection was recorded according to the hospital infection control practices advisory committee guideline.12 Also, early wound failure (wound dehiscence) without overt infection, i.e., draining pus, hyperemia, swelling, and inflammation, was assessed. Time to return to work or everyday activity for the unemployed was recorded. Patients were encouraged to visit at any time during follow-up if any problems occurred. They were routinely examined at the 1st and 6th postoperative months and yearly thereafter. During these visits, physical examinations were performed to identify any recurrence. Moreover, patient satisfaction, graded into 5 categories with the minimum score (0) being very dissatisfied and the maximum (4) being very satisfied, was also recorded. Patients were also asked if they were comfortable in performing everyday physical activities, would accept the same operation again if the need arose, and would suggest the same type of operation to an acquaintance.

CHAPTER 54-2  ■  Prospective Randomized Controlled Trial Comparing V–Y Advancement Flap  

Statistical Analysis Chi-square analysis and analysis of variance (ANOVA) were used for categorical variables and continuous variables, respectively, during assessment of possible differences between groups. Survival, i.e., time to recurrence between groups, was assessed by the Kaplan–Meier method with log-rank analysis to identify any significant difference. To identify the factors that correlated with recurrence, chi-square analysis and Fisher exact tests were used where appropriate for categorical variables. For assessing continuous variables, Student t test was used. A P value of III Ulceration SLN status

NS NS 0.03 30 to 35 cm H2O).32–35 Pathologically, this manifests as diffuse alveolar damage and is associated with cytokine release36 and bacterial translocation.37 In addition to being caused by simple overstretching of the lung, ventilator-induced lung injury (VILI) may have other determinants. Among these may be excessive tidal stretch (i.e., repetitive cycling of the lungs with tidal volumes larger than the normal 4–8 mL/kg ideal body weight)38 and a shear stress phenomenon that occurs when injured alveoli are repetitively opened and collapsed during the ventilatory cycle.22,35,39,40 VILI may also be worsened by increasing the frequency of excessive lung tidal stretch and from acceleration forces associated with rapid initial gas flow into the lung.41 VILI occurs clinically when low-resistance/high-compliance units receive a disproportionately high regional tidal volume in the setting of high alveolar distending pressures (see Figure 58-2-2). Concern about overdistention injury is the rationale for using “lung-protective” ventilator strategies that accept less than normal values for pH and O2 partial pressure in exchange for lower (and safer) distending pressures.

Cardiac Effects In addition to affecting ventilation and ventilation distribution, intrathoracic pressure changes resulting from positive-pressure ventilation can affect cardiovascular function.42 In general, as mean intrathoracic pressure is increased, right ventricular filling is decreased. This is the rationale for using volume repletion to maintain cardiac output in the setting of high intrathoracic pressure. Conversely, elevations in intrathoracic pressure can actually improve left ventricular function because of an effective reduction in afterload.43 Indeed, a sudden release of intrathoracic pressure (e.g., during a ventilator disconnect or spontaneous breathing trial) can sometimes precipitate flash pulmonary edema because of the acute increase in afterload coupled with increased venous return.44 Intrathoracic pressures can influence the distribution of perfusion. The relationship of alveolar pressures to perfusion pressures in the three-zone lung model can help explain this.45 Specifically, the supine human lung is generally in a zone 3 (distention) state. As intra-alveolar pressures rise, however,   units. Indeed, increases in dead space (i.e., zone 2 and zone 1 regions can appear, creating high V/Q zone 1 lung) can be a consequence of ventilatory strategies using high ventilatory pressures (e.g., IRV). Positive-pressure mechanical ventilation can affect other aspects of cardiovascular function. Specifically, dyspnea, anxiety, and discomfort from inadequate ventilatory support can lead to stress-related catechol release, with subsequent increases in myocardial O2 demands and risk of dysrhythmias. In addition, coronary blood vessel O2 delivery can be compromised by inadequate gas exchange from lung injury, coupled with low mixed venous O2 partial pressure due to high O2 consumption demands by the ventilatory muscles.

e1085

e1086   SECTION 15  ■  SURGICAL CRITICAL CARE

Patient-Ventilator Dyssynchrony As mentioned, patients can interact with all three phases of an assisted breath: trigger, target, and cycle. Patients dyssynchronous with any of these phases will have unnecessary loads placed on their respiratory muscles, thereby increasing the risk of muscle fatigue. Moreover, dyssynchronous interactions produce discomfort and a sense of dyspnea. When severe, patients are often noted to be “fighting the ventilator.” This leads to unnecessary sedation and a consequent prolongation of the need for ventilatory support.46

Intrinsic PEEP/Air Trapping The development of intrinsic PEEP can produce significant adverse events. In flow- and volumetargeted ventilation, all intrathoracic pressures are increased, which can lead to risk of VILI and reduction in cardiac filling. In pressure-targeted ventilation, buildup of intrinsic PEEP results in loss of tidal volume and minute ventilation. Intrinsic PEEP can also create a significant triggering load in patients, since inspiratory muscles must first overcome intrinsic PEEP before airway and circuit pressures and flows change sufficiently to initiate the assisted breath.47

Other Adverse Effects Oxygen concentrations approaching 100% are known to cause oxidant injury to airways and lung parenchyma.48 Many of the data supporting this concept, however, have come from animal studies, and animals and humans often have different O2 tolerances. It is unclear what the “safe” O2 concentration or duration of exposure is in sick humans. Most consensus groups have argued that Fio2 values less than 0.4 are safe for prolonged periods, and Fio2 values greater than 0.8 should be avoided if possible. Mechanically ventilated patients are at risk for pulmonary infections for several reasons.49,50 First, the natural protective mechanism of glottic closure is compromised by an endotracheal tube. This permits continuous seepage of oropharyngeal material into the airways. Second, the endotracheal tube itself impairs the cough reflex and serves as a potential portal for pathogens to enter the lungs. This is particularly important if the circuit is contaminated. Third, airway and parenchymal injury from both the underlying disease and management complications make the lung prone to infections. Fourth, the intensive care unit (ICU) environment itself, with its heavy antibiotic use and the presence of very sick patients in close proximity, poses a risk for a variety of infections. Preventing ventilator-associated pneumonias is critical because length of stay and mortality are heavily influenced by their development.49,50 Handwashing and carefully chosen antibiotic regimens for other infections can have important beneficial effects. Management strategies that avoid breaking the integrity of the circuit (e.g., circuit changes only when visibly contaminated) also appear to be helpful. Finally, continuous drainage of subglottic secretions may be a simple way of reducing lung contamination with oropharyngeal material.

APPLYING ASSIST-CONTROL MECHANICAL VENTILATION Tradeoffs To provide adequate support but minimize VILI, mechanical ventilation goals must involve tradeoffs. Specifically, the need for potentially injurious pressures, volumes, and supplemental O2 must be weighed against the benefits of gas exchange support. To this end, a rethinking of gas exchange goals has occurred over the last decade; pH goals as low as 7.15 to 7.20, and O2 partial-pressure goals as low as 55 mm Hg, are now considered acceptable if the lung can be protected from VILI.51,52 Ventilator settings are thus selected to provide at least this level of gas exchange support while at the same time meeting two mechanical goals: (1) provision of enough PEEP to enlist the recruitable alveoli and (2) avoidance of a PEEP–tidal volume combination that unnecessarily overdistends lung regions at end-inspiration. These goals embody the concept of a “lung-protective” mechanical ventilatory strategy, and these principles guide current recommendations for the specific management of parenchymal and obstructive lung disease.

Managing Parenchymal Lung Injury Parenchymal lung injury describes disease processes that involve the air spaces and interstitium of the lung. In general, parenchymal injury produces stiff lungs and reduced lung volumes.17 Functional

CHAPTER 58-2  ■  Mechanical Ventilation  

residual capacity is thus reduced, and the compliance curve is shifted to the right. It is important to realize, however, that in all but the most diffuse diseases (e.g., diffuse cardiogenic edema), there are often marked regional differences in the degree of inflammation present and thus the degree of mechanical abnormalities that exist. This heterogeneity can have a significant impact on the effects of a particular mechanical ventilation strategy. This is because delivered gases will preferentially go to the regions with the highest compliance and lowest resistance (i.e., the more normal regions) rather than to sicker regions with low compliance (see Figure 58-2-2). A “normal-sized” tidal volume may thus be distributed preferentially to the healthier regions, resulting in a much higher regional tidal volume and the potential for regional overdistention injury. Parenchymal injury can also affect the airways, especially the bronchioles and alveolar ducts.17 These narrowed and collapsible small airways can contribute to reduced regional ventilation to injured lung units. This can also lead to air trapping, and it may be a factor in subsequent cyst formation during the healing phase after lung injury. Gas exchange abnormalities in parenchymal lung injury are a consequence of alveolar flooding or   mismatching and shunts. collapse coupled with a maldistribution of ventilation that results in V/Q   = ∞) is not a major manifestation of parenchymal lung disease unless there Because dead space ( V/Q is severe or end-stage injury, hypoxemia tends to be a greater problem than CO2 clearance. Frequency–tidal volume settings for supporting a patient with parenchymal lung injury must focus on limiting end-inspiratory stretch. The importance of this limitation in improving outcome has been suggested by several recent clinical trials,53,54 but it was most convincingly demonstrated by the NIH ARDS Network trial, which showed a 10% absolute reduction in mortality with a ventilator strategy using a tidal volume calculated on ideal body weight of 6 mL/kg compared with 12 mL/kg.55 Because of this, initial tidal volume settings should start at 6 mL/kg ideal body weight. Moreover, strong consideration should be given to further reducing this setting if end-inspiratory plateau pressures, adjusted for any effects of excessive chest wall stiffness, exceed 30 cm H2O. Increases in tidal volume settings might be considered if there is marked patient discomfort or suboptimal gas exchange, provided the subsequent plateau pressures do not exceed 30 cm H2O. Respiratory rate settings are then adjusted to control pH. Unlike in obstructive diseases (see later), the potential for air trapping in parenchymal lung injury is low if the breathing frequency is less than 35 breaths per minute and may not develop even at frequencies exceeding 50 breaths per minute. The choice of pressure-targeted or volume-targeted breaths often depends more on clinician familiarity with the two modes than on important clinical differences between them. As noted earlier, both modes provide a comparable range of tidal volumes and inspiratory times. In general, pressure-targeted breaths are preferable when an absolute pressure limit is desired in the circuit or when patient effort is very active, with variable flow demands. In contrast, volume-targeted breaths are preferable when it is critical to maintain a certain level of minute ventilation. Setting the inspiratory time and the inspiratory-expiratory ratio in parenchymal injury involves several considerations. The normal ratio is roughly 1 : 2 to 1 : 4; such ratios produce the most comfort and are the usual initial ventilator setting. Assessment of the flow graphic should also be done to ensure that an adequate expiratory time is present to avoid air trapping. As noted earlier, inspiratoryexpiratory prolongation beyond the physiologic range of 1 : 1 (IRV) can be used as an alternative to   matching in severe respiratory failure.29,30 A variation on IRV is increasing PEEP to improve V/Q airway pressure release ventilation (also known as biphasic or bilevel ventilation).4 Airway pressure release ventilation incorporates the ability to spontaneously breathe during the long inflation period of a pressure-controlled breath – a feature that may enhance recruitment and comfort.4,56 IRV strategies are generally reserved for patients in whom the plateau pressure from the PEEP–tidal volume combination exceeds 30 cm H2O, and potentially toxic concentrations of Fio2 are being used without meeting arterial O2 saturation or O2 delivery goals. It must be emphasized, however, that although IRV strategies have physiologic appeal, good outcome studies supporting their use do not exist. There are both mechanical and gas exchange approaches to setting the PEEP-Fio2 combination to support oxygenation. Mechanical approaches often use either a static pressure-volume plot to set the PEEP–tidal volume combination between the upper and lower inflection points57 or step increases in PEEP to determine the PEEP level that gives the best compliance.58,59 A simpler mechanical approach involves analyzing the airway pressure waveform during a set constant flow breath (the “stress index”).60 If the pressure waveform shows a steady rise, this implies that no derecruitment or over­ distension is occurring during the breath. In contrast, if the pressure waveform is concave upward, it

e1087

e1088   SECTION 15  ■  SURGICAL CRITICAL CARE suggests overdistension is occurring; if the pressure waveform is concave downward, it implies derecruitment occurred during the previous exhalation. With any of these approaches, a recruitment maneuver could be used to recruit the maximal number of recruitable alveoli before setting the PEEP. Fio2 adjustments are then set as low as clinically acceptable. Because these mechanical approaches are time consuming and technically challenging, gas exchange criteria are often used to guide PEEP and Fio2 settings. These generally involve algorithms designed to provide adequate values for arterial partial pressure of O2 while minimizing Fio2 (see Table 582-1).61,62 Note that constructing a PEEP Fio2 algorithm is usually an empirical exercise in balancing arterial O2 saturation with Fio2 and depends on the clinician’s perception of the relative “toxicities” of high thoracic pressures, high Fio2, and low arterial O2 saturation. Of note, however, is that recent meta-analyses of three large trials comparing conservative versus aggressive PEEP-Fio2 tables (mean PEEP of 7–9 cm H2O versus mean PEEP of 14–16 cm H2O) suggested benefit to the more aggressive strategies in patients with more severe lung injury.62

Obstructive Airway Disease Respiratory failure from airflow obstruction is a direct consequence of increases in airway resistance. Airway narrowing and increased resistance lead to two important mechanical changes. First, the increased pressures required for airflow may overload ventilatory muscles, producing a “ventilatory pump failure,” with spontaneous minute ventilation inadequate for gas exchange. Second, the narrowed airways create regions in the lungs that cannot properly empty and return to their normal resting volume, and intrinsic PEEP is produced.14 These regions of overinflation create dead space and put inspiratory muscles at a substantial mechanical disadvantage, which further worsens muscle function. Overinflated   matching. Regions of air trapregions may also compress healthier regions of the lung, impairing V/Q ping and intrinsic PEEP also function as a threshold load to trigger mechanical breaths.47,63 Several gas exchange abnormalities can accompany worsening airflow obstruction. First, although there may be transient hyperventilation due to dyspnea in patients with asthma, worsening respiratory failure in those with obstructive lung disease is generally characterized by falling minute ventilation as respiratory muscles become fatigued in the face of airflow obstruction. The result of this clinical situation is termed hypercapnic respiratory failure. Second, as noted earlier, regional lung compression   mismatch, which results in progressive hypoxemia. Alveoand regional hypoventilation produce V/Q lar inflammation and flooding, however, are not characteristic features of respiratory failure due to pure airflow obstruction; thus, shunts are less of an issue than in parenchymal lung injury. Third, overdistended regions of the lungs, coupled with underlying emphysematous changes in some patients, result in capillary loss and increasing dead space. This wasted ventilation further compromises the inspiratory muscles’ ability to supply adequate ventilation for alveolar gas exchange. Emphysematous regions also have reduced recoil properties that can worsen air trapping. Fourth, hypoxemic pulmonary vasoconstriction, coupled with chronic pulmonary vascular changes in some airway diseases, overloads the right ventricle, further decreasing blood flow to the lung and making dead space worse. Setting the frequency–tidal volume pattern in obstructive lung disease involves many considerations that are similar to those in parenchymal lung injury. Specifically, tidal volumes should be sufficiently low (e.g., 6 mL/kg ideal body weight) to ensure that plateau pressure is less than 30 cm H2O. In obstructive disease, however, clinicians should be aware that high peak airway pressures, even in the presence of acceptable values for plateau pressure, may transiently subject regions of the lung to overdistention injury due to a pendelluft effect (see Figure 58-2-2). As with parenchymal lung injury, tidal volume reductions should be considered to meet plateau pressure goals. Tidal volume increases can be considered for comfort or gas exchange, provided plateau pressure values do not exceed 30 cm H2O. The set rate is used to control pH. Unlike parenchymal disease, however, the elevated airway resistance and often low recoil pressures of emphysema greatly increase the potential for air trapping, and this limits the range of breath rates available. The inspiratory-expiratory ratio in obstructive lung disease is generally set as low as possible to minimize the development of air trapping. For the same reason, approaches using IRV strategies are almost always contraindicated. Because alveolar recruitment is less of an issue in obstructive lung disease than in parenchymal lung injury, the PEEP-Fio2 steps in Table 58-2-1 should probably be shifted to emphasize Fio2 for oxygenation support. A specific role for PEEP in an obstructed patient occurs when intrinsic PEEP serves as an inspiratory threshold load on the patient’s attempting to trigger a breath. Under these conditions,

CHAPTER 58-2  ■  Mechanical Ventilation  

judicious application of circuit PEEP (up to 75% to 85% of intrinsic PEEP) can “balance” expiratory pressure throughout the ventilator circuitry to reduce this triggering load and facilitate the triggering process.47,63 In severe airflow obstruction, use of low-density helium can facilitate ventilator settings. Helium is available as 80 : 20, 70 : 30, or 60 : 40 helium-oxygen breathing gas mixtures and can both reduce patient inspiratory work and facilitate lung emptying (recall that driving pressure decreases and flow increases through a tube as gas density decreases).64 If using a helium-oxygen gas mixture, it must be remembered that many flow sensors must be recalibrated to account for the change in gas density.

Neuromuscular Respiratory Failure The risk of VILI is generally less in a patient with neuromuscular failure, because lung mechanics are often near normal, making regional overdistention less likely. More “generous” tidal volumes can thus be used to improve comfort, maintain recruitment, and prevent atelectasis. At the same time, however, maximal distending pressures should be monitored and kept as low as possible while still being compatible with the other goals noted earlier. Certainly, plateau pressure should always be kept well below 30 cm H2O. Low levels of PEEP are often beneficial in preventing derecruitment (atelectasis) in these patients, who are often supine and incapable of secretion clearance or spontaneous sigh breaths.

Recovering Respiratory Failure – The Ventilator Withdrawal Process Once the cause of respiratory failure stabilizes and begins to reverse, attention turns to the ventilator withdrawal process. Numerous evidence-based guidelines have focused on the pivotal role of spontaneous breathing trials (SBTs) in determining the need for continued mechanical ventilatory support.65 In patients failing SBTs, comfortable forms of interactive ventilatory support should be provided until the next attempt at an SBT. Although the pressure-support mode is often used for this purpose, pressure assist-control can also fill this role. When using pressure assist-control, the control rate is generally set quite low (or even to zero), and the inspiratory pressure is titrated to comfort. Like pressure support, this approach is patient triggered and pressure targeted but is time cycled as opposed to the flow cycling of pressure support.

CONCLUSION Mechanical ventilatory support is a critical component in the management of patients with respiratory failure. It must be remembered, however, that this technology is supportive, not therapeutic; it cannot cure lung injury. Indeed, the best we can hope for is to “buy time” by supporting gas exchange without harming the lungs. Assist-control ventilation is designed to provide substantial levels of respiratory support. The major goals of assist-control ventilation are to substantially unload ventilatory muscles, provide the bulk of required minute ventilation, and optimize ventilation-perfusion matching to ensure adequate oxygenation. Important complications include ventilator-induced lung injury, cardiac compromise, and patient discomfort. Applying assist-control ventilation requires tradeoffs as clinicians attempt to balance gas exchange needs with the risk of these complications. Future innovations cannot focus simply on physio­ logic endpoints. Rather, innovations need to show benefits in clinically relevant factors such as mortality, ventilator-free days, barotrauma, and costs. Only then can we properly assess the sometimes bewildering array of new approaches to this vital life-support technology. KEY POINTS 1. Ventilator breath delivery is characterized by the trigger, target, and cycle variables. 2. The interaction of a positive-pressure breath and respiratory system mechanics is summarized by the equation of motion: Driving pressure = (Flow ´ Resistance) + (Volume / System compliance) 3. The goal of assist-control ventilation is to provide adequate gas exchange while protecting the lung from overdistention and recruitment-derecruitment injury. 4. Assist-control ventilation in obstructive lung disease poses the additional risk of producing overdistention from air trapping. 5. High-frequency ventilation shows promise as a better lung-protective strategy in parenchymal lung injury.

e1089

e1090   SECTION 15  ■  SURGICAL CRITICAL CARE

References 1. Mushin M, Rendell-Baker W, Thompson PW, and Mapleson WW: Automatic Ventilation of the Lungs. Oxford: Blackwell, 1980 2. American Society for Testing and Materials: Standards specifications for ventilators intended for use in critical care. ASTM Standards 1991; 36: pp. 1123-1155 3. MacIntyre NR: Principles of mechanical ventilation. In Mason R, and Broaddus V (eds): Murray Nadel Textbook of Respiratory Medicine, 5. Philadelphia: Elsevier, 2010. 4. Habashi NM: Other approaches to open-lung ventilation: airway pressure release ventilation. Crit Care Med 2005; 33: pp. S228-S240 5. Branson RD, and MacIntyre NR: Dual control modes of mechanical ventilation. Respir Care 1996; 41: pp. 294-305 6. Lellouche F, and Brochard L: Advanced closed loops during mechanical ventilation (PAV, NAVA, ASV, SmartCare). Clinical Anaesthesiol 2009; 23: pp. 81-93 7. Mitrouska J, Xirouchaki N, Patakas D, Siafakas N, and Georgopoulos D: Effects of chemical feedback on respiratory motor and ventilatory output during different modes of assisted mechanical ventilation. Eur Respir J 1999; 13: pp. 873-882 8. Sinderby C, Navalesi P, Beck J, et al: Neural control of mechanical ventilation in respiratory failure. Nature Med 1999; 5: pp. 1433-1436 9. Sassoon CSH: Mechanical ventilator design and function: The trigger variable. Respir Care 1992; 37: pp. 1056-1069 10. Truwit JD, and Marini JJ: Evaluation of thoracic mechanics in the ventilated patient. Part I. Primary measurements. J Crit Care 1988; 3: pp. 133-150 11. Truwit JD, and Marini JJ: Evaluation of thoracic mechanics in the ventilated patient. Part II. Applied mechanics. J Crit Care 1988; 3: pp. 192-213 12. Ranieri VM, Brienza N, Santostasi S, et al: Impairment of lung and chest wall mechanics in patients with acute respiratory distress syndrome: role of abdominal distension. Am J Respir Crit Care Med 1997; 156: pp. 1082-1091 13. Prinianakis G, Kondili E, and Georgopoulos D: Patient-ventilator interaction: An overview. Respir Care Clin N Am 2005; 11: pp. 201-224 14. Marini JJ, and Crooke PS: A general mathematical model for respiratory dynamics relevant to the clinical setting. Am Rev Respir Dis 1993; 147: pp. 14-24 15. Macklen PT: Relationship between lung mechanics and ventilation distribution. Physiology 1973; 16: pp. 580-588 16. Milic-Emili J, Henderson JAN, Dolovich MB, et al: Regional distribution of inhaled gas in the lung. J Appl Physiol 1966; 21: pp. 749-759 17. Pratt PC: Pathology of the adult respiratory distress syndrome. In Thurlbeck WM, and Ael MR (eds): The Lung: Structure, Function and Disease. Baltimore: Williams & Wilkins, 1978. pp. 43-57 18. Caironi P, Cressoni M, Chiumello D, et al: Lung opening and closing during ventilation of acute respiratory distress syndrome. Am J Respir Crit Care Med 2010; 181: pp. 578-586 19. Gattinoni L, Caironi P, Cressoni M, et al: Lung recruitment in patients with the acute respiratory distress syndrome. New Engl J Med 2006; 354: pp. 1775-1786 20. Kacmarek RM, and Pierson DJ: AARC conference on positive end expiratory pressure. Respir Care 1988; 33: pp. 419-527 21. Gattinoni L, Pelosi P, Crotti S, et al: Effects of positive end expiratory pressure on regional distribution of tidal volume and recruitment in adult respiratory distress syndrome. Am J Respir Crit Care Med 1995; 151: pp. 1807-1814 22. Webb HH, and Tierney DF: Experimental pulmonary edema due to intermittent positive pressure ventilation with high inflation pressures: Protection by positive end-expiratory pressure. Am Rev Respir Dis 1974; 110: pp. 556-565 23. Wyszogrodski I, Kyei-Aboagye K, Taaeusch HW, and Avery ME: Surfactant inactivation by hyper ventilation: Conservation by end-expiratory pressure. J Appl Physiol 1975; 38: pp. 461-466 24. Grasso S, Stripoli T, De Michele M, et al: ARDSNet ventilatory protocol and alveolar hyperinflation: role of positive endexpiratory pressure. Am J Resp Crit Care Med 2007; 176: pp. 761-767 25. Crotti S, Mascheroni D, Caironi P, et al: Recruitment and derecruitment during acute respiratory failure. Am J Respir Crit Care Med 2001; 164: pp. 131-140 26. Rimensberger PC, Prisine G, Mullen BM, et al: Lung recruitment during small tidal volume ventilation allows minimal positive end expiratory pressure without augmenting lung injury. Crit Care Med 1999; 27: pp. 1940-1945 27. Lim SC, Adams AB, Simonson DA, et al: Intercomparison of recruitment maneuver efficacy in three models of acute lung injury. Crit Care Med 2004; 32: pp. 2371-2377 28. Pelosi P, Cadringher P, Bottino N, et al: Sigh in acute respiratory distress syndrome. Am J Respir Crit Care Med 1999; 159: pp. 872-880 29. Armstrong BW, and MacIntyre NR: Pressure controlled inverse ratio ventilation that avoids air trapping in ARDS. Crit Care Med 1995; 23: pp. 279-285 30. Cole AGH, Weller SF, and Sykes MD: Inverse ratio ventilation compared with PEEP in adult respiratory failure. Intensive Care Med 1984; 10: pp. 227-232 31. Kacmarek RM, Kirmse M, Nishimura M, Mang H, and Kimball WR: The effects of applied vs auto-PEEP on local lung unit pressure and volume in a four-unit lung model. Chest 1995; 108: pp. 1073-1079 32. Dreyfuss D, and Saumon G: Ventilator induced lung injury: Lessons from experimental studies. Am J Respir Crit Care Med 1998; 157: pp. 294-323 33. Dreyfuss D, Soler P, Bassett G, et al: High inflation pressure pulmonary edema. Am Rev Respir Dis 1988; 137: pp. 1159-1164 34. Muscedere JG, Mullen JB, Gan K, and Slutsky AS: Tidal ventilation at low airway pressures can augment lung injury. Am J Respir Crit Care Med 1994; 149: pp. 1327-1334

CHAPTER 58-2  ■  Mechanical Ventilation   35. Plotz FB, Slutsky AS, van Vught AJ, and Heijnen CJ: Ventilator-induced lung injury and multiple system organ failure: a critical review of facts and hypotheses. Intensive Care Med 2004; 30: pp. 1865-1872 36. Ranieri VM, Suter PM, Totorella C, et al: Effect of mechanical ventilation on inflammatory mediators in patients with acute respiratory distress syndrome. JAMA 1999; 282: pp. 54-61 37. Nahum A, Hoyt J, Schmitz L, et al: Effect of mechanical ventilation strategy on dissemination of intertracheally instilled. Crit Care Med 1997; 25: pp. 1733-1743 38. Hager DN, Krishnan JA, Hayden DL, and Brower RG: Tidal volume reduction in patients with acute lung injury when plateau pressures are not high. Am J Respir Crit Care Med 2005; 172: pp. 1241-1245 39. Benito S, and Lemaire F: Pulmonary pressure-volume relationship in acute respiratory distress syndrome in adults: Role of positive and expiratory pressure. J Crit Care 1990; 5: pp. 27-34 40. Gajic O, Lee J, Doerr CH, et al: Ventilator-induced cell wounding and repair in the intact lung. Am J Respir Crit Care Med 2003; 167: pp. 1057-1063 41. Rich BR, Reickert CA, Sawada S, et al: Effect of rate and inspiratory flow on ventilator induced lung injury. J Trauma 2000; 49: pp. 903-911 42. Pinsky MR, and Guimond JG: The effects of positive end-expiratory pressure on heart-lung interactions. J Crit Care 1991; 6: pp. 1-15 43. Marini JJ, Culver BH, and Butler J: Mechanical effect of lung inflation with positive pressure on cardiac function. Am Rev Respir Dis 1979; 124: pp. 382-386 44. Lemaire F, Teboul JL, Cinotti L, et al: Acute left ventricular dysfunction during unsuccessful weaning from mechanical ventilation. Anesthesiology 1988; 69: pp. 171-179 45. Hughes JM, Glazier JB, Maloney JE, and West JB: Effect of lung volume on the distribution of pulmonary blood flow in man. Respir Physiol 1968; 4: pp. 58-72 46. Thille AW, Rodriguez P, Cabello B, Lellouche F, and Brochard L: Patient-ventilator asynchrony during assisted mechanical ventilation. Int Care Med 2006; 32: pp. 1515-1522 47. MacIntyre NR, McConnell R, and Cheng KC: Applied PEEP reduces the inspiratory load of intrinsic PEEP during pressure support. Chest 1997; 1111: pp. 188-193 48. Jenkinson SG: Oxygen toxicity. New Horiz 1993; 1: pp. 504-511 49. Fagon J, Chastre J, Domart Y, et al: Nosocomial pneumonia in patients receiving continuous mechanical ventilation. Am Rev Respir Dis 1989; 139: pp. 877-884 50. Collard HR, Saint S, and Matthay MA: Prevention of ventilator-associated pneumonia: an evidence-based systematic review. Ann Intern Med 2003; 138: pp. 494-501 51. Hager DN, and Brower RG: Customizing lung-protective mechanical ventilation strategies. Crit Care Med 2006; 34: pp. 1554-1555 52. Slutsky AS: ACCP consensus conference: Mechanical ventilation. Chest 1993; 104: pp. 1833-1859 53. Amato MB, Barbas CSV, Medievos DM, et al: Effect of a protective ventilation strategy on mortality in ARDS. N Engl J Med 1998; 338: pp. 347-354 54. Villar J, Kacmarek R, Peres-Mendez L, et al: A high positive end expiratory pressure low tidal volume strategy improves outcome in persistent ARDS. Crit Care Med 2006; 34: pp. 1311-1318 55. NIH ARDS Network: Ventilation with lower tidal volumes as compared with traditional tidal volumes for acute lung injury and the acute respiratory distress syndrome. N Engl J Med 2000; 342: pp. 1301-1308 56. Myers T, and MacIntyre NR: Does airway pressure release ventilation offer important new advantages in mechanical ventilatory support? Resp Care 2007; 52: pp. 452-460 57. Putensen C, Bain M, and Hormann C: Selecting ventilator settings according to the variables derived from the quasi static pressure volume relationship in patients with acute lung injury. Anesth Analg 1993; 77: pp. 436-447 58. Suter PM, Fairley HB, and Isenberg MD: Optimal end expiratory pressure in patients with acute pulmonary failure. N Engl J Med 1975; 292: pp. 284-289 59. Caramez MP, Kacmarek RM, Helmy M, et al: A comparison of methods to identify open-lung PEEP. Intensive Care Med 2009; 35: pp. 740-747 60. Grasso S, Terragni P, Mascia L, et al: Airway pressure-time curve profile (stress index) detects tidal recruitment/hyperinflation in experimental acute lung injury. Crit Care Med 2004; 32: pp. 1018-1027 61. Phoenix SI, Paravastu S, Columb M, Vincent JL, and Nirmalan M: Does a higher positive end expiratory pressure decrease mortality in acute respiratory distress syndrome? A systematic review and meta-analysis. Anesthesiology 2009; 110: pp. 1098-2005 62. Briel M, Meade M, Mercat A, et al: Higher vs lower positive end-expiratory pressure in patients with acute lung injury and acute respiratory distress syndrome: systematic review and meta-analysis. JAMA 2010; 303: pp. 865-873 63. Reissmann HK, Ranieri VM, Goldberg P, and Gottfried SB: Continuous positive airway pressure facilitates spontaneous breathing in weaning chronic obstructive pulmonary disease patients by improving breathing pattern and gas exchange. Intensive Care Med 2000; 26: pp. 1764-1772 64. McConnell RR: Adjuncts to mechanical ventilation. In MacIntyre NR, and Branson RD (eds): Mechanical Ventilation. Philadelphia: WB Saunders, 2001. pp. 400-414 65. ACCP/AARC/SCCM Task Force: Evidence based guidelines for weaning and discontinuing mechanical ventilatory support. Also in Resp Care 2002; 47: pp. 20-35

e1091

58-3 

BASIC AIRWAY MANAGEMENT T. Barker  /  Y. Patel From Procedures Consult

The video for this procedure can be accessed here

e1092

OROTRACHEAL INTUBATION T.W. Thomsen  /  G.S. Setnik From Procedures Consult

58-4 

The video for this procedure can be accessed here

e1093

58-5 

SELF ASSESSMENT José M. Velasco  /  John Butsch  /  W. Christopher Croley  /  James A. Colombo  /  Chad E. Jacobs  /  Walter J. McCarthy  /  Crea Fusco From Velasco J: Rush University Medical Center Review of Surgery, 5th edition (Saunders 2011)

1. Regarding tracheal intubation in morbidly obese patients, which of the following statements is true? A. The body habitus of these patients (i.e., short, thick necks) makes intubation difficult. However, it does not compromise ventilation. B. The diagnosis of obstructive sleep apnea should not alter management of the airway. C. Hypoxemia following induction of anesthesia and during intubation is the result of diminished functional residual capacity. D. Awake intubation is contraindicated. E. All of the above. Ref.: 1 COMMENTS: Management of a morbidly obese patient’s airway can be difficult. The approach to intubation must take into consideration a number of factors. Such patients often have short thick necks, large tongues, limited mouth and neck mobility, and increased thoracic and abdominal pressure. For these reasons, both ventilation and intubation can be difficult. Patients with obstructive sleep apnea frequently have redundant soft tissue in the airway, which makes visualization of the vocal cords extremely difficult. In fact, obese patients with sleep apnea and abnormalities on airway examination should be considered for awake intubation. In experienced hands, awake fiberoptic intubation with prior topical application of local anesthetic and small amounts of sedation is ideal. If general anesthesia is induced and difficulty with intubation and ventilation ensues, obese patients can rapidly become desaturated and hypoxemic. This is due to both an increased rate of oxygen consumption and decreased functional residual capacity. Oxygenation with 100% oxygen before induction of anesthesia helps decrease the rate of desaturation but does not eliminate this risk.

ANSWER: C 2. During a tracheostomy, a flash is noted in the surgical field while using electrocautery. Which of the following is the correct sequence of steps in management of the patient? A. Extinguish the flames with saline solution or water; turn off all anesthetic gases, including O2; and then hyperventilate with 21% O2 through the endotracheal tube (ETT). B. Extinguish the flames with saline solution or water, turn off all anesthetic gases except O2, and then hyperventilate with 100% O2 through the ETT. C. Stop the ventilation; disconnect all anesthetic gas supply, including O2; extinguish the flames with saline solution or water; remove the ETT; ventilate the patient with a mask; and then reintubate. D. Extinguish the flames with saline solution or water; remove all draping immediately; stop the ventilation; disconnect all anesthetic gas supply, including O2; allow the patient to awaken; and then extubate. E. Stop the ventilation; disconnect all anesthetic gas supply, including O2; extinguish the flames with saline solution or water; and resume ventilation. Ref.: 2 e1094

CHAPTER 58-5  ■  Self Assessment  

COMMENTS: Airway fires occur most often during laser airway surgery but can take place in any O2-rich environment where igniting stimuli may exist. Any combustible material, including polyvinyl chloride tubing, surgical drapes, and human tissue, can ignite. Both the surgeon and the anesthesiologist should take the following steps simultaneously: stop all gas flow, including O2; extinguish the fire with water or saline solution; and remove the ETT or any foreign body present in the airway (e.g., bronchoscope or gauze). Mask ventilation is performed until the trachea is reintubated. Bronchoscopy is then performed to determine the extent of the airway damage and to remove any foreign bodies that may be present. The trachea should be left intubated for at least 24 hours after an airway fire, and humidified gases should be administered through the ETT or tracheostomy tube. The use of steroids is controversial and probably of no benefit.

ANSWER: C 3. Intubation of a spontaneously breathing but obtunded patient with a closed head injury is best accomplished by which of the following? A. Application of topical lidocaine to the nares, spontaneous ventilation, and “blind” nasal intubation B. Induction with thiopental, muscle relaxation with succinylcholine, and oral tracheal intubation C. Awake fiberoptic intubation D. Awake tracheostomy with local anesthesia E. Awake rigid laryngoscopy

Ref.: 2, 3, 4 COMMENTS: Securing an airway in a trauma patient can be difficult. Concurrent cervical injury should be suspected in a patient with a head injury. All airway management should be done while maintaining in-line axial cervical stabilization. All the methods listed are acceptable for intubating this patient, but the ideal method would attenuate increases in ICP via thiopental induction, which can decrease cerebral blood flow and the cerebral metabolic rate by 40% to 60%. Succinylcholine causes small, transient increases in ICP (approximately 4 mm Hg), but these increases are offset by the ICPreducing effect of thiopental. In addition, the muscle paralysis induced by succinylcholine prevents coughing, which can increase ICP by 50 to 70 mm Hg. Nasal intubation carries the risk of damage to the cribriform plate when pre-existing fractures are present, and epistaxis can cause airway compromise in an obtunded patient. Blind intubation under these circumstances is therefore less desirable than other methods of securing an artificial airway. Tracheostomy should be performed whenever airway distortion prevents prompt intubation by other methods. In some situations, tracheostomy is the appropriate initial approach to securing the airway.

ANSWER: B 4. Which of the following conditions is not usually associated with elevated dead space ventilation? A. 42-year-old female after MI with CHF and a CO of 1.5 L/min B. 28-year-old woman on partum day 1 with shortness of breath, a Pao2 of 60 mm Hg, and segmental clots bilaterally in the pulmonary arteries C. 52-year-old Hispanic immigrant with a long-standing ventricular septal defect and PA pressure of 80/52 mm Hg D. 22-year-old man after multiple gunshot wounds, massive transfusions, and a mean arterial to inspired oxygen ratio (PaO2/Fio2) of 180 E. 62-year-old woman smoker with the following ventilator settings: controlled mandatory ventilation (CMV) at a rate of 12 breaths/min, Fio2 of 60%, Vt of 600 mL, and PEEP of 5 cm H2O

Ref.: 5, 6

e1095

e1096   SECTION 15  ■  SURGICAL CRITICAL CARE COMMENTS: The most common causes of increased dead space in critically ill patients are decreased CO, PE, pulmonary hypertension, ARDS, and excessive PEEP, all of which directly cause decreased blood flow to the pulmonary vasculature. In dead space ventilation with a high   ) ratio, there is decreased blood flow to ventilated areas, which primarily ventilation/perfusion ( V/Q affects elimination of carbon dioxide. In ARDS, some areas of lung are perfused but not ventilated. Alveoli may be filled with secretions, exudate, blood, or edema, thereby increasing the shunt fraction. Other areas of the lung may be ventilated but not perfused, which accounts for the dead space ventilation. Positive end-expiratory pressure can cause dead space ventilation by decreasing CO and stenting alveoli open, which causes the surrounding capillaries to collapse and thereby decreases alveolar perfusion. Carbon dioxide production and the dead space–tidal volume ratio (VDS/VT) determine minute ventilation. The anatomic dead space includes the volume of the airways to the level of the bronchiole (150 mL). Dead space can also include alveoli that are well ventilated but poorly perfused. When combined, the anatomic and alveolar dead space constitutes the physiologic dead space, which is essentially the volume of gas moved during each tidal breath that does not participate in gas exchange.

ANSWER: E 5. A 17-year-old asthmatic girl is brought to the OR for ruptured ectopic pregnancy. Postoperatively on the floor, she is found to be profoundly dyspneic and in acute respiratory failure. She is intubated and transferred to the surgical ICU, where her ventilatory settings are AC mode, RR of 18 breaths/ min, Fio2 of 0.80, VT of 600 mL, and PEEP of 0 mm Hg. She was sedated and paralyzed for the intubation and is not breathing over the ventilator settings. After examining the patient and the flow pattern on the ventilator, changes in the ventilatory settings are made. Which change in ventilator setting would best limit intrinsic PEEP? A. Increase VT B. Decrease in the inspiratory flow rate C. Increase in PEEP D. Decreased RR E. Change from AC mode to SIMV Ref.: 7, 8 COMMENTS: Intrinsic positive end-expiratory pressure (commonly known as auto-PEEP) is a state at end exhalation in which there is incomplete gas emptying, which can elevate alveolar volume and pressure. It is the threshold pressure needed to be overcome to initiate inspiratory flow. Severe bronchospasm increases the expiratory time needed, and patients in status asthmaticus or severe chronic obstructive pulmonary disease (COPD) are at risk for intrinsic PEEP. If combined with narrowed airways, such as in asthma, and parenchymal noncompliance, the inspiratory work of breathing is increased. Therefore, there is an imbalance of respiratory muscle strength and work of breathing leading to respiratory failure. During mechanical ventilation, when the expiratory time is insufficient to allow full exhalation of a ventilator breath, expiratory flow is still occurring when the next ventilator breath is delivered. To best limit intrinsic PEEP, one can decrease the RR, thereby giving the patient more time to exhale between breaths. In addition, decreasing VT will allow minimal improvement. One should also limit the inspiratory time to leave more time in the respiratory cycle for exhalation. Avoidance of hyperinflation and overdistention at the expense of minute ventilation, otherwise known as permissive hypercapnia, is an important method of ventilatory management in asthmatics.

ANSWER: D 6. A family meeting is called for a 69-year-old man who was intubated 6 days earlier for pneumonia and respiratory distress. He is now awake, alert, and asking for the tube to come out. His family wants to know when and whether he will be extubated. Which of the following characteristics of this patient does not meet conventional weaning criteria?

CHAPTER 58-5  ■  Self Assessment  

A. Negative inspiratory force of −10 cm H2O B. A respiratory frequency/tidal volume (RF/Vt) ratio of 105 or less C. Correction of underlying pulmonary and nonpulmonary complications D. Pulse oximetry reading of 92% E. Vital capacity of 12 to 15 mL/kg and peak inspiratory pressure of less than 25 cm H2O Ref.: 9 COMMENTS: Many indices have been proposed to predict weaning outcome and success or failure of extubation. Most surgical patients (90%) are weaned from mechanical ventilation in less than 1 week. Conventional weaning criteria include (1) measurements of oxygenation with a pulse oximeter (best determined by arterial blood gas analysis, with an Sao2 >90% and any Fio2 usually being adequate for weaning) and (2) measurements of ventilation, such as an RR less than 24 breaths/min, Paco2 less than 50 mm Hg, peak inspiratory pressure below 30 cm H2O, Vt of at least 5 to 8 mL/kg, and a vital capacity double the Vt value. Failure to satisfy these conventional criteria is associated with unsuccessful weaning in as many as 63% of patients. The rapid, shallow breathing test (RF/Vt) is performed by having the patient breathe room air for 1 minute as quickly as possible. When RF/Vt is 105 or less, successful weaning occurs in 78% of patients, and when RF/Vt is less than 80, the success rate is 95%. Conversely, an RF/Vt value of 105 or higher is accompanied by a failure rate of 95%. Another method often described is the SOAP assessment: (1) ability to clear secretions, (2) adequate oxygenation (Pao2/Fio2 ratio >200 mm Hg, which requires an Fio2 of 0.4 to 0.5 and PEEP 5 million central venous catheters (in the IJ, SC, and FV) annually in the United States alone, with a mechanical complication rate of 5% to 19%. These complications may occur more often with less experienced operators, challenging patient anatomy (obesity, cachexia, distorted, tortuous or thrombosed vascular anatomy, congenital anomalies such as persistent left superior vena cava), compromised procedural settings (mechanical ventilation or emergency), and the presence of comorbidity (coagulopathy, emphysema). Central venous catheter mechanical complications include arterial puncture, hematoma, hemothorax, pneumothorax, arterial-venous fistula, venous air embolism, nerve injury, thoracic duct injury (left side only), intraluminal dissection, and puncture of the aorta. The most common complications of IJ vein cannulation are arterial puncture and hematoma. The most common complication of SC vein cannulation is pneumothorax. The incidence of mechanical complications increases sixfold when more than three attempts are made by the same operator. The use of ultrasound imaging before or during vascular cannulation greatly improves firstpass success and reduces complications. Practice recommendations for the use of ultrasound for vascular cannulation have emerged from numerous specialties, governmental agencies such as the National Institute for Health and Clinical Excellence and the Agency for Healthcare Research and Quality’s evidence report.

ULTRASOUND PRINCIPLES FOR NEEDLE-GUIDED CATHETER PLACEMENT Ultrasound modalities used for imaging vascular structures and surrounding anatomy include twodimensional (2D) ultrasound, Doppler color flow, and spectral Doppler interrogation. The operator must have an understanding of probe orientation, image display, the physics of ultrasound, and mechanisms of image generation and artifacts and be able to interpret 2D images of vascular lumens of interest and surrounding structures. The technique also requires the acquisition of the necessary handeye coordination to direct probe and needle manipulation according to the image display. The supplemental use of color flow Doppler to confirm presence and direction of blood flow requires an understanding of the mechanisms and limitations of Doppler color flow analysis and display. This skill set must then be paired with manual dexterity to perform the three-dimensional (3D) task of placing a catheter into the target vessel while using and interpreting 2D images. Two-dimensional images commonly display either the short axis (SAX) or long axis (LAX) of the target vessel, each with its advantage or disadvantage in terms of directing the cannulating needle at the correct entry angle and depth. Three-dimensional ultrasound may circumvent the spatial limitations of 2D imaging by providing simultaneous real-time SAX and LAX views along with volume perspective without altering transducer location, allowing simultaneous views of neck anatomy in three orthogonal planes. Detailed knowledge of vascular anatomy in the region of interest is similarly vital to both achieving success and avoiding complications from cannulation of incorrect vessels. Ultrasound probes used for vascular access vary in size and shape. Probes with smaller footprints are preferred in pediatric patients. Higher frequency probes (≥7 MHz) are preferred over lower frequency probes ( .05). A limitation of the needle guide is that the needle trajectory is limited to orthogonal orientations from the SAX imaging plane. Although helpful in limiting lateral diversion of the needle path, sometimes oblique angulation of the needle path may facilitate target vessel cannulation. In addition, there may be considerable costs associated with the use of needle guides. Depending on the manufacturer, they may cost as little as several dollars to >$100 each. Importantly, although the needle guide facilitated prompt cannulation with ultrasound in the novice operator, it offered no additional protection against arterial puncture. However, one in vitro simulation study has refuted these in vivo results. Arterial puncture during attempted venous cannulation with ultrasound generally occurs because of a misalignment between the needle and imaging screen. It may also occur as a result of a

e1127

e1128   SECTION 15  ■  SURGICAL CRITICAL CARE

FIGURE 60-2-2  Various needle guides, used to direct the needle at the center of the probe (and image) and at an appropriate angle and depth beneath the probe. IJV, IJ vein. (From Troianos CA. Intraoperative monitoring. In: Troianos CA, ed. Anesthesia for the Cardiac Patient. New York: Mosby; 2002.)

through-and-through puncture of the vein into a posteriorly positioned artery. The first scenario is due to improper direction of the needle, while the latter occurs because of a lack of needle depth control. Needle depth control is also an important consideration because the anatomy may change as the needle is advanced deeper within the site of vascular access. The ideal probe should have a guide that not only directs the needle to the center of the probe but also directs the needle at the appropriate angle beneath the probe (Figure 60-2-2). This type of guide compensates for the limitation of using 2D ultrasound to perform a 3D task of vascular access. The more experienced operator with a better understanding of these principles and better manual dexterity may find the needle guide cumbersome, choosing instead the “maneuverability” of a freehand technique. Although the routine use of a needle guide requires further study, novice operators are more likely to improve their first-pass success. Vascular structures can be imaged in SAX, LAX, or oblique orientation (Figures 60-2-3A, 60-2-3B, and 60-2-3C). The advantage of the SAX view is better visualization of surrounding structures and their relative positions to the needle. There is usually an artery in close anatomic proximity to most central veins. Identification of both vascular structures is paramount to avoid unintentional cannulation of the artery. In addition, it may be easier to direct the cannulating needle toward the target vessel and coincidentally away from surrounding structures when both are clearly imaged simultaneously. The advantage of the LAX view is better visualization of the needle throughout its course and depth of insertion, because more of the needle shaft and tip are imaged within the ultrasound image plane throughout its advancement, thereby avoiding insertion of the needle beyond the target vessel. A prospective, randomized observational study of emergency medicine residents evaluated whether the SAX or LAX ultrasound approach resulted in faster vascular access for novice ultrasound users. The SAX approach yielded a faster cannulation time compared with the LAX approach, and the novice operators perceived the SAX approach as easier to use than the LAX approach. The operator’s handeye coordination skill in aligning the ultrasound probe and needle is probably the most important variable influencing needle and target visibility. Imaging in the SAX view enables the simultaneous visualization of the needle shaft and adjacent structures, but this view does not image the entire needle pathway or provide an appreciation of insertion depth. Although novice users may find ultrasound guidance easier to adopt using SAX imaging, ultrasound guidance with LAX imaging should be promoted, because it enables visualization of the entire needle and depth of insertion, thereby considering

CHAPTER 60-2  ■  Guidelines for performing ultrasound-guided vascular cannulation  

e1129

FIGURE 60-2-3  Two-dimensional imaging of the right IJ vein (IJV) and CA from the head of the patient over their right shoulder. A, SAX, B, LAX, C, oblique axis. SAX imaging displays the lateral-right side of the patient on the right aspect of the display screen and the medial structures on the left aspect of the display screen. LAX imaging displays the caudad structures on the right aspect of the display screen and cephalad structures on the left aspect of the display screen. If the transducer is rotated counterclockwise about 30–40 degrees, oblique imaging displays more lateral-right caudad structures on the right aspect of the display screen, while more medial-left cephalad structures are on the left aspect of the display screen.

anatomic variations along the needle trajectory as the needle is advanced deeper within the site of vascular access. The oblique axis is another option that may allow better visualization of the needle shaft and tip and offers the safety of imaging surrounding structures in the same view, thus capitalizing on the strengths of both the SAX and LAX approaches.

REAL-TIME IMAGING VERSUS STATIC IMAGING Ultrasound guidance for vascular access is most effective when used in real time (during needle advancement) with a sterile technique that includes sterile gel and sterile probe covers. The needle is observed on the image display and simultaneously directed toward the target vessel, away from surrounding structures, and advanced to an appropriate depth. Static ultrasound imaging uses ultrasound imaging to identify the site of needle entry on the skin over the underlying vessel and offers the appeal of nonsterile imaging, which obviates the need for sterile probe coverings, sterile ultrasound gels, and needle guides. If ultrasound is used to mark the skin for subsequent cannulation without real-time (dynamic) use, ultrasound becomes a vessel locator technique that enhances external landmarks rather than a technique that guides the needle into the vessel. Both static and real-time ultrasound-guided approaches are superior to a traditional landmark-guided approach. Although the real-time ultrasound guidance outperforms the static skin-marking ultrasound approach, complication rates are similar. Venous puncture using real-time ultrasound was faster and required fewer needle passes among neonates and infants randomly assigned to real-time ultrasound-assisted IJ venous catheterization versus ultrasound-guided skin marking. Fewer than three attempts were made in 100% of patients in the real-time group, compared with 74% of patients in the skin-marking group (P < .01). In this study, a hematoma and an arterial puncture occurred in one patient each in the skin-marking group. One operator can usually perform real-time ultrasound-guided cannulation. The nondominant hand holds the ultrasound probe while the dominant hand controls the needle. Successful cannulation

e1130   SECTION 15  ■  SURGICAL CRITICAL CARE of the vessel is confirmed by direct vision of the needle entering the vessel and with blood entering the attached syringe during aspiration. The probe is set aside on the sterile field, the syringe removed, and the wire is inserted through the needle. Further confirmation of successful cannulation occurs with ultrasound visualization of the guide wire in the vessel. Difficult catheterization may benefit from a second person with sterile gloves and gown assisting the primary operator by either holding the transducer or passing the guide wire.

VESSEL IDENTIFICATION Morphologic and anatomic characteristics can be used to distinguish a vein from an artery with 2D ultrasound. For example, the IJ vein has an elliptical shape and is larger and more collapsible with modest external surface pressure than the carotid artery (CA), which has rounder shape, thicker wall, and smaller diameter (Figure 60-2-4). The IJ vein diameter varies depending on the position and fluid status of the patient. Patients should be placed in Trendelenburg position to increase the diameter of the jugular veins and reduce the risk for air embolism when cannulating the SC vein, unless this maneuver is contraindicated. A Valsalva maneuver will further augment their diameter and is particularly useful in hypovolemic patients. Adding Doppler, if available, can further distinguish whether the vessel is a vein or an artery. Color flow Doppler demonstrates pulsatile blood flow in an artery in either SAX or LAX orientation. A lower Nyquist scale is typically required to image lower velocity venous blood flows. At these reduced settings, venous blood flow is uniform in color and present during systole and diastole with laminar flow, whereas arterial blood flow will pulse and be detected predominantly during systole (Figure 60-2-5) in patients with unidirectional arterial flow (absence of aortic regurgitation). A small pulsed-wave Doppler sample volume within the vessel lumen displays a characteristic systolic flow within an artery, while at the same velocity range displays biphasic systolic and diastolic flow and reduced velocity in a vein. A lower pulsed-wave Doppler velocity range makes this distinction more apparent (Figure 60-2-6). Misidentification of the vessel with ultrasound is a common cause of unintentional arterial cannulation. Knowledge of the relative anatomic positions of the artery and vein in the particular location selected for cannulation is essential and is discussed below in the specific sections. Ultrasound images

FIGURE 60-2-4  Vessel identification. Right IJ vein (top) and CA (bottom) in SAX and LAX orientation. Slight external pressure compresses the oval-shaped vein but not the round-shaped artery.

CHAPTER 60-2  ■  Guidelines for performing ultrasound-guided vascular cannulation  

FIGURE 60-2-5  A-D, Vessel identification with color flow Doppler. Arterial flow is visible in systole only, irrespective of Nyquist limit. Venous flow is visible in systole and diastole, but only if the Nyquist limit is sufficiently decreased.

FIGURE 60-2-6  Vessel identification with pulsed-wave Doppler will distinguish artery A from vein B at a Nyquist limit of ±50 cm/sec. Arterial blood flow has a predominately systolic component and higher velocity A compared with venous blood flow B, C, which has systolic and diastolic components and much lower velocity, better delineated with a lower Nyquist scale (±9 cm/sec) C.

of veins and arteries have distinct characteristics. Veins are thin walled and compressible and may have respiratory-related changes in diameter. In contrast, arteries are thicker walled, not readily compressed by external pressure applied with the ultrasound probe, and pulsatile during normal cardiac physiologic conditions. Obviously, arterial pulsatility cannot be used to identify an artery during clinical conditions such as cardiopulmonary bypass, nonpulsatile ventricular circulatory assistance, and cardiac or circulatory arrest. Confirmation of correct catheter placement into the intended vascular structure is covered later.

e1131

e1132   SECTION 15  ■  SURGICAL CRITICAL CARE

INTERNAL JUGULAR VEIN CANNULATION Anatomic Considerations The IJ is classically described as exiting the external jugular foramen at the base of the skull posterior to the internal carotid and coursing toward an anterolateral position (in relation to the carotid) as it travels caudally. Textbook anatomy does not exist in all adult and pediatric patients. Denys and Uretsky showed that the IJ was located anterolateral to the CA in 92%, >1 cm lateral to the carotid in 1%, medial to the carotid in 2%, and outside of the path predicted by landmarks in 5.5% of patients. The anatomy of the IJ is sufficiently different among individual patients to complicate vascular access with a “blind” landmark method (Figure 60-2-7). Therefore at a minimum, a clear and intuitive advantage of using static ultrasound imaging for skin marking is the ability to identify patients in whom the landmark technique is not likely to be successful.

Cannulation Technique The traditional approach to IJ vein cannulation uses external anatomic structures to locate the vein. A common approach identifies a triangle subtended by the two heads of the sternocleidomastoid muscle and the clavicle (Figure 60-2-8). A needle placed at the apex of this triangle and directed toward the ipsilateral nipple should encounter the IJ 1.0 to 1.5 cm beneath the skin surface. The use of external landmarks to gain access to the central venous system is considered a safe technique in experienced hands. A failure rate of 7.0% to 19.4% is due partly to the inability of external landmarks to precisely correlate with the location of the vessel. Furthermore, when initial landmark-guided attempts are unsuccessful, successful cannulation diminishes to 75% overlap among 54% of all patients whose heads were rotated to the contralateral side (image plane positioned in the direction of the cannulating needle; Figure 60-2-9). Additionally, two thirds of older patients (age ≥ 60 years) had >75% overlap of the IJ and CA. Age was the only demographic factor that was associated with vessel overlap. The concern is that vessel overlap increases the likelihood of unintentional CA puncture by a through-and-through puncture of the vein. The accidental penetration of the posterior vessel wall can occur despite the use of ultrasound when the SAX imaging view is used for guidance. Typically, the anterior wall of the vein is compressed as the needle approaches the vein (Figure 60-2-10). The compressive effect terminates as the needle enters the vein (heralded by the aspiration of blood into the syringe) and the vessel assumes its normal shape. A low-pressure IJ may partially or completely compress during needle advancement, causing puncture of the anterior and posterior walls without blood aspiration into the syringe. IJ-CA overlap increases the possibility of unintentional arterial puncture as the “margin of safety” decreases. Some authors have described the “margin of safety” as the distance between the midpoint of the IJ and the lateral border of the CA. This zone represents the area of nonoverlap between the IJ and CA. The margin of safety decreases, and the percentage overlap increases from 29% to 42% to 72% as the head is turned to the contralateral side from 0° (neutral) to 45° to 90°, respectively. Vessel overlap increasing with head rotation is most apparent among patients with increased body surface areas (>1.87 m2) and increased body mass indexes (>25 kg/m2). Ultrasound can be used to alter the approach angle to avoid this mechanism of CA puncture by directing the advancing needle away from the CA (Figure 60-2-11). Vascular anomalies and anatomic variations of the IJ and surrounding tissues have been observed in up to 36% of patients. Ultrasound identifies the vein size and location, anomalies, and vessel patency, thus avoiding futile attempts in patients with absent or thrombosed veins and congenital anomalies such as persistent left superior vena cava. Denys et al observed small fixed IJs in 3% of patients. An ultrasound vein diameter

e1133

e1134   SECTION 15  ■  SURGICAL CRITICAL CARE

FIGURE 60-2-10  The anterior wall of the IJ vein (IJV) recesses as the needle approaches the vein (left). The vein assumes its normal shape after the needle penetrates its wall (right).

FIGURE 60-2-11  A-D, Under ultrasound guidance, the needle approach to the IJ vein (IJV) can be altered to avoid CA puncture. (From Cardiovasc Intervent Radiol.)

< 7 mm (cross-sectional area < 0.4 cm2) is associated with decreased cannulation success, prompting redirection to another access site, thus reducing cannulation time and patient discomfort. Ultrasound also identifies disparity in patency and size between the right IJ and the left IJ (the right IJ usually larger than the left IJ). Maneuvers that increase the size of the IJ and thus potentially improve the cannulation success include the Valsalva maneuver (Figure 60-2-12) and the Trendelenburg position.

Complications Several factors contribute to the success rate, risk, and complications associated with central venous cannulation, including patient characteristics, comorbidities, and access site. Although the landmark method is associated with an arterial puncture risk of 6.3% to 9.4% for the IJ, 3.1% to 4.9% for the SC, and 9.0% to 15.0% for the FV, Ruesch et al demonstrated a higher incidence of arterial puncture during attempted IJ versus SC central venous access. Obese patients with their attendant short thick necks and others with obscured external landmarks derive a particular benefit from ultrasound guidance by decreasing the incidence of arterial puncture, hematoma formation, and pneumothorax. The recognition and avoidance of pleural tissue during real-time ultrasound imaging could potentially decrease the risk for pneumothorax for approaches that involve a needle entry site closer to the clavicle.

CHAPTER 60-2  ■  Guidelines for performing ultrasound-guided vascular cannulation  

FIGURE 60-2-12  The size of the IJ vein (IJV) is increased with a Valsalva maneuver B compared with apnea A.

High-risk conditions include hemostasis disorders, uncooperative or unconscious patients, critically ill patients who may be hypovolemic, and patients who have had multiple previous catheter insertions. Oguzkurt et al prospectively reviewed 220 temporary IJ dialysis catheters placed sonographically by interventional radiologists in 171 high-risk patients (27.7% with bleeding tendency, 10% uncooperative, 2% obese, 37% with previous catheters, and 21.3% with bedside procedure because their medical conditions were not suitable for transport to the radiology suite). The success rate was 100%, with only seven complications among the 171 procedures. The carotid puncture rate was 1.8%, while oozing around the catheter, small hematoma formation, and pleural puncture without pneumothorax occurred at rates of 1.4%, 0.4%, and 0.4%, respectively. In summary, ultrasound imaging of the IJ and surrounding anatomy during central venous cannulation both facilitates identification of the vein and improves first-pass cannulation but also decreases the incidence of injury to adjacent arterial vessels.

Recommendation for IJ Vein Cannulation It is recommended that properly trained clinicians use real-time ultrasound during IJ cannulation whenever possible to improve cannulation success and reduce the incidence of complications associated with the insertion of large-bore catheters. This recommendation is based on category A, level 1 evidence. The writing committee recognizes that static ultrasound (when not used in real time) is useful for the identification of vessel anatomy by skin-marking the optimal entry site for vascular access and for the identification of vessel thrombosis and is superior to a landmark-guided technique.

SUBCLAVIAN VEIN CANNULATION Anatomic Considerations Landmark-guided SC vein access uses the anatomic landmarks of the midpoint of the clavicle, the junction between the middle and medial border of the clavicle, and the lateral aspect of a tubercle palpable on the medial part of the clavicle. The most common approach is to insert the needle 1 cm inferior to the junction of the middle and medial third of the clavicle at the deltopectoral groove. The degree of lateral displacement of the entrance point is based on the patient’s history and anatomic considerations.

Cannulation Technique The landmark-guided approach to the central venous circulation via the SC vein is generally considered by many clinicians to be the simplest method to access this vein. Several million SC vein catheters are placed each year in the United States. The risk factors for complications and failures are poorly understood, with the exception of physician experience. Advantages of using the SC vein for central venous access include consistent surface anatomic landmarks and vein location, patient comfort, and lower potential for infection. In contrast to attempted IJ vein cannulation, in which unintentional

e1135

e1136   SECTION 15  ■  SURGICAL CRITICAL CARE injury to the adjacent CA can compromise circulation to the brain, unintentional injury to the adjacent SC artery during SC vein cannulation carries a less morbid sequela. The physician’s experience and comfort level with the procedure are the main determinants for successful placement of a SC vein catheter, when there are no other patient-related factors that increase the incidence of complications. The SC vein may be cannulated from a supraclavicular or an infraclavicular approach. The infraclavicular approach is the most common approach and hence is the focus of this discussion. The supraclavicular approach (without ultrasound) has largely been abandoned because of a high incidence of pneumothorax. As experience with ultrasound-guided regional anesthesia for upper extremity blocks has increased imaging and identification of the supraclavicular vessels and nerves, clinicians are gaining more familiarity with imaging the supraclavicular approach to the SC vein using ultrasound for vessel cannulation. Whether this approach will continue to gain popularity remains to be demonstrated. Tau et al analyzed anatomic sections of the clavicle and SC vein and determined that the supine position with neutral shoulder position and slight retraction of the shoulders was the most effective method to align the vein for a landmark-based technique. Although many clinicians place patients in the Trendelenburg (head-down) position to distend the central venous circulation, there is less vessel distention of the SC vein than the IJ vein because the SC vein is fixed within the surrounding tissue, so relative changes in size are not realized to the same degree as with the IJ vein. Thus, the primary reason for the Trendelenburg position is to reduce the risk for air embolism in spontaneously breathing patients. Ultrasound-directed vascular cannulation may lead inexperienced operators to use needle angle approaches that lead to an increased risk for complications. It is important that traditional approaches and techniques are not abandoned with ultrasound guidance, particularly during cannulation of the SC vein, in which a steeper needle entry angle may lead to pleural puncture. The needle is directed toward the sternal notch in the coronal plane. The bevel of the needle should be directed anteriorly during insertion and gentle aspiration applied with a syringe, as the needle enters the skin at a very low (nearly parallel) angle to the chest wall. An increased or steeper angle increases the likelihood of creating a pneumothorax. The needle bevel may be turned caudally upon venopuncture to direct the guide wire toward the right atrium. The wire is advanced, leaving enough wire outside the skin for advancement of the entire catheter length over the wire (i.e., the wire should extend beyond the catheter outside the skin). The electrocardiogram should be closely monitored for ectopy that may occur when the wire is advanced into the right atrium or right ventricle. Chest radiography is mandatory not only to confirm proper line placement but also to rule out pneumothorax. Similar preparation of the patient occurs with ultrasound-guided cannulation as with the landmarkguided approach with respect to positioning, skin preparation, and vascular access kits. The use of a smaller footprint transducer probe for SC vein access for realtime ultrasound imaging is recommended because larger probes make imaging of the vein more challenging. It is generally more difficult to position the larger footprint probe between the clavicle and rib to obtain an adequate SC vein image. Despite some loss of resolution in the far field that inherently occurs with phased-array transducers, smaller probes may allow better maneuverability underneath the clavicle. Similar to the landmark technique, the middle third of the clavicle is chosen as the site used for ultrasound imaging and subsequent needle insertion. The transducer is oriented to image the SC vein in the SAX view with a coronal imaging plane. The vein appears as an echo-lucent structure beneath the clavicle (Figure 602-13). It is important to distinguish between pulsatility on the vein due to respiratory variation and pulsatility of the artery. Confirmation of the venous circulation can be facilitated by the injection of agitated saline “echo contrast” into a vein of the ipsilateral arm (if available) with subsequent imaging of the microbubbles in the vein. Confirmation can also be achieved by addition of color flow Doppler to the ultrasound assessment. When positioning the transducer marker toward the left shoulder (during right SC vein cannulation), arterial flow will be the color that indicates flow away from the transducer, while venous flow will be the color that indicates flow toward the transducer. It is important to ensure correct transducer orientation before using color flow Doppler to determine the identification of artery or vein. Considerable skin pressure is required to obtain adequate imaging planes (windows) that may incur some patient discomfort. A prospective randomized SC vein cannulation study favored the ultrasound-guided over the landmark-guided approach, with a higher success rate (92% vs 44%), fewer minor complications (1 vs 11), and fewer venopunctures (1.4 vs 2.5) and catheter kits (1.0 vs 1.4) per attempted cannulation. A more recent study of 1,250 attempted central venous catheter placements included 354 SC vein

CHAPTER 60-2  ■  Guidelines for performing ultrasound-guided vascular cannulation  

FIGURE 60-2-13  Two-dimensional ultrasound image of the left SC vein and left SC artery obtained from the left side of the patient during ultrasound-guided cannulation of the left SC vein.

attempts. The incidence and success rates with ultrasound guidance during central venous catheter placement supported the impression of many clinicians that the added benefit of ultrasound for cannulation of the SC vein is less than the benefit of ultrasound during attempted cannulation of the IJ vein. Although ultrasound use was uncommon for cannulation of the SC vein, either as the primary technique or as a rescue technique, success rates were high with ultrasound guidance even when surface techniques were unsuccessful.

Complications Complication rates for the landmark-guided approach to SC vein cannulation are 0.3% to 12% and include pneumothorax, hematoma, arterial puncture, hemothorax, air embolism, dysrhythmia, atrial wall puncture from the wire, lost guide wire, anaphylaxis in patients who are allergic to antibiotics upon the insertion of an antibiotic-impregnated catheter, catheter malposition, catheter in the wrong vessel, and thoracic duct laceration (left side only). Kilbourne et al reported the most common errors during failed SC vein catheter placement attempts by resident physicians were inadequate landmark identification, improper insertion position, advancing the needle through periosteum, a shallow or cephalad needle angle, and loss of intravenous needle position while attempting to place the guide wire. Factors associated with cannulation failure were previous major surgery, radiation therapy, prior catheterization, prior attempts at catheterization, high body mass index, more than two needle passes, only 1 year of postgraduate training, lack of classic anatomy, and previous first rib or clavicle fracture. If only one needle pass was attempted, the failure rate for subsequent catheter placement was 1.6%, compared with 10.2% for two passes and 43.2% for three or more passes. In the 8.7% of patients in whom initial attempts at catheterization failed, subsequent attempts by second physicians were successful in 92%, with a complication rate of 8%. Similar success was demonstrated in a study of patients undergoing SC vein cannulation with and without ultrasound guidance. The ultrasound group had fewer attempts, better patient compliance, and a zero incidence of pneumothorax, while the incidence of pneumothorax in the landmark group was 4.8%. Identification of risk factors before catheter insertion may decrease complication rates by altering the approach to include ultrasound guidance. Additionally, in patients with body mass indexes > 30 kg/ m2 or < 20 kg/m2, history of previous catheterization, prior surgery, or radiotherapy at the site of venous access, experienced physicians should attempt catheter placement rather than physicians who are learning the procedure. Obese patients with their attendant short thick necks and others with obscured external landmarks derive a particular benefit from ultrasound guidance by decreasing the incidence of arterial puncture, hematoma formation, and pneumothorax. Mansfield et al noted that a body mass index > 30 kg/m2 resulted in a cannulation failure rate of 20.1% for attempted SC vein cannulation. These investigators found no benefit of ultrasound guidance for SC vein catheterization, but in comparing ultrasound with

e1137

e1138   SECTION 15  ■  SURGICAL CRITICAL CARE landmark-guided techniques, Hind et al’s meta-analysis found that the landmark technique had a higher relative risk for failed catheter placements and mean time to successful cannulation. As operators gain more experience with the use of ultrasound for guiding catheterization and diagnostic procedures, it is likely that an incremental benefit with the use of ultrasound for SC vein cannulation will also be realized. Orihashi et al found a benefit to the use of ultrasound in SC venopuncture in a small cohort of 18 patients. Although Gualtieri et al demonstrated improved success and fewer minor complications with use of ultrasound for SC vein cannulation, there were no major complications in either group. The overwhelming evidence in the literature supports the routine use of ultrasound for IJ access, but the data on the SC approach warrant consideration of anatomic landmarks and interference of the clavicle as impediments to the use of real-time ultrasound for this approach.

Recommendation for SC Vein Cannulation Current literature does not support the routine use of ultrasound for uncomplicated patients undergoing SC vein cannulation. Individual operators should not attempt cannulation more than twice, as the incidence of complication, particularly pneumothorax, rises significantly with additional attempts. High-risk patients may benefit from ultrasound screening of the SC vein before attempted cannulation to identify vessel location and patency and to specifically identify thrombus before attempted cannulation. The recommendation for ultrasound guidance during SC vein cannulation is based on category A (supportive), level 3 evidence.

FEMORAL VEIN CANNULATION Anatomic Considerations The femoral vessels are often used to provide access for left-sided and right-sided cardiac procedures. In addition, the common FV is often used for central venous access during emergency situations, because of its relative safe and accessible location with predictable anatomic landmarks (i.e., lying within the femoral triangle in the inguinal-femoral region). A detailed understanding of the regional anatomy is important for performing FV cannulation using a landmark-guided technique. The common femoral artery and FV lie within the femoral triangle in the inguinal-femoral region. The superior border of this triangle is formed by the inguinal ligament, the medial border by the adductor longus muscle, and the lateral border by the sartorius muscle. Another important landmark is the femoral artery pulse, because the common FV typically lies medial to the common femoral artery within the femoral sheath. The femoral artery lies at the midpoint of the inguinal ligament connecting the anterior superior iliac spine to the pubic tubercle, while the common FV is typically located medial to the common femoral artery. This side-by-side relationship of the common femoral artery and FV occurs in close proximity to the inguinal ligament, but significant vessel overlap may occur, particularly in children. In addition, it is essential to understand that the relationship between the inguinal crease and the inguinal ligament is highly variable, so the inguinal crease is not always a useful surface landmark. The femoral site has numerous advantages both with elective vascular access and in critically ill patients. The femoral site remains the most commonly accepted site for vascular access for cardiac procedures because of relatively short access times and few complications. For critically ill patients, it is relatively free of other monitoring and airway access devices, allowing arm and neck movement without impeding the access line. Femoral access avoids the risks of hemothorax and pneumothorax, which is particularly important in patients with severe coagulopathy or profound respiratory failure. In addition, the femoral site permits cannulation attempts without interruption of cardiopulmonary resuscitation during cardiac arrest. However, the femoral approach is associated with complications, including bleeding and vascular injury, such as pseudoaneurysms, arteriovenous fistulas and retroperitoneal bleeding (see section “Complications”).

Cannulation Technique Similar to other central venous cannulation sites, the modified Seldinger technique is the most common method used to access the common FV. The procedure requires patient positioning with the hip either in the neutral position or with slight hip abduction and external rotation. Abduction and

CHAPTER 60-2  ■  Guidelines for performing ultrasound-guided vascular cannulation  

external rotation increases the accessibility of the common FV from 70% to 83% in adults and increases the vessel diameter in children compared with a straight-leg approach. The reverse Trendelenburg position increases common FV cross-sectional area by >50%. The surface landmarks are identified and the FV located by palpating the point of maximal femoral artery pulsation 1 to 2 cm below the midpoint of the inguinal ligament. The FV is located by inserting a needle 1 cm medial to the maximal pulsation, directed cephalad and medially at a 45° angle to the skin. Many clinicians advocate the use of a small (25-gauge) exploratory or “finder” needle to initially identify the vein location. A larger 20-gauge to 22-gauge needle is subsequently placed directly adjacent to the finder needle along a parallel path to the FV. The vein is normally 2 to 4 cm beneath the skin in most adults.

Complications There are a number of complications associated with FV cannulation. Infection remains one of the most common problems with femoral catheters because of their close proximity to the perineal region, which is the reason that this site is not typically recommended for long-term catheters. Some investigators, who have demonstrated that the incidence of catheter-related blood-stream infection with femoral catheters is not significantly different from the incidence with the supraclavicular access sites, dispute this risk. The number of attempts to gain access may increase the risk for infection but seems to be primarily related to the duration of catheter use at the site. The complications of FV cannulation directly related to catheter insertion technique are most often due to unintentional femoral artery puncture. Because of the close proximity to the common femoral artery, arterial puncture may occur if the needle is directed too laterally. This may result in hematoma, retroperitoneal bleeding, pseudoaneurysm, and arteriovenous fistula formation. In addition, thrombus may develop within the FV or iliac vein because of the presence of the catheter or during compression upon removal. If the needle is directed too laterally, the patient may experience paresthesia with the potential for femoral nerve injury. Other rare but serious complications include bowel penetration and bladder puncture. Complications occur despite the optimal use of surface landmark-guided techniques (Figure 602-14). Ultrasound imaging at the femoral site has demonstrated that surface anatomic landmarks are less useful in projecting the underlying anatomy, although surface anatomy is more reliable when the

FIGURE 60-2-14  Femoral vascular anatomy illustrating that the femoral nerve is lateral, while the FV is medial to the femoral artery; top of the figure is cephalad.

e1139

e1140   SECTION 15  ■  SURGICAL CRITICAL CARE cannulation site is closer to the inguinal ligament. Ultrasound-guided femoral artery and FV cannulation most likely reduces the incidence of complications because the anatomy is better defined. Iwashima et al and Seto et al demonstrated reduction in vascular-related complications due to inadvertent femoral artery or FV puncture with the use of ultrasound guidance during femoral vessel cannulation.

Recommendation for FV Cannulation The scientific evidence for real-time ultrasound-guided FV cannulation is category C, level 2: equivocal with insufficient scientific evidence to support a recommendation for routine use. In addition, complications during FV cannulation are less severe than those that occur with SC and IJ vein cannulation. It is therefore the recommendation of this writing committee that real-time ultrasound be used only for examining the FV to identify vessel overlap and patency when feasible.

PEDIATRIC ULTRASOUND GUIDANCE The United Kingdom’s National Institute for Health and Clinical Excellence guidelines recommend real-time use of ultrasound during central vein cannulation in all patients, children and adults. Data to support this practice in pediatrics are limited. In a meta-analysis that included pediatric studies, Hind et al confirmed a higher success rate with 2D ultrasound compared with anatomic landmark techniques for the IJ vein cannulation among infants. Hosokawa et al demonstrated in a randomized trial of 60 neonates weighing 10.7 Cardiac output (CO) is measured by thermodilution. Traditionally, a cooled saline bolus was injected proximally, and the temperature of the bolus was measured by a thermistor at the catheter tip. CO is calculated by determining the area under the curve and plotting the integral. An average of three to five injections were used to minimize variability. Newer catheters measure CO continuously by integrating a thermal coil, which gently heats the proximal blood, measures the temperature drop at the catheter tip and uses a similar method to calculate CO. In either method, intracardiac shunt and tricuspid regurgitation will lead to unpredictable error in cardiac output measurement. At least one set of authors suggest that greater reliability can be had by calculating CO through plotting of the Fick curve and extrapolating CO.

Pressure, Volume, and Work Measures While modern PAC monitoring units or ICU nurses provide continuous data readouts of hemodynamic parameters, it remains essential to have a grounded understanding of the physiologic principles and calculations involved in PAC monitoring. Actual measured parameters begin with CVP. CVP is directly transduced from the proximal or CVP port of the catheter. Without obstruction between the right atrium and the vena cava, CVP is analogous to right atrial pressure (RAP), which is again analogous to right ventricle end-diastolic pressure (RVEDP) and right ventricle end-diastolic volume (RVEDV). Hence, CVP ~ RAP ~ RVEDP ~ RVEDV . CVP is a direct measured extrapolation of RVEDV or cardiac preload. Just as CVP is a measure of right heart filling, PCWP measures left-sided pressure and volume. As described previously when

e1269

e1270   SECTION 15  ■  SURGICAL CRITICAL CARE the catheter is “wedged” in the pulmonary artery, there is a continuous fluid column between the PA and LV. Hence, PCWP ~ LAP ~ LVEDP ~ LVEDV . Cardiac output is also measured directly through the previously mentioned thermodilution. CO is converted to a more normalized cardiac index (CI) by dividing CO by body surface area (BSA). BSA is derived from height and weight nomograms, but can also be easily calculated as follows: BSA (m 2 ) = {Ht (cm ) + wt (kg ) − 60} 100 ( IB). Once CO is known, stroke volume (SV) can be calculated: SV = CO HR , where HR is heart rate. Right-sided ejection fraction (RVEF) is then calculated as follows: RVEF = SV RVEDV . Calculation of RVEDV requires a continuous thermistor, which is capable of dividing CI into systolic and diastolic components. Knowing RVEF allows RVEDV to be calculated: RVEDV = SV RVEF . Right-sided heart work estimates the work of the right heart as it pumps blood through the lungs back to the left ventricle. The right ventricle stroke work index (RVSWI) is estimated as follows: RVSWI = (PAP − CVP ) × SVI × 0.0136, where PAP is pulmonary artery pressure, and SVI is stroke volume index. The left heart volume and work are calculated similarly to the right. The left ventricular work is the work of the blood circulating through the systemic circulation. This is estimated by LVSWI = ( MAP − PCWP ) × SVI × 0.0136, where LVSWI is left ventricle stroke work index, and MAP is mean arterial pressure. Among the most useful calculated measures provided by the PAC is the pulmonary and systemic vascular resistance. Each estimates the resistance across the pulmonary or systemic circulation and is an extrapolation of vascular tone. Systemic vascular resistance (SVR) is the drop in pressure across the systemic circulation multiplied by the flow and is calculated as SVR = ( MAP − RAP ) × 80 CI , where RAP is renal arterial pressure. Pulmonary vascular resistance (PVR) is the drop in pressure across the pulmonary circulation multiplied by right-sided flow: PVR = (PAP − PCWP ) × 80 CI .

Goal-Directed Therapy Using Pulmonary Artery Catheter There is little disagreement regarding the large amount of data provided by a properly placed and interpreted PA catheter. Whether the data provided result in better patient outcome is the source of considerable debate. Much clinical research has been done in an attempt to determine which PACderived physiologic parameters should be measured and which are predictive of outcome. Shoemaker and his group provided the initial work toward defining and grouping patient physiologic response to trauma.8 As early as 1973 Shoemaker and colleagues showed in several studies that PAWP measured reduced cardiac output. They further showed that decreased oxygen delivery along with increased peripheral vascular resistance characterized non-survivors after trauma.9 Based on these and other similar study results, many groups targeted resuscitation of severely injured trauma patients to achieve physiologic values retrospectively associated with survival. Ten years after his initial studies, Shoemaker and colleagues showed an overall survival benefit in patients who achieved normal physiologic values.10,11 Finally, this group showed that there was less oxygen debt as measured by VO2 in surviving patients.12 Interestingly, there was no difference in oxygen consumption between the groups. In similar research,

CHAPTER 67-2  ■  The Pulmonary Artery Catheter and the Meaning of its Readings  

Bishop et al. examined physiologic parameters in 90 trauma patients and found that patients who achieved higher levels of CI, oxygen delivery, and oxygen consumption within the first 24 hours after admission had a lower incidence of ARDS and reduced overall mortality.13 Based on these and similar findings, several groups then randomized patients to supernormal resuscitation versus traditional resuscitation with the intent of proving that invasive monitoring of aggressive resuscitation would allow achievement of supranormal physiology and better outcome. These studies showed mixed results. Bishop’s group randomized patients to be supranormally resuscitated to a cardiac index of more than 4.5, DO2I greater than 670, and a VO2I greater than 166, versus traditional resuscitation as defined by achievement of normal urine output and CVP.14 The supranormally resuscitated group showed significantly lower mortality and decreased organ failure. Interestingly, optimal parameters were reached in 70% of the study group and 29% of the control group. Similarly, Flemming et al. randomized trauma patients with blood loss of more than 2000 cc to supernormal CI, DO2, and VO2, and found that attainment of these goals resulted in statistically significant decreases in organ failure, ICU stay, and ventilator days, and a trend toward decreased mortality.15 Other studies, however, pointed out that attainment of resuscitation was more important than the means by which it was achieved and many more recent studies have been unable to show similar benefit to monitoring or goal-directed resuscitation. Velmahos et al. randomized severely injured patients to supranormal resuscitation versus traditional hemodynamic monitoring. While there was no difference in mortality, organ failure, sepsis, ICU days, or hospital stay between the two groups, they found that 70% of the supranormal group and 40% of the traditional group achieved supranormal physiologic parameters, and that attainment of these optimal values was associated with better outcome independent of treatment protocol. There were no outcome differences between the two groups. What was perhaps most interesting in this study, however, was the finding that nonresponders (those who failed to reach the set goals despite volume and ionotropic manipulation) fared significantly worse than the control group. This suggests the likelihood that aggressive resuscitation toward supranormal physiology will be harmful in those whose physiology cannot attain it, and is in fact exhausted by such attempts. Other groups have reported similar results, which indicate that the ability to achieve normal parameters is associated with the mortality benefit, and is ultimately more important than whatever treatment was designed to achieve them.16 Taken together, these results cast doubt regarding the usefulness of interventional monitoring and resuscitation. Indeed, many believe that aggressive resuscitation should be instituted in all patients without any need for invasive monitoring. Others argue for limited supraphysiologic resuscitation. Some data point to poorer outcome from PAC use. Indeed, deleterious consequences of aggressive resuscitation and the PAC have been reported. Hayes et al. reported that increasing oxygen delivery with ionotropic augmentation with dobutamine was associated with increased mortality.17 Others have reported similar increased mortality in supranormally resuscitated patients. At least one group has purported to show that the PAC is itself a predictor of morbidity independent of protocol-driven resuscitation. Rhodes et al. performed a prospective randomized trial of 200 patients where no protocol-driven resuscitation was instituted.18 In this study, patients were randomized to PAC placement versus no PAC where patients were resuscitated based on the clinical judgment of the ICU staff. Data were obtained prospectively and examined retrospectively, and revealed that the PAC group received significantly more fluid in the first 24 hours and exhibited increased morbidity. Evidence for the deleterious effects of PAC protocol–driven resuscitation were seen in a study by Hayes et al. that showed an increased mortality in PAC-treated patients (54% vs. 34%).17 In this study there was a rigorous goal-directed attempt to achieve supranormal physiologic parameters. To achieve these goals, aggressive fluid resuscitation and ionotropic support were used. In an attempt to make sense of the huge amount of data regarding PACs and endpoints of resuscitation, Kern and Shoemaker performed a meta-analysis on 21 randomized clinical trials and found that early hemodynamic optimization achieved significant mortality reduction only if optimized parameters were achieved prior to the onset of sepsis or organ dysfunction.19 Heyland and associates analyzed seven studies and found no significant reductions in mortality achieved with optimization.20 Lastly, an analysis by Poeze et al. showed that physiologic optimization resulted in decreased mortality. In this analysis the entire benefit was from patients who were hemodynamically optimized perioperatively. Patients with established sepsis or organ failure prior to treatment were unlikely to benefit from “optimization.”21 Others have questioned the accuracy of the actual PAC measurements. Epstein et al. studied the accuracy of VO2 measurements by the calculated reverse Fick method, versus direct measurement by

e1271

e1272   SECTION 15  ■  SURGICAL CRITICAL CARE indirect calorimetry in an attempt to determine the accuracy of PAC-determined oxygen consumption measures. Their study showed a bias of 41 ml/min/m2, indicative of an undermeasurement of VO2 by PAC measures.22 Finally, Luchette and colleagues studied the accuracy of continuous cardiac output measurements and found that there was higher signal-to-noise ratio and degraded accuracy when patient temperature was above 38.5° C, resulting in further questioning of the accuracy and usefulness of PAC measurements in severely injured or septic patients.23

Mixed Venous Saturation: Monitoring Tissue Metabolism While the results of supranormal resuscitation are mixed, many believe that the best use of a PAC is as a measure of tissue perfusion. PAC sampling of mixed venous saturation should serve as an adjunctive measure of tissue-level perfusion. Even if supranormal resuscitation remains controversial with mixed data, surely Svo2 should provide useful data? Unfortunately, this measure of tissue perfusion, while seemingly useful as a measure of tissue oxygenation and subsequent outcome, has also ultimately not been shown to be useful as the targeted endpoint of resuscitation. Gattinoini and associates published a randomized prospective trial involving 762 patients across 56 ICUs who were randomized to either traditional resuscitation or targeting of CI > 4.5 or SvO2 > 70%. There was no difference in mortality or organ dysfunction among the three groups. Like the studies discussed above, subgroup analysis showed that patients who achieved hemodynamic targets had similar mortality rates independent of the group to which they were randomized.

Right Ventricle End-Diastolic Pressure as Measure of Cardiac Index and Cardiac Function As predicted by the Starling curve, end-diastolic volume is the best predictor of preload. Unfortunately, PAC pressure measurements as discussed previously are confounded in the setting of changing intrinsic ventricular compliance and extrinsic PEEP. RVEDV monitoring provides the first actual measurement of preload. Rather than being a pressure surrogate for volume, RVEDV utilizes a PAC with a fast thermistor to give a directly measured volume measure. In an attempt to find an alternative to PAOP as a measure of cardiac function, Chetham et al. showed that CI correlated better with RVEDVI than PAOP.24 The same group then studied 64 patients with respiratory failure and high PEEP requirements, and suggested that RVEDVI was a better measure of CI than PAOP at all levels of PEEP. CI was inversely correlated with PCWP at PEEP levels over 15. Taken together, these findings caused this group to conclude that RVEDVI should be used in lieu of PAOP as a measure of cardiac function. Despite initial reservations that mathematical coupling was responsible for the close correlation between RVEDV and CI, several groups have shown that the directly measured volume measures are superior to extrapolations from pressure (CVP and PAWP) measures. Chang and colleagues showed that splanchnic malperfusion was better predicted by RVEDV than PCWP, which was in turn associated with MODS and mortality.25 Other correlations among RVEDV, tissue perfusion, and outcome have been similarly reported by several groups,26 Lastly Kincaid et al.27 presented data to show that optimal RVEDV could be calculated for each individual patient. Taken together, these data support the use of RVEDV as a superior measure of cardiac function and goal-directed resuscitation.

CONCLUSIONS: USE THE PULMONARY   ARTERY CATHETER WISELY Since its inception, the PAC has been a data tool of the critical care physician. As can be seen from a careful review of the literature, little prospective, conclusive data exist to support its continued use. At the same time, however, there is a recent literature and clinical support for early goal-driven resuscitation of critically ill patients. Trauma patients are often hyper-protocolized based on their mechanism and injuries. The PAC is also subject to extreme difficulties in proper measurement and interpretation of collected data, and it is impossible to know the precise accuracy of collected data and how well they were interpreted in the many PAC and resuscitation trials. In the midst of controversy, however, it is important to remember that the PAC is not a drug or intervention, and its utility therefore cannot be directly assessed. The potential effectiveness of the PAC depends on a number of other factors, many of which are not measured or assessed in clinical studies. These include proper

CHAPTER 67-2  ■  The Pulmonary Artery Catheter and the Meaning of its Readings  

indications and risk assessment, proper placement and risk reduction measures, knowledgeable use and interpretation, and the contextual relevance and utility of the data obtained from the PAC. Because of these confounders and the powerful data that can be procured from measurement, we advocate continued selected and diligent use of the PAC as a tool for proper resuscitation in the critically injured and ill patient. While the proper resuscitative parameters and goals may not be absolutely known, and may change from minute to minute in our patients, more frequent and better data collection properly filtered through clinical judgment should only benefit patient care. To be used correctly, however, perfect attention to proper and safe placement and use must be assumed. From there, continued data collection and monitoring corroborated with patient state is also essential. It is not enough to intermittently examine a few select physiologic variables. Indeed, at any given moment a patient’s physiology and subsequent predicted outcome are the sum of a huge number of measured and unmeasured variables and the interactions between these patterns of variables. As technology becomes sufficiently advanced to allow for pattern recognition of more than a few variables, the utility of each measure should increase and be revealed. To discount the important measures derived from the PAC based on studies that failed to find utility of these independent measures is probably shortsighted, and ignores the utility of these otherwise unobtainable data. Indeed, the continued use of the PAC despite multiple studies reflects the clinician’s belief that the PAC data combined with other monitoring and clinical acumen will be beneficial on a patient-to-patient basis, which might not be measured in a large trial. That said, use of the PAC requires continual attention, monitoring, and intervention, and we believe that much of the confusion and negative data regarding PA catheterization results from a once-a-day glance at the Swan numbers (usually without regard to their accuracy) without careful perusal of the ongoing flux of the patient’s state. Discounting the PAC is foolish. It is a tool that can only be made useful and better through physician education and diligence.

References 1. Chen YY, et al: Comparison between replacement at 4 days and 7 days of the infection rate for pulmonary artery catheters in an intensive care unit. Crit Care Med 31:1353–1358, 2003. 2. Blot F, et al: Mechanisms and risk factors for infection of pulmonary artery catheters and introducer sheaths in cancer patients admitted to an intensive care unit. J Hosp Infect 48:289–297, 2001. 3. Kac G, et al: Colonization and infection of pulmonary artery catheter in cardiac surgery patients: epidemiology and multivariate analysis of risk factors. Crit Care Med 29:971–975, 2001. 4. Rello J, Coll P, Net A, Prats G: Infection of pulmonary artery catheters. Epidemiologic characteristics and multivariate analysis of risk factors. Chest 103:132–136, 1993. 5. Horowitz HW, Dworkin BM, Savino JA, Byrne DW, Pecora NA: Central catheter-related infections: comparison of pulmonary artery catheters and triple lumen catheters for the delivery of hyperalimentation in a critical care setting. J Parenter Enteral Nutr 14:588–592, 1990. 6. Eyer S, Brummitt C, Crossley K, Siegel R, Cerra F: Catheter-related sepsis: prospective, randomized study of three methods of long-term catheter maintenance. Crit Care Med 18:1073–1079, 1990. 7. Marino PL: The ICU Book. Baltimore, Williams and Wilkins, 1998. 8. Shoemaker WC, Reinhard JM: Tissue perfusion defects in shock and trauma states. Surg Gynecol Obstet 137:980–986, 1973. 9. Shoemaker WC, Montgomery ES, Kaplan E, Elwyn DH: Physiologic patterns in surviving and nonsurviving shock patients. Use of sequential cardiorespiratory variables in defining criteria for therapeutic goals and early warning of death. Arch Surg 106:630–636, 1973. 10. Shoemaker WC, Hopkins JA: Clinical aspects of resuscitation with and without an algorithm: relative importance of various decisions. Crit Care Med 11:630–639, 1983. 11. Shoemaker WC, Appel P, Bland R: Use of physiologic monitoring to predict outcome and to assist in clinical decisions in critically ill postoperative patients. Am J Surg 146:43–50, 1983. 12. Shoemaker WC, Appel PL, Kram HB, Waxman K, Lee TS: Prospective trial of supranormal values of survivors as therapeutic goals in high-risk surgical patients. Chest 94:1176–1186, 1988. 13. Bishop MH, et al: Relationship between supranormal circulatory values, time delays, and outcome in severely traumatized patients. Crit Care Med 21:56–63, 1993. 14. Bishop MH, et al: Prospective, randomized trial of survivor values of cardiac index, oxygen delivery, and oxygen consumption as resuscitation endpoints in severe trauma. J Trauma 38:780–787, 1995. 15. Fleming A, et al: Prospective trial of supranormal values as goals of resuscitation in severe trauma. Arch Surg 127:1175– 1179, discussion 1179–1181, 1992. 16. McKinley BA, et al: Normal versus supranormal oxygen delivery goals in shock resuscitation: the response is the same. J Trauma 53:825–832, 2002. 17. Hayes MA, et al: Elevation of systemic oxygen delivery in the treatment of critically ill patients. N Engl J Med 330:1717– 1722, 1994. 18. Rhodes A, Cusack RJ, Newman PJ, Grounds RM, Bennett ED: A randomised, controlled trial of the pulmonary artery catheter in critically ill patients. Intensive Care Med 28:256–264, 2002.

e1273

e1274   SECTION 15  ■  SURGICAL CRITICAL CARE 19. Kern JW, Shoemaker WC: Meta-analysis of hemodynamic optimization in high-risk patients. Crit Care Med 30:1686– 1692, 2002. 20. Heyland DK, Cook DJ, King D, Kernerman P, Brun-Buisson C: Maximizing oxygen delivery in critically ill patients: a methodologic appraisal of the evidence. Crit Care Med 24:517–524, 1996. 21. Poeze M, Solberg BC, Greve JW, Ramsay G: Monitoring global volumerelated hemodynamic or regional variables after initial resuscitation: What is a better predictor of outcome in critically ill septic patients? Crit Care Med 33:2494–2500, 2005. 22. Epstein CD, Peerless JR, Martin JE, Malangoni MA: Comparison of methods of measurements of oxygen consumption in mechanically ventilated patients with multiple trauma: the Fick method versus indirect calorimetry. Crit Care Med 28:1363–1369, 2000. 23. Luchette FA, et al: Effects of body temperature on accuracy of continuous cardiac output measurements. J Invest Surg 13:147–152, 2000. 24. Cheatham ML, Nelson LD, Chang MC, Safcsak K: Right ventricular end-diastolic volume index as a predictor of preload status in patients on positive end-expiratory pressure. Crit Care Med 26:1801–1806, 1998. 25. Chang MC, Meredith JW: Cardiac preload, splanchnic perfusion, and their relationship during resuscitation in trauma patients. J Trauma 42: 577–582, discussion 582–584, 1997. 26. Miller PR, Meredith JW, Chang MC: Randomized, prospective comparison of increased preload versus inotropes in the resuscitation of trauma patients: effects on cardiopulmonary function and visceral perfusion. J Trauma 44:107–113, 1998. 27. Kincaid EH, Meredith JW, Chang MC: Determining optimal cardiac preload during resuscitation using measurements of ventricular compliance. J Trauma 50:665–669, 2001.

PULMONARY ARTERY CATHETERIZATION L.A. Fleisher  /  R. Gaiser From Procedures Consult

67-3 

The video for this procedure can be accessed here

e1275

67-4 

SELF ASSESSMENT Crea Fusco  /  José M. Velasco From Velasco J: Rush University Medical Center Review of Surgery, 5th edition (Saunders 2011)

1. A 70-kg, 72-year-old man known to suffer from congestive heart failure (CHF), arthritis, diabetes mellitus, and a first-degree heart block is intubated in the ICU on postoperative day 2 after exploratory laparotomy for perforated sigmoid diverticulitis. His urine output has dropped to 10 mL/h for the last shift, and he is hypotensive despite several fluid boluses. A PA catheter is placed through the right internal jugular vein with some difficulty. As the line is advanced to 50 cm, the patient has a 14-beat run of ventricular tachycardia, which resolves when the catheter is pulled back. It is finally advanced to 62 cm and the balloon is inflated with 3 cc of air by the resident. As the line is being secured, a large amount of blood is noted in the endotracheal tube and the patient becomes hypotensive. Select the best intervention for this patient: A. Place external pacing wires and administer lidocaine to treat the ventricular tachycardia. B. Place a double-lumen endotracheal tube and occlude the appropriate bronchus with a Fogarty catheter. C. Pull the PA catheter back 2 cm with the balloon inflated. D. Suction the endotracheal tube while deflating the balloon by 2 cc of air. E. Obtain a chest radiograph to confirm correct placement of the line. Ref.: 2, 3 COMMENTS: The indications for pulmonary artery catheters and their value in patients with sepsis or hemodynamic instability are uncertain, but they may be useful in the management of patients unresponsive to the use of fluids and vasoactive agents. Dysrhythmias occur in 12% to 67% of patients undergoing catheterization but are usually self-limited, premature ventricular contractions. Complete heart block can develop in patients with pre-existing left bundle branch block. A prophylactic pacing wire should be used in these patients. Prophylactic lidocaine and full inflation of the balloon may prevent ventricular ectopy. Hemoptysis in patients with a PA catheter suggests the diagnosis of perforation or rupture. Mechanisms involved in PA rupture include (1) overinflation of the balloon, (2) incomplete balloon inflation ( 0.5 : 1 • Pleural fluid lactate dehydrogenase (LDH)–to–serum LDH ratio > 0.6 : 1 • Pleural fluid LDH greater than two thirds the upper limit of normal for serum LDH (a cutoff value of 200 IU/L was used previously) Pleural fluid is classified as an exudate if it meets any of the aforementioned criteria. Conversely, if all three characteristics are absent, the fluid is classified as a transudate. These researchers achieved a diagnostic sensitivity of 99% and specificity of 98% for classification of an exudate.

management decisions for complicated effusions because it can distinguish empyema from lung abscess, detect pleural masses, outline loculated fluid collections, and identify additional problems such as contusions, blebs, and infiltrates.5

Pleural Fluid Analysis Ideally, evaluation of a pleural effusion should begin with diagnostic thoracentesis and proceed to classification of the pleural fluid as either a transudate or an exudate. The currently accepted benchmark for classifying pleural fluid, developed by Light et al.,6 is shown in Box 68-1-1. A number of later studies used modifications of the Light criteria but had poorer diagnostic accuracy.7 Normal pleural fluid pH has been estimated to be approximately 7.64. A pH below 7.30 suggests the presence of an inflammatory or infiltrative process,8 such as parapneumonic effusion, empyema, malignancy, connective tissue disease, tuberculosis, or esophageal rupture. According to the current American College of Chest Physicians consensus statement on the treatment of parapneumonic effusions, pH is the preferred pleural fluid chemistry test for classifying the category of a parapneumonic effusion for subsequent management (Box 68-1-2).9,10 Additional testing considerations for pleural fluid include cholesterol, glucose, amylase, and adenosine deaminase.7,9,10 RED FLAGS Failure to establish a working differential diagnosis or identify the underlying cause Failure to initiate prompt source management for an underlying infectious cause Failure to identify and manage tension physiology caused by large effusions on an emergency basis

TREATMENT Acute medical management of pleural effusions is based on both therapeutic and diagnostic considerations. In the emergency department (ED), therapeutic thoracentesis is indicated for relief of acute

e1281

e1282   SECTION 15  ■  SURGICAL CRITICAL CARE Box 68-1-2  FLUID ANALYSIS OF PLEURAL EFFUSIONS Exudates Protein content > 3 g/dL High lactate dehydrogenase (LDH) content Pleural fluid–to–serum LDH ratio > 0.6 : 1 Pleural fluid–to–serum protein ratio > 0.5 : 1 Differential Clues Gross blood in pleural fluid suggests tumor, trauma, or infarction. Pleural fluid amylase elevation is associated with pancreatic disease, esophageal rupture, or malignancy. Pleural fluid pH is normally higher than 7.30; a pH lower than 7.2 suggests an infectious process such as empyema. Consider pulmonary emboli as the cause of loculated pleural effusions, particularly if the pleural fluid predominantly contains lymphocytes.

TABLE 68-1-1  Management of Patients with Parapneumonic Effusions Pleural Anatomy Minimal effusion: 10 mm but < 1 2 the hemithorax on lateral decubitus radiograph No loculations Large effusion: > 1 2 the hemithorax or associated loculations or pleural thickening Other

Pleural Fluid Microbiology

Pleural Fluid Chemical Analysis

Perform Drainage?

Unknown culture and Gram stain results Negative culture and Gram stain results

Unknown pH

No

pH > 7.20

No

Positive culture or Gram stain results

pH < 7.20

Yes

Purulent

pH < 7.0

Yes

respiratory or cardiovascular distress. Diagnostic thoracentesis should be used in the ED to diagnose immediately life-threatening conditions in toxic-appearing patients. Circumstances outside these situations should not necessitate emergency thoracentesis, but appropriate monitoring and further medical management are essential. Table 68-1-1 summarizes findings dictating the appropriateness of intervention.9

Approach to Unilateral Pleural Effusion Thoracentesis should be performed for new and unexplained pleural effusions when sufficient fluid is present to allow a safe procedure. Conventional wisdom holds that if a 10-mm layer of fluid is visible on a radiograph, sufficient fluid is present for thoracentesis to be successful. Treatment of the underlying disorder is generally all that is required for effusions caused by renal, cardiac, or rheumatologic diseases.

Approach to Pleural Effusion Associated with Malignancy Malignancy-induced pleural effusions should undergo therapeutic thoracentesis as needed. If effusions are undiagnosed, cytologic evaluation is required for patients with oncologic risks. Completion of evaluation may require CT assessment and subsequent tissue biopsy.

Approach to Parapneumonic Pleural Effusions Parapneumonic effusion and empyema are treated initially with empiric antibiotics according to the patient’s age and the probable organisms and sensitivities commonly present in the community. Parapneumonic effusion usually progresses through three stages. The exudative stage is associated with capillary leak during the first 3 days; the fibrinopurulent stage is associated with bacterial invasion of the pleura between days 3 and 7; and the organizational stage is characterized by fibroblast growth, occurring for 2 to 3 weeks, if the effusion is not treated properly. Lack of early diagnosis and drainage

CHAPTER 68-1  ■  Pleural Effusion  

of empyema, especially in younger children, may worsen the course of disease. In a hospitalized patient with a complicated parapneumonic effusion, antibiotics are administered intravenously and a thoracostomy tube is left in place until the patient is afebrile and improving clinically. Oral antibiotics are frequently continued for weeks after these procedures.9,11

PRIORITY ACTIONS Identify the cause of the pleural effusion. Therapeutic thoracentesis is recommended for symptomatic patients. Assess respiratory status before and after any intervention. Inform patients about the complications associated with pleural effusions and thoracentesis.

THORACENTESIS: THE IDEAL APPROACH Thoracentesis is an elective procedure requiring informed consent. Sterile technique and procedural experience lower the incidence of complications. Drainage of a pleural effusion is indicated for the following reasons: Diagnostic fluid or cellular analysis Therapeutic relief of symptomatic dyspnea Evaluation of complicated parapneumonic effusions or empyema

Positioning Ideally for thoracentesis, the patient should sit on the edge of the bed, lean forward slightly, and rest on an adjustable table. If the patient cannot sit up because of hemodynamic status, mental status, or the presence of tubes and indwelling lines, thoracentesis can be performed with the patient supine. In this case, the patient should turn onto the side with the effusion and move to position the back at the edge of the bed.

Procedural Approach Diagnostic thoracentesis is used to determine the cause of a pleural effusion, and therapeutic thoracentesis is performed to relieve symptoms of respiratory distress. Therapeutic thoracentesis may be repeated if indicated, but more definitive therapy such as sclerosis is usually needed to treat recurrent symptomatic pleural effusions. If more than 1.0 to 1.5 L of fluid is removed at one time, re-expansion pulmonary edema (RPE) and post-thoracentesis shock should be anticipated in the postprocedural period. Supplemental oxygen should be provided because post-thoracentesis decreases in arterial oxygenation have been reported. The magnitude and duration of this decline roughly correlate with the amount of fluid removed. If removal of a large volume of fluid is anticipated, concurrent fluid resuscitation should be considered to blunt post-thoracentesis shock. Depending on the causative process, reaccumulation of pleural fluid may occur. After appropriate positioning, the patient is prepared in standard sterile fashion. The effusion can be identified along the posterior infrascapular line by either clinical examination (auscultation and percussion) or bedside ultrasonography. Ultrasonography is recommended because it identifies not only the level of effusion but also the subdiaphragmatic organs that should be avoided and the depth of the fluid pocket.1,2 As the angiocatheter is advanced, the neurovascular bundles located on the inferior aspect of the ribs should be avoided. Aspiration of fluid can continue until enough fluid is obtained for diagnostic purposes or therapeutic relief. Needle thoracentesis is adequate for both diagnostic evaluation and therapeutic management of most parapneumonic pleural effusions. When the effusion has progressed to the fibrinopurulent or organizational stage, needle thoracentesis is often inadequate. Thoracoscopy offers the advantages of visual evaluation of the pleura, direct tissue sampling, and therapeutic intervention such as dissecting loculations and pleurodesis. Appropriate consultation for medical thoracoscopy and video-assisted thoracoscopy is indicated in these circumstances.

e1283

e1284   SECTION 15  ■  SURGICAL CRITICAL CARE DOCUMENTATION Identify the size and location of the effusion. Consider the cause and duration of the effusion. Discuss the appropriate intervention and associated risks. Assess respiratory function before and after any intervention. Document repeated physical examination and vital signs during the postprocedural period. Ensure appropriate outpatient follow-up or inpatient evaluation.

Box 68-1-3  CONTRAINDICATIONS TO THORACENTESIS Absolute Contraindications Adhesions, pleurodesis Coagulopathy Dysrhythmia Known medical noncompliance or lack of established outpatient care Relative Contraindications Bullous disease Concurrent chest wall infection Mechanical ventilation

Contraindications Procedural contraindications to pleurocentesis or thoracostomy are listed in Box 68-1-3. Absolute contraindications include coagulopathy, known adhesions, and a history of pleurodesis. If the patient is symptomatic and coagulopathic, correction of coagulopathy and ultrasonographic guidance are recommended to minimize bleeding risks. Pleurodesis, or the introduction of a chemical or medication (talc, tetracycline, or bleomycin sulfate) into the chest cavity, triggers an inflammatory reaction over the surface of the lung and inside the chest cavity that causes the pleurae to adhere to each other and prevents or reduces further accumulation of pleural fluid. Thoracentesis should be avoided in patients at increased risk for adverse reactions as a result of unstable angina or arrhythmia or known medical noncompliance, including lack of established outpatient care. Relative contraindications include mechanical ventilation because of an increased risk for lung collapse and difficulty with positioning. In intubated patients, use of ultrasonography or CT for thoracentesis or postponing the procedure is recommended if the indication is not urgent. Patients with known bullous lung disease are at increased risk for postprocedural pneumothorax. Placement of the thoracentesis needle through a concurrent chest wall infection should be avoided because the pleural space may become seeded. A postprocedure radiograph should always be obtained to assess for pneumothorax.9,12

COMPLICATIONS Adverse outcomes associated with pleural effusions can be characterized as iatrogenic or pathologic. Thoracentesis can predispose patients to pneumothorax, acute RPE, shock, subsequent fluid reaccumulation, bleeding, infection, and solid organ injury. If untreated, the parapneumonic effusion can progress to fibrinopurulent empyema, which frequently requires surgical intervention. If the patient complains of increasing respiratory distress within the first hour after thoracentesis, RPE or pneumothorax may be occurring, and an emergency chest radiograph should be obtained. RPE is a syndrome associated with hypotension and hypoxemia. It is thought to be a result of combined alveolar-capillary membrane disruption initiated by distention, reperfusion-mediated injury, and increased pulmonary flow. Risk factors include previous atelectasis and rapid re-expansion of the lung parenchyma. Typically, a patient with significant RPE becomes symptomatic within 15 minutes to 2 hours after rapid re-expansion of the lung. Treatment is based on adequate oxygenation and circulation, generally with positive end-expiratory pressure. Concern about the potential for RPE after thoracentesis is important because mortality in patients with this condition is consistently 15% to 20% despite mechanical ventilation.13,14

CHAPTER 68-1  ■  Pleural Effusion  

FOLLOW-UP, NEXT STEPS IN CARE,   AND PATIENT EDUCATION In many cases, small pleural effusions are identified during evaluation of the patient’s chief complaint. Although not all effusions require immediate drainage, when thoracentesis is performed, stable patients with a clear etiology and well-established care plan may be discharged after an appropriate observation period of 3 to 6 hours. Patients with large-volume evacuation or exudative effusions or those who require further evaluation and stabilization should be admitted to the hospital.9,13 Close outpatient follow-up and management are required for all patients evaluated for pleural effusion, regardless of whether they underwent thoracentesis, to further address the underlying cause and monitor the effusion. PATIENT TEACHING TIPS Pleural effusions represent an underlying disease process that must be addressed. Possible complications of pleural effusions include pneumothorax, respiratory failure secondary to massive fluid reaccumulation, septicemia, bronchopleural fistula, and pleural thickening. Follow-up is recommended for all patients undergoing thoracentesis. Some experts recommend serial chest radiographs to ensure clearing. Some perform computed tomography after plain radiographs show clearing.

TIPS AND TRICKS Establish bedside ultrasonography as part of assessment for pleural effusions. Consider pulmonary embolism with an uncertain pleural effusion etiology. Ultrasound-guided thoracentesis enhances visualization and minimizes complications. Complicated pleural effusions may require surgical intervention. “Two-test” and “three-test” rules for pleural fluid analysis exist but are not as specific as the Light criteria.

Suggested Reading Neustein SM. Re-expansion pulmonary edema. J Cardiothorac Vasc Anesth 2007;21:887–91. Sahn SA. The value of pleural fluid analysis. Am J Med Sci 2008;335:7–15. Sartori S, Sombesi P. Emerging roles for transthoracic ultrasonography in pleuropulmonary pathology. World J Radiol 2010;2:83–90. Thomsen TW, DeLaPena J, Setnik GS. Thoracentesis. N Engl J Med 2006;355:e16.

References 1. Sartori S, Sombesi P. Emerging roles for transthoracic ultrasonography in pleuropulmonary pathology. World J Radiol 2010;2:83–90. 2. Piccoli M, Trambaiolo P, Salustri A, et al. Bedside diagnosis and follow-up of patients with pleural effusion by a handcarried ultrasound device early after cardiac surgery. Chest 2005;128:3413–20. 3. Tayal VS, Nicks BA, Norton HJ. Emergency ultrasound evaluation of symptomatic nontraumatic pleural effusions. Am J Emerg Med 2006;24:782–6. 4. Ramnath RR, Heller RM, Ben-Ami T, et al. Implications of early sonographic evaluation of parapneumonic effusions in children with pneumonia. Pediatrics 1998;101:68–71. 5. Desai SR, Wilson AG. Pleura and pleural disorders. In: Hansell DM, Lynch DA, McAdams HP, et al, editors. Imaging of diseases of the chest, 5th ed. London: Mosby; 2010. 6. Light RW, MacGregor MI, Luchsinger PC, et al. Pleural effusions: the diagnostic separation of transudates and exudates. Ann Intern Med 1972;77:507–13. 7. Sahn SA. The value of pleural fluid analysis. Am J Med Sci 2008;335:7–15. 8. Mishra EK, Rahman NM. Factors influencing the measurement of pleural fluid pH. Curr Opin Pulm Med 2009;15:353–7. 9. Colice GL, Curtis A, Deslauriers J, et al. Medical and surgical treatment of parapneumonic effusions: an evidence-based guideline [ACCP consensus statement]. Chest 2000;118:1158–71. 10. Heffner JE, Highland K, Brown LK. A meta-analysis derivation of continuous likelihood ratios for diagnosing pleural fluid exudates. Am J Respir Crit Care Med 2003;167:1591. 11. Celli RB. Diseases of the diaphragm, chest wall, pleura and mediastinum. In: Cecil RL, Goldman L, Ausiello D, editors. Cecil textbook of medicine. 22nd ed. Philadelphia: Saunders; 2007. 12. Thomsen TW, DeLaPena J, Setnik GS. Thoracentesis. N Engl J Med 2006;355:e16. 13. Neustein SM. Reexpansion pulmonary edema. J Cardiothorac Vasc Anesth 2007;21:887–91. 14. Feller-Kopman D, Berkowitz D, Boiselle P, et al. Large-volume thoracentesis and the risk of re-expansion pulmonary edema. Ann Thorac Surg 2007;84: 1656–61.

e1285

68-2 

THORACENTESIS Terry S. Ruhl  /  Jennifer L. Good From Pfenninger JL, Fowler GC: Pfenninger & Fowler’s Procedures for Primary Care, 3rd edition (Saunders 2010)

FIGURE 68-2-1  Position for fluid removal.

FIGURE 68-2-2  A, Anesthetizing the intercostal muscle layers. Needle is “walked” over the rib. B, Clamp is placed to mark the depth to the effusion. Continued

e1286

CHAPTER 68-2  ■  Thoracentesis  

FIGURE 68-2-2, cont’d C, Needle in pleural space with fluid draining into evacuated bottle.

FIGURE 68-2-3  Position for air removal.

e1287

e1288   SECTION 15  ■  SURGICAL CRITICAL CARE

FIGURE 68-2-4  Diagnostic scheme for pleural effusion. LDH, lactate dehydrogenase. (From Light RW: Pleural effusion. N Engl J Med 346:1971–1977, 2002.)

THORACENTESIS T.W. Thomsen  /  G.S. Setnik  /  S.V. Gaufberg From Procedures Consult

68-3 

The video for this procedure can be accessed here

e1289

68-4 

SELF ASSESSMENT Matthew J. Graczyk  /  Anthony W. Kim From Velasco J: Rush University Medical Center Review of Surgery, 5th edition (Saunders 2011)

1. A 20-year-old tall, thin man experiences spontaneous pneumothorax. On further questioning and examination, it is revealed that approximately 1 year earlier he was hospitalized and had a right chest tube placed for a similar problem. What is the optimal treatment option for this patient at this time? A. Observation and discharge B. Repeated tube thoracostomy maintained until resolution C. Needle aspiration and discharge D. Lobectomy and hospitalization E. Thoracoscopic resection Ref.: 1, 2 COMMENTS: Primary spontaneous pneumothorax occurs in young patients without significant lung disease, whereas secondary spontaneous pneumothorax occurs in patients with chronic obstructive pulmonary disease. The most common cause of primary spontaneous pneumothorax is rupture of small apical blebs. In the United States, tube thoracostomy with water seal drainage is the usual first-line treatment of a moderate to large pneumothorax in a patient with a first-time occurrence. Needle aspiration of air from the pleural space is more commonly done in Europe. Patients with small firsttime pneumothoraces can be safely observed. Approximately 20% to 30% of patients will have a recurrence within 2 years of the first episode. After three or more episodes of spontaneous pneumo­ thorax, the rate of recurrence rises to 50% to 70% within the following 2 years. It is for this reason that surgery is indicated if there is a recurrence. Operative intervention may be considered after a first episode of spontaneous pneumothorax in patients with previous pneumonectomy, a history of untreated bilateral pneumothorax, or an occupation that poses an elevated risk for the development of pneumo­ thorax, such as an airline pilot or underwater diver.

ANSWER: E 2. The location of the thoracic duct at the level of the diaphragm is best described as: A. Extrapleural along the right anterior surface of the vertebral bodies, posterior to the esophagus, between the aorta and azygous vein B. Extrapleural along the left anterior surface of the vertebral bodies, posterior to the esophagus, between the aorta and azygous vein C. Intrapleural along the right anterior surface of the vertebral bodies, posterior to the esophagus, between the aorta and azygous vein D. Intrapleural along the left anterior surface of the vertebral bodies, posterior to the esophagus, between the aorta and azygous vein E. Extrapleural along the right anterior surface of the vertebral bodies, anterior to the esophagus, between the aorta and azygous vein Ref.: 1, 2 COMMENTS: See Question 3.

ANSWER: A e1290

CHAPTER 68-4  ■  Self Assessment  

3. Management of chylothorax includes all of the following except: A. Drainage of the pleural space B. Fluid, electrolyte, and nutritional support C. External beam radiation therapy D. Surgical ligation of the thoracic duct E. Reduction of chyle production Ref.: 1, 2 COMMENTS: Chylothorax is the accumulation of excess lymphatic fluid in the pleural space. It is usually a result of injury to the thoracic duct or one of its major branches and occasionally results from obstruction of the duct. The thoracic duct at the level of the diaphragm runs extrapleurally along the left anterior surface of the vertebral bodies, posterior to the esophagus, between the aorta and azygous vein. A triglyceride level of 110 mg/dL has a 99% likelihood of being chylous versus 5% when the drainage is 50 mg/dL. The most common causes are trauma, neoplasms, tuberculosis, and venous thrombosis. Treatment options are divided into operative and nonoperative categories. Drainage of the pleural space is the basic treatment of any significant accumulation of fluid. Prevention of dehydration and malnutrition and correction of electrolyte imbalance are important for higher-output chyle leaks. The most effective means of reducing chyle production is limitation or elimination of oral intake and institution of total parenteral nutrition. Somatostatin, octreotide, etilefrine, mechanical ventilation with positive end-expiratory pressure, and embolization of the thoracic duct have been used with variable success. In general, 25% to 50% of chyle leaks will close spontaneously within 2 weeks of nonoperative treatment. Surgical treatment is recommended for persistent leaks and can be performed with a variety of described techniques. Successful operative management relies on anatomic understanding of the course of the thoracic duct. Prolonged drainage only results in dehydration, malnutrition, and immunologic compromise secondary to the loss of fluid, fats, proteins, and T lymphocytes.

ANSWER: C 4. A 64-year-old woman has recurrent malignant pleural effusion that is not responding to repeated thoracentesis. She underwent mastectomy for stage II carcinoma of the breast 10 years earlier. In managing this malignant pleural effusion, the most effective agent for chemical pleurodesis is which one of the following? A. Tetracycline B. Talc C. Bleomycin D. Doxycycline E. Erythromycin Ref.: 1, 2 COMMENTS: Malignant pleural effusion is a common clinical problem that can lead to significant morbidity and reduction in quality of life in patients with advanced cancer. Lung cancer and breast cancer are the most common underlying primary malignancies. Relief of shortness of breath and improvement in quality of life are the mainstays of treatment. The diagnosis is usually made by cytologic evaluation of pleural fluid obtained via thoracentesis or thoracoscopy. Treatment options include repeated thoracentesis, tube thoracostomy with bedside pleurodesis, and placement of an indwelling pleural catheter. The most effective agent for achieving pleurodesis is talc. Talc can be instilled via closed tube thoracostomy as a “slurry” in solution or as an aerosol “poudrage” delivered at the time of thoracoscopy. Talc has been shown to control malignant pleural effusions in more than 90% of patients, generally with a success rate of between 85% and 96%. Although other series have reported high success rates with other agents, on average, the success rates are not as consistent as with talc pleurodesis. The other sclerosing agents listed are associated with success rates of 50% to 75%. It is important to point out that pleurodesis can be achieved only if the lung is not trapped and can re-expand to allow apposition of the visceral and parietal pleural surfaces. For patients with trapped

e1291

e1292   SECTION 15  ■  SURGICAL CRITICAL CARE lung associated with a malignant pleural effusion, an indwelling pleural catheter is a good option for ongoing drainage of the pleural space and control of symptoms. Erythromycin has no current role in pleurodesis.

ANSWER: B 5. Two months following right-sided pneumonectomy, empyema develops in a 60-year-old man. Conservative measures followed by lesser invasive measures to address this problem are not successful. Therefore, the patient is scheduled for open drainage of his empyema. Which of the following is true? A. Typically used in any patient able to tolerate general anesthesia B. Best suited if the underlying lung will re-expand after drainage and is not well adherent to the surrounding chest wall C. Particularly useful if a BPF is present D. Cannot be used in the setting of a BPF E. Can be considered as an alternative first-line therapy in lieu of a chest tube Ref.: 1, 2 COMMENTS: Empyema is defined as a purulent pleural effusion. The most common source of infection is the lung, but bacteria may enter the pleural space through the chest wall, from below the diaphragm, or through the mediastinum. The mainstays of treatment are antibiotics and drainage of the pleural space. Diagnostic thoracentesis is often performed, but definitive pleural drainage must be undertaken to control the infection. This can be performed by closed tube thoracostomy, pigtail catheter, VATS, or open thoracotomy. Open drainage of an empyema is best suited for chronic empyema with fixed underlying lung parenchyma that will not re-expand. Open drainage is particularly useful if a bronchopleural fistula is present. The drainage technique of open-window thoracostomy is credited to Dr. Leo Eloesser. In 1935 he described an open thoracic window for tuberculous empyemas in which a U-shaped flap of skin and subcutaneous tissue was created and sewn to the most dependent portion of the empyema cavity after removing two to three underlying ribs and intercostal muscles. The Eloesser flap is performed with a thoracotomy incision over the empyema, rib resection, and marsupialization of the skin edges to the parietal pleura to prevent closure of the incision. The Clagett procedure can be used if a BPF is not present and consists of open pleural drainage, serial operative débridement, and eventual chest closure after filling the chest cavity with antibiotic solution. It is most often used for an infected pneumonectomy space.

ANSWER: C 6. A 24-year-old woman is a passenger in a motor vehicle collision. On arrival to the trauma bay, a chest radiograph is suggestive of a left hemothorax. A tube thoracostomy is performed and a copious amount of bloody effusion is evacuated. Blood continues to be drained from her chest. In blunt chest trauma with hemothorax, what is an indication for surgical exploration that is listed among the following choices? A. Initial chest tube output exceeding 1500 mL B. Hourly chest tube output of up to 100 mL for 3 consecutive hours C. Declining hemoglobin or hematocrit D. Increasing opacities on chest radiography E. Presence of pneumothorax Ref.: 1, 2 COMMENTS: Blunt injury to the thorax often results in varying degrees of hemothorax, pneumo­ thorax, or both. Tube thoracostomy is the initial management of all injuries, both blunt and penetrating, to the thoracic cavity that result in hemothorax or pneumothorax. Many such injuries can be managed with tube thoracostomy alone. With regard to hemothorax, two metrics are generally

CHAPTER 68-4  ■  Self Assessment  

accepted as indications for thoracotomy: (1) volume of initial drainage after tube thoracostomy of 1500 mL or more or (2) ongoing bloody chest tube output of greater than 200 mL/h. A drop in hemoglobin or hematocrit is often associated with a multiply injured trauma victim and is not by itself an indication for thoracotomy. Opacities on a chest radiograph in trauma patients are relatively nonspecific and may represent lung contusion, atelectasis, pneumonia, pleural effusion, or retained hemothorax. Their presence, although important to ongoing care of the patient, is not an indication for surgical exploration. Most pneumothoraces from trauma will resolve with tube thoracostomy.

ANSWER: A

References 1. Patterson GA, Cooper JD, Deslauriers J, et al, editors: Pearson’s thoracic and esophageal surgery, ed 3, Philadelphia, 2008, Churchill Livingstone. 2. Sellke FW, del Nido PJ, Swanson SJ, editors: Sabiston and Spencer surgery of the chest, ed 7, Philadelphia, 2005, WB Saunders.

e1293

69 

Ultrasound Use for Intravascular Access

GOALS/OBJECTIVES • • • • •

BASIC PRINCIPLES ANATOMY PHYSIOLOGIC CONSIDERATIONS TECHNICAL CONSIDERATIONS MANAGEMENT OF COMPLICATIONS

ULTRASOUND-GUIDED VASCULAR ACCESS Andrew W. Shannon  /  Christine Butts  /  Justin Cook From Adams JG, et al: Emergency Medicine Clinical Essentials, 2nd edition (Saunders 2012)

69-1 

KEY POINTS

• The use of ultrasound for vascular access is now standard. • Although bedside ultrasound improves the overall success of venous access and decreases complications, it is not without potential pitfalls.

INTRODUCTION Emergency physician expertise in the use of ultrasound for obtaining vascular access is widespread because of its clinical benefit. Patients may not have accessible superficial veins. Obesity and decreased intravascular volume further increase the challenges. Central venous access has known complications that include pneumothorax and injury to great vessels. Bedside ultrasound may decrease the complication rate by allowing direct, real-time visualization of vascular targets, decreasing the need for multiple attempts, and avoiding arterial injury.1,2 The application of ultrasound for invasive and therapeutic procedures has become standard as reflected in the 2006 policy statement of the American College of Emergency Physicians on emergency ultrasound.3 Over the past decade, wide acceptance of the benefits of ultrasound-guided vascular access has led to the recommendation that ultrasound guidance be used routinely in obtaining central vascular access.4,5 Debate regarding the role of ultrasound has shifted to a focus on implementation of these recommendations and their cost-effectiveness.6–9 Research is now largely focused on improving education and training techniques or documenting the adoption of ultrasound to augment central venous access in a wider variety of settings.10,11

HOW TO SCAN AND SCANNING PROTOCOLS Ultrasound-guided central venous access is accomplished with many of the same techniques as used by the traditional landmark approach. Patient positioning, informed consent, use of sterile technique with full draping, and selection of the anatomic site should be undertaken in the usual manner. Either a two- or single-operator technique is acceptable.12 A single operator will use the dominant hand to advance and aspirate the needle while manipulating the transducer with the opposite hand. In a two-operator procedure, the cannulating operator will concentrate on the needle and syringe, and the probe will be held steady by the second operator. Two techniques are commonly accepted for achieving ultrasound-guided vascular access. In the static technique, ultrasound is used to identify vascular structures in relation to external landmarks, and then the ultrasound device is set aside and cannulation performed in the usual manner. The dynamic technique involves real-time, direct visualization of entry of the needle into the vein by ultrasound and seems to be preferable, particularly when the venous structures are small.13 In this case, once the vein has been accessed (or a “flash” of blood is seen), the ultrasound device is set aside. The probe most conducive to central venous access is a linear-array high-frequency (5- to 12-MHz) probe (Figure 69-1-1). Care should be taken to identify the side of the probe bearing the indicator mark that corresponds to the on-screen indicator. This will allow the most intuitive positioning of the e1295

e1296   SECTION 15  ■  SURGICAL CRITICAL CARE

FIGURE 69-1-1  A high-frequency, or linear, transducer.

FIGURE 69-1-2  Position of the transducer to obtain a transverse image of the internal jugular vein. Note that as the operator is standing at the patient’s head, the indicator is pointing toward the patient’s left. This ensures that when the operator is looking at the screen, the patient’s left and the operator’s left are the same. This minimizes confusion if the needle track needs to be adjusted.

probe during venous access such that medial on the patient is medial on the screen of the machine as viewed by the operator when attempting cannulation (Figure 69-1-2). Once a site has been chosen, usually the internal jugular or femoral, it should be evaluated with ultrasound to identify the artery and the vein (Figure 69-1-3). When compared with their accompanying veins, arteries appear thick walled, more circular, and pulsatile on ultrasound. Arteries do not compress with light pressure. Veins are more irregular in shape, sometimes appearing triangular rather than round, and compress with light pressure. Use of color Doppler can also aid in identification. It is often easiest to begin with the probe in a transverse orientation. In this view, the vessels appear in cross section as round or oval structures (see Figure 69-1-3). The depth of the target vessel and its relationship to surrounding structures can be determined. The vein should then be centered on the screen. This allows an external landmark, the center of the transducer, to be established. Pressure over this area with a blunt object, such as a fingertip, can confirm the correct location. The needle should then be inserted at a 45-degree angle to the skin at a distance from the probe equal to the depth of the target vessel (Figure 69-1-4). Immediately after entering the skin, the needle tip should be identified on the screen. It will appear as a hyperechoic (white) object within subcutaneous tissue. The needle tip should be followed with the transducer as it advances toward the vein. As the needle tip reaches

CHAPTER 69-1  ■  Ultrasound-Guided Vascular Access  

FIGURE 69-1-3  Transverse anatomy of the vessels of the neck. In this image, the internal jugular vein (IJ) is seen lying on top of the carotid artery. The IJ is large and oval in shape. Direct pressure over it should cause slight collapse. The carotid remains stable in size, appearance, and compressibility. It is usually small, round, and noncompressible. Overlying the vessels is the sternocleidomastoid muscle.

FIGURE 69-1-4  Demonstration of the method for judging the angle and placement of entry for ultrasound-guided vascular access. Because the vessel is measured to be 2 cm below the surface, introducing the needle 2 cm from the transducer at an angle of 45 degrees will result in the correct trajectory to visualize the needle tip as it approaches the target vessel.

the vein, the wall of the vessel will be seen to deform. A flash of blood in the syringe confirms that the needle has entered the vein (Figure 69-1-5). The longitudinal approach is somewhat more challenging to master but allows better visualization of the needle along its entire length. The longitudinal view is obtained by rotating the probe 90 degrees from the transverse position to line up in parallel with the course of the vein (Figure 69-1-6). Extra care should be taken to differentiate venous from arterial vessels in this view and to avoid accidental migration of the probe. In this approach the needle should enter the skin at one end of the probe (Figure 69-1-7) – and therefore the ultrasound screen – and be advanced in plane toward the underlying vein along the long axis (Figure 69-1-8). Similar pressure deformity and indentation of the vessel wall should be noted before it is punctured, and a flash of blood should again be sought. An oblique approach has been described in which the vessels are imaged with an orientation inbetween the transverse and longitudinal views.14 The probe is aligned obliquely over the vessel so that it appears between the structure typically seen in the transverse and longitudinal views. The needle can then be introduced from the end of the transducer and followed in plane as it advances toward the vessel.15 The oblique approach combines the familiar view of the vessel with the reassurance of being able to view the length of the needle.

e1297

e1298   SECTION 15  ■  SURGICAL CRITICAL CARE

FIGURE 69-1-5  A needle tip (seen as the hyperechoic structure on the right of the vessel) entering the internal jugular vein. This image should correspond to a flash of blood seen in the syringe attached to the cannulating needle.

FIGURE 69-1-6  The internal jugular (IJ) vein and carotid artery in longitudinal orientation. Note how closely opposed these vessels are to one another. The IJ is the more superficial of the two and has thinner walls. Its size should vary with respiration and compression. The carotid artery is deep to the IJ, has thicker walls, and should not vary in size. At the left of the screen, a catheter or wire is seen within the lumen of the IJ.

FIGURE 69-1-7  The in-plane technique used for longitudinal or oblique placement of a catheter. The indicator in this image is pointing toward the patient’s feet and the needle is inserted from this end. This causes the needle to appear from the left side of the screen as shown on the right of this figure. It can then be followed in plane as it advances toward the vessel.

CHAPTER 69-1  ■  Ultrasound-Guided Vascular Access  

FIGURE 69-1-8  A needle advancing toward the internal jugular in longitudinal orientation. The needle is seen on the right of the figure as a hyperechoic object. It can be seen entering the vessel as shown by the arrow.

FIGURE 69-1-9  The peripheral veins of the upper extremity. The basilic vein is a frequent target of ultrasound-guided access because it is easy to find, relatively easy to access, and frequently available.

Ultrasonography is also commonly used for peripheral approaches to intravenous access, particularly in patients with difficult access, such as those undergoing dialysis or chemotherapy.16 The basilic vein is usually a good option, even when other peripheral veins are unusable (Figure 69-1-9). The extremity chosen should be positioned comfortably and a tourniquet applied to facilitate an initial ultrasound survey to identify candidate veins (Figure 69-1-10). The operator localizes the vein (Figure 69-1-11) and performs cannulation via the transverse or longitudinal approach, as described for central access. Because peripheral veins requiring ultrasound guidance for cannulation are often deeper structures, the use of longer catheters should be considered. It should also be appreciated that peripheral veins are much more likely to collapse with even light pressure from the ultrasound transducer.

e1299

e1300   SECTION 15  ■  SURGICAL CRITICAL CARE

FIGURE 69-1-10  A sonographer scanning the area of the basilic vein. Note that the indicator is pointed toward the patient’s right. This ensures that the image that is seen on screen is true to the surface anatomy. In other words, the orientation of the anatomy seen on screen is the same as the orientation encountered by the operator.

FIGURE 69-1-11  Transverse image of the basilic vein. Note its close proximity to the brachial artery and vein.

RED FLAGS Although bedside ultrasound improves the overall success of venous access and decreases complications, it is not without potential pitfalls. When viewing vessels in the transverse orientation, only a small part of the needle can be visualized. Identifying and following the needle tip immediately after it enters the skin will avoid inadvertent arterial puncture. In the longitudinal orientation, the vein and artery are very closely opposed (see Figure 69-1-4). Extra care should be taken to ensure that the vessel on screen is the target vessel. Both the transverse and longitudinal orientations have limitations in localizing the needle tip. In the transverse orientation, the medial-to-lateral position of the tip can best be determined (Figure 69-1-12), but the slope of the needle path may be difficult to appreciate. Conversely, in the longitudinal orientation, the slope can be appreciated, but the medial-to-lateral position may not be apparent (Figure 69-1-13). A combination of these two approaches, or the oblique approach, may minimize these potential shortcomings. It is also important to avoid reliance on any one aspect of the image to identify the structures. Variant vascular anatomy may make landmarks less reliable, and severe volume depletion may lead to a completely collapsed internal jugular vein with a compressible carotid. Multiple characteristics should be examined to confirm that the vessel in question is venous. Even though visualization of the anatomy does make successful cannulation more likely, it is no guarantee. Inadvertent carotid puncture while using ultrasound guidance is well described, in particular as a result of a through-and-through venous puncture.17,18 Prudence and careful technique are always appropriate.

CHAPTER 69-1  ■  Ultrasound-Guided Vascular Access  

FIGURE 69-1-12  The advantages of transverse orientation for vascular access. In this orientation, the left-to-right (or medialto-lateral) placement of the needle can be identified. However, the slope of the angle of the needle is out of the plane of this orientation and cannot easily be appreciated. Failure to appreciate this shortcoming may result in inadvertent arterial puncture.

FIGURE 69-1-13  The advantages of longitudinal orientation for vascular access. In this orientation, the slope of the angle of the needle can be identified. However, the right-to-left (or medial-to-lateral) placement of the needle tip cannot easily be appreciated.

Suggested Reading 1. Keyes LE, Frazee BW, Snoey ER, et al. Ultrasound-guided brachial and basilic vein cannulation in emergency department patients with difficult intravenous access. Ann Emerg Med 1999;34:711–14. 2. Leung J, Duffy M, Finckh A. Real-time ultrasonographically-guided internal jugular vein catheterization in the emergency department increases success rates and reduces complications: a randomized, prospective study. Ann Emerg Med 2006;48:540–7. 3. Phelan M, Hagerty D. The oblique view: an alternative approach for ultrasound- guided central line placement. J Emerg Med 2009;37:403–8. 4. Moon CH, Blehar D, Shear MA, et al. Incidence of posterior vessel wall puncture during ultrasound-guided vessel cannulation in a simulated model. Acad Emerg Med 2010;17:1138–41.

References 1. Leung J, Duffy M, Finckh A. Real-time ultrasonographically-guided internal jugular vein catheterization in the emergency department increases success rates and reduces complications: a randomized, prospective study. Ann Emerg Med 2006;48:540–7. 2. Hind D, Calvert N, McWilliams R, et al. Ultrasonic locating devices for central venous cannulation: meta-analysis. BMJ 2003;327(7411):361. 3. ACEP Board of Directors. ACEP policy statement: emergency ultrasound imaging criteria and compendium. 2006. Available at http://www.acep.org/policystatements/. Accessed June 12, 2015.

e1301

e1302   SECTION 15  ■  SURGICAL CRITICAL CARE 4. Rothschild JM. Ultrasound guidance of central vein catheterization. In: Shojania KG, Duncan BW, McDonald KM, et al, editors. Making health care safer: a critical analysis of patient safety practices. Agency for Healthcare Research and Quality; 2001. Available at http://archive.ahrq.gov/clinic/ptsafety/chap21.htm. Accessed June 12, 2015. 5. National Institute for Clinical Excellence. Guidance on the use of ultrasound locating devices for placing central venous catheters. London: National Health Service; Issue date: September 2002, Review date: August 2005. Technology Appraisal No. 49. Available at http://www.nice.org.uk/guidance/ta49. Accessed June 12, 2015. 6. Neustein SM. Mandating ultrasound usage for internal jugular vein cannulation. Can J Anaesth 2010;57:868; author reply 868–9. 7. Chalmers N. Ultrasound guided central venous access. NICE has taken sledgehammer to crack nut. BMJ 2003;326(7391):712. 8. Matera JT, Egerton-Warburton D, Meek R. Ultrasound guidance for central venous catheter placement in Australasian emergency departments: potential barriers to more widespread use. Emerg Med Australas 2010;22:514–23. 9. Keenan SP. Use of ultrasound to place central lines. J Crit Care 2002;17:126–37. 10. Wells M, Goldstein L. The polony phantom: a cost-effective aid for teaching emergency ultrasound procedures. Int J Emerg Med 2010;3:115–18. 11. Agarwal A, Singh DK, Singh AP. Ultrasonography: a novel approach to central venous cannulation. Indian J Crit Care Med. 2009;13:213–16. 12. Milling T, Holden C, Melniker L, et al. Randomized controlled trial of single-operator vs. two-operator ultrasound guidance for internal jugular central venous cannulation. Acad Emerg Med 2006;13:245–7. 13. Milling Jr TJ, Rose J, Briggs WM, et al. Randomized, controlled clinical trial of point-of-care limited ultrasonography assistance of central venous cannulation: the Third Sonography Outcomes Assessment Program (SOAP-3) Trial. Crit Care Med 2005;33:1764–9. 14. Phelan M, Hagerty D. The oblique view: an alternative approach for ultrasound-guided central line placement. J Emerg Med 2009;37:403–8. 15. Schofer JM, Nomura JT, Bauman MJ, et al. The “ski lift”: a technique to maximize needle visualization with the long-axis approach for ultrasound-guided vascular access. Acad Emerg Med 2010;17(7):e83–4. 16. Keyes LE, Frazee BW, Snoey ER, et al. Ultrasound-guided brachial and basilic vein cannulation in emergency department patients with difficult intravenous access. Ann Emerg Med 1999;34:711–14. 17. Moon CH, Blehar D, Shear MA, et al. Incidence of posterior vessel wall puncture during ultrasound-guided vessel cannulation in a simulated model. Acad Emerg Med 2010;17:1138–41. 18. Blaivas M. Video analysis of accidental arterial cannulation with dynamic ultrasound guidance for central venous access. J Ultrasound Med 2009;28:1239–44.

Further Reading Hind D, Calvert N, McWilliams R, et al. Ultrasonic locating devices for central venous cannulation: meta-analysis. BMJ 2003; 327:361.

ULTRASOUND GUIDANCE FOR VASCULAR ACCESS Paul-André C. Abboud  /  John L. Kendall From Abboud PA, Kendall JL: Ultrasound guidance for vascular access. Emerg Med Clin North Am 2004;22:749

69-2 

The ability to establish central venous access efficiently is a fundamental skill for emergency physicians. Central venous access is essential for hemodynamic monitoring, volume resuscitation, and the delivery of vasoactive drugs.1 It is important in the management of shock and other conditions such as renal failure and complete heart block because it contributes to temporizing and life-saving therapies. Traditionally, central venous access has been guided only by palpable anatomic landmarks such as bony prominences, muscle surfaces, and arterial pulsations. This “blind” approach to the central veins assumes anatomic homogeneity, does not account for the possibility of thrombosis, and depends on correct discernment of the relationship among multiple anatomic landmarks.2 Research in emergency department (ED) and intensive care settings has supported the efficacy of traditional landmark approaches to the internal jugular vein (IJV), subclavian vein (SV), and femoral vein (FV) in adult3–12 and pediatric patients.13–16 Failure rates, however, have been reported as high as 30% in some series.17 The failure rate has been demonstrated to be greater for emergent cases and highest for patients in cardiopulmonary arrest.18 Nonrandomized studies of central venous cannulation specifically for critical trauma4,6,11 or cardiopulmonary resuscitation8,10,11 have reported success rates ranging from 62% to 99%. One study of failed cardiopulmonary resuscitation cases demonstrated that 31% of attempted FV catheters were not in the FV.19 Complication rates related to central line placement are reported to range from 0.3% to 18.8%, depending on the site of insertion, patient population, and definition of complications.17,20–22 Acute complications associated with the landmark approach commonly include pneumothorax, arterial puncture, hemothorax, hematoma (subcutaneous or mediastinal), misplaced catheter tip, nerve injury, and dysrhythmia.16,17,23–25 Cases of transient Horner syndrome and dysphonia after IJV catheterization have been reported in some series.21,26 Death due to complications from a central venous line also has been reported.27 The complication rate depends on the time needed for catheter insertion,22 the number of needle passes,28 the extremes of body habitus, previous central venous cannulation, prior surgery or radiation therapy in the area of the vein, and operator inexperience.20 Characteristics associated with difficult or complicated central access include limited sites for access attempts (other catheters already in place, pacemaker, local surgery, or infection), known vascular abnormality, coagulopathy, mechanical ventilation or severely diminished pulmonary function (leading to worse morbidity from a possible pneumothorax), severe peripheral vascular disease, soft tissue edema, chronic intravenous drug use, and patient intolerance of supine position (orthopnea, increased intracranial pressure).29–34 Under these circumstances in which the margin for error is small, central venous access must be undertaken carefully. Emergency medicine has developed an expanding familiarity with portable two-dimensional (2-D) real-time ultrasound (US) over the past decade.35 In that time, a body of research has developed that supports US for guidance of central venous cannulation. Descriptions of US guidance for central venous access first were published in the anesthesiology literature and, subsequently, in the surgery, radiology, nephrology, critical care, and emergency medicine literature. In 1978, Ullman and Stoelting36 first described the use of a “pencil-shaped Doppler probe” to identify the “windstorm” sounds of the IJV to mark the overlying skin site for cannulation. Legler and Nugent37 published the first experience with Doppler localization of the IJV before catheterization. In 1986, Yonei et al38 first reported the use of 2-D real-time US guidance for cannulation of the IJV. e1303

e1304   SECTION 15  ■  SURGICAL CRITICAL CARE The first case series of 2-D US for central venous access in the ED was published in 1997.39 The reported technique involved two operators: one for line placement and one to hold the US probe. Since then, emergency physicians have published four studies on the use of US for vascular access in the ED.33,40–42 These studies, which are reviewed below, reported favorable experiences and improved success rates for venous access with US guidance. In 1997, the American College of Emergency Physicians (ACEP) published a policy statement on the use of US imaging by emergency physicians. In 2001, a revised policy statement and accompanying ACEP guidelines specifically included US guidance for central venous access in a list of “primary applications for emergency ultrasound”.43

EVIDENCE FOR ULTRASOUND-GUIDED   CENTRAL VENOUS ACCESS In 2001, the Agency for Healthcare Research and Quality published an evidence-based report on patient safety practices. This report, which was been highly publicized in professional and lay media, includes a chapter on US guidance for central venous access. US guidance for central venous access was listed among 11 practices with the most highly rated “strength of evidence for supporting more widespread implementation”.44 This report based its findings on much of the same literature that previously had been reviewed in a meta-analysis by Randolph et al.1 The meta-analysis, published in 1996, reviewed eight randomized controlled trials of 2-D or Doppler US guidance versus the landmark method for central venous access. No studies of FV access were included. A significant decrease in the failure rate, complication rate, and number of attempts for successful access of the SV and the IJV were reported. A subsequent meta-analysis commissioned by the British National Institute for Clinical Excellence (NICE) was published in 2003.45 It included 18 randomized controlled trials published through October 2001. These trials compared 2-D real-time or Doppler US with the landmark method for central venous access. The meta-analysis considered risk of failed placement, complications, failure on the first attempt, number of attempts to successful access, and time to successful access as outcome measures. These outcome measures were analyzed by type of vein studied (IJV, SV, and FV), by US method (2-D and Doppler), and by age category (adult and infant). This meta-analysis concluded that 2-D US guidance was more effective than the landmark method for all outcomes for IJV access in adults. The relative risks of failed attempts, complications, and failed first attempts were reduced by 86%, 57%, and 41%, respectively. Significantly fewer attempts were required for success, and the IJV was successfully accessed more quickly when using US. Limited evidence suggested that 2-D US guidance reduced the relative risk of failed access in the SV and FV. The three studies of IJV access in infants included in the meta-analysis were limited by small sample size;46–48 however, the analysis suggested that 2-D US was more effective in these studies. Using US, the relative risk of failed placements and complications in infants was reduced by 85% and 73%, respectively. No studies of SV or FV access among infants were included in the meta-analysis. The investigators45 also undertook a cost-effectiveness analysis of 2-D US guidance based on the evidence from their systematic review of the literature. The analysis of a simple decision analytic model suggested that US guidance avoided 90 arterial punctures for every 1000 patients and reduced costs by a negligible amount (approximately $5) per patient. Given the evidence for its superior efficacy, the recent Agency for Healthcare Research and Quality mandate for improving patient safety, and the 2001 ACEP emergency US guidelines, US guidance for central venous access has been transformed from an interesting novelty to an important skill for emergency physicians to acquire.

GENERAL TECHNICAL ISSUES There are several commonly accepted variations of US guidance: indirect, direct or real-time, freehand, mechanical guide, and Doppler. Choosing among these approaches mostly depends on the location of the vessel to be cannulated and the specific characteristics of the operator, patient, and the equipment at hand. In addition, vessel visualization can be obtained in two different ways: in the long axis or the short axis. A solid understanding of these technical issues is necessary to successfully cannulate a vessel, regardless of the approach used or the location of the vessel.

CHAPTER 69-2  ■  Ultrasound Guidance for Vascular Access  

FIGURE 69-2-1  IJV and carotid artery viewed in their short axes.

FIGURE 69-2-2  Subcutaneous location of IJV is identified on the skin surface.

FIGURE 69-2-3  Subcutaneous location of IJV is marked on the skin surface.

Indirect Method The indirect method employs the least amount of actual guidance. With this approach, US is used only to identify the vessel and then center it on the US screen (Figure 69-2-1). Next, a temporary mark is placed on the skin that corresponds to the vessel’s subcutaneous position. This mark is used for the puncture site after US identifies the target vessel’s location, dimensions, and depth below the skin. The easiest way to accurately make this mark is to identify the point where the center of the transducer overlies the skin surface just above the center of the vessel (Figures 69-2-2 and 69-2-3). There is no direct visualization of the needle as it enters the vessel, however. This technique has been used for localization and cannulation of larger structures but has been criticized for not taking

e1305

e1306   SECTION 15  ■  SURGICAL CRITICAL CARE

FIGURE 69-2-4  Sterile barrier covering transducer and cable.

FIGURE 69-2-5  Advancing needle and ring-down artifact.

full advantage of the potential of US for greater precision.49 Mansfield et al20 compared the indirect method of US guidance with the standard landmark approach for SV cannulation. This study was closed after an interim analysis of 824 patients showed that US guidance by the indirect method had no effect on the rate of complications or failures.

Real-Time Visualization The alternative to the indirect method is to perform needle placement under direct, or real-time, US guidance so that the entire procedure is visualized continuously. With this technique, a sterile sheath whose tip is filled with transmission gel is unrolled over the transducer (Figure 69-2-4). The transducer is then placed on the skin and the target vessel is identified and centered on the viewing screen. With the other hand, local anesthetic is injected at a point corresponding to the middle of the US transducer. After anesthesia is achieved, the cannulation needle is advanced through the skin. After the skin has been punctured, the operator can switch visual focus to the US monitor where the needle will appear sonographically as an echogenic line with a ring-down artifact (Figure 69-2-5). Advancement of the needle is then guided by viewing its progression on the US monitor. When the operator visualizes the needle piercing the anterior wall of the vessel (Figure 69-2-6) and after the subsequent flash of blood into the syringe, the transducer is placed aside and the remaining aspects of the procedure can be completed normally. Few studies have compared indirect and real-time US guidance methods for insertion of venous catheters. Nadig et al50 randomized 73 patients to an external landmark or a real-time US guidance of IJV cannulation. There were 87 unsuccessful attempts among 37 patients in whom cannulation was performed using the indirect method. In comparison, there were only 10 unsuccessful attempts among the 36 patients who underwent real-time US guidance.

Mechanical Guides A mechanical guide is an attachment to the US transducer that controls the depth, angle, and trajectory of the needle during cannulation (Figure 69-2-7). In addition to venous cannulation, mechanical

CHAPTER 69-2  ■  Ultrasound Guidance for Vascular Access  

FIGURE 69-2-6  Needle visualized advancing through anterior (ant.) vessel wall.

FIGURE 69-2-7  Ultrasound transducer with needle inserted through a mechanical guide.

guides are used for many other US-guided invasive procedures, including amniocentesis, follicle retrieval, cordocentesis, biopsies, and fluid aspiration.51–54 The approach uses an attachment to the transducer that provides a fixed trajectory for the needle. Advancement of the needle through the designated path ensures a predictable, uniform trajectory of the needle relative to the transducer. This stability may be particularly advantageous for inexperienced operators because it incorporates control of the transducer’s placement with the needle’s angle of entry.55 The use of mechanical guides has some notable disadvantages. It requires investing in an additional piece of equipment that makes large, linear transducers even bulkier. Mechanical guides restrict the angle of the needle and the skin entry point. This restriction prevents the operator from continuously redirecting the needle as may be needed in certain cases.56 Lastly, the fixed angle of entry may make some superficial structures difficult to access.57 The largest study to evaluate needle-guided US was published by Denys et al.58 This nonrandomized study reported a 100% success rate for US guidance among 928 patients. In comparison, the success rate for the landmark approach was 88% among 302 patients. With needle-guided US, there also was a significant improvement in venous access time (9.8 versus 44.5 seconds), carotid puncture rate (1.7% versus 8.3%), brachial plexus irritation (0.4% versus 1.7%), hematoma development (0.2% versus 3.3%), and average number of attempts to success (1.3 versus 2.5).

Free-Hand Method The alternative to using a needle-guide system is to perform the procedure using the free-hand technique. In this situation, the transducer and the advancing needle are positioned and stabilized with the operator’s or an assistant’s hands (Figure 69-2-8). Continuous fine adjustments can be made in the needle’s direction and in the transducer’s view. Although it generally is considered to be a more

e1307

e1308   SECTION 15  ■  SURGICAL CRITICAL CARE

FIGURE 69-2-8  Free-hand technique.

FIGURE 69-2-9  Short-axis approach to needle insertion.

technically demanding procedure, this approach offers more flexibility for the operator. In addition, if an assistant is available, he or she can hold the transducer on a site distant from the needle entry point, thereby removing it from the sterile field57 and potentially increasing the speed with which the target vein can be cannulated.

Short-Axis Versus Long-Axis Approach Another variable to consider is the axis of visualization for the target vessel. For the short-axis approach, the vessel is identified in the transverse plane and centered under the transducer (see Figure 69-2-1). The midpoint of the transducer then becomes a reference point for insertion of the needle. The needle is inserted at a 45° angle to the transducer (Figure 69-2-9). As the needle is advanced, the tip is visualized as it approaches the anterior wall of the vessel. After contacting the anterior wall of the vessel, further insertion of the needle will cause posterior displacement of the vessel wall (Figure 69-2-10). A flash of blood in the syringe signifies that the needle has entered the vessel. At this point, the transducer can be set aside and the rest of the procedure performed normally. In contrast, the long-axis approach identifies the vessel in its long axis and involves lining up the transducer over the greatest anterior–posterior diameter of the vessel (Figure 69-2-11). The needle is then inserted through the skin just off one end of the transducer in a plane that is in line with the long axis of the transducer and at an approximate 30° angle to the skin surface (Figure 69-2-12). As the needle is advanced, its progress through the subcutaneous tissue is monitored in real-time on the US screen (Figure 69-2-13). After the needle has punctured the anterior wall of the vessel and a flash of blood is apparent in the syringe, the transducer can be set aside and the rest of the procedure completed normally. In the authors’ experience, the advantages of the short-axis over the long-axis approach are that it is easier to perform in anatomic areas where space is limited (eg, neck), inexperienced users can acquire proficiency more quickly, and smaller vessels (eg, deep brachial vein) are more consistently visualized. On the other hand, performing the procedure along the vessel’s long axis allows much better

CHAPTER 69-2  ■  Ultrasound Guidance for Vascular Access  

FIGURE 69-2-10  Posterior displacement of vessel wall by advancing needle viewed in the short axis.

FIGURE 69-2-11  IJV visualized in the long axis.

FIGURE 69-2-12  Long-axis approach to needle insertion.

e1309

e1310   SECTION 15  ■  SURGICAL CRITICAL CARE

FIGURE 69-2-13  Real-time visualization of needle through subcutaneous tissue.

FIGURE 69-2-14  Needle visualized perforating posterior (post.) vessel wall.

visualization of the advancing needle tip and, therefore, may avoid inadvertent punctures of the posterior vessel wall (Figure 69-2-14). A study that compared the short- and long-axis approaches to vascular access found that novice users could perform cannulation more quickly using the short-axis approach. There was, however, no statistically significant difference in terms of mean difficulty, number of skin breaks, and mean number of needle redirections.59

Doppler Method Historically, Doppler US for venous access has not been used in many American EDs, most likely because these EDs purchased real-time 2-D US machines. These multipurpose machines have been favored over purchasing additional equipment that is needed to implement the Doppler method. In the interim, research has shown 2-D real-time US to be superior to Doppler for central venous access.45 The authors expect most future research on US-guided techniques to focus on 2-D real-time US.

INTERNAL JUGULAR VEIN APPROACH Anatomic Considerations The IJV is the most studied vein for US-guided catheterization. It usually lies anterior and slightly lateral to the carotid artery. It normally is larger in diameter than the carotid artery and expands in

CHAPTER 69-2  ■  Ultrasound Guidance for Vascular Access  

diameter with a Valsalva maneuver.60 Many studies have documented its anatomic variations with regard to potential complications. Yonei et al38 first reported the use of real-time US guidance for IJV cannulation in 1986. In this series, central lines were placed successfully in 160 intensive care patients without complication. In 1991, Denys and Uretsky61 reported a series of 200 patients who underwent IJV cannulation in the cardiac catheterization laboratory, coronary care unit, and intensive care unit. The investigators found anatomic anomalies of the IJV (small diameter, unresponsiveness to Valsalva maneuver, and unexpected lateral or medial displacement) in 8% of patients. Troianos et al62 reported the largest case series for determining the anatomic relationship between the IJV and carotid artery. Among 1009 patients admitted for surgery, 54% had an IJV overlying the carotid artery, rather than coursing it laterally, as expected. This anomalous anatomy might predispose the patients to arterial punctures if the needle traversed the IJV. In a prospective series of 31 patients with known difficult central venous access, Hatfield and Bodenham29 reported a success rate of 100% using real-time US guidance for 22 patients. Among the remaining 9 patients, for whom indirect US guidance was used, 66% were cannulated successfully within three attempts. Of 23 patients who had been referred specifically because of prior difficulties with or complications from cannulation, 16 had an anatomic reason for difficulty that was determined by US. Docktor et al63 reported a 100% success rate with real-time US guidance in a prospective series of 150 patients referred for nonemergent central venous access. Using US, the investigators were able to document the phenomenon of double wall puncture among 30 patients. Double wall puncture occurs in cases of an IJV with low pressure. The anterior wall is pushed against the posterior wall before the needle punctures it. If the carotid artery is located underneath the vein along the needle tract, then a double wall puncture can extend into the carotid artery. In this study, the IJV was visualized directly over the carotid artery in 25% of cases. The investigators surmised that this high rate was partially due to variable transducer positioning and different degrees of head rotation. The only complications were two cases of carotid artery puncture. These occurred among patients whose IJV was visualized to lie directly over the carotid artery. Based on their findings, the investigators recommended using real-time US to visualize the anatomic relationship between the IJV and carotid artery and then determining the optimal needle track that would miss the carotid artery in the event of a double wall puncture. Denys et al58 compared real-time US guidance that used a transducer needle guide with the landmark approach in 1230 patients who had IJV cannulation. Among patients in the US-guidance group, 3.4% had a right IJV that was not visualized or was deemed too small to cannulate. In all of these cases, the left IJV was successfully cannulated. The investigators also recognized the double wall phenomenon, commenting, “the IJV is actually compressed completely by the needle before the vessel is penetrated. The needle must be advanced a little deeper and then retracted slightly to be positioned in the center of the lumen.” This finding underscores the need to identify an underlying carotid artery.

Evidence-Based Analysis Eleven of the 18 articles included in the 2003 NICE-sponsored meta-analysis investigated the IJV approach in adults.45 Of these 11 studies, 7 used real-time US guidance and 4 used Doppler US guidance. Notable improvements over the landmark approach were demonstrated with real-time US guidance. The relative risk of failed first attempts was 0.59 (95% confidence interval: 0.39–0.88, P = 0.009) and the relative risk of failed catheter placement was 0.14 (95% confidence interval: 0.06–0.33, P < 0.0001). The relative risk of complications decreased by 57%, the mean number of attempts to successful cannulation decreased by 1.5, and the mean time to successful cannulation decreased by almost 70 seconds (all P < 0.02). Subsequent to the meta-analysis, another randomized trial that compared real-time US with the landmark approach among intubated patients undergoing elective surgery was published.64 This study investigated the use of indirect US guidance with two different frequency transducers: 7.5 MHz and 3.75 MHz. In measuring the number of successful first attempts, mean number of attempts to success, and rate of complications, the investigators discovered no significant differences between the two frequencies. They noted that the 7.5-MHz transducer visualized structures with higher resolution but that “the image quality of 3.75 MHz was acceptable for the purpose of locating the IJV and CA.” When the results of two US groups were pooled for comparison to the landmark group, the rate of successful first attempts showed improvement (73% versus 86%, P < 0.05); however, no statistically

e1311

e1312   SECTION 15  ■  SURGICAL CRITICAL CARE significant differences were found regarding the overall success or complication rates. The investigators noted that in a subset of 52 patients who lacked the anatomic landmark of respiratory jugular venodilation (visible bulging of the vein beneath the skin synchronized with inspiration of positive-pressure ventilation) on which they traditionally depended, US guidance was superior to the landmark approach for all outcomes. The number of successful first attempts was almost tripled (86% versus 30%, P < 0.001) and the number of successful cannulations was greatly improved (100% versus 78%, P < 0.05). There were no complications among the US-guided attempts, whereas the landmark approach was associated with three carotid artery punctures. These results, although not surprising, further support the use of US guidance, especially for patients with poor percutaneous landmarks.

Emergency Medicine Literature Hudson and Rose39 first reported the use of US guidance for IJV cannulation specifically in the ED in 1997. In this article, they described their successful experience in 2 patients with challenging percutaneous landmarks due to severe skin graft scarring or morbid obesity. Since then, two prospective studies of ED patients have been published. Hrics et al41 reported a small case series in which patients who needed central venous access within 1 hour of arrival to the ED underwent cannulation either with real-time or indirect US guidance. The success rates were 87.5% among the 8 patients in the real-time US group and 71% among the 24 patients in the indirect US group. Miller et al33 have published the only trial as yet of real-time US versus a landmark approach in the ED setting. This trial was pseudorandomized. It used odd- and even-day assignment of 122 patients to real-time US guidance or to the landmark approach to the IJV, SV, FV, or peripheral vein. It did not analyze results by the type of vein cannulated; however, the largest subgroup (55%) of the US-guided cases were for IJV cannulation. The investigators noted an overall decrease in the mean time to successful cannulation and number of attempts when using US. These improvements occurred across the range of operator experience.

SUBCLAVIAN VEIN APPROACH Anatomic Considerations As with the IJV, the SV offers ideal size for central access. Its proximity to structures such as the lung, subclavian artery, and brachial plexus, however, can lead to significant morbidity. Other challenges for accessing the SV with US guidance are its deeper location and the presence of the clavicle bone. Because bone does not transmit US waves, placing the transducer over it provides no information to guide the operator. US guidance can be used to cannulate the SV in its midportion as is most commonly taught with the landmark approach. In this case, the apex of the lung can be less than 1 cm away from the SV.65 Attempting US visualization of the SV in short axis also can be challenging in this location because it involves holding the transducer on top of the clavicle. Maintaining appropriate pressure for the transducer over the clavicle can be uncomfortable for the patient and may add to the technical difficulty of this approach.

Supraclavicular Approach Due to its inherent anatomy, the supraclavicular approach to the SV is troublesome to achieve with US guidance. In most patients, little space is available for the transducer to be placed concurrently with needle insertion, which makes real-time US guidance difficult at best. Two alternatives for successful US-guided supraclavicular approach exist. The first is to use the indirect method (previously described). Another alternative is to use the low-IJV approach. This approach has been found to be a safe and direct route to the superior vena cava and right atrium for US-guided central venous access. Silberzweig and Mitty66 investigated 116 low-IJV punctures among 109 patients in the interventional radiology suite. They reported no complications and an average of 1.2 attempts (range, 1–3) needed for success.

Axillary Approach Yet another approach is to access the SV more laterally on the shoulder so that the needle cannulates the axillary vein. This more distal approach eliminates the problem of holding the transducer over the

CHAPTER 69-2  ■  Ultrasound Guidance for Vascular Access  

uneven surface of the clavicle and removes the potential for the placed catheter to be pinched between the subclavius muscle and the costoclavicular ligament complex associated with the standard approach.67 It also decreases the risk of pneumothorax because the lung generally is farther away from the vascular structures in the lateral shoulder. A misplaced needle passing through the axillary vein will travel posteriorly through the axillary fat and layers of muscle and, finally, to the scapula, thereby missing the pleural space.68 The landmark approach to the axillary has been demonstrated to be safe in adults69 and critically ill infants and children.70 Potential advantages for using the axillary approach specifically in the ED include easier access for patients in cervical collars or with neck trauma. Using this more lateral approach in critical trauma patients may also allow for more efficient simultaneous management of the airway and acquisition of central access. In addition, the landmark approach to the axillary vein has been reported to be efficacious among severe burn victims who often present with burns of the face, neck, and proximal shoulders.71 Drawbacks of the axillary approach include the decreased diameter and deeper location of the axillary vein compared with the SV. Smaller caliber and deeper location may lead to difficulty visualizing it with a high-frequency transducer, especially in larger patients, and may increase the need for a longer catheter to reach the vena cava in larger patients. No studies specifically comparing the axillary and SV approaches have been published.

Evidence-Based Analysis Four studies of SV access were included in the 2003 NICE-sponsored meta-analysis.45 Of these, three used Doppler US guidance and one used real-time US guidance. The real-time US study evaluated 53 cannulation attempts among 32 critically ill patients in a combined trauma and medical intensive care unit.55 The investigators used the axillary vein approach for US guidance and reported an improved success rate compared with the landmark method (92% versus 44%, P = 0.003). With US guidance, there was also a decrease in the complication rate (4% versus 41%, P = 0.002), the mean number of attempts (1.4 versus 2.5, P = 0.0007), and the mean number of insertion kits used (1.0 versus 1.4, P = 0.0003). Malpositioned catheters in the landmark group also led to the need for additional chest radiographs. Fry et al30 reported 100% success with the US-guided placement of 43 SV catheters in patients who had relative contraindications to the landmark approach. The investigators subjectively noted that awake patients who have US-guided access “seem less apprehensive.” The investigators further suggested that “the ability to watch what is going on via the ultrasound video screen, a decrease in the number of attempts, and better local anesthesia along the intended needle path” contribute to improved patient satisfaction with US guidance.

FEMORAL VEIN APPROACH Anatomic Considerations Modern study of the FV has discovered variations from generally accepted anatomy. Reviewing CT scans of the pelvis in 100 patients, Baum et al72 discovered that a portion of the FV and the femoral artery overlaps in the anteroposterior plane 65% of the time. A subsequent study that used US in 50 intensive care unit patients confirmed this finding: “in most patients there was overlap of the artery over the vein far closer to the inguinal ligament than conventional anatomical textbooks would indicate”.73 Furthermore, landmarks were not predictive of the underlying anatomy that was documented on US.

Evidence-Based Analysis Only one randomized trial of US guidance for FV access has been published. This trial was undertaken among 20 patients undergoing cardiopulmonary resuscitation in the ED.40 Compared with the landmark approach, real-time US had a higher success rate (90% versus 65%, P = 0.058), a lower mean number of needle passes (2.3 ± 3 versus 5.0 ± 5, P = 0.006), and a lower rate of arterial catheterization (0% versus 20%, P = 0.025). The investigators suggested that the better performance of US was due to the ability to localize a nonpalpable FV by visualizing it instead. They also noted that chest compressions were associated with changes in FV diameter visualized on US. This finding is contrary to

e1313

e1314   SECTION 15  ■  SURGICAL CRITICAL CARE the expectation of arterial pulsations with chest compressions. It implies that palpating for a pulse during cardiopulmonary resuscitation as part of the landmark approach may be misleading. Furthermore, the color and pulsatility of returning blood may be unreliable for predicting arterial or venous source, especially in patients whose oxygen saturation is low.74 This study demonstrated the ability of US to accurately identify the FV without dependence on these traditional signs. In a prospective series, Kwon et al75 reported a 100% success rate among 28 patients who needed acute hemodialysis access. Compared with 38 patients with the landmark approach, these patients experienced a higher rate of successful first attempts (92.9% versus 55.3%, P < 0.05) and improved mean total procedure time (45.1 ± 18.8 versus 79.4 ± 61.7 seconds, P < 0.05). Femoral artery puncture occurred in 7.1% of US cases compared with 15.8% of landmark cases.

PERIPHERAL VEINS The subset of ED patients with poor peripheral access is well known to many emergency nurses and physicians. US offers a potential alternative to central venous access, surgical cutdowns, and blind, deep brachial vein catheterization for patients who need simple intravenous access but have no palpable or visible peripheral veins. Studies of peripherally inserted central venous catheter lines have shown that real-time US guidance is safe and successful in adult76,77 and pediatric populations.78 A case series of US-guided brachial and basilic vein cannulation among 100 ED patients with difficult intravenous access demonstrated a 91% overall success rate and a 73% rate of success on first attempt.42 Two cases of brachial artery puncture were reported. The mean time to successful cannulation was 77 ± 129 seconds (range, 4–600 seconds). No trials that have compared US guidance with the landmark approach have been published.

ISSUES IN PEDIATRIC PATIENTS Procedural Challenges Central venous access in infants and children is challenging under any circumstances. There are various possible reasons for the greater morbidity associated with central venous cannulation in pediatric patients: superficial anatomic landmarks may be less distinct, vessel diameters are generally smaller, the proximity of important anatomic structures may be greater, there may be less patient cooperation than among adults, anatomic anomalies may be present, and nonpediatric specialists may not be as experienced with pediatric vascular access.46,79 For these reasons, US guidance should be integrated into methods of central venous access for pediatric patients in the ED.

Evidence for Ultrasound Guidance in Pediatric Patients Studies of real-time US guidance for central vein cannulation in pediatric patients currently are found mainly in the anesthesia literature. Three randomized trials of US guidance versus landmark method for IJV cannulation among infants and children have been published.46–48 These trials and at least four case series34,79–81 constitute the current body of evidence that supports the application of US guidance in pediatric patients. The 2003 NICE-sponsored meta-analysis used the three trials to determine overall relative risk reductions of 85% for failed placement and 73% for complications of IJV cannulation in pediatric patients.45 Anatomic factors contribute to complications of the landmark approach to IJV cannulation in children. Alderson et al46 determined that 18% of patients aged 3 days to 5.5 years had anatomic variations of the IJV. The investigators reported a superior success rate (100% versus 80%), shorter mean time to successful cannulation (27.4 versus 48.9 seconds), and lower mean number of attempts (1.37 versus 2.0) with US guidance. Two patients in the landmark group and one in the US group suffered a carotid artery puncture. In comparing real-time US to the landmark approach for IJV cannulation among 95 infants aged 12 months or less who underwent elective cardiovascular surgery, Verghese et al47 found that the US approach significantly improved success and complication rates. US guidance was successful and had no associated carotid artery punctures. In contrast, the landmark method had a 77% success rate and a 25% rate of carotid artery puncture. Almost half of the patients with carotid artery puncture sustained

CHAPTER 69-2  ■  Ultrasound Guidance for Vascular Access  

additional complications, including hemothorax, pneumothorax, jugular venous hematoma, catheter kinking, and threading difficulty. Among the subset of patients with unsuccessful landmark attempts, 25% were subsequently successfully catheterized under US guidance. In a subsequent study, Verghese et al48 found statistically significant improvements in the success rate and median number of attempts with real-time US guidance over the landmark approach. The small sample size of 16 patients in each group, however, limited the statistical evaluation of trends toward improved time to successful cannulation and complication rates. Three carotid artery punctures occurred using the landmark approach compared with one using real-time US guidance.

Limitations of Pediatric Emergency Department Ultrasound-Guided Venous Cannulation The three existing trials have been criticized for their relatively small sample sizes of less than 100 patients each. Because these trials reported results only with IJV access, definitive conclusions regarding other venous sites presently are impossible. No studies have been reported on SV cannulation and only one small study reported the successful use of real-time US to facilitate FV catheterization.34 Although the pediatric studies consistently have reported positive findings for IJV cannulation, the importance of their success in the well-controlled, elective setting of scheduled surgery to the acute circumstances in the ED has yet to be demonstrated. To date, no studies of US-guided venous cannulation conducted in the pediatric ED setting have been published.

LIMITATIONS OF EMERGENCY DEPARTMENT ULTRASOUND-GUIDED VENOUS ACCESS Transducer Type Most US-guided vascular access is performed using a linear, high-frequency (6–10 MHz) transducer. The linear transducer provides a larger field of view compared with a sector transducer. This larger field of view allows visualization of the advancing needle through its entire course. In addition, because most of the target vessels are superficial, a high-frequency transducer can be used that yields superior resolution of the subcutaneous tissues, the advancing needle, and the vessels to be cannulated or avoided. As a separate purchase, the cost of a linear, high-frequency transducer can be prohibitive. Some EDs may not have budgeted for additional transducers when they originally acquired US equipment. Consequently, many emergency physicians do not have access to this transducer. One alternative is to use the large curvilinear transducer commonly used for abdominal imaging; however, this transducer has some drawbacks for real-time US guidance. Typically, the large curvilinear probe is bulkier than a linear transducer, and its lower-frequency images can make the procedure more technically challenging. The curvilinear transducer’s greatest obstacle is its curved visual field. Although the center of the visual field is relatively linear, the lateral aspects of the screen are curved to the extent that advancement of the needle under real-time US guidance is distorted. One possibility is to use the curvilinear transducer for indirect guidance, but this approach has not been formally studied. Another approach is to use an endovaginal transducer for US-guided vascular access. This transducer is a common component of many ED US systems and its use for venous access has been promoted in the gynecologic and emergency medicine literature.82,83 It has been described only for the short-axis approach to the IJV, and efficacy was not studied. For physicians with access to only the curvilinear or endovaginal transducer, the endovaginal transducer may be the superior choice.

Sterile Barrier Another equipment issue unique to US-guided vascular access is that a sterile barrier typically is needed. Such barriers usually are designed to cover both the transducer and its cable (see Figure 69-2-4) and allow sterile performance by a single operator. Although there are many relatively inexpensive varieties of sterile transducer covers, occasionally, one may not be available. In this case, other alternatives can be employed. The easiest method is to use a sterile glove. A conducting agent is placed inside the glove wherever the largest uninterrupted flat surface is located. An assistant can then place the transducer inside the glove. The operator then folds back the fingers of the glove and holds the transducer so that the flat surface of the glove forms

e1315

e1316   SECTION 15  ■  SURGICAL CRITICAL CARE the scanning surface for the transducer. It is important to eliminate any air bubbles that may be interposed between the glove and the transducer’s scanning surface because they would compromise image quality severely.

Mechanical Guides Mechanical guides typically come in two forms. The first is a built-in needle slot within the central or side portion of the transducer that directs the needle at a predetermined angle within the plane of view of the US beam. Another form is a separate guide that can be fitted to a transducer (see Figure 69-2-7). Presently, these guides are not interchangeable among different transducers. Most companies manufacture them for each linear transducer that they produce. Mechanical guides may not be necessary, especially for experienced operators.55

Education and Training Aids A significant issue pertaining to US-guided vascular access is the time and cost of training new operators. It is unfortunate that practical education for US-guided venous access currently is not available, and standard methods for teaching other US examinations, such as normal or dialysis models, cadavers, swine, or simulators, have significant limitations. Because the procedure is invasive, practicing on normal or dialysis models is problematic. Although the anatomy of cadavers may demonstrate vascular structures well,84 the entry site would be revealed after the initial puncture, thereby limiting the educational benefits for subsequent students. Swine or other animal models not only have unique vascular anatomy but ethical and cost issues also limit their use. Lastly, although simulation would seem to be an attractive alternative, only one vascular model currently exists. This model is limited to teaching peripheral vein cannulation using landmarks, not US guidance.85 A newer development in the area of US-guided vascular access education is the use of phantoms. Phantoms are generally easy and inexpensive to produce. They simulate vessels well and, hence, the mechanics of US-guided cannulation.

Time to Cannulation and Operator Experience The time required to set up and complete the procedure commonly is considered a drawback to US-guided vascular access. As with any novel procedure, there is a learning curve; however, this curve has been shown to be short and steep.58 Improved results have been demonstrated across the spectrum of operator experience.86 Furthermore, due to fewer failed attempts, the average time to vessel cannulation is the same as or is decreased with US guidance versus the landmark technique. Hilty et al40 compared US-guided FV cannulation with the landmark approach in 20 patients presenting in cardiac arrest. The average time to flash of blood under US guidance was 30.8 ± 32 seconds versus 33.8 ± 35 seconds for the landmark technique. Time to cannulation also was decreased with US guidance (121.0 ± 60 versus 124.2 ± 69 seconds). In another study that compared the short- and long-axis approaches on a US phantom, the mean time to cannulation was 2.36 minutes versus 5.02 minutes, which was statistically significant.59 Although it did not compare landmark and US-guided approaches, this study suggests that shorter intervals to vessel cannulation can be obtained, especially for inexperienced operators, when the vessel is approached in the short axis.

SUMMARY The evidence that supports the general application of US guidance for venous access in the ED has reached a critical mass. The increasing familiarity of emergency physicians with US and the recent focus on patient safety and clinical outcomes has intensified attention on the capacity for US to improve patient care in the ED. US guidance can increase the safety and efficiency of venous access procedures and offers improved outcomes. The potential for these improvements is compelling, especially among certain types of ED patients such as those with difficult or complicated access. Varying levels of evidence support the use of US guidance over the traditional landmark approach for venous access in adult and pediatric populations and for central and peripheral veins. Many different techniques may be applied, depending on the clinical situation and equipment available.

CHAPTER 69-2  ■  Ultrasound Guidance for Vascular Access  

References 1. Randolph AG, Cook DJ, Gonzales CA, Pribble CG. US guidance for placement of central venous catheters: a meta-analysis of the literature. Crit Care Med 1996;24:2053–8. 2. Tripathi M, Tripathi M. Subclavian vein cannulation: an approach with definite landmarks. Ann Thorac Surg 1996;61:238–40. 3. Simpson ET, Aitchison JM. Percutaneous infraclavicular subclavian vein catheterization in shocked patients: a prospective study in 172 patients. J Trauma 1982;22:781–4. 4. Scalea TM, Sinert R, Duncan AO, Rice P, Austin R, Kohl L, et al. Percutaneous central venous access for resuscitation in trauma. Acad Emerg Med 1994;1:525–31. 5. Williams MR, Dunn EL. Percutaneous central venous catheters – a continuum of use. J Emerg Med 1985;2:335–9. 6. Westfall MD, Price KR, Lambert M, Himmelman R, Kacey D, Dorevitch S, et al. Intravenous access in the critically ill trauma patient: a multicentered, prospective, randomized trial of saphenous cutdown and percutaneous femoral access. Ann Emerg Med 1994;23:541–5. 7. Parsa MH, Tabora F. Central venous access in critically ill patients in the emergency department. Emerg Med Clin N Am 1986;4:709–44. 8. Swanson RS, Uhlig PN, Gross PL, McCabe CJ. Emergency intravenous access through the femoral vein. Ann Emerg Med 1984;13:244–7. 9. Getzen LC, Pollak EW. Short-term femoral vein catheterization: a safe alternative venous access? Am J Surg 1979;138:875–8. 10. Emerman CL, Bellon EM, Lukens TW, May TE, Effron D. A prospective study of femoral versus subclavian vein catheterization during cardiac arrest. Ann Emerg Med 1990;19:26–30. 11. Dronen S, Thompson B, Nowak R, Tomlanovich M. Subclavian vein catheterization during cardiopulmonary resusci­ tation: a prospective comparison of the supraclavicular and infraclavicular percutaneous approaches. JAMA 1982;247: 3227–30. 12. Durbec O, Viviand X, Potie F, Vialet R, Albanese J, Martin C. A prospective evaluation of the use of femoral catheters in critically ill adults. Crit Care Med 1997;25:1986–9. 13. Chiang VW, Baskin MN. Uses and complications of central venous catheters inserted in a pediatric emergency department. Pediatr Emerg Care 2000;16:230–2. 14. Brunnette DD, Fischer R. Intravascular access in pediatric cardiac arrest. Am J Emerg Med 1988;6:577–9. 15. Newman BM, Jewett TC Jr, Karp MP, Cooney DR. Percutaneous central venous catheterization in children: first line choice for venous access. J Pediatr Surg 1986;21:685–8. 16. Casado-Flores J, Barja J, Martino R, Serrano A, Valdvielso A. Complications of central venous catheterization in critically ill children. Pediatr Crit Care Med 2001;2:57–62. 17. Sznajder JI, Zveibil FR, Bitterman H, Weiner P, Bursztein S. Central vein catheterization: failure and complication rates by three percutaneous approaches. Arch Intern Med 1986;146:259–61. 18. Bo-Lin GW, Andersen DJ, Andersen KC, McGoon MD. Percutaneous central venous catheterization performed by medical house officers: a prospective study. Cathet Cardiovasc Diagn 1982;8:23–9. 19. Jastremski MS, Matthias HD, Randell PA. Femoral venous catheterization during cardiopulmonary resuscitation: a critical reappraisal. J Emerg Med 1984;1:387–91. 20. Mansfield PF, Hohn DC, Fornage BD, Gregurich MA, Ota DM. Complications and failures of subclavian-vein catheterization. N Engl J Med 1994;331:1735–8. 21. Goldfarb G, Lebrec D. Percutaneous cannulation of the internal jugular vein in patients with coagulopathies: an experience based on 1000 attempts. Anesthesiology 1982;56:321–3. 22. Merrer J, De Jonghe B, Golliot F, Lefrant J, Raffy B, Barre E, et al. Complications of femoral and subclavian venous catheterization in critically ill patients. JAMA 2001;286:700–7. 23. Cook TL, Dueker CW. Tension pneumothorax following internal jugular cannulation and general anesthesia. Anesthesiology 1976;45:554–5. 24. McEnany MT, Austen WG. Life-threatening hemorrhage from inadvertent cervical arteriotomy. Ann Thorac Surg 1997;24:233–6. 25. Mason MS, Wheeler JR, Jaffe AH, Gregory RT. Massive bilateral hydrothorax and hydromediastinum: an unusual complication of percutaneous internal jugular vein cannulation. Heart Lung 1980;9:883–6. 26. Parikh RK. Horner’s syndrome: a complication of percutaneous catheterisation of internal jugular vein. Anaesthesia 1972;27:327–9. 27. Digby S. Fatal respiratory obstruction following insertion of a central venous line. Anaesthesia 1994;40:1013–4. 28. Johnson FE. Internal jugular vein catheterization: prospective study. N Y State J Med 1978;78:2168–71. 29. Hatfield A, Bodenham A. Portable US for difficult central venous access. Br J Anaesth 1999;82:822–6. 30. Fry WR, Clagett GC, O’Rourke PT. US-guided central venous access. Arch Surg 1999;134:738–41. 31. Gilbert TB, Seneff MG, Becker RB. Facilitation of internal jugular venous cannulation using an audio-guided Doppler US vascular access device: results from a prospective, dual center, randomized, crossover clinical study. Crit Care Med 1995;23:60–5. 32. Bold RJ, Winchester DJ, Madary AR, Gregurich MA, Mansfield PF. Prospective, randomized trial of Doppler-assisted subclavian vein catheterization. Arch Surg 1998;133:1089–93. 33. Miller AH, Roth BA, Mills TJ, Woody JR, Longmoor CE, Foster B. US guidance versus the landmark technique for the placement of central venous catheters in the emergency department. Acad Emerg Med 2002;9:800–5. 34. Sheridan RL, Petras L, Lydon M. Ultrasonic imaging as an adjunct to femoral vein catheterization in children. J Burn Care Rehabil 1997;18:156–8.

e1317

e1318   SECTION 15  ■  SURGICAL CRITICAL CARE 35. Cook T, Roepke T. Prevalence and structure of US curricula in emergency medicine residencies. J Emerg Med 1998;16:655–7. 36. Ullman JI, Stoelting RK. Internal jugular vein location with the US Doppler blood flow detector. Anesth Analg 1978; 57:118. 37. Legler D, Nugent M. Doppler localization of the internal jugular vein facilitates central venous cannulation. Anesthesia 1984;60:481–2. 38. Yonei A, Nonoue T, Sari A. Real-time ultrasonic guidance for percutaneous puncture of the internal jugular vein. Anesthesiology 1986;64:830–1. 39. Hudson PA, Rose JS. Real-time ultrasound-guided internal jugular vein catheterization in the emergency department. Am J Emerg Med 1997;15:79–82. 40. Hilty WM, Hudson PA, Levitt MA, Hall JB. Real-time US-guided femoral vein catheterization during cardiopulmonary resuscitation. Ann Emerg Med 1997;29:331–7. 41. Hrics P, Wilber S, Blanda MP, Gallo U. US-assisted internal jugular catheterization in the ED. Am J Emerg Med 1998;16:401–3. 42. Keyes LE, Frazee BW, Snoey ER, Simon BC, Christy D. US-guided brachial and basilic vein cannulation in emergency department patients with difficult intravenous access. Ann Emerg Med 1999;34:711–4. 43. American College of Emergency Physicians. Use of US imaging by emergency physicians. Ann Emerg Med 2001;38:469–70. 44. Agency for Healthcare Research and Quality. Making health care safer: a critical analysis of patient safety practices [summary]. Evid Rep Technol Assess 2001;43:i–x, 1–668. 45. Hind D, Calvert N, McWilliams R, Davidson A, Paisley S, Beverley C, et al. Ultrasonic locating devices for central venous cannulation: meta-analysis. BMJ 2003;327:361–7. 46. Alderson PJ, Burrows FA, Stemp LI, Holtby HM. Use of ultrasound to evaluate internal jugular vein anatomy and to facilitate central venous cannulation in pediatric patients. Br J Anaesth 1993;70:145–8. 47. Verghese ST, McGill WA, Patel RI, Sell JE, Midgely FM, Ruttiman UE. Ultrasound-guided internal jugular venous cannulation in infants: a prospective comparison with the traditional palpation method. Anaesthesiology 1999;91: 71–7. 48. Verghese ST, McGill WA, Patel RI, Sell JE, Midgely FM, Ruttiman UE. Comparison of three techniques for internal jugular vein cannulation in infants. Paediatr Anaesth 2000;10:505–10. 49. Nemcek AA. The use of ultrasound as an adjunct to the performance of vascular procedures. J Vasc Interv Radiol 1996;7:869–75. 50. Nadig C, Leidig M, Schmiedeke T, Hoffken B. The use of ultrasound for placement of dialysis catheters. Nephrol Dial Transplant 1998;13:978–81. 51. Williamson RA, Varner MW, Weiner CP. Use of needle guide to improve sonographically directed amniocentesis. Am J Obstet Gynecol 1984;149:107–8. 52. Williamson RA, Varner MW, Grant SS. Reduction in amniocentesis risks using a real-time needle guide procedure. Obstet Gynecol 1985;65:751–5. 53. Petrikovsky B, Schneider EP, Klein VR, Wyse LJ. Cordocentesis using the combined technique: needle guide-assisted and free-hand. Fetal Diagn Ther 1997;12:252–4. 54. Hatada T, Ishii H, Ichii S, Okada K, Yamamura T. Ultrasound-guided fine-needle aspiration biopsy for breast tumor: needle guide versus freehand technique. Tumori 1999;85:12–4. 55. Gualtieri E, Deppe SA, Sipperly ME, Thompson DR. Subclavian venous catheterization: greater success rate for less experienced operators using ultrasound guidance. Crit Care Med 1995;23:692–7. 56. Parker SH, Stavros AT, Dennis MA. Needle biopsy techniques. Radiol Clin N Am 1995; 33:1171–86. 57. Matalon TAS, Silver B. US guidance of interventional procedures. Radiology 1990;174:43–7. 58. Denys BG, Uretsky BF, Reddy PS. Ultrasound-assisted cannulation of the internal jugular vein: a prospective comparison to the external landmark-guided technique. Circulation 1993;87:1557–62. 59. Blaivas M, Brannam L, Fernandez E. Short axis versus long axis approaches for teaching ultrasound guided vascular access [abstract]. Acad Emerg Med 2003;10:572b. 60. Weissleder R, Elizondo G, Stark DD. Sonographic diagnosis of subclavian and internal jugular vein thrombosis. J Ultrasound Med 1987;6:577–87. 61. Denys BG, Uretsky BF. Anatomical variations of internal jugular vein location: impact on central venous access. Crit Care Med 1991;19:1516–9. 62. Troianos CA, Kuwik RJ, Pasqual JR, Lim AJ, Odasso DP. Internal jugular vein and carotid artery anatomic relation as determined by ultrasonography. Anesthesiology 1996;85:43–8. 63. Docktor B, So B, Saliken JC, Gray R. Ultrasound monitoring in cannulation of the internal jugular vein: anatomic and technical considerations. Can Assoc Radiol J 1996;47:195–201. 64. Hayashi H, Amano M. Does ultrasound imaging before puncture facilitate internal jugular vein cannulation? Prospective randomized comparison with landmark-guided puncture in ventilated patients. J Cardiothorac Vasc Anesth 2002;16: 572–5. 65. Skolnick ML. The role of sonography in the placement and management of jugular and subclavian central venous catheters. AJR 1994;163:291–5. 66. Silberzweig JE, Mitty HA. Central venous access: low internal jugular vein approach using imaging guidance. AJR 1998;170:1617–20. 67. Galloway S, Bodenham A. Ultrasound imaging of the axillary vein – anatomical basis for central venous access. Br J Anaesth 2003;90:589–95. 68. Nickalls RWD. A new percutaneous infraclavicular approach to the axillary vein. Anaesthesia 1987;42:151–4.

CHAPTER 69-2  ■  Ultrasound Guidance for Vascular Access   69. Taylor BL, Yellowlees I. Central venous cannulation using the infraclavicular axillary vein. Anesthesiology 1990;72:55–8. 70. Metz RI, Lucking SE, Chaten FC, Williams TM, Mickell JJ. Percutaneous catheterization of the axillary vein in infants and children. Pediatrics 1990;85:531–3. 71. Andel H, Rab M, Felfernig M, Andel D, Koller R, Kamolz LP, et al. The axillary vein central venous catheter in severely burned patients. Burns 1999;25:753–6. 72. Baum PA, Matsumoto AH, Teitelbaum GP, Zuuribier RA, Barth KH. Anatomic relationship between the common femoral artery and vein: CT evaluation and clinical significance. Radiology 1989;173:775–7. 73. Highes P, Scott C, Bodenham A. Ultrasonography of the femoral veins in the groin: implications for vascular access. Anesthesia 2000;55:1198–202. 74. Todd MR, Barone JE. Recognition of accidental arterial cannulation after attempted central venipuncture. Crit Care Med 1991;19:1081–3. 75. Kwon TH, Kim YL, Cho DK. Ultrasound-guided cannulation of the femoral vein for acute haemodialysis access. Nephrol Dial Transplant 1997;12:1009–12. 76. Sofocleous CT, Schur I, Cooper SG, Quintas JC, Brody L, Shelin R. Sonographically guided placement of peripherally inserted central venous catheters: review of 355 procedures. AJR 1998;170:1613–6. 77. Chrisman HB, Omary RA, Nemcek AA, Ryu RK, Saker MB, Vogelzang RL. Peripherally inserted central venous catheters: guidance with the use of US versus venography in 2650 patients. J Vasc Interv Radiol 1999;10:473–5. 78. Donaldson JS, Morello FP, Junewick JJ, O’Donovan JC, Lim-Dunham J. Peripherally inserted central venous catheters: US-guided vascular access in pediatric patients. Radiology 1995;197:542–4. 79. Etheridge SP, Berry JM, Krabill KA, Braunlin EA. Echocardiographic-guided internal jugular venous cannulation in children with heart disease. Arch Pediatr Adolesc Med 1995;149:77–80. 80. Asheim P, Mostad U, Aadahl P. Ultrasound-guided central venous cannulation in infants and children. Acta Aneasthesiol Scand 2002;46:390–2. 81. Liberman L, Hordof AJ, Hsu DT, Pass RH. Ultrasound-assisted cannulation of the right internal jugular vein during electrophysiologic studies in children. J Inter Card Electrophysiol 2001;5:177–9. 82. Jelsema RD, Deppe G, Isada NB. Vaginal probe ultrasound guidance for internal jugular catheterization. J Gynecol Surg 1992;8:243–5. 83. Phelan MP. A novel use of the endocavity (transvaginal) ultrasound probe: central venous access in the ED. Am J Emerg Med 2003;21:220–2. 84. Nip IL, Haruno MM. A systematic approach to teaching insertion of a central venous line. Acad Med 2000;75:552. 85. Reznek MA, Rawn CL, Krummel TM. Evaluation of educational effectiveness of a virtual reality intravenous insertion simulator. Acad Emerg Med 2002;9:1319–25. 86. Geddes CC, Walbaum D, Fox JG, Mactier RA. Insertion of internal jugular hemodialysis cannulae by direct US guidance – a prospective comparison of experienced and inexperienced operators. Clin Nephrol 1998;50:320–5.

Further Reading Troianos CA, et al. Guidelines for performing ultrasound guided vascular cannulation: recommendations of the American Society of Echocardiography and the Society of Cardiovascular Anesthesiologists. Anesth Analg;114(1): 46–71, 2012.

e1319

69-3 

CENTRAL VENOUS LINE PLACEMENT L.A. Fleisher  /  R. Gaiser From Procedures Consult

The video for this procedure can be accessed here

e1320

SELF ASSESSMENT Joseph R. Durham  /  José M. Velasco  /  Vikram D. Krishnamurthy  /  Tina J. Hieken From Velasco J: Rush University Medical Center Review of Surgery, 5th edition (Saunders 2011)

69-4 

1. Concerning acoustic impedance, which of the following statements is true? A. It can be amplified by increasing the gain on the ultrasound equipment. B. It is influenced by the density of the tissue and the velocity of the sound wave. C. It permits the operator to distinguish between two structures even if their densities are the same. D. It is calculated by multiplying the amplitude of the waves by the density of the tissue. E. The greater the difference in impedance between two tissues, the less energy is reflected to the transducer. Ref.: 1 COMMENT: Diagnostic ultrasonography is centered on the analysis of sound waves that have been reflected back to the ultrasound transducer. Impedance is the acoustic resistance to sound traveling in a medium. Acoustic impedance is dependent on the speed of sound in the tissue and the density of the tissue and can be calculated as acoustic impedance = density × velocity. Sound wave properties are not the only parameters that shape ultrasound physics. The medium (tissue) carrying the sound is a major contributor to events. The compressibility of a material determines, in part, the way that sound is carried along within that material. Because sound forms compressions and rarefactions, the ability of the tissue to be compressed and stretched determines just how well sound can be propagated through the tissue. Hard tissues (e.g., bone) are difficult to compress and thus impede the formation of compressions and rarefactions when they carry sound waves. As a result, hard materials have high acoustic impedance when compared with softer tissues (e.g., muscle), which have low acoustic impedance. Therefore, the ease with which sound is transmitted through a substance is termed impedance. The interface between two adjacent tissues serves as a major source for reflecting sound waves back to the transducer. When two adjacent tissues have different impedance values, the sound wave reflects back to the transducer. The greater the difference in impedance between the two tissues, the less energy is transferred to the next tissue and more energy is reflected back. Fortunately, differences in impedance between most soft tissues are small. These small differences are enough to cause a reflection of the sound waves to provide the information for generating an image; at the same time, the differences are small enough to allow enough amplitude for passage of some sound waves past the tissue interface into deeper tissues. These small differences in impedance are sufficient to make ultrasound a workable diagnostic modality. Increasing the gain on the machine does not affect any of these parameters.

ANSWER: B 2. Which of the following statements regarding transducers is false? A. Higher-frequency transducers have poor penetration and good resolution B. The higher the frequency, the shorter the wavelength. C. Longer wavelengths result in deeper penetration. D. Axial resolution is independent of frequency. E. The piezoelectric effect is defined as the “conversion of electrical to mechanical energy.” Ref.: 1 COMMENT: Ultrasound transducers contain crystals. When a sound wave mechanically deforms one of the crystals, voltage is produced. The corollary is also true: when a crystal has voltage applied e1321

e1322   SECTION 15  ■  SURGICAL CRITICAL CARE to it, it deforms and a sound wave is generated. This is described as the piezoelectric effect and has practical applications to the field of ultrasonography. The crystals used in ultrasound machines initially act as speakers that send out and receive sound waves. The returning sound that is reflected back causes the crystals to vibrate and generate voltage. High-frequency transducers provide high-resolution images at the expense of tissue penetration. In ultrasonography, three types of resolution exist: axial resolution, lateral resolution, and temporal resolution. Axial resolution is the ability to distinguish one object from another object below it. It is dependent on frequency. By definition, a higher frequency means a shorter wavelength. Because the depth of penetration is dependent on the wavelength, a higher frequency results in less tissue penetration. Lateral resolution is the ability to differentiate between two objects that are next to each other. It is independent of frequency and is dependent on the width of the beam. Temporal resolution is the perception of real-time movement and is dependent on the frame rate.

ANSWER: D 3. Regarding vascular arterial ultrasound imaging, which of the following statements is true? A. In Doppler ultrasound of blood flow, the reflected wave returning to the transducer has the same frequency as the transmitted wave. B. For Doppler ultrasound, the transducer should be held at a 90-degree angle to the body. C. Arterial stenosis leads to decreased flow velocity. D. Carotid artery duplex ultrasound scanning allows assessment of arterial plaque morphology, as well as estimation of the degree of carotid artery stenosis caused by the plaque. E. In dialysis access patients, duplex ultrasonography does not generally assess arterial inflow for arteriovenous (AV) fistulas or grafts accurately. Ref.: 1 COMMENTS: Doppler ultrasound relies on the fact that the sound wave that has been reflected back to the transducer from a moving object has a different frequency than does the transmitted wave. The change in frequency is known as the Doppler shift, named after the Austrian physicist Christian Doppler who described it in 1842. If the transducer is held at a 90-degree angle while performing Doppler ultrasonography, regardless of the actual velocity in a blood vessel, the ultrasound machine will read zero velocity. This is because the theoretical velocity is calculated by the equation V = Δf c/2f cos θ (where V is velocity, c is the speed of sound in soft tissue, Δf is the change in frequency of reflected versus transmitted sound waves, f is the frequency of transmitted sound, and cos θ is the angle between the ultrasound wave and the direction of motion of the target). Because the cosine of 90 degrees is zero, the theoretical velocity would be zero if the transducer is held at a 90-degree angle to the target. The ideal angle of insonation is 60 degrees. Analysis of the Doppler shift is used to determine the speed and direction of blood flow. Unless the arterial stenosis is so severe that blood flow is slowed almost to zero, the velocity increases in arterial stenosis. As its name implies, duplex ultrasonography uses two diagnostic modalities: (1) high-resolution gray-scale B-mode imaging (anatomic information) and (2) Doppler spectral analysis of blood flow patterns (physiologic information). B-mode imaging allows visualization of plaque location, composition, and morphology. Soft plaques and plaques with an irregular intimal surface (ulceration) may be relatively unstable and pose more risk for cerebral thromboembolic events and stroke than might dense fibrous plaques with a smooth intimal lining. Doppler spectral measurement of flow velocities allows accurate assessment of the degree of carotid artery stenosis. As with other ultrasound diagnostic modalities, the accuracy and reliability of vascular ultrasound imaging are dependent on the skill and experience of the operator. Ultrasound imaging is very useful in patients requiring hemodialysis access. Vein mapping can be performed preoperatively to determine the best vessels to support an AV fistula and postoperatively to assess fistula maturation. It can also be used to assess the arterial inflow for a fistula or graft. Ultrasound of new AV grafts may be limited by air entrained in the wall of the prosthetic graft.

ANSWER: D

Reference 1. Machi J, Staren ED: Ultrasound for surgeons, ed 2, Philadelphia, 2005, Lippincott Williams & Wilkins.

Urinary Catheterization

GOALS/OBJECTIVES • • •

INDICATIONS TECHNIQUE (MALE VS FEMALE) COMPLICATIONS

70 

70-1 

URINARY BLADDER CATHETERIZATION Dan Vetrosky From Dehn RW, et al: Essential Clinical Procedures, 3rd edition (Saunders 2013)

PROCEDURE GOALS AND OBJECTIVES GOAL: To perform urinary bladder catheterization on a patient safely and accurately. OBJECTIVES: The student will be able to: • Describe the indications, contraindications, and rationale for performing urinary bladder catheterization. • Identify and describe common complications associated with performing urinary bladder catheterization. • Describe the essential anatomy and physiology associated with the performance of urinary bladder catheterization. • Identify the materials necessary for performing urinary bladder catheterization and their proper use.

BACKGROUND AND HISTORY Disease processes that require urinary bladder catheterization have existed since ancient times. Urethral strictures, bladder stones, and prostatism are among the first diseases that necessitated urinary bladder decompression by catheterization. The approach to urinary catheterization remains the same today as it was in ancient times. It is the technique of passing a hollow tube through the urethra into the urinary bladder for purposes of circumventing an obstructed urinary bladder or obtaining a sample of urine for analysis, or both. The first known urologic instruments would be considered somewhat barbaric by today’s standards. Ancient and medieval “urologists-lithot-omists” used perineal incision and metal and glass tubes to circumvent urinary obstruction. Today’s approach often uses a local anesthetic and urethral catheters made of rubber, latex, polytetrafluoroethylene (Teflon), or silicone polymers. Urethral catheterization is currently used for relief of bladder outlet obstruction or when measurement of urinary output must be precise (e.g., in multiple trauma, surgery, intensive care, renal failure).

INDICATIONS Reasons for passing a catheter into the urinary bladder are many. The most common uses of bladder catheterization are the following: To obtain a sterile urine sample, especially in the female patient To monitor urinary output closely in critically ill patients To facilitate urinary drainage in patients who are incapacitated (stroke, advanced Alzheimer disease, spinal transection, etc.) To bypass obstructive processes in the urethra, prostate, or bladder neck caused by disease or trauma until surgical repair can be performed To hold urethral skin grafts in place after urethral stricture repair To act as a traction device for the purpose of controlling bleeding after prostate surgery Specialized three-way Foley catheters are used after bladder or prostate surgery to allow continuous bladder irrigation. Continuous irrigation and drainage help prevent the formation of blood clots, e1324

CHAPTER 70-1  ■  Urinary Bladder Catheterization  

FIGURE 70-1-1  A, Three-way Foley-irrigation Foley catheter. B, Robinson, or straight, catheter. C, Coudé catheter. D, Foley catheter.

which can occlude a catheter and cause bladder obstruction. Three-way Foley catheters also allow easier evacuation of formed blood clots (Figure 70-1-1). The main reasons for using the one-time, straight, or Robinson catheter are as follows: To obtain a sterile urine sample or to decompress a distended bladder caused by an acute obstructive process As a protocol of intermittent catheterization in persons with neurogenic bladder: Catheterizing patients with neurogenic bladder at regular intervals with the Robinson catheter facilitates complete bladder emptying, routine urine sampling, and bladder training. After a time, some of these patients may be able to decrease the frequency of their catheterization, regain complete bladder control, or both. To deliver topical antineoplastic medication to the bladder in patients who have bladder cancer or deliver other topical medication to patients who suffer from interstitial cystitis Assess postvoid residual urine through catheterization; however, this is being replaced by postvoid ultrasound of the bladder

CONTRAINDICATIONS The only contraindication to inserting a catheter (either Robinson or Foley) is the appearance of blood at the urethral meatus in a patient who has sustained pelvic trauma. This finding can be an indication that the urethra has been partially or totally transected. Attempting to pass a catheter in this situation could cause a partial urethral transection to become total. A urologist should be consulted when blood at the urethral meatus is present in a patient with pelvic trauma. Allergy to materials used in the procedure, such as latex, rubber, tape, and lubricants, is also a contraindication.

e1325

e1326   SECTION 15  ■  SURGICAL CRITICAL CARE

POTENTIAL COMPLICATIONS Most of the complications with catheterization are seen in the male patient. Female patients rarely have urethral strictures, caused by traumatic catheterization. Because the female urethra is comparatively short, false passages are rarely created. Complications can include the following: Urethral dilation resulting from placement of long-term indwelling Foley catheter in women. Leaking can occur because of bladder spasm. Instead of treating the spasm, progressively larger diameter catheters are placed, causing urethral dilation and continuation of leaking. Urinary structural trauma may occur as a result of catheterization. Urinary tract infection may occur as a result of organisms on the catheter or transmitted during the procedure. Inflammation of the urinary tract may occur secondary to the procedure. Catheterizing a male patient with urethral stricture disease, bladder neck contracture, or an enlarged prostate; this may present some technical difficulties for the unsuspecting health care provider Passage of a Robinson or Foley catheter in a patient with urethral stricture disease or an enlarged prostate. This increases the danger of creating false passages in the urethra if excessive force is applied when resistance is met during the catheterization. The mechanism of injury occurs when the obstructive process deflects the catheter into the side wall of the urethra. If the clinician meets these types of obstructive processes and continues to apply excessive pressure in an attempt to bypass the blockage, the catheter can act like a drill and undermine the lining of the urethra, thus creating a false passage. The worst scenario in this situation would be pushing the catheter completely through the urethra into the surrounding tissue. This results in copious bleeding from the urethra and creates the possibility of urine and blood extravasating into the surrounding tissues. Having the catheter double back or make a U-turn at the site of obstruction. It is not uncommon to have the catheter tip reappear at the urethral meatus when a significant obstruction or bladder neck spasm is present. Improper securing or taping of the Foley catheter. Patient-caused trauma. Patients who are confused can pull out a fully inflated Foley catheter.

ESSENTIAL ANATOMY AND PHYSIOLOGY Urine is produced by the kidneys and transported to the bladder by the ureters, where it is stored for transport through the urethra during urination. Bladder catheterization involves the passage of a mechanical device into the bladder through the urethra. To accomplish this without damage requires an understanding of the anatomy of the lower urinary tract. Figure 70-1-2 illustrates the anatomy in relation to the location at which a urinary catheter would be placed for males and females. In females, the distance from the distal end of the urethra to the bladder is relatively short (1.5 to 2 inches) and the course through the urethra is relatively unobstructed. Because of this, bladder catheterization in the female patient is typically accomplished faster and with less discomfort than it is in the male patient. In males, the distance from the distal tip of the urethra to the bladder is longer (typically 6 to 7 inches; however, it can vary considerably) and is more circuitous than in females, thus making catheter insertion potentially more difficult. In males, the path to the bladder typically includes curves that may be encountered while traversing the penis as well as a sharp bend through the prostate. Occasionally, prostatic hypertrophy can make catheter insertion difficult because the pressure of the hypertrophic prostate can add a curvature to the urethra as well as produce urethral obstruction.

STANDARD PRECAUTIONS Practitioners should use Standard Precautions at all times when interacting with patients. Determining the level of precaution necessary requires the practitioner to exercise clinical judgment based on the patient’s history and the potential for exposure to body fluids or aerosol-borne pathogens.

CHAPTER 70-1  ■  Urinary Bladder Catheterization  

FIGURE 70-1-2  Anatomy of the female (left) and male (right) lower urinary tracts with catheters in place. (Redrawn from Potter PA, Perry AG. Fundamentals of Nursing, ed 4. St. Louis: Mosby; 1997, p 1324.)

PATIENT PREPARATION The following must be considered in preparation of the patient for bladder catheterization: Before the procedure, inform the patient how the catheterization will be performed and what he or she might expect to feel during the procedure. This will help secure the patient’s trust and cooperation. Do not tell the patient that he or she will not feel anything, because this would be untruthful and counterproductive during the procedure. Inform the patient that the passage of the catheter may feel as though he or she must urinate and that it will be slightly uncomfortable. Patient comfort must be a primary consideration if a sterile, atraumatic catheterization is to be accomplished. Explain to the patient the importance of being reasonably still and not touching your gloved hands or sterile implements. Typically, the patient is positioned in the supine position. Drapes should be placed to cover all but the genitalia. The female patient will need to abduct the legs laterally to allow easy access to the urethra.

MATERIALS Sterile tray or working area Vessel for collecting urine (sometimes included with tray) Sterile gloves Sterile lubricant or anesthetic jelly lubricant Antiseptic cleansing solution (typically povidone-iodine [Betadine]) Sterile gauze or cotton balls for cleansing the external exit of the urethra and the surrounding skin Sterile forceps Syringe filled with sterile water for catheter balloon, 5 to 30 mL depending on the balloon capacity of the catheter selected Urine collection tubing, bags, hardware, and specimen collection containers Sterile drapes to protect the sterile field and nonsterile drapes to maintain patient modesty Robinson or Foley catheter, 14, 16, or 18 Fr. If a Foley catheter is used, the kit will also contain a prefilled 10-mL Luer-tipped syringe to inflate the Foley balloon and can contain a preattached drainage bag (attached to the Foley catheter). The advantage of a preattached drainage bag is that once in place, the Foley catheter and the drainage bag are considered a sterile “closed

e1327

e1328   SECTION 15  ■  SURGICAL CRITICAL CARE system.” The disadvantage is the inability to obtain a specimen or irrigate the bladder without “breaking the seal” and making what was once a sterile closed system a “contaminated” open system.

TYPES OF CATHETERS Urinary catheters (Robinson, coudé, and Foley types) are made of various materials and are soft and flexible (see Figure 70-1-1). The most common, the Robinson or straight type, catheter is made of rubber. Catheters can be made of pure rubber, rubber with synthetic coatings such as latex, or pure latex. Pure silicone and silicone-coated catheters are also manufactured, although they are much more expensive than rubber or latex catheters. These coated catheters are more commonly seen in indwelling or Foley catheter lines. The coatings are touted to resist encrustation when left in the bladder for prolonged periods. Patients with latex allergies should not be catheterized with rubber or latex catheters. In such cases, catheters made of pure silicone are an acceptable alternative.

Robinson Catheter The Robinson catheter is also known as the straight catheter and is sterile if the package seal is not broken. It has a soft, rounded tip and one or two drainage eyelets on the tip side walls. The catheter is hollow, and the distal end is flared to facilitate urinary drainage. These catheters are designed for one-time use, hence the term in-and-out catheter (see Figure 70-1-1).

Coudé Catheter Coudé catheters have a bend at the distal tip that causes the catheter to follow the anterior surface of the male urethra. This bent tip facilitates the insertion of the catheter in patients with false passages, which typically occur on the posterior surface of the urethra.

Foley Catheter The Foley catheter is designed to remain in place in the bladder. It also is sterile, and its appearance is similar to that of the Robinson catheter, with a few exceptions. At the tip, behind the drainage eyelets, is an inflatable balloon. The balloon is inflated after the catheter is properly placed in the bladder to help keep the catheter seated in the bladder. The flared end of the catheter is located at the distal end and can be attached to a drainage bag. Also at the distal end is an elbow with a Luer-Lok cap attached. This elbow is the end of an extremely small lumen, which traverses the length of the catheter and ends in the balloon at the tip. The Luer-Lok cap allows the balloon to be inflated once the catheter is in place and deflated once the catheter must be removed. The balloon is typically inflated with sterile water. Use of saline is discouraged because of the possibility of crystal formation along the balloon’s lumen. Should this occur, the balloon might not deflate when the catheter must be removed. The two sizes of Foley catheter balloons are 5 and 30 mL. The most commonly used is 5 mL, and it is typically inflated with 10 mL of sterile water, which accounts for the lumen volume and the balloon volume; 30-mL balloons are used to ensure that the Foley catheter does not migrate into the prostatic fossa or out of the urinary bladder altogether. In addition, the 30-mL balloon can be inflated with 50 mL of sterile water and used as a traction stent after certain urologic procedures (e.g., radical prostatectomy, transurethral prostatectomy).

CATHETER SIZE REQUIREMENTS Urinary catheters come in various sizes and are measured according to the Charriére French scale (0.33 mm equals 1 Fr). A 3-Fr catheter is 1 mm in diameter; a 30-Fr catheter is 10 mm in diameter. The French size of the catheter depends on the patient and the catheter’s purpose. For example, pediatric boys will need a French size between 5 and 12 Fr. Adult men should be catheterized with a 16- or 18-Fr catheter. These sizes are slightly stiffer and will follow the anatomic curvature of the male urethra easier and better than smaller French catheters (14 Fr or smaller). Smaller French

CHAPTER 70-1  ■  Urinary Bladder Catheterization  

catheters have a tendency to turn around in the male urethra if the slightest resistance is met (especially at the bladder neck). The adult woman should also be catheterized with 16- or 18-Fr catheters, although a 14 Fr should be used most of the time to facilitate comfort. Larger French catheters (20 to 30 Fr) are used to evacuate blood clots in postoperative prostate surgery patients or in patients who are bleeding from the kidney or bladder.

FOLLOW-UP CARE AND INSTRUCTIONS Short-Term Catheterization or In-and-Out Catheterization Aftercare for the short-term and In-and-Out catheterization procedures are as follows: Complications are unlikely. The most common complications include irritation of the urinary tract and infection. Patients will most likely experience a burning sensation the first few times they urinate after catheter removal. Reassurance is usually all that is needed. Instruct the patient to monitor urination for continuous dysuria, urinary frequency, hematuria, and pyuria, as well as for systemic signs of urinary tract infection such as fever or back pain.

Indwelling Catheterization Aftercare instructions for the indwelling catheterization procedure are as follows: The two major risks associated with an indwelling urinary catheter are trauma and infection. After successful catheter placement, trauma is typically a result of not protecting the catheter properly. Instruct the patient that the catheter should be secured with tape at all times and that care should be taken not to snag the tubing on clothing or furniture in a way that would pull on the catheter. Infection prevention measures include the following: Advise the patient to always position the drainage bag below the bladder to prevent urine flowing back into the bladder. Instruct the patient to be careful to avoid kinks in the tubing system. Instruct the patient to monitor the bag often and ensure it is emptied before it becomes completely full. Caution the patient to be careful when emptying the bag or manipulating the drainage system, to avoid introducing contaminants. Instruct the patient to wash hands frequently and use latex gloves (if not allergic; if allergic to latex, indicate which type of gloves to obtain). Be careful not to have the drainage system come into contact with contaminated objects such as toilet bowls. Caution the patient to be aware of signs of infection, such as changes in the appearance of the urine or symptoms of a urinary tract infection and to call the office.

Bibliography Potter PA, Perry AG. Fundamentals of Nursing, ed 4. St. Louis: Mosby; 1997. Tanagho EM, McAninch JW, eds. Smith’s General Urology, ed 14. Norwalk, CT: Appleton & Lange; 1995.

e1329

70-2 

MALE BLADDER CATHETERIZATION From Tuggy ML, Garcia J: Atlas of Essential Procedures, 1st edition (Saunders 2010)

FIGURE 70-2-1  Bladder catheterization kit.

FIGURE 70-2-2  Open the kit and drape the patient.

e1330

CHAPTER 70-2  ■  Male Bladder Catheterization  

FIGURE 70-2-3  Cleanse urethral meatus.

FIGURE 70-2-4  Lubricate the catheter with 2% lidocaine jelly.

FIGURE 70-2-5  Gently insert the catheter.

e1331

e1332   SECTION 15  ■  SURGICAL CRITICAL CARE

FIGURE 70-2-6  Inflate the balloon tip (while it is inside the patient’s bladder).

FIGURE 70-2-7  Collect urine in a specimen cup.

FEMALE BLADDER CATHETERIZATION From Tuggy ML, Garcia J: Atlas of Essential Procedures, 1st edition (Saunders 2010)

70-3 

FIGURE 70-3-1  Bladder catheterization equipment.

FIGURE 70-3-2  Lubricate the catheter.

e1333

e1334   SECTION 15  ■  SURGICAL CRITICAL CARE

FIGURE 70-3-3  Insert the catheter through the urethra.

FIGURE 70-3-4  Collect a urine sample.

CHAPTER 70-3  ■  Female Bladder Catheterization  

FIGURE 70-3-5  Inflate the balloon.

e1335

70-4 

URETHRAL CATHETERIZATION: MALE T.W. Thomsen  /  G.S. Setnik From Procedures Consult

The video for this procedure can be accessed here

e1336

URETHRAL CATHETERIZATION: FEMALE T.W. Thomsen  /  G.S. Setnik From Procedures Consult

70-5 

The video for this procedure can be accessed here

e1337

70-6 

SELF ASSESSMENT Michael R. Abern  /  Kalyan C. Latchamsetty From Velasco J: Rush University Medical Center Review of Surgery, 5th edition (Saunders 2011)

M.N. Kulaylat  /  M.T. Dayton From Townsend CM, et al: Sabiston Textbook of Surgery, 19th edition (Saunders 2012)

1. Which of the following is not a principle of repair of an intraoperative ureteral injury? A. Use of nonabsorbable suture material B. Spatulation of the transected ends C. Foley catheter drainage D. Drainage E. Intraureteral stent Ref.: 1 COMMENTS: Ureteral injuries are usually iatrogenic and occur during the course of retroperitoneal dissection for various abdominal and pelvic conditions. In cases of transection, repair should be carried out with absorbable suture material and an indwelling intraureteral stent. Nonabsorbable sutures should be avoided because they may serve as a nidus for calculus formation. Extensive ureteral dissection should be avoided to preserve the segmental blood supply. Spatulation reduces the incidence of anastomotic stricture in the severed ureter. Drains should be placed to accommodate any anastomotic leak. When injury involves the pelvic ureteral segment, ureteroneocystostomy may be preferable. Percutaneous (or open) nephrostomy serves to divert urine from the repair site, thereby facilitating healing at the anastomotic site. Foley catheter drainage is important in the immediate postoperative period because an intraureteral stent allows reflux of bladder urine to the anastomosis.

ANSWER: A 2. Regarding bladder trauma, which of the following statements is true? A. Rupture is usually extraperitoneal when associated with pelvic fracture. B. A single-view retrograde cystogram in the emergency department demonstrates most significant bladder injuries. C. Primary closure is generally indicated for extraperitoneal ruptures. D. Intraoperative injury usually requires repair with a suprapubic cystostomy. E. Injuries at the dome of the bladder are typically extraperitoneal. Ref.: 2, 3 COMMENTS: Bladder injury may result from blunt or penetrating trauma or may occur during pelvic operations. When associated with pelvic fracture, the site of injury is usually extraperitoneal because it has been caused by the shearing force of the pelvic fracture. Extraperitoneal rupture without pelvic fracture is an infrequent occurrence. Isolated extraperitoneal bladder rupture is treated with 7 to 10 days of Foley catheter drainage. Blunt injury without pelvic fracture is associated with intraperitoneal rupture, particularly if the bladder is full at the time of injury, and results in perforation, typically at the dome of the bladder. Bladder injury should be suspected in any patient with lower abdominal trauma if there is any hematuria or the patient is unable to void. Single-view cystography may miss a significant injury. Anterior, posterior, lateral, oblique, and in particular, postvoid films are necessary. e1338

CHAPTER 70-6  ■  Self Assessment  

Alternatively, a CT cystogram may be performed by injecting 300 to 400 ml of contrast material through a Foley catheter followed by CT of the pelvis. The usual treatment of intraperitoneal rupture involves a two-layer, watertight closure with absorbable suture and transurethral or suprapubic bladder drainage. Iatrogenic injury recognized at the time of an operation does not generally require suprapubic cystotomy but does require repair with absorbable suture and urethral catheter drainage for 5 to 7 days. It is also necessary to be vigilant that the Foley catheter does not become obstructed, such as with blood, and cause the bladder to become distended.

ANSWER: A 3. A 65-year-old man is unable to void after an abdominoperineal resection. Postvoid residuals have been 600 to 800 mL. The treatment of choice is which of the following? A. Chronic Foley catheterization B. Transurethral resection of prostate (TURP) C. Clean intermittent catheterization D. Transurethral sphincterotomy E. α-Blockers alone Ref.: 4 COMMENTS: Bladder dysfunction has been reported in 10% to 50% of patients following abdominal perineal resection or other major pelvic surgery. The type of voiding dysfunction that occurs is dependent on the specific nerve involved and the degree of injury. Patients with urinary retention are best treated by clean intermittent catheterization. Most (>80%) resolve over a period of 3 to 6 months. The use of a chronic indwelling catheter is a reasonable choice in some patients, but the risk for infection is higher with chronic catheterization than with intermittent catheterization. The use of α-blockers alone or TURP is unlikely to be successful. Transurethral sphincterotomy does not treat the underlying problem and may result in incontinence.

ANSWER: C 4. Postrenal causes of acute renal failure include all the following except: A. Ureteral obstruction caused by stones B. Bladder dysfunction caused by nerve injury C. Urethral obstruction caused by prostatic enlargement D. A blocked Foley catheter E. Myoglobinuria COMMENTS: Myoglobin obstructs the collecting tubules with the kidney and is an intrarenal cause of acute renal failure. All others are examples of postrenal causes.

ANSWER: E

References 1. McAninch JW, Santucci RA: Renal and ureteral trauma. In Wein AR, Kavoussi LR, Novick AC, et al, editors: CampbellWalsh urology, ed 9, Philadelphia, 2007, WB Saunders. 2. Morey AF, Rozanski TA: Genital and lower urinary tract trauma. In Wein AR, Kavoussi LR, Novick AC, et al, editors: Campbell-Walsh urology, ed 9, Philadelphia, 2007, WB Saunders. 3. Olumi AF, Richie JP: Urologic surgery. In Townsend CM, Beauchamp RD, Evers BM, et al, editors: Sabiston textbook of surgery: the biological basis of modern surgical practice, ed 18, Philadelphia, 2008, Saunders. 4. Schaeffer AJ, Schaeffer EM: Infections of the urinary tract. In Wein AR, Kavoussi LR, Novick AC, et al, editors: CampbellWalsh urology, ed 9, Philadelphia, 2007, WB Saunders.

e1339

Focused Abdominal Sonography for Trauma (FAST)

GOALS/OBJECTIVES • •

INDICATIONS TECHNIQUE

71 

71-1 

DIAGNOSTIC PERITONEAL LAVAGE AND THE FOCUSED ASSESSMENT WITH SONOGRAPHY IN TRAUMA Gerald R. Fortuna, Jr. From Stehr W: Mont Reid Surgical Handbook, 6th edition (Saunders 2008)

FAST Sonography in trauma was first introduced by Kristensen and colleagues in 1971. It was popularized in the 1990s with several studies reported in the emergency medicine and surgery literature.

Indications Same as in DPL. Useful in all blunt abdominal traumas. Less useful in penetrating trauma. Currently, no literature exists to support its use in this setting; however, this is still dependent on operator and institutional preferences. Sensitivity, specificity, and accuracy are comparable with CT and DPL in experienced hands. There is a steep learning curve and is very much operator dependent. Expertise is gained between 50 and 200 examinations.

Advantages Rapid, noninvasive means to diagnose intra-abdominal injuries May be repeated quickly at the bedside, allowing ongoing investigation Not limited by contraindications of DPL Useful in diagnosing cardiac injuries with pericardial fluid through the subxiphoid window ○ Could aid in deciding which organ cavity to explore first in a patient with multisystem injuries

Disadvantages Not reliable in pediatric blunt trauma Operator dependence and experience

Complicating Factors Obesity Subcutaneous air

Technique – Same as That for Formal Ultrasound Includes subxiphoid pericardial window, hepatorenal fossa (Morison’s pouch), splenorenal fossa, and the bladder/pelvis; some institutions add windows of both the right and left paracolic gutters. Perform a second control scan after 30 minutes, used to detect progressive hemoperitoneum if there is initial doubt.

CT Scan Should be used in hemodynamically stable patients only ○ Time consuming, used when no immediate need for celiotomy ○ Provides most specific information about individual organ injuries and their extent ○ May diagnose retroperitoneal and pelvic organ injuries, which may be missed by DPL or physical examination e1342

CHAPTER 71-1  ■  DIAGNOSTIC PERITONEAL LAVAGE AND THE FOCUSED ASSESSMENT WITH SONOGRAPHY IN TRAUMA  

○ Relative contraindications ○ Delay time to use scanner if not immediately available ○ Uncooperative patient who cannot adequately be sedated ○ Inability to give intravenous contrast agent because of allergy or renal insufficiency

CT may miss some gastrointestinal, diaphragmatic, and pancreatic injuries. Presence of free fluid in a patient with no liver or splenic injury mandates early exploration for suspected injury to the gastrointestinal tract or its mesentery.

e1343

71-2 

THE ROLE OF FOCUSED ASSESSMENT WITH SONOGRAPHY FOR TRAUMA: INDICATIONS, LIMITATIONS AND CONTROVERSIES Michael Dunham  /  Mark McKenney  /  David Shatz From Asensio JA, Trunkey DD: Current Therapy of Trauma and Surgical Critical Care, 1st edition (Mosby 2008)

1 3 2

4

FIGURE 71-2-1  Transducer positions for focused assessment with sonography for trauma (FAST): 1, pericardial; 2, right upper quadrant; 3, left upper quadrant; and 4, pelvis.

e1344

CHAPTER 71-2  ■  The Role of Focused Assessment with Sonography for Trauma  

FIGURE 71-2-2  Positive pericardial view showing anechoic hemopericardium between liver and heart.

FIGURE 71-2-3  Sagittal view of right upper quadrant showing minimal hemoperitoneum between the right kidney and liver.

FIGURE 71-2-4  Sagittal view of right upper quadrant with moderate hemoperitoneum.

e1345

e1346   SECTION 16  ■  TRAUMA

FIGURE 71-2-5  Normal sagittal view of left upper quadrant showing the hyperechoic left kidney/spleen interface.

FIGURE 71-2-6  Sagittal view of left upper quadrant showing anechoic hemoperitoneum between left kidney and spleen.

CHAPTER 71-2  ■  The Role of Focused Assessment with Sonography for Trauma  

FIGURE 71-2-7  Positive longitudinal view of pelvis for hemoperitoneum posterior to bladder.

FIGURE 71-2-8  Algorithm for penetrating thoracoabdominal trauma. FAST, Focused assessment with sonography for trauma; OR, operating room.

e1347

71-3 

SELF ASSESSMENT Edie Y. Chan  /  Jamie Elizabeth Jones From Velasco J: Rush University Medical Center Review of Surgery, 5th edition (Saunders 2011)

1. A 28-year-old woman is an unrestrained driver in a motor vehicle crash. She has stable vital signs and left upper quadrant tenderness without signs of peritonitis. Select the most appropriate next step in management of the abdominal pain. A. Computed tomographic (CT) scan of the abdomen and pelvis B. Diagnostic peritoneal lavage (DPL) C. Admission for observation and serial abdominal examinations D. Abdominal ultrasound E. Exploratory laparotomy Ref.: 1 COMMENTS: Evaluation of any trauma patient should follow the advanced trauma life support (ATLS) principles, with a complete primary and secondary survey. It is during the secondary survey that a thorough abdominal examination, including inspection and palpation, is performed. CT is the best radiographic tool for diagnosing blunt abdominal injury; it has higher sensitivity than focused assessment with sonography for trauma patients (FAST) and is noninvasive, in contradistinction to diagnostic peritoneal lavage. In addition, CT is useful for evaluating the retroperitoneum, which DPL cannot. Absolute indications for immediate laparotomy in patients with blunt abdominal trauma are abdominal distention with cardiovascular instability despite resuscitation and the presence of peritonitis.

ANSWER: A 2. A 58-year-old man is a restrained passenger in a high-speed motor vehicle collision. On arrival at the emergency department his pulse is 118 beats/min with a blood pressure of 90/58 mm Hg. After infusion of 2 L of a crystalloid solution, his pulse decreases to 95 and blood pressure increases to 120/62. An Abdominal CT scan shows an isolated splenic injury with a laceration 2 cm in parenchymal depth. Which of the following statements is true regarding this type of injury? A. Approximately 60% of all splenic injuries in adults are successfully managed nonoperatively. B. The type of injury in this patient has a 5% failure rate with nonoperative management. C. This patient’s age is associated with a higher failure rate with nonoperative management. D. This patient’s initial tachycardia and hypotension preclude him from nonoperative management. E. Nonoperative management of a grade V splenic injury is associated with an approximate 25% success rate. Ref.: 1, 2 COMMENTS: See Question 3. e1348

CHAPTER 71-3  ■  Self Assessment  

TABLE 71-3-1  Spleen Organ Injury Scale – 1994 Revision by the American Association for the Surgery of Trauma Grade I

Injury Description

III

Hematoma Laceration Hematoma Laceration Hematoma

IV

Laceration Laceration

II

V

Laceration Vascular

Subcapsular, 3 cm in parenchymal depth or involving a trabecular vessel Laceration involving the segmental or hilar vessels and producing major devascularization (>25% of spleen) Completely shattered spleen Hilar vascular injury that devascularizes the spleen

AIS-90 2 2 2 2 3 3 4 5 5

AIS, Abbreviated injury scale.

ANSWER: E 3. The same patient as in Question 2 is admitted for observation and serial hemoglobin tests. On hospital day 2, the patient’s heart rate increases to 120 beats/min and systolic blood pressure decreases to 100 mm Hg. His hemoglobin is now noted to be 7 g/dL. Select the next step in management. A. Transfusion of 2 units of packed red blood cells and serial hemoglobin determinations B. Angiography C. Repeated CT D. DPL E. Immediate laparotomy Ref.: 1, 2 COMMENTS: The spleen is the most commonly injured organ after blunt abdominal trauma. The patient described in this question has a grade II splenic laceration as diagnosed on CT (Table 71-3-1). Approximately 30% of all splenic injuries are treated operatively on arrival at the hospital. Of the remaining 60% to 70%, 80% to 90% of these are treated successfully with nonoperative management. Failure rates increase with the grade of injury, with a failure rate of 10% for grade I and II injuries, which increases to 75% for grade V injuries. Contraindications to conservative management include hemodynamic instability after adequate resuscitation, requirement for transfusion, and peritonitis. Independent risk factors for failure of nonoperative management include age older than 55 years, the presence of a pseudoaneurysm, and the amount of hemoperitoneum present on the initial CT. This patient fails conservative management because of continued hemodynamic instability and a requirement for transfusion. This necessitates laparotomy for splenectomy. The indication for angiography is the presence of an arteriovenous fistula or pseudoaneurysm on either initial or repeated CT, and it is usually performed 24 to 48 hours after admission with a splenic injury of grade III or higher.

ANSWER: E 4. Which of the following statements regarding FAST is true? A. A 2.5-MHz convex-array transducer should be used. B. The hepatorenal space, known as the Morison pouch, is viewed between the eleventh and twelfth ribs in the right midaxillary line. C. The splenorenal space is evaluated between the ninth and eleventh ribs in the left midaxillary line. D. The bladder should preferentially by emptied before examination to allow better visualization of fluid in the pelvis. E. FAST is an important part of the primary survey. Ref.: 3

e1349

e1350   SECTION 16  ■  TRAUMA COMMENTS: Focused assessment for the sonographic examination of trauma patients is performed as part of the ATLS secondary survey. A 3.5-MHz convex-array transducer is used to evaluate for the presence of fluid in the abdomen. Four areas are to be examined. The first is the pericardial window, which is viewed with the transducer placed subxiphoid. The hepatorenal space is evaluated in the right midaxillary line, between the eleventh and twelfth ribs. The splenorenal space is evaluated in the right posterior axillary line, between the ninth and eleventh ribs. The last area examined is the pouch of Douglas in the pelvis. This rectouterine/rectovesical space is evaluated with the transducer placed approximately 3 cm above the pubic symphysis. A full bladder actually helps elucidate the presence of blood in this space, and Foley catheters should be placed after FAST has been performed.

ANSWER: B

References 1. Hoyt DB, Coimbra R, Acosta J: Management of acute trauma. In Townsend CM, Beauchamp RD, Evers BM, et al, editors: Sabiston textbook of surgery: the biological basis of modern surgical practice, ed 18, Philadelphia, 2008, WB Saunders. 2. Edmonds RD, Peitzman AB: Injury to the spleen. In Cameron JL, editor: Current surgical therapy, ed 9, Philadelphia, 2008, CV Mosby. 3. Dente CJ, Rozycki GS: The surgeon’s use of ultrasound in thoracoabdominal trauma. In Cameron JL, editor: Current surgical therapy, ed 9, Philadelphia, 2008, CV Mosby.

Gastrointestinal Tract Injury – Operation

GOALS/OBJECTIVES • •

MANAGEMENT OF SMALL BOWEL INJURY ENTERIC FISTULA

72 

72-1 

MANAGEMENT OF ACUTE TRAUMA R. Shayn Martin  /  J. Wayne Meredith From Townsend CM: Sabiston Textbook of Surgery, 19th edition (Saunders 2012)

In contradistinction to spleen injuries, the operative intervention for liver trauma is less definitive and can be challenging. Therefore, hemodynamic decline requires operation but slow decreases in hemoglobin levels are at times tolerated and even occasionally treated with transfusion. This is especially true when there are other injuries that may account for some blood loss, and the decline in hemoglobin level may not be reflective of ongoing hepatic bleeding. Because many liver injuries are associated with some degree of hemoperitoneum, it is possible that a hollow visceral injury could be present but overlooked if the intra-abdominal fluid is attributed solely to the liver injury. Therefore, serial abdominal examinations to detect evidence of intestinal injury are an important part of nonoperative management of any solid abdominal organ. In some cases, CT reveals a liver injury that demonstrates the extravasation of IV contrast from a disrupted vascular structure. These appear as a blush of high-density contrast, often within the injuredappearing hepatic parenchyma. In the setting of hemodynamic stability, this extravasation is usually contained within a pseudoaneurysm. The natural history of hepatic pseudoaneurysms is not exactly known but it is believed that they may be associated with an increased risk of delayed bleeding, especially when caused by hepatic arterial branches. A more recent advance in the management of hepatic pseudoaneurysms is the use of hepatic angiography, with embolization of blood vessels that demonstrate extravasation. Even with successful embolization, patients need standard surveillance, which is required for all hepatic injuries managed nonoperatively. When selected appropriately, the use of angioembolization has improved the rate of successful nonoperative management by reducing the number of conversions to operative therapy.3,4 This has also allowed many higher grade injuries that historically might have required operation to be managed without surgery. The evolution of nonoperative approaches to liver trauma has required advances in evaluating and managing complications that arise. In addition to delayed rebleeding, these include bile leaks with biloma formation, hemobilia, and development of liver abscesses. Frequently, these are suggested by the development of abdominal symptoms, with or without evidence of systemic infection or inflammation. CT or, at times, ultrasound will identify the liver injury–related pathology. Percutaneous drainage guided by CT or ultrasound is usually successful in managing abscess or biloma. Endoscopic retrograde cholangiopancreatography (ERCP) with stent placement is occasionally required to decompress the biliary tree and promote healing of a bile leak. Occasionally, a laparoscopy or laparotomy is necessary to manage biliary ascites not amenable to percutaneous drainage. Operative management begins in the same fashion as with other abdominal injuries. A midline laparotomy is the most versatile approach for managing any liver injury that might be encountered. The falciform ligament is divided and perihepatic sponges are placed to manage bleeding from the liver temporarily. A fixed retractor is placed to expose the right upper quadrant structures. With perihepatic packing and manual compression, bleeding can be temporarily controlled and resuscitation provided. On patient stabilization, the packs are removed and the hepatic lacerations evaluated. Mild injuries with little or no ongoing bleeding may be managed with further compression, topical hemostatic agents, or suture hepatorrhaphy. Addressing these injuries may sometimes be facilitated by mobilizing the right or left hepatic lobes by dividing the triangular ligaments. This will allow injuries to be better exposed for interventions but may also allow better packing by optimizing anterior-toposterior compression. Occasionally, however, the risks of mobilization should be carefully considered if there is the possibility that the attachments of the liver are providing lifesaving tamponade of e1352

CHAPTER 72-1  ■  Management of Acute Trauma  

retrohepatic bleeding. This combination of superficial techniques will successfully manage most liver injuries encountered. In the setting of more severe bleeding, a Pringle maneuver is a valuable adjunct. The hepatoduodenal ligament is encircled with a vessel loop or vascular clamp to occlude hepatic blood flow from the hepatic artery and portal vein. This maneuver helps distinguish hepatic venous bleeding, which persists from a portal vein, and hepatic artery bleeding that slows, allowing identification of sources of hemorrhage. The hepatic laceration can then be explored and any actively bleeding vessels controlled with suture ligation. Grossly devitalized hepatic parenchyma should be débrided when accessible and drains should be placed when injuries appear to be at risk for a bile leak. When feasible, a vascularized pedicle of omentum may be packed within the liver injury to reduce parenchymal bleeding and promote healing of the laceration. Liver injuries in the vicinity of the retrohepatic vena cava that are not actively bleeding may benefit most from packing alone, without operative exploration. There are many heroic techniques seen in the literature that describe methods of repairing retrohepatic vena cava injuries, but it is likely that the approach with the greatest likelihood of success is maintaining the body’s natural tamponade of this low-pressure region when feasible. An atriocaval shunt (Shrock shunt) is one method that entails isolation of the retrohepatic vena cava by placing an intracaval shunt between the right atrium and infrahepatic vena cava. Isolation of the liver with an atriocaval shunt with the addition of a Pringle maneuver allows repair of the vena cava or hepatic veins without ongoing associated blood loss. Damage control techniques are often of great value because many patients who require operative intervention for liver injuries have already deteriorated physiologically. This approach includes control of surgical bleeding followed by aggressive perihepatic packing and temporary abdominal closure. It is fruitless to leave surgical bleeding and hope that packing alone will provide control. Similarly, it is futile to continue surgical attempts with sutures to control diffuse liver bleeding from coagulopathy. Patients are then resuscitated in the intensive care unit until hypothermia, coagulopathy, and acidosis resolve, at which time the abdomen is re-explored and the packs removed. Angiography with embolization after damage control may provide additional assistance with managing ongoing bleeding from hepatic artery branches, although the mortality rate in this patient cohort remains high.3 Gastric Injuries.  Gastric injuries most commonly occur after penetrating abdominal trauma, with the stomach being the injured organ in approximately 17% of cases identified in two separate series from busy urban trauma centers.2 This is similar to contemporary data obtained from the NTDB in which 18.1% of penetrating abdominal trauma involving the stomach were associated with a mortality rate of 19.7%. Penetrating injuries are frequently full-thickness perforations resulting in the spillage of gastric contents. Conversely, blunt gastric injuries are rare, occurring in 0.05% of all blunt trauma patients and 4.3% of patients with a blunt hollow visceral injury.5 These injuries are associated with a significant mortality rate, reaching 28.2% in an EAST multi-institutional trial. In this series, gastric injury was independently associated with death when analyzed by regression analysis (relative risk [RR], 2.8; 95% confidence interval [CI], 1.8 to 4.4).5 Blunt gastric injuries are equally as rare in the NTDB and are associated with a mortality rate of 28.3%. The proposed mechanism of blunt gastric rupture is an acute increase in intraluminal pressure from external forces that results in bursting of the gastric wall. Because of the high-energy nature of this mechanism, associated injuries are common and often include the liver, spleen, pancreas, and small bowel. Mortality is frequently attributed to these associated injuries. Gastric injuries will often be identified on physical examination by the presence of peritonitis. Some gastric injuries are identified by CT or DPL but the value of these modalities is limited. The evaluation of gastric injuries follows the approach to that for other hollow abdominal viscera (see earlier). Repair of gastric injuries is based on severity and injury location. Large intramural hematomas should be evacuated to ensure the absence of perforation, followed by control of bleeding and closure of the seromusculature with nonabsorbable suture. Full-thickness perforations should be débrided to remove nonviable gastric tissue and then closed with one or two layers. The perforation is generally closed with an absorbable suture, followed by inversion of the suture line with nonabsorbable seromuscular stitches. Because of the size and redundancy of the stomach, this can also be repaired with a stapling device. Perforations involving the gastroesophageal junction, lesser curve, fundus, and posterior wall may be more challenging to approach and require better exposure of the upper abdomen. Rarely, destructive injuries to the stomach involving large portions of the gastric wall require a partial or even

e1353

e1354   SECTION 16  ■  TRAUMA total gastrectomy. Reconstruction options include a Billroth I or II gastroenterostomy or creation of a Roux-en-Y esophagojejunostomy. Duodenal Injuries.  Duodenal injuries are uncommon after blunt and penetrating trauma but can be challenging to diagnose and manage. Most are caused by penetrating mechanisms occurring in 6.7% of penetrating abdominal cases, most of which being the result of gunshot wounds. The associated mortality rate is significant, 22.1% in the NTDB. Only 0.1% of patients experiencing blunt trauma sustain a duodenal injury. In those that present with a blunt hollow visceral injury, 12% are located in the duodenum.5 The mortality rate after blunt duodenal injury ranges from 11.4% to 14.8%. Blunt injuries are presumably caused by a blow to the epigastrium by a narrow object, resulting in contusion of the wall or a blowout secondary to acute elevation of intraluminal pressure. The classic description is the abdomen being struck by a steering wheel or, in children, a bicycle handlebar. Although duodenal injuries after penetrating trauma are found at laparotomy, their identification after a blunt mechanism can be challenging and therefore require a high index of suspicion to avoid missed injuries. Because of the retroperitoneal location of a significant portion of the duodenum, physical examination findings may be limited. Even full-thickness perforations of the duodenum may not demonstrate peritoneal signs unless the perforation involves an intraperitoneal segment. The mainstay of evaluation for duodenal injury has become abdominal CT, with a low threshold for operative exploration. Findings on CT that reflect possible duodenal injury include thickened duodenal wall, air or fluid outside the bowel lumen, and contrast extravasation if oral contrast was administered. Some authors advocate the administration of oral contrast whereas others have found that it is not necessary with current imaging capabilities.1 Low-grade injuries resulting in a duodenal hematoma can be identified by CT, although it is important also to evaluate the pancreas because of a high rate of concomitant injury. Any indication of duodenal perforation on examination or imaging should prompt operative exploration. At times, the findings are subtle but a low threshold for exploration should be maintained because of the potential for false-negative interpretations of the CT scan. Upper GI contrast studies, DPL, and laboratory studies such as serum amylase level determination, have at most a limited role in the evaluation of duodenal injuries. Management of duodenal injuries depends on the severity and location of the injury. Hematomas of the duodenal wall typically require no treatment unless they are large and result in a gastric outlet obstruction. Treatment of obstructing hematomas consists of gastric decompression and initiation of total parenteral nutrition, with re-evaluation of gastric emptying with a contrast study after 5 to 7 days. If after 2 weeks of upper GI bowel rest the obstruction persists, exploration is warranted to evaluate for perforation, stricture, or associated pancreatic injury. Duodenal hematomas identified at the time of laparotomy for another indication require careful evaluation for perforation. Frequently, they decompress during duodenal mobilization, although intentionally opening the serosa to drain an incidentally identified hematoma should generally be avoided in the absence of a full-thickness injury. Most full-thickness injuries of the duodenal wall can be repaired primarily using a single- or doublelayer approach, depending on the amount of tissue available. Adequate mobilization of the duodenum with a wide Kocher maneuver is required to provide necessary exposure and ensure a tension-free repair. Duodenal transection can be managed with primary anastomosis as long as the ampulla is not involved and the segment is short. Larger segments of duodenal destruction may require more complex reconstruction, frequently using bypass around the injured duodenum. Any repair can be protected from the enteric contents by performing a pyloric exclusion and creating a gastroenterostomy. In the damage control setting, the use of a duodenostomy tube or resection leaving the GI tract in discontinuity is highly effective for controlling contamination temporarily. Pancreatic Injuries.  Because of their adjacent location, injuries to the duodenum are frequently associated with pancreatic injuries. These are rare in blunt and penetrating mechanisms, occurring in only 0.09% of the patients in the NTDB. Of those that sustain penetrating injuries to the abdomen, the pancreas is involved in 6.6% of the cases. Despite the infrequency of these injuries, they remain a serious problem, resulting in mortality rates of 23.4% and 30.2% for blunt and penetrating mechanisms, respectively. These high mortality rates can frequently be attributed to delays in diagnosis and treatment. Because of the caustic nature of pancreatic enzymes, delays in managing pancreatic injuries result in massive systemic inflammation, with subsequent poor outcomes. Pancreatic injuries can result from direct penetration of the organ or through the transmission of blunt force energy to the

CHAPTER 72-1  ■  Management of Acute Trauma  

FIGURE 72-1-1  Pancreatic injury on abdominal CT scan. The injury involves the pancreatic neck and appears as a 2-cm segment of nonperfused pancreas tissue, with surrounding edema (arrow).

retroperitoneum. A commonly identified mechanism involves the crushing of the body of the pancreas between a rigid structure such as a steering wheel or seatbelt and the vertebral column. This can cause injury to the gland, ranging from mild contusion to complete transection with ductal disruption. The diagnosis of pancreatic injuries can be extremely challenging and no single imaging modality has been found to be highly effective. As with the duodenum, the retroperitoneal location of the pancreas makes physical examination less helpful for diagnosis. Abdominal imaging with IV-enhanced CT can indicate the pancreatic injury but the sensitivity is limited for parenchymal injury and pancreatic duct disruption, as identified recently in a large multicenter trial.6 Depending on the generation of scanner used, the sensitivity for detecting parenchymal or ductal injury did not surpass 60%. Peitzman and colleagues7 have evaluated the usefulness of CT prospectively and found a somewhat better sensitivity, approximately 80%, likely reflecting the variations in radiologic interpretation among centers. Nevertheless, CT alone may not be satisfactory to rule out a pancreatic injury and a high index of suspicion must be maintained. Findings on CT that suggest pancreatic injury include malperfusion of the pancreatic parenchyma indicating disruption, surrounding fluid, or hematoma and stranding in the adjacent soft tissue. Figure 72-1-1 demonstrates an injury at the neck of the pancreas on an abdominal CT scan. Given the limitations of imaging pancreatic trauma, the detection of injuries may require the use of other modalities. Although these injuries are uncommon, there is great value in minimizing the time to diagnosis because any delays could be associated with worse outcomes. Patients who are not responding appropriately to their known injuries require further evaluation for missed injuries. In this setting, repeat CT scanning may suggest a pancreatic injury that required time to develop radiographically evident pancreatic inflammation. Although not predictive as a screening tool, elevated serum amylase levels may reflect pancreatic trauma when obtained more than 3 hours after admission. Serum amylase levels may be sensitive but little is known about their specificity; therefore, the use of this indicator is limited and should not be routinely used. Imaging of the pancreatic ducts with ERCP and magnetic resonance cholangiopancreatography (MRCP) may be helpful, especially for those patients who have a suggestion of pancreatic injury but a lack of supporting studies. These modalities continue to be evaluated, but they may occasionally be of assistance in planning therapy and determining an operative approach. The mainstay of therapy for pancreatic injuries is surgical. Exposure of the entire gland to evaluate the pancreas comprehensively is required to exclude injury or select appropriate management. This exposure includes mobilization of the hepatic flexure of the colon and division of the gastrocolic ligament to retract the transverse colon and mesocolon inferiorly. A wide Kocher maneuver will mobilize the pancreatic head and facilitate evaluation. Assessment of the injury includes determining the degree of parenchymal involvement, location of the injury within the gland, and presence of pancreatic ductal

e1355

e1356   SECTION 16  ■  TRAUMA involvement. The management of pancreatic injuries with ductal involvement depends on the location of the injury. Injuries to the left of the superior mesenteric vessels are managed with a distal pancreatectomy. The proximal stump can be managed by individually ligating the duct and oversewing the parenchyma or using a stapling device. Covering the stump with omentum may be advantageous and a closed suction drain should be placed. Managing injuries of the ductal system within the head of the pancreas can be more challenging. Although some advocate resection in this setting, the associated morbidity can be great, often necessitating a more conservative approach. Managing these injuries with drainage alone often successfully diverts the leakage of pancreatic fluid externally, creating a controlled fistula that frequently will close spontaneously. This healing may also be promoted with biliary decompression through the placement of stents via ERCP. Massive destruction of the pancreatic head with devitalized parenchyma or combined pancreatic and duodenal injuries may require a pancreaticoduodenectomy (Whipple procedure). This can be extremely challenging in this setting and is associated with a high postoperative complication rate. Performing a Whipple procedure in the setting of trauma requires ongoing patient stability or the operation should be abbreviated, with later reconstruction after the physiologic condition improves. Damage control for pancreatic injury includes hemorrhage control, external drainage, and temporary abdominal closure with plans for re-exploration. Adequate external drainage is an important principle in the management of most pancreatic injuries. The diversion of leaking pancreatic enzymes is required to prevent the devastating effects of uncontrolled accumulation of highly caustic digestive fluid, which will provoke a massive inflammatory response and progressive organ dysfunction. Pancreatic injuries not involving the pancreatic duct, including hematomas, parenchymal contusions, and lacerations of the capsule or superficial parenchyma, should be managed with external drainage alone. External drainage should be with a closed suction system because these are associated with a reduced rate of abscess development.8 Distal feeding access should be considered based on the overall clinical picture. Figure 72-1-2 depicts an approach to the operative management of pancreatic injuries. Small Bowel Injuries.  Depending on the series reviewed, the small intestine is one of the most frequently injured organs after penetrating abdominal trauma, likely secondary to the large percentage of the abdomen it occupies. Although the incidence of small bowel injury after penetrating abdominal trauma has been described as high as 60%, these injuries are less common in the NTDB, identified in 21.8% of cases. Mortality rates range from 10% to 25%, with most caused by associated vascular injuries. Penetrating injuries can vary from tiny perforations to large destructive injuries that destroy circumferential segments of small bowel. Blunt, small intestinal injuries are less common, present in 2.7% of all blunt abdominal injuries in the NTDB, although these injuries are associated with a significant mortality rate of 16.3%. Mechanisms of blunt small bowel injury include crushing, rupture, and shearing types of patterns. The small bowel can be crushed between the steering wheel or seatbelt and a rigid structure such as the vertebral column, resulting in direct tissue injury. Similar forces can result in a rupture-type injury during which the intraluminal pressure rapidly increases, causing a blowout along the antimesenteric border. Finally, deceleration mechanisms can result in a shearing of the serosa or muscularis throughout a segment of small bowel. Mesenteric injuries can cause devascularization of sections of small bowel without direct tissue injury. Small intestinal injuries are often identified at the time of laparotomy. Otherwise, the evaluation can be challenging and is similar to the approach to other hollow abdominal viscera. The use of imaging and other modalities has been described earlier. The repair of small bowel injuries depends on the extent of intestinal wall destruction in relation to the luminal circumference. Serosal tears can be reinforced with interrupted nonabsorbable suture, which imbricates the injury. Small perforations that can be closed without compromising the intestinal lumen can be débrided and repaired with one or two layers. This can safely be performed for multiple perforations as long as closure will not result in obstruction of the enteric contents, although many choose resection when several injuries are close together. Injuries occupying over 50% of the intestinal wall circumference should be addressed with resection and anastomosis. There has been no difference demonstrated between stapled and hand-sewn anastomoses for intestinal resections. Selection of the anastomosis technique should be based on the experience of the surgeon, with the method of greatest comfort used. Hand-sewn anastomoses are frequently constructed in two layers but single-layer methods are equally efficacious. The damage control approach to small bowel injuries includes rapid

CHAPTER 72-1  ■  Management of Acute Trauma  

FIGURE 72-1-2  Algorithm for the operative management of pancreatic injury.

closure of perforations to control contamination and/or stapled resection of injured segments. Patients in shock may benefit from resection without immediate anastomosis because of related delays and a higher risk of anastomotic dehiscence. The abdomen is temporarily closed and the patient is resuscitated to correct physiologic derangements. Intestinal continuity can then be re-established on return to the operating room, following resuscitation. Colon Injuries.  Similar to other hollow viscera, colon and rectal injuries occur most commonly after penetrating abdominal trauma and rarely after blunt mechanisms. The colon is one of the most frequently involved organs after penetrating abdominal trauma, occurring in 36% to 40% of patients in a series of 250 cases.2 This incidence is similar to data from the NTDB, in which 34.3% of all cases of penetrating abdominal trauma involved the colon or rectum. The associated mortality for colon and rectal injuries is the lowest of all the abdominal viscera. Penetrating injuries can range in the degree of colonic wall destruction, depending on the level of energy associated with the mechanism. Penetrating injuries can also be obscured by the retroperitoneal location of some segments of colon. Blunt colon and rectal injury occur in less than 1% of all blunt trauma patients but, in those patients with blunt hollow visceral injury, the colon or rectum is involved in 30.2% of cases.5 Mortality after blunt colon or rectal injury equals 16.3%, with much of this caused by associated injuries. Injuries to the colon can result from similar biomechanical mechanisms as those that occur in the small bowel. The colonic wall can be crushed by physical forces or rupture when the impact results in a rapid elevation

e1357

e1358   SECTION 16  ■  TRAUMA

FIGURE 72-1-3  Blunt left-sided colon injury at the time of laparotomy. The injury mechanism resulted in a deserosalizing-type injury that involved a several-centimeter-long segment of colon.

in intraluminal pressure. Depending on the colonic segment involved, this perforation can occur into the retroperitoneum. The colon is also vulnerable to shearing forces, which can cause a separation of the serosa or muscularis over a long segment. Figure 72-1-3 demonstrates a segment of colon that was injured secondary to a shearing-type mechanism. Injury to the rectum can also occur when severe pelvic fractures result in a laceration by sharp bone fragments. As with other hollow organ injuries, colonic injuries may be identified first at the time of laparotomy that was prompted by hemodynamic instability or the appropriate penetrating mechanism. Otherwise, evaluation is as described earlier for other hollow abdominal viscera. Care must be taken to assess segments of the colon that are retroperitoneal in location adequately. Blood identified on rectal examination or a penetrating trajectory that suggests rectal involvement requires further evaluation. Rigid proctosigmoidoscopy can visualize the rectum and distal sigmoid colon to assist in determining the presence or absence of a rectal injury. This can be performed prior to laparotomy in hemodynamically stable patients to help plan the operative approach. Endoscopy may clearly reveal an injury to the rectum or only demonstrate hematoma in the rectal wall or a large amount of blood in the rectal vault. When possible, determining the size of the injury and location on the rectal wall may be valuable when planning the necessary management. Upper rectal injuries, especially those on the anterior or lateral surfaces, may be identified during examination of the pelvis during laparotomy. Operative repair of colon injuries depends on the severity of the colonic wall injury and the patient’s overall condition. Historically, it was believed that all colon injuries required resection with the creation of a colostomy because of a high risk of anastomotic dehiscence. A substantial amount of work has been dedicated to determining whether proximal fecal diversion was necessary to manage colonic perforation. Several randomized prospective trials have concluded that primary repair or resection with primary anastomosis is safe in select patients, resulting in a leak rate that was not significantly greater than that for colonic diversion.9,10 Therefore, injuries that involve less than 50% of the colonic wall circumference can be repaired with one or two layers, being sure to imbricate the mucosal edge. Usually, compromising the colonic lumen is not as common a concern as in the small bowel. Destructive colon injuries that involve more than 50% of the colonic wall should be resected; many can then be anastomosed immediately. Injuries proximal to the middle colic artery are managed with a right hemicolectomy with creation of ileocolostomy, because this has been found to be a durable anastomosis. Distal injuries require segmental resection with colocolostomy anastomosis. In the setting of shock, immediate anastomosis may be associated with an unacceptably high leak rate and should be carefully considered. There are two other options in the setting of hemodynamic instability to manage colon injuries. First, the injured segment can be resected and a diverting colostomy created. The second option is to resect the injured segment of colon and leave the GI tract in discontinuity until after the patient has been adequately resuscitated. On return visit to the operating room, delayed primary anastomosis or

CHAPTER 72-1  ■  Management of Acute Trauma  

creation of a colostomy can be completed. Leak rates after delayed primary anastomosis have been found to be equivalent to immediate anastomosis performed in the setting of hemodynamic stability.11 Other concerns that may suggest colostomy instead of primary repair or anastomosis include significant associated injuries, underlying medical disease, and delayed injury recognition with the development of severe peritoneal inflammation. Rectal injuries that result in perforation present a significant risk of developing pelvic sepsis and thus require operative management. The mainstays of treatment for rectal injuries are fecal diversion and presacral drainage until healing has occurred, at which time the colostomy is reversed. This can be achieved with an end colostomy or a loop configuration as long as complete fecal diversion can be achieved. Historically, drainage of the presacral space has been considered an important part of managing rectal perforations because of data generated in the military theater. More recently, some have countered this edict, concluding that presacral drainage is an unnecessary component, especially in the setting of low-energy, nonmilitary types of penetrating rectal trauma.12 Without definitive studies, one approach is to drain injuries that occur posteriorly or laterally, if in the lower third of the rectum, because these have likely entered the presacral space and are at greater risk of abscess formation. Other injuries to the extraperitoneal rectum can be managed with fecal diversion alone. Destructive rectal injuries that involve more than 50% of the rectal wall circumference may require resection of the rectum above the injury with the creation of an end colostomy. Abdominal Great Vessel Injuries.  The great vessels of the abdomen are located within the retro­ peritoneum and abdominal mesenteries. Injuries to these vessels can be challenging to manage given the amount of blood loss that can be present when these structures are injured. Although these injuries frequently occur after blunt trauma, it is most commonly during penetrating injury that exploration of this region is required. Often, hematoma within the retroperitoneum is secondary to a pelvic fracture because hemorrhage from the pelvic vessels can dissect superiorly through the surrounding tissue. Those concepts related to initial assessment and management will be presented here. Vascular injuries in the abdomen are often first recognized at the time of laparotomy for penetrating abdominal trauma. Frequently, these injuries are associated with significant ongoing blood loss and hemodynamic instability. Exploration of penetrating injuries to the retroperitoneum results in a definitive diagnosis. Penetrating injuries to the back frequently benefit from three-dimensional imaging, given that most do not enter the peritoneal cavity. Current CT can often identify the path of the injury and therefore suggest possible injury to adjacent structures. After blunt trauma, injuries to the abdominal vasculature with associated hematoma are often identified via contrast-enhanced CT. Occasionally, blunt trauma to the retroperitoneum with vascular injury is identified during urgently performed laparotomy, although further identification of specific injury depends on the location of the hematoma. Usually, penetrating injuries to the retroperitoneum identified during laparotomy require explora­ tion. A knowledge of the basic approach to and exposure of the abdominal vasculature is important. Hematomas in the vicinity of the right renal hilum or infrarenal vasculature benefit from a right medial visceral mobilization, also known as the Cattel-Brasch maneuver. A wide Kocher maneuver is performed and the peritoneal dissection is continued inferiorly to mobilize the right colon. The dissection is continued around the cecum and then superiorly up the mesenteric root, allowing all the abdominal viscera to be retracted to the left, thus exposing the midline vascular structures. Basic tenets of vascular repair are paramount, including proximal and distal control of the injured vessel, when feasible. Injuries to the left renal hilum or the suprarenal vessels can be exposed by performing a left medial visceral mobilization (the Mattox maneuver). This is performed by dividing the left lateral peritoneum from above the spleen to the distal left colon. The plane posterior to the colonic mesentery and the pancreas is developed and the abdominal viscera are retracted to the right to expose the superior retroperitoneal vasculature. Blunt abdominal vascular injuries that are not actively bleeding may require operation to repair or, as more recently found, may be considered for endovascular therapy. When confronted with a retroperitoneal hematoma during laparotomy for blunt trauma, the location of the hematoma suggests the appropriate treatment. Figure 72-1-4 depicts three zones used to classify these hematomas. Zone 1 hematomas require exploration because these frequently involve the aorta, proximal visceral vessels, or inferior vena cava. An exception may be the dark hematoma behind the liver, which suggests a retrohepatic vena cava injury. These injuries may be best served by not exposing the contained low-pressure

e1359

e1360   SECTION 16  ■  TRAUMA

FIGURE 72-1-4  Zones of the retroperitoneum visualized at the time of laparotomy. Zone 1 includes the central vascular structures, such as the aorta and vena cava. Zone 2 includes the kidneys and adjacent adrenal glands; zone 3 describes the retroperitoneum associated with the pelvic vasculature.

injury or by gently packing the surrounding area; heroic management techniques can be extremely challenging. A hematoma in the region of zone 2 should only be explored if it appears that the hematoma is expanding and continuing to lose blood. Finally, a hematoma in zone 3 is usually secondary to pelvic fracture bleeding and should not be explored unless exsanguinating hemorrhage is present.

References 1. Holmes JF, Offerman SR, Chang CH, et al: Performance of helical computed tomography without oral contrast for the detection of GI injuries. Ann Emerg Med 43:120–128, 2004. 2. Nicholas JM, Parker Rix E, Easley KA, et al: Changing patterns in the management of penetrating abdominal trauma: The more things change, the more they stay the same. J Trauma 55:1095–1110, 2003. 3. Duane TM, Como JJ, Bochicchio GV, et al: Reevaluating the management and outcomes of severe blunt liver injury. J Trauma 57:494–500, 2004. 4. Asensio JA, Roldàn G, Petrone P, et al: Operative management and outcomes in 103 AAST-OIS grades IV and V complex hepatic injuries: Trauma surgeons still need to operate, but angioembolization helps. J Trauma 54:647–654, 2003. 5. Watts DD, Fakry SM; EAST Multi-Institutional Hollow Viscus Injury Research Group: Incidence of hollow viscus injury in blunt trauma: An analysis from 276,557 trauma admissions from the EAST multi-institutional trial. J Trauma 54:289– 294, 2003. 6. Phelan HA, Velmahos GC, Jurkovich GJ, et al: An evaluation of multidetector computed tomography in detecting pancreatic injury: results of a multicenter AAST study. J Trauma 66:641–647, 2009. 7. Peitzman AB, Makaroun MS, Slasky BS, et al: Prospective study of computed tomography in the initial management of blunt abdominal trauma. J Trauma 26:585–592, 1986. 8. Fabian TC, Kudsk KA, Croce MA, et al: Superiority of closed suction drainage for pancreatic trauma. A randomized, prospective study. Ann Surg 211:724–728, 1990. 9. Stone HH, Fabian TC: Management of perforating colon trauma: Randomization between primary closure and exteriorization. Ann Surg 190:430–436, 1979. 10. Demetriades D, Murray JA, Chan L, et al: Penetrating colon injuries requiring resection: Diversion or primary anastomosis? An AAST prospective multicenter study. J Trauma 50:765–775, 2001. 11. Miller PR, Chang MC, Hoth JJ, et al: Colonic resection in the setting of damage control laparotomy: Is delayed anastomosis safe? Am Surg 73:606–610, 2007. 12. Gonzalez RP, Falimirski ME, Holevar MR: The role of presacral drainage in the management of penetrating rectal injuries. J Trauma 45:656–661, 1998.

SONOGRAPHY FOR TRAUMA C. Butts From Adams JG, et al: Emergency Medicine Clinical Essentials, 2nd edition (Saunders 2012)

72-2 

The video for this procedure can be accessed here

e1361

72-3 

SELF ASSESSMENT Alicia Growney  /  Steven D. Bines  /  Edie Y. Chan  /  Jacquelyn Turner  /  Theodore J. Saclarides  /  Daniel J. Deziel  /  Shaun Daly  /  Ai-Xuan L. Holterman From Velasco J: Rush University Medical Center Review of Surgery, 5th edition (Saunders 2011)

1. Which of the following regarding complicated intra-abdominal infections is true? A. They can be treated with intravenous antibiotics to eliminate the need for more invasive interventions. B. Isolated bacteria from colonic perforations are often aerobic gram-negative organisms. C. The infectious isolates from acute necrotizing pancreatitis and colonic perforation are similar. D. Bowel injuries secondary to penetrating, blunt, or iatrogenic trauma repaired within 12 hours of injury require no more than 72 hours of antibiotics. E. Single-agent therapy is often inadequate. Ref.: 1 COMMENTS: Complicated intra-abdominal infections are defined as infections that extend beyond the hollow viscus of origin into the peritoneal space and result in either peritonitis or abscess formation. These infections require either operative or percutaneous intervention in addition to the administration of antibiotics for resolution. For community-acquired infections, the pathogen isolated varies according to the location of the perforation along the gastrointestinal tract. More proximal infections are due to facultative and aerobic gram-positive and gram-negative organisms. Terminal ileum/colonic perforations result from facultative and obligate anaerobes such E. coli, enterococci, and streptococci. Infections resulting from necrotizing pancreatitis are due to pathogens similar to those found in colonic perforation infections. Bowel injuries caused by penetrating, blunt, or iatrogenic trauma and repaired within 12 hours of injury are adequately treated with 24 hours or less of antibiotics. Randomized prospective trials have demonstrated that single-agent therapies with β-lactam/βlactamase inhibitor combinations, carbapenems, or cephalosporins are adequate in the treatment of complicated intra-abdominal infections.

ANSWER: C 2. A 44-year-old man suffers a gunshot wound to his abdomen. He is hemodynamically stable and taken to the operating room. On exploration, his injuries are found to be limited to two small bowel injuries 7 cm apart, each with destruction of 70% of the bowel wall, and a through-and-through injury to the ascending colon with destruction of 30% of the bowel wall. How should these injuries be managed? A. Resection and anastomosis of the small bowel injuries and primary repair of the colon injury B. Primary repair of both the small bowel and colon injuries C. Primary repair of the small bowel injuries, primary repair of the colon injury, and creation of a diverting ileostomy D. Resection of the small bowel injuries and exteriorization of the colon injury as a colostomy E. Resection and anastomosis of all injuries Ref.: 2 e1362

CHAPTER 72-3  ■  Self Assessment  

COMMENTS: Historically, all colon injuries were treated by diversion. However, with the progression of surgical technique, resuscitative and critical care, and antibiosis, many colon injuries can be repaired primarily. Patients who are hemodynamically stable and have injuries that involve less than 50% of the circumferential bowel and no vascular disruption can undergo primary repair. In regard to small bowel injury, resection is indicated for injuries involving greater than 50% of the wall circumference, multiple injuries in a short segment, or both.

ANSWER: A 3. A 56-year-old woman underwent pelvic radiation therapy 5 years ago for cervical cancer. Now, 5 days after a right hemicolectomy for villous adenoma of the cecum, her surgical wound is red and tender. The surgeon opens her wound, and the initial drainage is obviously purulent. The drainage persists as a continuous brown, liquid discharge. Which of the following is the most likely diagnosis? A. Simple wound infection B. Clostridial infection C. Anastomotic leakage with an enterocutaneous fistula D. Dehiscence E. Cellulitis Ref.: 3 COMMENTS: Most fistulas are iatrogenic and result from anastomotic leakage, inadvertent injury to the bowel during the operation, laceration of the bowel during abdominal closure, or retained foreign bodies. Less than 2% of fistulas are the result of diseased bowel. When they are, the most common contributing factors are preoperative radiation therapy, intestinal obstruction, and inflammatory bowel disease. Although small bowel fistulas occasionally lead to generalized peritonitis, they most commonly produce a walled-off abscess manifested as an infection of the operative incision. The initial drainage may be purulent, but if the infection is caused by anastomotic leakage of the small bowel, the drainage becomes enteric within 1 to 2 days.

ANSWER: C 4. For the patient described in Question 3, which of the following is the most appropriate initial management? A. Packing of subcutaneous tissue with wet-to-dry dressings B. Packing of subcutaneous tissue with dry, absorbent dressings C. Immediate return to the operating room for exploration D. Protecting the skin around the fistula with Stomahesive karaya powder, aluminum paste, or zinc oxide and collecting the drainage fluid in an attached plastic bag E. Antibiotic and wet-to-dry dressing changes Ref.: 3 COMMENTS: The initial management of a small bowel fistula includes the administration of appropriate intravenous fluids, proximal decompression with nasogastric suction, control and quantification of the output of the fistula, and protection of the surrounding skin. Fistulas are classified according to their locations and the volumes of their output. Proximal fistulas tend to have higher output and lead to more severe electrolyte and fluid imbalances. Nasogastric suction can be helpful in diminishing the output of proximal intestinal fistulas, but the output of those more distal in the gut may not be influenced by this maneuver. Sump catheters can provide a means of controlling and quantifying high-output fistulas, especially early in their formation. Maintaining proper position of the catheter in the wound can be problematic. Once the fistula tract is established, suction catheters should be promptly replaced with a stoma appliance fixed to the edges of the fistula. Enteric contents are highly corrosive, and the skin surrounding the fistula opening should be protected carefully. Gauze

e1363

e1364   SECTION 16  ■  TRAUMA dressings are generally ineffective at absorbing all the drainage and protecting the skin. Therefore, their use is generally avoided. Most well-established fistulas do not produce sepsis, but in patients with persistent fever, systemic administration of antibiotics and a careful search for an undrained abdominal abscess are indicated. Early in the work-up of this patient and before the GI tract has been filled with contrast material (from conventional GI radiographs), CT should be performed to look for areas of abscess formation or fluid accumulation. It may also identify the site of the fistula. If CT does not show the site, fistulography is helpful. If one is concerned about distal obstruction, a small bowel follow-through study may provide information if this issue was not satisfactorily answered by CT or fistulography.

ANSWER: D 5. Diagnostic work-up of the woman described in Question 3 reveals that she has a distal ileal fistula that is communicating with a small cavity. Which of the following is appropriate therapy? A. Prompt exploration and interruption of the fistula tract B. Prompt exploration and bypass of the fistula C. Prompt exploration and resection of the portion of ileum involved in the fistula and primary reanastomosis D. A 4- to 6-week trial of intravenous hyperalimentation E. A 2-week trial of low-residue or elemental enteral alimentation Ref.: 3 COMMENTS: Knowing the location of the fistula is of important prognostic and therapeutic value. The overall mortality rate for small bowel fistulas is 20%, and the rate is higher for jejunal fistulas and lower for those of the ileum. With proper supportive care, such as intravenous or enteral alimentation, and in the absence of distal obstruction, up to 40% of small bowel fistulas close spontaneously. Enteral alimentation has the advantage of avoiding the possible hepatic and septic complications associated with prolonged TPN. Even if there is a slight increase in fistula output after the start of enteral nutrition, the fistula may still close. Fistulas of the proximal jejunum may require transnasal insertion of a long tube through the stomach and duodenum and just beyond the fistula before starting enteral alimentation. Surgery should be avoided for 4 to 6 weeks to permit spontaneous closure and to allow the local inflammation to subside, thereby facilitating subsequent surgery. The preferred operation for correcting a persistent fistula is resection of the fistula in continuity with the segment of involved bowel, followed by primary anastomosis. Alternative therapies include complete or partial exclusion with primary anastomosis.

ANSWER: E 6. With regard to the management of a patient with gallstone ileus, which of the following statements is true? A. Initial tube decompression and nonoperative management allow spontaneous stone passage in one third of patients. B. Operative treatment attempts to displace the stone into the colon without enterotomy. C. Operative treatment involves enterotomy proximal to the site of obstruction. D. Cholecystectomy and fistula repair at the time of stone removal are contraindicated. E. Standard treatment is initial laparotomy for stone removal and reoperation for cholecystectomy when the patient is stable. Ref.: 4, 5 COMMENTS: Gallstone ileus is mechanical obstruction of the gastrointestinal tract caused by a gallstone that has entered the intestine via an acquired biliary enteric fistula. Although gallstone ileus accounts for only 1% to 3% of all small bowel obstructions, it is associated with a higher mortality rate than other nonmalignant causes of bowel obstruction because it tends to occur in the elderly

CHAPTER 72-3  ■  Self Assessment  

population and typical cases are characterized by diagnostic delay as a result of waxing and waning of symptoms (“tumbling obstruction”). Pathognomonic radiologic features include a gas pattern of small bowel obstruction with pneumobilia and an opaque stone outside the expected location of the gallbladder. Not all of these radiologic features are usually present, however. The most common site of obstruction is the terminal ileum. Infrequently, sigmoid obstruction occurs in an area narrowed by intrinsic colonic disease. Initial therapy is appropriate resuscitation followed by surgery. Spontaneous passage is a rare phenomenon, and nonoperative management is associated with a prohibitive mortality rate. Stone removal is best accomplished with an enterotomy placed proximal to the site of obstruction. Care must be taken to search for additional intestinal stones, which are present in 10% of patients. Attempts to crush the stone extraluminally or to milk it distally are contraindicated because they may cause bowel injury. In rare instances, small bowel resection is necessary if there is ischemic compromise or bleeding at the site of impaction. The main controversy regarding surgical treatment of gallstone ileus is whether a definitive biliary tract operation with cholecystectomy, fistula repair, and possible common duct exploration should be performed at the time of stone removal. This decision must be based on sound surgical judgment and consideration of the underlying physiologic status of the patient and the anatomic status of the right upper quadrant. Up to one third of patients who do not undergo definitive biliary surgery experience recurrent biliary symptoms, including cholecystitis, cholangitis, and recurrent gallstone ileus. Furthermore, the rate of spontaneous fistula closure is open to question. For these reasons, a definitive one-stage procedure should be considered in physiologically fit patients if right upper quadrant dissection does not prove unduly hazardous from a technical standpoint, particularly if residual stones can be demonstrated in the right upper quadrant. In properly selected patients, a definitive one-stage procedure is not associated with higher operative morbidity or mortality rates. However, because most of these patients are elderly and have a high incidence of comorbid disease, surgical therapy has been limited to stone removal in most instances. Interval cholecystectomy should be considered for patients with postoperative biliary symptoms and for those with residual right upper quadrant stones, provided that they are physiologically fit. In reality, because of the compromised underlying status of many of these patients, interval elective procedures are not commonly performed.

ANSWER: C 7. Which of the following statements concerning necrotizing enterocolitis (NEC) is true? A. The initial insult in NEC is to the intestinal mucosa. B. The jejunum is the most frequently involved site. C. Operative intervention is indicated after resuscitation. D. Progression of NEC is halted after surgical therapy. E. The cecum is frequently not involved. Ref.: 6, 7 COMMENTS: Necrotizing enterocolitis is a disease that affects the intestinal tract of neonates. Clinical and experimental data have shown that the cause of NEC is multifactorial, with risk factors including perinatal stress, maternal cocaine use, and prematurity. The initial injury with NEC is observed in the intestinal mucosa. The spectrum of severity ranges from isolated mucosal injury to transmural bowel necrosis. Although the terminal ileum and right colon are the most commonly affected sites, the disease can be segmental or affect the entire gastrointestinal tract. The initial treatment of infants with NEC is nonsurgical and consists of nasogastric decompression, bowel rest, broadspectrum antibiotics, optimal fluid management, and parenteral nutrition. Operative intervention, such as bowel resection with proximal enterostomy, is indicated to treat the acute complications of NEC, which include intestinal perforation, necrosis, persistent bleeding, or obstruction. Operative intervention, however, does not prevent progression of the disease, which can continue after resection and may require additional surgical therapy.

ANSWER: A

e1365

e1366   SECTION 16  ■  TRAUMA

References 1. Solomkin JS, Mazuski JE, Baron EJ, et al: Guidelines for the selection of anti-infective agents for complicated intraabdominal infections, Clin Infect Dis 37:997–1005, 2003. 2. Britt LD, Rushing GD: Penetrating abdominal trauma. In Cameron JL, editor: Current surgical therapy, ed 9, Philadelphia, 2008, CV Mosby. 3. Tavakkolizadeh A, Whang EE, Ashley SW, et al: Small intestine. In: Brunicardi FC, Andersen DK, Billiar TR, et al, editors: Schwartz’s principles of surgery, ed 9, New York, 2010, McGraw-Hill. 4. Chari RS, Shah SA: Biliary system. In Townsend CM, Beauchamp RD, Evers BM, et al, editors: Sabiston textbook of surgery: the biological basis of modern surgical practice, ed 18, Philadelphia, 2008, WB Saunders. 5. Blumgart LH: Surgery of the liver, biliary tract and pancreas, ed 4, Edinburgh, 2006, Churchill-Livingstone. 6. Warner BW: Pediatric surgery. In Townsend CM, Beauchamp RD, Evers BM, et al, editors: Sabiston textbook of surgery: the biological basis of modern surgical practice, ed 18, Philadelphia, 2008, WB Saunders. 7. Hackman D, Grikscheit TC, Wang KS, et al: Pediatric surgery. In Brunicardi FC, Andersen DK, Billiar TR, et al, editors: Schwartz’s principles of surgery, ed 9, New York, 2010, McGraw-Hill.

73 

Temporary Closure the Abdomen

GOALS/OBJECTIVES • • •

INDICATIONS ANATOMIC CONSIDERATIONS TECHNICAL CONSIDERATIONS

of

THE ABDOMEN THAT WILL NOT CLOSE Adil H. Haider From Cameron JL, Cameron AM: Current Surgical Therapy, 10th edition (Mosby 2011)

73-1 

TEMPORARY ABDOMINAL CLOSURE Once the decision has been made to leave the abdomen open, the most optimal TAC technique should be used. This technique should be based on the experience of the surgeon and the practice environment. An ideal TAC should have the following attributes: Easily encompasses the bowel and abdominal viscera Allows enlargement of the abdominal cavity in situations of massive bowel, tissue, or retroperitoneal edema without inducing IAH and while preventing ACS Is expansible but also sturdy enough to permit the tamponade effect of packing the liver or other bleeding surfaces Does not damage the fascia and prevents fascial retraction Contains and quantifies fluid loss Prevents adhesion formation between viscera and abdominal fascia Promotes removal of infectious materials Is quick to apply and remove Has a good primary fascial closure rate Table 73-1-1 compares the most common methods of TAC. Although the earliest forms of TAC, the towel clip closure and Bogotá bag closure, are mentioned in the table, these have been largely replaced by more improved options. Similarly, polypropylene (Prolene; Ethicon, Somerville, NJ); polytetrafluoroethylene (PTFE) (Gore-Tex; W.L. Gore & Associates, Flagstaff, Ariz.); biologic mesh, such as human acellular dermal matrix, or HADM (Alloderm; Lifecell, Branchburg, NJ); and bovine acellular collagen matrix (Surgimend; TEI Biosciences, Boston, Mass.) are rarely used anymore for TAC. These materials were abandoned for a number of reasons. Polypropylene is notorious for developing fistulas and is difficult to remove from the bowel; PTFE, although relatively inert, limits tissue granulation, is not well incorporated in the fascia, and carries a high risk of mesh infection. The biologic mesh matrix products are too expensive for temporary use and may not provide enough tensile strength, especially in heavily infected fields, where they have been known to “melt away.”

Commercial and “Homemade” Vacuum Closure Devices In the United States, most surgeons use a vacuum closure device, such as the commercially available VAC abdominal dressing (Kinetic Concepts Inc., San Antonio, Tex.) or the vacuum-pack closure device described by Barker and colleagues (see Fig. 73-1-5). These devices create a negative-pressure silo that contains the abdominal contents, is somewhat expansible, and enables measurable fluid removal. The negative pressure is applied medially up through the open abdomen, minimizing fascial retraction and loss of abdominal domain. These dressings are also very quick and easy to apply, and they can be used in situations of massive bowel swelling. Although several variations of the negativepressure TACs exist, most include these key features: A fenestrated inner layer of an inert, pliable material (e.g., polyethylene) that is placed on top of the viscera to prevent it from forming adhesions to the abdominal wall, contain the abdominal viscera, and allow fluid movement. e1369

e1370   SECTION 16  ■  TRAUMA TABLE 73-1-1  Various Methods of Temporary Abdominal Closure Closure Technique

Description

Advantages

Disadvantages

Skin only (towel clip closure, running suture of skin) Bogotá bag

Serial application of towel clips or suture

Rapid

3 L IV bag, Steri-drape (3M; St Paul, Minn.), Silastic bag, plastic bag rapidly sutured to skin Suturing of absorbable mesh to skin or fascial edges

Inexpensive, inert, nonadherent

Does not prevent IAH; may interfere with radiography or angiography Risk of evisceration, loss of abdominal domain, risk for IAH; fluid losses difficult to quantify Rapid loss of tensile strength (in the setting of infection), potentially large-volume late ventral hernia; risk for bowel fistula; damage to fascial edges from repeated suturing Poor control of third-space fluid, adherence of bowel to abdominal wall, potential for fistulas

Absorbable mesh

Wittmann patch (Star Surgical, Burlington, Wis.)

Vacuum-pack closure

Suturing of artificial burr (i.e., Velcro) to fascia, staged abdominal closure by application of controlled tension Bowel covered with plastic sheet and towel or laparotomy pads; flat drains attached to wall suction and outer adhesive layer

Modified “sandwich” vacuum pack

3 L irrigation bag placed on bowel; three fascial sutures placed to retain “domain”; NG tubes used for suction, ostomy bag used to bring NG tube through outer adhesive dressing

Negative-pressure therapy; VAC (vacuum-assisted closure) abdominal dressing system (Kinetic Concepts, Inc., San Antonio, Tex.)

Reticulated polyurethane foam dressing over the plastic covering of the bowel; negative pressure is controlled with a computer-controlled vacuum pump that applies a constant, regulated pressure to the wound surface and a sensing device to prevent uncontrolled fluid (e.g., blood) drainage

Can be applied directly over bowel; allows for drainage of peritoneal fluid

Good tensile strength, allows for easy re-exploration and eventual primary fascial closure Inexpensive, uses available materials; moderate control of fluid; suction provides constant medial traction, preventing loss of domain; high success in fascial closure Same as vacuum-pack closure but is thought to retain fascial domain and further improve primary fascial closure; inert innermost layers help prevent fistulas Increase in blood flow, a reduction on abdominal wall tension, reduction in size of the abdominal wall defect, decreased bowel edema, and potential removal of inflammatory substances that accumulate in the abdomen during inflammatory states; edema and third-space losses can be controlled

Difficult to quantify suction; unknown whether full benefits of negativepressure therapy are realized

Does not use innermost, perforated layer, which may make fluid removal somewhat difficult; unknown whether benefits of negativepressure therapy are realized Expensive; not available at all institutions, in austere environments, or in most less developed countries; mechanism of action not fully understood, but does lead to hyperossification; full relationship to fistula not studied well enough

IAH, Intra-abdominal hypertension; NG, nasogastric.

A middle layer of foam or towels that helps generate suction, keeps the bowel moist, and provides some support to the dressing. The suction helps prevent the fascia from retracting and provides a mechanism for fluid and infectious effluent removal. A suction mechanism with major force applied to the middle layer. An outer layer made up of an adhesive polyurethane sheet to create an airtight seal around the entire apparatus and enable measurement of fluid loss and generation of negative pressure through the dressing. Figures 73-1-1 to 73-1-4 illustrate the ABThera VAC device (Kinetic Concepts Inc.). Figure 73-1-5 illustrates the “homemade” Barker dressing, which uses materials readily available in the operating room (OR) to create a vacuum pack.

CHAPTER 73-1  ■  The Abdomen that Will not Close  

e1371

FIGURE 73-1-1  The ABThera VAC system (Kinetic Concepts, Inc.) that is currently replacing their VAC open abdomen system. Recently introduced in the United States, the ABThera features a foam that extends out laterally to assist in volume removal. (Used with Permission. Courtesy of KCI, an Acelity Company.)

FIGURE 73-1-2  Placement of innermost layer for the ABThera VAC, which uses an innermost layer with an embedded foam that fans out laterally, which makes it easier to apply and also assists in lateral paracolic fluid and effluent removal.

e1372   SECTION 16  ■  TRAUMA

FIGURE 73-1-3  VAC dressing on patient in Surgical ICU. This dressing was easily applied despite an extremely swollen bowel and massive retroperitoneal bulging.

FIGURE 73-1-4  Application of a VAC dressing after two take-backs and construction of an iliostomy. Note the ostomy is placed very laterally. (Photo courtesy Dr Elliott Haut, Johns Hopkins Acute Care Surgery, Baltimore, MD.)

Technical Tips for Vacuum Dressing Placement Although it appears to be quite simple, experience in proper placement and application of the dressing does help avoid subsequent complications, the most dreadful of which is enteroatmospheric fistula formation. It is essential to place polyurethane between the bowel and the middle layer to prevent direct suction pressure on the bowel, which will cause fistula formation. One technique is to insert this innermost layer before releasing the Bookwalter, Omni, or other abdominal wall retractor. This practice ensures that the polyurethane sheet has been placed in far enough laterally. In addition, the use of ostomies or feeding tubes must be avoided in patients with an open abdomen. A nasoduodenal feeding tube can be used until the patient is ready to be closed. If an ostomy must be done, it should be placed as laterally in the flanks as possible to preserve enough normal abdominal wall for subsequent mobilization and closure. An airtight seal of the outermost layer is essential for all vacuum dressings. It is important to keep the skin area dry prior to application of this outer adhesive layer. In cases of substantial fluid spillage, cut strips of adhesive dressing can be placed horizontally across the middle layer to stabilize the open abdomen, then the skin can be quickly dried again, and the outer adhesive layer can be applied. Special attention must be given to the suprapubic area to ensure that negative pressure is generated. This can be confirmed by ensuring that the entire abdominal dressing is “sucked in,” as soon as suction is applied.

CHAPTER 73-1  ■  The Abdomen that Will not Close  

e1373

FIGURE 73-1-5  The Barker vacuum-pack technique. A, The polyethylene sheet is perforated multiple times with a scalpel blade. B, The sheet is placed over viscera and beneath the peritoneum/abdominal wall. C, A moist towel is placed over the polyurethane and positioned below the skin edges. D, Suction drains are placed. E, An outer adhesive dressing is applied, and the drains are hooked up to wall suction. (Courtesy Donald H. Barker, MD, Department of Surgery, University of Tennessee, Chattanooga.)

If an airtight seal is not accomplished, the adhesive drape must be removed, the skin redried, and outer dressing reapplied. It is important to get an excellent, dry seal in the OR, because it is usually difficult to reinforce this dressing in the intensive care unit (ICU).

Outcomes of Vacuum-Based Temporary Abdominal Closures In most cases both the VAC and vacuum pack have excellent outcomes, with a primary fascial closure rate reported between 70% and 80% and a mean closure time between 6 and 10 days. Both methods have a reported 15% complication rate. The most common complications are fistula formation (5% to 7%), intra-abdominal abscesses (4% to 6%), and delayed small bowel obstruction (4%). Most recently, Kinetic Concepts has introduced a newer dressing, called the KCI ABThera negative-pressure dressing. This product has a sponge embedded in the innermost layer that reaches out to the paracolic gutters, which makes it easier to apply and improves fluid removal. Outcomes for this newer product have not been reported. Another vacuum-based TAC that has excellent reported outcomes is the modified sandwich vacuum pack, described by Navasaria and Nicols from the University of Cape Town in South Africa. This technique uses an inner layer made up of an emptied 3 L irrigation bag and nasogastric tubes used to create a vacuum suction. It also uses a few large fascial sutures to keep medial traction on the wound and preserves abdominal domain. Results comparable to the VAC have been reported, with minimal complications. The technique is depicted in Figure 73-1-6, and the entire apparatus costs less than $20.

e1374   SECTION 16  ■  TRAUMA

FIGURE 73-1-6  A, B, Modified sandwich vacuum-pack technique from South Africa. A sturdier inner plastic material is used, and suction is generated using nasogastric tubes. Three large fascial sutures are also placed. To ensure an airtight seal, the nasogastric tubes are brought through the outer adhesive layer through ostomy bags. (Courtesy Dr. P. Navasaria, Department of Surgery, University of Cape Town, South Africa.)

ABDOMINAL COMPARTMENT SYNDROME, DAMAGE CONTROL, AND THE POSTTRAUMATIC OPEN ABDOMEN

73-2 

Richard S. Miller  /  John A. Morris From Asensio JA, Trunkey DD: Current Therapy of Trauma and Surgical Critical Care, 1st edition (Mosby 2008)

FIGURE 73-2-1  A cycle of ischemia producing intra-abdominal hypertension and the abdominal compartment syndrome. (Reproduced with permission from Michael Rotondo, MD.)

FIGURE 73-2-2  High-risk patient for abdominal compartment syndrome.

e1375

e1376   SECTION 16  ■  TRAUMA

FIGURE 73-2-3  Shock bowel.

FIGURE 73-2-4  Progressive organ dysfunction with increasing intra-abdominal hypertension.

FIGURE 73-2-5  Bedside decompressive celiotomy for abdominal compartment syndrome (note massive small bowel edema).

CHAPTER 73-2  ■  Abdominal Compartment Syndrome, Damage Control, and The Post-Traumatic Open Abdomen  

FIGURE 73-2-6  Algorithm for damage control.

FIGURE 73-2-7  Host factors define physiologic reserve.

e1377

e1378   SECTION 16  ■  TRAUMA

FIGURE 73-2-8  Depletion of physiologic reserve.

FIGURE 73-2-9  Physiologic reserve as related to Injury Severity Score (ISS).

FIGURE 73-2-10  Vacuum pack temporary abdominal closure: A, bowel bag, B, sterile surgical towel/two large sump drains/large adhesive drape.

CHAPTER 73-2  ■  Abdominal Compartment Syndrome, Damage Control, and The Post-Traumatic Open Abdomen  

FIGURE 73-2-11  Three potential stages of the post-traumatic abdomen.

FIGURE 73-2-12  Outcome of 344 open abdomen patients with a temporary abdominal closure.

FIGURE 73-2-13  Percent of patients with wound complications versus days to fascial closure.

e1379

e1380   SECTION 16  ■  TRAUMA

FIGURE 73-2-14  Wound closure complications associated with the post-traumatic open abdomen. *p = 0.0001 when comparing fistula formation between primary and temporizing groups. (From Miller RS et al: Complications after 344 damage-control open celiotomies. J Trauma 59:1365-1371, 2005.)

FIGURE 73-2-15  Enteroatmospheric fistula.

CHAPTER 73-2  ■  Abdominal Compartment Syndrome, Damage Control, and The Post-Traumatic Open Abdomen  

FIGURE 73-2-16  A, B, Mature granulation tissue in an open abdomen, which is then covered with a split-thickness skin graft.

FIGURE 73-2-17  Vacuum-assisted closure – KCI VAC. Note external fixator for damage control orthopedics.

e1381

e1382   SECTION 16  ■  TRAUMA

FIGURE 73-2-18  Alloderm with overlying skin flaps for early closure of abdominal fascia.

FIGURE 73-2-19  A, B, Alloderm used to bridge fascial gap in contaminated open abdomen wound with KCI VAC placed over the biologic fascial replacement.

CHAPTER 73-2  ■  Abdominal Compartment Syndrome, Damage Control, and The Post-Traumatic Open Abdomen  

FIGURE 73-2-20  Pinch sign.

e1383

FIGURE 73-2-21  Fascial edges freed from surrounding tissue and skin flaps raised past edge of lateral rectus muscle.

FIGURE 73-2-22  A, B, Component separation closure.

e1384   SECTION 16  ■  TRAUMA

FIGURE 73-2-23  Algorithm for damage control and open abdomen management.

MANAGING THE OPEN ABDOMEN Daniel Vargo From Rosen MJ: Atlas of Abdominal Wall Reconstruction, 1st edition (Saunders 2012)

73-3 

FIGURE 73-3-1 

FIGURE 73-3-2 

FIGURE 73-3-3 

FIGURE 73-3-4 

e1385

e1386   SECTION 16  ■  TRAUMA

FIGURE 73-3-5  Figures 73-3-1 to 73-3-5 show temporary coverage of the intestine is accomplished with a plastic drape, iodine impregnated drape, or a commercially available visceral drape.

FIGURE 73-3-6  Placement of drape.

FIGURE 73-3-7  Commercially available drape with foam.

CHAPTER 73-3  ■  Managing the Open Abdomen  

e1387

FIGURE 73-3-8 

FIGURE 73-3-9 

FIGURE 73-3-10 

FIGURE 73-3-11  Figures 73-3-8 to 73-3-11 show placement of wound vacuum device.

FIGURE 73-3-12 

FIGURE 73-3-13  Figures 73-3-12 to 73-3-13 show the fascia can be gradually re approximated.

73-4 

REFINEMENT IN THE TECHNIQUE OF PERIHEPATIC PACKING From Baldoni F, et al: Refinement in the technique of perihepatic packing: a safe and effective surgical hemostasis and multidisciplinary approach can improve the outcome in severe liver trauma. Am J Surg 2011;201(1):e5–e14

The video for this procedure can be accessed here

e1388

SELF ASSESSMENT Edward F. Hollinger  /  Troy Pittman From Townsend CM: Sabiston Textbook of Surgery, 19th edition (Saunders 2012)

73-5 

1. Abbreviated laparotomy is the initial phase of damage control surgery. Which of the following indications are correct? A. Temperature less than 35° C B. Medical bleeding C. Arterial pH less than 7.20 D. Urine output less than 30 mL/kg/hr E. A, B, and C are correct COMMENTS: Indications for damage control surgery during trauma include the impending development of loss of physiologic reserve. This condition is manifested in the operating room as hypothermia with core body temperature less than 35° C, acidosis with arterial pH less than 7.20, and medical coagulopathy shown by bleeding from cut surfaces.

ANSWER: E 2. The most common indications for the use of the open abdomen technique in general surgery are as follows: A. Abdominal compartment syndrome B. Ruptured abdominal aortic aneurysm C. Trauma–damage control D. Acute pancreatitis E. All of the above COMMENTS: The open technique is a lifesaving heroic technique in the management of intraabdominal pathology. Decompressive laparotomy is lifesaving in a patient with abdominal compartment syndrome. After repair of a ruptured abdominal aortic aneurysm, if intraoperative resuscitation exceeds 6 units of packed red blood cells or 10 liters of crystalloid or both, the abdominal viscera may have developed massive edema. Closure of the abdomen in this setting may lead to abdominal compartment syndrome. During damage control trauma surgery, an abbreviated operation would have been performed. Leaving the abdomen open would decrease the incidence of abdominal compartment syndrome and allow for a quick re-exploration. The open abdomen technique is used in acute necrotizing pancreatitis for serial debridements of necrotic pancreas.

ANSWER: E 3. There are several techniques for creating a temporary abdominal closure for the open abdomen. The key to all techniques must include the following: A. Quick application B. Seal in moisture and temperature C. Quickly removable D. High tensile strength E. A, B, and C are correct e1389

e1390   SECTION 16  ■  TRAUMA COMMENTS: The most desirable qualities of a temporary abdominal closure for the management of an open abdomen are its quick and repeatable application, that it seals in moisture and decreases loss of body temperature, and can easily be removed for quick re-entrance into the abdomen.

ANSWER: E 4. The open abdomen technique has a high rate of nonclosure because of the following complications: A. Intra-abdominal abscess and intra-abdominal sepsis B. Acute lung injury C. Atmospheric intestinal fistula D. Urinary tract infection E. A and C are correct COMMENTS: The rate of abdominal closure is affected most by the development of intra-abdominal abscess and atmospheric intestinal fistula.

ANSWER: E 5. Which of the following mesh products should not be used in the open abdomen setting because they have very high rates of intestinal fistula formation and mesh infection? A. Human dermal acellular dermis B. PTFE C. Porcine dermal matrix D. Polypropylene E. B and D are correct COMMENTS: The use of permanent mesh such as polypropylene and PTFE is associated with a high rate of mesh infection and intestinal fistula formation.

ANSWER: E

Wounds, Major – Debride/Suture

GOALS/OBJECTIVES • • •

ANATOMY AND PHYSIOLOGY REVIEW INDICATIONS TECHNICAL CONSIDERATIONS

74 

74-1 

SOFT TISSUE INFECTIONS Sharon Henry From Asensio JA, Trunkey DD: Current Therapy of Trauma and Surgical Critical Care, 1st edition (Mosby 2008)

Soft tissue infections occur frequently and account for approximately 48.3 in 1000 outpatient visits. The severity of these infections varies from trivial to life-threatening. Severe soft tissue infections have been described throughout the medical literature since ancient times. Necrotizing fasciitis was described in the fifth century bc by Hippocrates, though Wilson coined the term in 1952. Much of our knowledge regarding the treatment of soft tissue infection has been based on the experience gained during military conflicts. For instance, hospital gangrene was described first by Joseph Jones, a Confederate surgeon during the Civil War. The treatment of battlefield infections has influenced civilian practice. This review will begin with a description of the anatomically more superficial infections and progress to the deeper, life-threatening infections (Figure 74-1-1).

SUPERFICIAL INFECTIONS Superficial infections are limited anatomically to the epidermis and the dermis. These infections can occur spontaneously or secondary to minor trauma. Impetigo usually presents with vesicles that leak,

FIGURE 74-1-1  Skin structures with corresponding infection.

e1392

CHAPTER 74-1  ■  Soft Tissue Infections  

producing a thick yellow crust. These lesions are typically located on the face, neck, and extremities. Staphylococcus aureus and Streptococcus pyogenes are the most common causative agents. Bullous impetigo, like impetigo, is most commonly caused by S. aureus. It is characterized by small vesicles that coalesce to form large bullae. Folliculitis develops as an infection of the hair follicle. The lesions appear as pustules or papules, commonly on the extremities, scalp, or beard. Whirlpool folliculitis is associated with immersion in inadequately chlorinated pools, whirlpools, or hot tubs. Diffuse pustules are seen. Pseudomonas aeruginosa is the classic causative agent. Swimmer’s itch is a folliculitis that develops after freshwater exposure. Furuncles, deeper inflammatory nodules, can develop from folliculitis. S. aureus is often the causative agent. Carbuncles are coalescing furuncles formed by connecting sinuses. The nape of the neck is the most common anatomic location. S. aureus is most often isolated. Patients often have comorbidities such as diabetes mellitus, alcoholism, immunosuppression, or malnutrition. Systemic infection may result from these lesions. Cellulitis is an inflammation of the subcutaneous tissue. There is erythema, pain, and edema of varying severity. A portal of entry for the bacteria is usually present. It may be as mundane as a crack in the skin from dryness or athlete’s foot. More substantial trauma may be involved, such as shotgun or shrapnel penetrations. Systemic symptoms may manifest and include fever, chills, malaise, and (infrequently) organ failure. Streptococcus (Groups A, B, C, and G) as well as Staphylococcus are frequent culprits. Areas of compromised venous or lymphatic drainage are prone to this infection, and recurrence is common. Pelvic irradiation or having had lymphadenectomy, mastectomy, or venectomy makes the development of cellulitis more likely.

DEEP INFECTION Deeper infections are more frequently life- and limb-threatening. The literature concerning the potentially life-threatening infections is confusing. Multiple terms are used to describe the same disease, depending on the clinical setting in which it arises. The treatment is the same regardless of the term used to describe it. It therefore seems that anatomic classification is more logical and easily remembered. This review uses anatomic characterizations and relates them to older terminology as necessary. In the medical literature, deep structure infection has masqueraded under a variety of pseudonyms. The term “necrotizing soft tissue infection” (NSTI) is used here. Table 74-1-1 lists a variety of terms that may appear in the medical literature to describe this infective process. All the terms describe an infection involving the subcutaneous fat and fascia, with variable skin involvement. The incidence of this infection is not known. A recent report quotes World Health Organization statistics of 500–1500 cases of necrotizing fasciitis annually. Though uncommon, it is not rare. The Centers for Disease Control has monitored Group A streptococcal infections and estimated 10–20 cases per 100,000 population. In Ontario, Canada, a population study estimated an incidence of 0.6 per 100,000.

TABLE 74-1-1  Terms Used to Describe NSTI Meleney’s synergistic gangrene Streptococcal gangrene Gas gangrene Suppurativa fasciitis Phagedena “Flesh-eating disease”

Clostridial cellulitis Hospital gangrene Fournier’s gangrene Acute dermal gangrene Necrotizing erysipelas Phagedena gangrenosum Necrotizing fasciitis

e1393

e1394   SECTION 16  ■  TRAUMA

CLINICAL PRESENTATION Signs and symptoms of NSTI can be quite nonspecific (Table 74-1-2). Pain, erythema, and swelling of the affected area are most frequently present. This same constellation of symptoms may be seen in pathologic processes that have a much more benign course and respond effectively to antibiotic therapy alone. “Hard signs” of necrotizing infection include tense erythema, bullae, skin discoloration, and crepitus, pain out of proportion to examination, or anesthesia of the affected area. Unfortunately, many of these are late signs and indicate that the infective process is well established or they occur only in a small percentage of patients. Signs of systemic toxicity may also be present. These may include pyrexia, tachycardia, hypotension, and organ dysfunction. The progression of symptoms may be rapid over the course of hours to days or more indolent over the course of days to weeks. The rate of progression of symptoms may be ameliorated by partial treatment. Some suggest classifying the disease by its clinical course. Fulminant disease presents in patients with acute onset and rapid progression over the course of hours with shock. Acute disease presents with large surface area involvement and over the course of days. Subacute disease presents for weeks and is usually localized. Differentiating this process from cellulitis or simple abscess can be a challenge. Clinically, failure to improve with appropriate antibiotics or worsening systemic toxicity portends this diagnosis (Table 74-1-3). Laboratory data are equally nonspecific. Leukocytosis, hyponatremia, and elevated creatinine phosphokinase have been evaluated in clinical studies and may be markers of the disease. Wall et al. matched 21 patients with necrotizing fasciitis with controls and attempted to identify parameters that would distinguish the groups. White blood cell count (WBC) > 15.4 × 109/l, serum sodium (Na) less than 135 mmol/l, or both, were the best factors to distinguish necrotizing fasciitis from non-necrotizing fasciitis. The sensitivity was 90%, and the specificity was 76%. In this study, 40% of the patients with necrotizing fasciitis lacked “hard signs.” Wong and colleagues developed the Laboratory Risk Indicator

TABLE 74-1-2  Frequency of Clinical Signs Author Callahan McHenry Brook Elliot Tang Theis Wong

Erythema (%) 77 72 89 66 50 54 100

Crepitans (%) 3 12 39 45 – – 13

Edema (%)

Year Published

# Patients in Study

20 75 77 75 58 – 92

1988 1995 1995 1996 2001 2002 2002

30 65 83 197 24 13 89

TABLE 74-1-3  Bacteriology Rapid Progression: Incubation 600 IU/l in the GAS group.

DIAGNOSTIC IMAGING Diagnostic imaging may be useful in diagnosis or in defining the extent of disease. The hallmark of necrotizing fasciitis – air in subcutaneous tissues – can be seen with a variety of imaging techniques. Plain radiographs will demonstrate subcutaneous air in only 16% of patients. Plain films may also demonstrate foreign bodies. This is especially important when treating intravenous drug users. Ultrasonography, computed tomography (CT scan), and magnetic resonance imaging (MRI) are more sensitive than plain radiography in demonstrating air in the tissues. These modalities may additionally identify fluid collections in the subcutaneous tissues or within the muscle. They may be very helpful in cases without significant skin changes in planning incisions. MRI has the advantage of not requiring the administration of intravenous contrast material which may be toxic to the kidneys. This is especially helpful in patients who already have compromised renal function. However, systemically compromised patients are often logistically poor candidates for MRI scanning. In most cases, diagnostic imaging is only confirmatory, as patients with nonspecific findings on evaluation, such as edema, may still harbor the disease. Failure to improve with appropriate antibiotics, development of systemic toxicity, or profound elevation of the WBC should markedly raise the index of suspicion.

PATHOPHYSIOLOGY The ability of an organism to cause infection is dependent on the virility of the organism and the resistance of the host. This disease is unique in that it affects both the compromised and the uncompromised host. Multiple reviews identify multiple risk factors associated with increased susceptibility; however, exceptionally virulent bacteria such as GAS are able to affect otherwise uncompromised hosts. These bacteria produce exotoxins that activate cytokines, leading to a robust systemic response. At the cellular level, the toxins and enzymes (hyaluronidases and lipases) facilitate the spread of bacteria along fascial planes and through the subcutaneous tissue. Necrosis of the superficial fascia and fat often produce a thin, watery, foul-smelling fluid (“dishwater pus”). The skin necrosis that may accompany this infection results when thrombosis of the skin’s nutrient vessels occurs. Microscopic findings are usually severe subcutaneous fat necrosis, severe inflammation of the dermis and subcutaneous fat, vasculitis, endarteritis, and local hemorrhage. The fascia may be edematous and suppurative, and thrombosis of vessels may be seen. Myonecrosis may also be seen in advanced cases.

SURGICAL TREATMENT Necrotizing fasciitis is a surgical emergency. The mainstay of treatment is surgical debridement. These procedures can be quite deforming, requiring the removal of large amounts of skin, subcutaneous fat, fascia, and possibly muscle or bone. Explorations on the extremities are usually begun by making generous vertical incisions. When involvement is diffuse, fasciotomy incisions on the extremities are often a useful starting place. The dissection must extend down to the level of the deep fascia. The muscle should also be inspected to confirm its viability (Figure 74-1-2). Formal fasciotomies may be necessary in cases with very intense edema in order to prevent myonecrosis (Figure 74-1-3). The integrity of the tissue plane between the subcutaneous fat or superficial fascia and the deep fascia is tested with either a finger or clamp (Figure 74-1-4). Lack of resistance to this probing is the hallmark of the diagnosis in early cases. “Dishwater pus” may be encountered as this plane is opened.

e1395

e1396   SECTION 16  ■  TRAUMA

FIGURE 74-1-2  Lower extremity fasciotomy.

FIGURE 74-1-3  Upper extremity fasciotomy.

FIGURE 74-1-4  Loss of tissue plane integrity.

CHAPTER 74-1  ■  Soft Tissue Infections  

FIGURE 74-1-5  Thorough debridement.

In more advanced cases, frankly necrotic or purulent material is encountered. Cultures for stat Gram stain and aerobic and anaerobic cultures should be obtained. If the patient is immunosuppressed, cultures for Mycobacterium and fungus should also be sent. It is imperative to widely open all affected tissue planes and debride all obviously devitalized tissue (Figure 74-1-5). When the presentation is more focal, incisions can be placed over the area of maximal skin abnormality and the incision extended as abnormalities of the deeper tissue are encountered. A colostomy may be helpful in cases with extensive perianal involvement to prevent ongoing stool contamination. Surgical feeding tube placement should be considered in critically ill patients or in patients with large surface area wounds that are likely to remain open for some time. Plans should be made at the end of the case for return to the operating room for re-evaluation of the wounds within 48 hours. Adequate evaluation of these deep and often painful wounds is not possible at the bedside. A variety of wound dressings can be applied. Negative pressure wound therapy (NPWT) is optimally suited for this purpose. It allows removal of exudate, which may further decrease bacterial counts. It prevents maceration of the surrounding tissue and keeps the patient and their bed linens dry. It is often helpful to be able to quantify the fluid losses that are occurring. Replacement of these losses may be necessary. The skin surrounding the wound can be evaluated for advancing cellulitis or edema. The character of the fluid can also be assessed. It is imperative if this type of wound care is to be used that hemostasis is meticulous and coagulopathy corrected. Parameters must be given to the nursing staff regarding volume and character of the fluid loss that is acceptable. When this type of dressing is not available or prudent, the wounds may be packed with Kerlix (Kendall Kerlix AMD) or gauze impregnated with antimicrobial material. Although some of these materials may be cytotoxic, the antimicrobial properties may outweigh the negative effect of the cytotoxicity on normal tissues. Once the infection is controlled, non-cytotoxic products should be used. Silver ion-based products are a good choice.

BACTERIOLOGY Necrotizing fasciitis is usually divided into two categories. Type I is a polymicrobial infection and is the most common variety. Typically this infection is seen in patients with comorbidities. Gramnegative enteric bacteria are seen in combination with Gram positives and anaerobes. Type II is GAS either alone or in combination with S. aureus. This type takes a more virulent course and occurs in younger, healthier patients. Clostridial infections classically produce air in the soft tissues, although the polymicrobial type I infections may also produce GAS. Vibrio vulnificus and Aeromonas hydrophilia should be suspected when the patient has been exposed to shellfish or freshwater.

PHARMACOLOGIC THERAPY Initial treatment of the patient in shock should be aimed at resuscitation. Fluids and blood products are given as needed to replace deficits. Coagulation abnormalities should be corrected. It must be

e1397

e1398   SECTION 16  ■  TRAUMA stressed that resuscitation is begun, but all abnormalities will not and need not be fully corrected before proceeding to surgery. Resuscitation must continue throughout the preoperative, operative, and postoperative phases. Many abnormalities will not fully correct until there is adequate source control. Pressors and steroids may be necessary in severe cases. When GAS is involved, intravenous immunoglobulin (IVIG) may also be given. Broad spectrum antibiotic therapy is initiated to cover the most likely causative organisms. Extended spectrum penicillins are a good first-line choice. Clindamycin should be added in cases that may involve GAS. Methicillin resistance is emerging in the community, and this must be considered in choosing antibiotics. Vancomycin or daptomycin should be considered when resistance is suspected or in cases where the patient is severely compromised. Patients with penicillin allergies can be treated with fluoroquinolones in combination with Gram-negative and anaerobic therapy or carbapenems.

HYPERBARIC OXYGEN Hyperbaric oxygen (HBO) is an adjunct to resuscitation, surgical debridement, and broad spectrum antibiotics. A variety of salutary affects have been attributed to HBO therapy (Table 74-1-4). There are no prospective randomized studies to scientifically validate the efficacy of hyperbaric oxygen therapy. There are a number of retrospective studies that seem to support its use in severe necrotizing infections. Several studies demonstrated decreased mortality in patients treated with HBO compared with historic controls. Other studies demonstrated improved preservation of tissue as evidenced by a decrease in the number of debridements to achieve control of the infection. Some studies, however, have questioned the efficacy of HBO. These studies showed no statistically significant difference in mortality between patients treated with HBO and those who received only surgical debridement. HBO therapy is not uniformly available throughout the country, so it is not an option for every patient who presents with this problem. When available, given the relatively high mortality and morbidity, use of this modality as an adjunct makes sense.

MORTALITY, MORBIDITY, AND   COMPLICATIONS MANAGEMENT The morbidity following the treatment of severe necrotizing soft tissue infections can be severe. Most of the morbidity results from the tissue destruction brought about by the infection. The extent of morbidity depends on the area involved and the extent of the necessary debridement. During debridement, iatrogenic injury to nerves and blood supply can compound dysfunction. This is most common in the management of extremity infections. Tissues may be distorted by the inflammation and necrosis or scarred by prior surgery, making the identification of vital structures a challenge. It is vitally important to be familiar with the anatomy of the area. Patients may be left with significant deficits in function solely related to the magnitude of the surgical debridement. Amputation rates from 17%–33% have been documented in patients with severe extremity soft tissue infections. Patients with abdominal sites of infection, who require debridement of their abdominal wall, are subject to the development of enterocutaneous fistulae. Management of the output of the fistula and

TABLE 74-1-4  Beneficial Effects of HBO Inhibits growth of anaerobic organisms Reduces the production of clostridial toxin Improves leukocyte bacterial killing Bacteriocidal and bacteriostatic effects on a variety of organisms Enhances efficacy of certain antibiotics Modulates cytokine levels Decreases tissue edema Increases collagen formation

CHAPTER 74-1  ■  Soft Tissue Infections  

providing for adequate nutrition are the primary management issues in the early management of this problem. Mortality rates as high as 76% have been recorded, although contemporary studies report overall mortality in the range of 20%. Multiple factors affect mortality. Clinical presentation and speed to surgery appear to be the two most important determinants. Patients who suffer delays in obtaining surgical treatment had higher mortalities. Patients who present with organ failure or increased serum lactates also have higher mortalities. Comorbidities such as diabetes mellitus, renal failure, and advanced age, and the need for large surface area debridements have also been noted to increase mortality.

CONCLUSIONS NSTI can occur under a variety of clinical conditions. It should be considered in post-surgical, posttraumatic wounds as well as in wounds from insect or animal bites. Differentiation from superficial infection is mandatory to ensure appropriate surgical therapy is performed. Patients who fail to respond to appropriate medical therapy or who present with evidence of shock or organ dysfunction often harbor deeper infections. Patients with leukocytosis, hyponatremia, hyperglycemia, elevated creatinine, C-reactive protein, or CPK on laboratory evaluation should elicit an aggressive evaluation. This may include radiographic workup or surgical exploration. Progression of physical findings, including worsening edema, blistering, and crepitans or skin necrosis, mandates surgical exploration. Delays in treatment result in increased mortality and morbidity.

Suggested Readings Bosshardt TL, Henderson VJ, Organ CH: Necrotizing soft-tissue infections. Arch Surg 131:846–854, 1996. Brook I, Fraxier EH: Clinical and microbiological features of necrotizing fasciitis. J Clin Microbiol 33:2382–2387, 1995. Callahan TE, Schecter WP, Horn JK: Necrotizing soft tissue infection masquerading as cutaneous abscess following illicit drug injection. Arch Surg 133:812–818, 1998. Elliott DC, Kufera FA, Meyers RA: Necrotizing soft tissue infections: risk factors for mortality and strategies for management. Ann Surg 224:672–683, 1996. Simonart T, Nakafusa J, Narisawa Y: The importance of serum creatinine phosphokinase level in the early diagnosis and microbiological evaluation of necrotizing fasciitis. Eur Acad Dermatol Venereol 18:687–690, 2004. Tang WM, Ho PL, Fung KK, Yuen KY, Leong JC: Necrotizing fasciitis of a limb. J Bone Joint Surg 83:709–714, 2001. Theis FC, Rietveld J, Danesh-Clough T: Severe necrotising soft tissue infections in orthopaedic surgery. J Orthop Surg 10:108–113, 2002. Wall DB, de Virgilio C, Black S, Klein S: Objective criteria may assist in distinguishing necrotizing fasciitis from nonnecrotizing soft tissue infection. Am J Surg 179:17–21, 2000. Wall DB, Klein SR, Black S, de Virgilio C: A simple model to help distinguish necrotizing fasciitis for non-necrotizing soft tissue infection. J Am Coll Surg 191:227–300, 2000. Wilkinson D, Doolette D: Hyperbaric oxygen treatment and survival from necrotizing soft tissue infections. Arch Surg 139:1339–1345, 2004. Wong CH, Chang HC, Pasupathy S, Khin LW, Tan JL, Low CO: Necrotizing fasciitis: clinical presentation, microbiology, and determinants of mortality. J Bone Joint Surg 85A:1454–1460, 2003. Wong CH, Khin LW, Heng KS, Tan KC, Low CO: The LRINEC (Laboratory Risk Indicator for Necrotizing Fasciitis) score: a tool for distinguishing necrotizing fasciitis from other soft tissue infections. Crit Care Med 32:1535–1541, 2004. Wu W, Scannell C, Lieber MJ, Huang W: Hyperbaric oxygen therapy: current status in the management of severe nonclostridial necrotizing soft tissue infections. Curr Treat Opin Infect Dis 3:217–225, 2001.

e1399

74-2 

LACERATION AND INCISION REPAIR Richard P. Usatine  /  Wendy C. Coates From Pfenninger JL, Fowler GC: Pfenninger & Fowler’s Procedures for Primary Care, 3rd edition (Saunders 2010)

FIGURE 74-2-1  Irrigation of a dirty wound using a syringe and plastic shield.

FIGURE 74-2-2  Débridement. A, Irregular jagged wound. B, Excise a jagged wound or crush injury to create a more readily reparable wound.

e1400

CHAPTER 74-2  ■  Laceration and Incision Repair  

FIGURE 74-2-3  A, When skin margins approximate with tension, this can be relieved by undermining the margins through the use of a blade or B, C, scissors. The usual plane is at the dermal–adipose junction. Undermine twice as far back as the wound is wide, if possible. D, The proper level of undermining to mobilize the skin is shown. (D, Courtesy of The Medical Procedures Center, PC, Midland, Mich, John L. Pfenninger, MD.)

FIGURE 74-2-4  Closing the dead space. A, Improper closure with dead space not closed. B, Proper closure with dead space closed by deep sutures.

e1401

e1402   SECTION 16  ■  TRAUMA

FIGURE 74-2-5  Simple interrupted suture. A, Proper spacing. B, Interrupted sutures after excision of a basal cell carcinoma of the elbow. (B, Courtesy of The Medical Procedures Center, PC, Midland, Mich, John L. Pfenninger, MD.)

FIGURE 74-2-6  Wound margin appearance after closure. A, Proper eversion of the skin edges on closure (“build pyramids, not ditches”). B, Acceptable, but not optimal, closure. C, Improper closure because healing will lead to further contraction and scar depression.

FIGURE 74-2-7  Needle should enter the skin surface at a 90-degree angle. (Revised from Moy R: Suturing techniques. In Usatine RP, Moy RL, Tobinick EL, Siegel DM [eds]: Skin Surgery: A Practical Guide. St. Louis, Mosby, 1998, pp 88–100.)

CHAPTER 74-2  ■  Laceration and Incision Repair  

FIGURE 74-2-8  Use the Erlenmeyer flask–shaped pathway to promote eversion of skin edges. (Revised from Moy R: Suturing techniques. In Usatine RP, Moy RL, Tobinick EL, Siegel DM [eds]: Skin Surgery: A Practical Guide. St. Louis, Mosby, 1998, pp 88–100.)

FIGURE 74-2-9  Running stitch. A, This is a good stitch to use if there is no tension on the wound or after deep stitches were already placed with good approximation of the wound edges. B, Always keep the depth of the suture placement the same on each side. (A, Courtesy of Richard P. Usatine, MD, San Antonio, Tex; B, From Moy R: Suturing techniques. In Usatine RP, Moy RL, Tobinick EL, Siegel DM [eds]: Skin Surgery: A Practical Guide. St. Louis, Mosby, 1998, pp 88–100.)

FIGURE 74-2-10  Deep stitch with absorbable suture material. A, Needle should enter deep in the skin below the dermis where the undermining was accomplished (1) and exit in the upper dermis (2). The needle enters in the upper dermis (3) and exits below the dermis where the undermining was accomplished (4). B, The deep inverted buried stitch is tied at the bottom of the wound to avoid having the knot stick out of the incision. C, Placing the deep stitch. (From Moy R: Suturing techniques. In Usatine RP, Moy RL, Tobinick EL, and Siegel DM [eds]: Skin Surgery: A Practical Guide, St Louis, Mosby, 1998, pp 88–100.)

e1403

e1404   SECTION 16  ■  TRAUMA

FIGURE 74-2-11  Vertical mattress suture. A, Cross-section. B, Overhead view. Begin at a, and go under skin to b. Come out, go in at c, and exit at d. C, Photograph of two vertical mattress sutures used to obtain wound eversion. (Courtesy of Richard P. Usatine, MD, San Antonio, Tex.)

FIGURE 74-2-12  Horizontal mattress suture. A, Needle is passed 0.5 to 1 cm away from wound edge deeply into the wound. B, Needle is passed through the opposite side and re-enters the wound parallel to the initial suture. C, Re-enter the skin perpendicularly to provide some eversion of the wound edges. Enter and exit both the wound and skin at the same depth; otherwise, “buckling” and irregularities occur in the wound margin. D, Suture is then tied as shown.

CHAPTER 74-2  ■  Laceration and Incision Repair  

FIGURE 74-2-13  Three-point or half-buried mattress suture to repair a V-flap laceration.

FIGURE 74-2-14  T-laceration repair using half-buried mattress suture technique.

FIGURE 74-2-15  Dog-ear repair. A, Note site of initial incision of bulging dog ear. B, Pull the tip over and excise. C, Close the new incision for skin to lie flat.

e1405

e1406   SECTION 16  ■  TRAUMA

FIGURE 74-2-16  C-flap repair. A, Laceration. B, The problem: The point X is often very thin and may necrose. Even if it does not, contracture will occur after healing and the slim margin along the X will be depressed, causing a more visible scar. C, If small enough, convert the wound to an ellipse for easier repair. D, Alternatively, excise the angled margins of skin to obtain “square” borders. E, Undermine. F, Close with interrupted sutures. Because side a is smaller than side b, a small wedge of tissue may need to be removed. G, Complete closure.

FIGURE 74-2-17  The red rectangles illustrate where Mastisol is applied to the skin.

SELF ASSESSMENT Edward F. Hollinger  /  Troy Pittman From Velasco J: Rush University Medical Center Review of Surgery, 5th edition (Saunders 2011)

74-3 

1. A 41-year-old woman undergoes complex repair of a deep laceration in her hand. When removing the dressing on postoperative day 2, a large clot with mild surrounding erythema is encountered. Which of the following statements regarding the inflammatory phase of wound healing is true? A. It lasts up to 24 hours after the injury is incurred. B. Initial vasodilation is followed by subsequent vasoconstriction. C. Bradykinin causes vasoconstriction, which inhibits migration of neutrophils to the healing wound. D. The complement component C5a and platelet factor attract neutrophils to the wound. E. The presence of neutrophils in the wound is essential for normal wound healing. Ref.: 1-5 COMMENTS: The inflammatory phase starts immediately after the injury occurs and lasts up to 72 hours. After the injury, there is a transient period (about 10 minutes) of vasoconstriction followed by active vasodilation. These events are mediated by substances released secondary to the local tissue injury. Vasoactive components such as histamine cause brief periods of vasodilation and increased vascular permeability. The kinins (bradykinin and kallidin) are released by the enzymatic action of kallikrein, which is formed after activation of the coagulation cascade. These components, in addition to those of the complement system, stimulate the release of prostaglandins (particularly PGE1 and PGE2), which work in concert to maintain more prolonged vessel permeability, not only of capillaries but also of larger vessels. In addition, these substances, particularly the complement component C5a and platelet-derived factors such as platelet-derived growth factor (PDGF), act as chemotactic stimuli for neutrophils to enter the wound. Although neutrophils can phagocytize bacteria from a wound, the results of studies involving clean wound healing show that healing can proceed normally without them. Monocytes, however, must be present for normal wound healing because in addition to their role in phagocytosis, they are required to trigger a normal fibroblast response. The later phases of wound healing include the proliferative or regenerative phase and the remodeling phase. The proliferative phase is marked by the appearance of fibroblasts in the wound, which leads to the formation of granulation tissue. The remodeling phase involves an increase in wound strength secondary to collagen remodeling and lasts up to 1 year after the initial injury. The three main phases of wound healing may occur sequentially or simultaneously.

ANSWER: D 2. A 55-year-old woman with a history of venous stasis ulcers is evaluated for a nonhealing ulcer on the medial aspect of the lower part of her leg. Application of topical ointment to the ulcer and compression stockings have allowed partial healing. However, she states that regardless of the various interventions, the ulcer never completely heals. Which of the following statements regarding wound epithelialization is true? e1407

e1408   SECTION 16  ■  TRAUMA A. Integrins act as a key modulator of the interaction between epithelial cells and the surrounding environment. B. Structural support and attachment between the epidermis and dermis are provided by tight cell junctions. C. Early tensile strength of the wound is a direct result of collagen deposition. D. A re-epithelialized wound develops hair follicles and sweat glands like those seen in normal skin. E. Contact inhibition can prevent collagen deposition and result in a chronic (nonhealing) wound. Ref.: 2, 4-6 COMMENTS: Migration of epithelial cells is one of the earliest events in wound healing. Shortly after injury and during the inflammatory phase, basal epithelial cells begin to multiply and migrate across the defect, with fibrin strands being used as the support structure. Integrins are the main cellular receptors involved in epithelial migration; they act as sensors and integrators between the extracellular matrix and the epithelial cell cytoskeleton. Tight junctions within the epithelium contribute to its impermeability, whereas the basement membrane contributes to structural support and attachment of the epidermis to the dermis. Surgical incisions seal rather promptly and after 24 hours are protected from the external environment. Early tensile strength is a result of blood vessel ingrowth, epithelialization, and protein aggregation. After covering the wound, the epithelial cells keratinize. The re-epithelialized wound has no sweat glands or hair follicles, which distinguishes it from normal skin. Control of the cellular process during wound epithelialization is not completely understood, but it appears to be regulated in part by contact inhibition, with growth being arrested when two or more similar cells come into surface contact. Derangements in the control of this process can result in epidermoid malignancy. Malignancy is more frequently observed in wounds resulting from ionizing radiation or chemical injury, but it can occur in any wound when the healing process has been chronically disrupted. For example, squamous cell carcinoma may develop in patients with chronic burn wounds or osteomyelitis (Marjolin ulcer).

ANSWER: A 3. A 31-year-old man undergoes his second exploratory laparotomy for bowel obstruction secondary to Crohn’s disease. The patient expresses concern regarding the long-term complications related to his midline incision since he has taken steroids for the last year. Which of the following statements regarding the role of collagen in wound healing is true? A. Collagen synthesis in the initial phase of injury is the sole responsibility of endothelial cells. B. Net collagen content increases for up to 2 years after injury. C. At 3 weeks after injury, more than 50% of the tensile strength of the wound has been restored. D. Tensile strength of the wound increases gradually for up to 2 years after injury; however, it generally reaches a level of only about 80% of that of uninjured tissue. E. Tensile strength is the force necessary to reopen a wound. Ref.: 2, 3, 6 COMMENTS: Synthesis of collagen by fibroblasts begins as early as 10 hours after injury and increases rapidly; it peaks by day 6 or 7 and then continues more slowly until day 42. Collagen continues to mature and remodel for years. Its solubility in saline solution and the thermal shrinkage temperature of collagen reflect the intermolecular cross-links, which are directly proportional to collagen age. After 6 weeks, there is no measurable increase in net collagen content. However, synthesis and turnover are ongoing for life. Historical accounts of sailors with scurvy (with impaired collagen production) who experienced reopening of previously healed wounds illustrate this fact. Tensile strength correlates with total collagen content for approximately the first 3 weeks of wound healing. At 3 weeks, the tensile strength of skin is 30% of normal. After this time, there is a much slower increase in the content of collagen until it plateaus at about 6 weeks. Nevertheless, tensile strength continues to increase as a result of intermolecular bonding of collagen and changes in the physical arrangement of collagen fibers. Although the most rapid increase in tensile strength occurs during the first 6 weeks of healing, there is slow gain for at least 2 years. Its ultimate strength, however, never

CHAPTER 74-3  ■  Self Assessment  

equals that of unwounded tissue, with a level of just 80% of original skin strength being reached. Tensile strength is measured as the load capacity per unit area. It may be differentiated from burst strength, which is the force required to break a wound (independent of its area). For example, in wounds of the face and back, burst strength is different because of differences in skin thickness, even though tensile strength may be similar. Corticosteroids affect wound healing by inhibiting fibro­ blast proliferation and epithelialization. The latter effect can be reversed by the administration of vitamin A.

ANSWER: D 4. Which of the following is correct regarding cell signaling? A. Cytokines are exclusively peptide mediators. B. Autocrine mediators are secreted by a cell and act on adjacent cells of a different type. C. Cytokines are usually produced by cells specialized for only that purpose. D. The effects of hormones are generally local rather than global. E. Growth factors are frequently mediated by second messenger systems such as diacylglycerol (DAG) and cyclic adenosine monophosphate (cAMP). Ref.: 7-9 COMMENTS: Cytokines are proteins, glycoproteins, or peptides that bind to target cell surface receptors to stimulate a cellular response. They are important mediators of wound healing. Cytokines can reach target cells by paracrine, autocrine, or intracrine routes. Paracrine mediators are produced by one cell and act on an adjacent target cell. Autocrine mediators are secreted by a cell and act on cell surface receptors on the same cell. Intracrine mediators act within a single cell. Hormones are released by cells and act on a distant target (endocrine route). Although the distinction between cytokines and hormones has blurred, in general, hormones are secreted from specialized glands (e.g., insulin, parathyroid hormone), and cytokines are secreted by a wide variety of cell types. Hormones typically induce body-wide effects, whereas the effects of cytokines may be more localized (e.g., wound healing at the site of an injury). Generally, growth factors are named according to their tissue of origin or their originally discovered action. Growth factors interact with specific membrane receptors to initiate a series of events that ultimately lead to stimulation of cell growth, proliferation, or differentiation. The intermediate events activate a variety of second messenger systems mediated by agents such as inositol 1,4,5-triphosphate (IP3), DAG, and cAMP.

ANSWER: E 5. A 25-year-old man is seen in the office with complaints of contracture of his left index finger after a burn injury. Which of the following statements is true about growth factors? A. Epidermal growth factor (EGF) stimulates the production of collagen. B. Vascular endothelial growth factor (VEGF) and PDGF both stimulate angiogenesis by binding to a common receptor. C. Fibroblast growth factor (FGF) stimulates wound contraction. D. Transforming growth factor-β (TGF-β) is stored in endothelial cells. E. Tumor necrosis factor-α (TNF-α) inhibits angiogenesis. Ref.: 3, 6, 10, 11 COMMENTS: Epidermal growth factor was the first cytokine described. It is a potent mitogen for epithelial cells, endothelial cells, and fibroblasts. EGF stimulates synthesis of fibronectin, angiogenesis, and collagenase activity. Platelet-derived growth factor is released from the alpha granules of platelets and is responsible for the stimulation of neutrophils and macrophages and for increasing production of TGF-β. PDGF is a mitogen and chemotactic agent for fibroblasts and smooth muscle cells and stimulates angiogenesis, collagen synthesis, and collagenase activity. Vascular endothelial growth factor is similar to PDGF but does not bind to the same receptors. VEGF is mitogenic for endothelial cells. Its role in promoting angiogenesis has led to interest in anti-VEGF therapies for cancer.

e1409

e1410   SECTION 16  ■  TRAUMA Fibro­blast growth factor has acidic and basic forms whose actions are identical but whose strengths differ (basic FGF is 10 times stronger than acidic FGF). FGF is mitogenic for endothelial cells, fibroblasts, keratinocytes, and myoblasts; stimulates wound contraction and epithelialization; and induces the production of collagen, fibronectin, and proteoglycans. It is an important mediator of angiogenesis. Transforming growth factor-β is released from the alpha granules of platelets and has been shown to regulate its own production in an autocrine manner. TGF-β stimulates fibroblast proliferation and the production of proteoglycans, collagen, and fibrin. It is an important mediator of fibrosis. Administration of TGF-β has been suggested as an approach to reduce scarring and reverse the inhibition of wound healing by glucocorticoids. Tumor necrosis factor-α is a mitogen for fibroblasts and is produced by macrophages. It stimulates angiogenesis and the synthesis of collagen and collagenase.

ANSWER: C 6. A 34-year-old man sustained a gunshot wound to his abdomen that necessitated exploratory laparotomy and small bowel resection. Two weeks after the initial operation, he was re-explored for a large intra-abdominal abscess. Which of the following will result in the most rapid gain in strength of the new incision? A. A separate transverse incision is made. B. The midline scar is excised with a 1-cm margin. C. The midline incision is reopened without excision of the scar. D. The midline incision is left to heal by secondary intention. E. The rate of gain in strength is not affected by the incision technique. Ref.: 2, 3, 6 COMMENTS: When a normally-healing wound is disrupted after approximately the fifth day and then reclosed, return of wound strength is more rapid than with primary healing. This is termed the secondary healing effect and appears to be caused by elimination of the lag phase present in normal primary healing. If the skin edges more than about 7 mm around the initial wound are excised, the resulting incision is through essentially uninjured tissue, so accelerated secondary healing does not occur.

ANSWER: C 7. A 21-year-old graduate student has a large hypertrophic scar on the lower part of her face. The patient had sustained a laceration on her face 2 years previously after hitting her face on the side of a swimming pool. Which of the following statements regarding scar revision is true? A. Scar maturation refers to the change in size of the wound in the first 1 to 2 months. B. Scar revision should have been performed in the first 3 months after injury to minimize fibrosis. C. Revision should be performed earlier in children than in adults. D. It corrects undesirable pigmentation. E. Scar revision should be delayed approximately 1 year to allow maturation. Ref.: 2, 3, 6 COMMENTS: Changes in pliability, pigmentation, and configuration of a scar are known as scar maturation. This process continues for many months after an incision, so it is generally recommended that revision not be carried out for approximately 12 to 18 months because natural improvement can be anticipated within this period. In general, scar maturation occurs more rapidly in adults than in children. Most erythematous scars show little improvement after revision, therefore scar revision should not be undertaken for correction of undesirable scar color alone.

ANSWER: E

CHAPTER 74-3  ■  Self Assessment  

8. A 25-year-old ballet dancer with a history of anorexia nervosa arrives at the emergency department with right lower quadrant pain. After an appendectomy, a wound infection at the surgical site requires débridement. The patient is placed on an antibiotic regimen, and the wound is packed with wet-to-dry dressings. Regarding wound healing and malnutrition, which of the following statements is true? A. Hypoproteinemia leads to decreased levels of arginine and glutamine, which are essential in wound healing. B. Cell membranes rapidly become dehydrated in the absence of vitamin E, resulting in delayed wound healing. C. Zinc is essential to the fibroblast’s ability to cross-link collagen. D. Vitamin D serves an immunomodulatory role in wound healing. E. The patient should be treated with high-dose vitamin C, vitamin A, and zinc. Ref.: 2, 12 COMMENTS: Adequate amounts of protein, carbohydrates, fatty acids, and vitamins are essential for wound healing. Hypoproteinemia results in decreased delivery of the essential amino acids used in the synthesis of collagen. Carbohydrates and fats provide energy for wound healing, and in their absence, proteins are rapidly broken down. Fatty acids are vital components of cell membranes. Vitamin C is a cofactor for hydroxylation of lysine and proline during collagen synthesis, and deficiency leads to decreased collagen cross-linking by fibroblasts. Vitamin C is also effective in providing resistance to infection. Vitamin A is essential for normal epithelialization, proteoglycan synthesis, and enhanced immune function. Vitamin D is required for normal calcium metabolism, but it is also involved in promoting immune function in the skin. Vitamin E has not been shown to play a role in wound healing. Zinc deficiency leads to deficient formation of granulation tissue and inhibition of cellular proliferation. Increased administration of vitamins and minerals does not accelerate wound healing and often has a deleterious effect.

ANSWER: D

References 1. Alarcon LH, Fink MP: Mediators of the inflammatory response. In Townsend CM, Beauchamp RD, Evers BM, et al, editors: Sabiston textbook of surgery: the biological basis of modern surgical practice, ed 18, Philadelphia, 2008, WB Saunders. 2. Ethridge RT, Leong M, Phillips LG: Wound healing. In Townsend CM, Beauchamp RD, Evers BM, et al, editors: Sabiston textbook of surgery: the biological basis of modern surgical practice, ed 18, Philadelphia, 2008, WB Saunders. 3. Fine NA, Mustoe TA: Wound Healing. In Mulholland MW, Lillemoe KD, Doherty GM, et al, editors: Greenfield’s surgery: scientific principles and practice, ed 4, Philadelphia, 2006, Lippincott Williams & Wilkins. 4. Simmons RL, Steel DL: Basic science review for surgeons, Philadelphia, 1992, WB Saunders. 5. Gupta S, Lawrence WT: Wound healing: normal and abnormal mechanisms and closure techniques. In O’Leary JP, Tabuenca A, editors: The physiologic basis of surgery, ed 4, Philadelphia, 2008, Lippincott Williams & Wilkins. 6. Barbul A, Efron DT: Wound healing. In Brunicardi FC, Andersen DK, Billiar TR, et al, editors: Schwartz’s principles of surgery, ed 9, New York, 2010, McGraw-Hill. 7. Ko TC, Evers BM: Molecular and cell biology. In Townsend CM, Beauchamp RD, Evers BM, et al, editors: Sabiston textbook of surgery: the biological basis of modern surgical practice, ed 18, Philadelphia, 2008, WB Saunders. 8. Williams JA, Dawson DC: Cell structure and function. In Mulholland MW, Lillemoe KD, Doherty GM, et al, editors: Greenfield’s surgery: scientific principles and practice, ed 4, Philadelphia, 2006, Lippincott Williams & Wilkins. 9. Rosengart MR, Billiar TR: Inflammation. In Mulholland MW, Lillemoe KD, Doherty GM, et al, editors: Greenfield’s surgery: scientific principles and practice, ed 4, Philadelphia, 2006, Lippincott Williams & Wilkins. 10. Peacock EE Jr.: Symposium on biological control of scar tissue, Plast reconstr surg 41:8–12, 1968. 11. Barbul A: Immune aspects of wound repair, Clin Plast Surg 17:433–442, 1990. 12. Martindale RG, Zhou M: Nutrition and metabolism. In O’Leary JP, Tabuenca A, editors: The physiologic basis of surgery, ed 4, Philadelphia, 2008, Lippincott Williams & Wilkins.

e1411

75 

Amputations – Lower Extremity

GOALS/OBJECTIVES • • •

BASIC PRINCIPLES ANATOMY TECHNICAL CONSIDERATIONS

LOWER EXTREMITY AMPUTATION: GENERAL CONSIDERATIONS Wayne W. Zhang  /  Ahmed M. Abou-Zamzam, Jr.

75-1 

From Cronenwett JL, Johnston KW: Rutherford’s Vascular Surgery, 7th edition (Saunders 2010)

Addressing Associated Heart Disease The evaluation of any potential amputee depends on the urgency of the intervention. In patients with acute ischemia, embolic disease directs the focus of the investigation toward embolic sources – most often cardiac. Any arrhythmias or recent myocardial infarction with ventricular thrombus must be investigated and addressed. Echocardiography and anticoagulation are key aspects of perioperative management. Preoperatively, however, the focus is on stabilizing an unstable patient. The nonviable limb should be amputated in a timely manner, with most diagnostic testing taking place in the post­ operative period. Patients with acute ischemia related to thrombosis of chronically diseased arteries have the same risk factors as patients presenting with chronic ischemia. In these patients, the role of extensive peri­ operative cardiac evaluation is unclear. Indeed, most reports of preoperative cardiac evaluation have excluded patients undergoing amputation, focusing instead on those undergoing aneurysm repair or lower extremity revascularization.6,8–10 Comparable perioperative mortality rates regardless of extensive preoperative cardiac evaluation call the practice of preoperative “cardiac clearance” into question.11 Guidelines issued in 2007 by the American College of Cardiology and the American Heart Associa­ tion clarified that the purpose of preoperative cardiac evaluation is not to give medical clearance but rather to evaluate the patient’s current medical status and cardiac risks over the entire perioperative period. These guidelines recommend that no test be performed unless it is likely to influence the patient’s treatment.7

Planning Rehabilitation Throughout the perioperative evaluation, it is important to involve rehabilitation medicine specialists, physical therapists, nurses, and prosthetists in the care of these patients. Centers with dedicated multi­ specialty teams have much more successful rehabilitation outcomes.12 Addressing the patient’s concerns regarding postoperative recovery, the timing of prosthetic use, and ultimate functional goals is best done in the preoperative period.

AMPUTATION LEVEL SELECTION The goals of amputation are (1) to eliminate all infected, necrotic, and painful tissue; (2) to have a wound that heals successfully; and (3) to have an appropriate remnant stump that can accommodate a prosthesis. The length of the preserved limb has important implications for rehabilitation. Prosthetic use following major amputation puts an increased energy demand on the patient. Unilateral belowknee amputees require a 10% to 40% increase in energy expenditure for ambulation, and above-knee amputees require 50% to 70% more energy to ambulate.1 This differential may explain why the suc­ cessful rehabilitation rate is much lower following above-knee amputation (AKA) than below-knee amputation (BKA). Prosthetic use is reportedly 50% to 100% following BKA but only 10% to 30% following AKA.4,12–14 Interestingly, the true rate of ambulation is significantly lower than that of pros­ thetic use, and it shows a steady attrition in the 5 years following amputation.4,13 Partial foot or toe amputations are minor procedures that preserve the majority of the extremity and allow ambulation without the need for bulky prostheses. Most minor amputations, including toe and ray amputations, lead to minimal increases in energy expenditure and require simple orthotic inserts. e1415

e1416   SECTION 17  ■  VASCULAR - ARTERIAL DISEASE Failure of an amputation to heal is multifactorial. Much emphasis has been placed on assessing blood flow at the level of the amputation to predict wound healing. However, failure may be caused not just by ischemia but also by infection, hematoma, or trauma. This explains why no single test can predict with 100% accuracy the ability of an amputation to heal or, conversely, its inability to heal. Most tests are better at predicting wound healing than failure to heal. Thus, using any single test may lead to unnecessarily proximal amputation. The importance of optimizing level selection is underlined by the need to revise BKAs to AKAs in 15% to 25% of patients.1,3,4,15 This revision rate is frequently accompanied by a perioperative mortal­ ity rate of greater than 5%.1 Such events also lead to increased patient anxiety and fear of repeated, more proximal amputations.

Objective Testing and Clinical Judgment The drive to maximize limb length in amputees and to minimize the need for revisions has led to a search for the optimal modality for selecting an amputation level. Physical findings (pulses, skin quality, extent of foot ischemia or infection, skin temperature), noninvasive hemodynamic tests (segmental arterial pressures, Doppler waveforms, toe pressures), invasive anatomic tests (angiographic scoring systems), and physiologic tests (skin blood flow, skin perfusion pressure, muscle perfusion, transcutane­ ous oxygen measurements) have all been extensively investigated.

Pulse Palpation and Physical Findings Physical examination is the essential first step in determining the level of amputation. The extent of gangrene and infection dictates the maximal length attainable. In this evaluation, the presence of depend­ ent rubor should be considered analogous to gangrene because this tissue is ischemic. The presence of pulses should be accurately assessed. The presence of a palpable pulse immediately proximal to a proposed amputation level predicts successful healing in nearly 100% of patients undergoing either major or minor amputation.16,17 However, the absence of a pulse does not necessarily lead to failure of wound healing; therefore, sole reliance on the presence of a pulse leads to unnecessarily proximal amputations. Using “clinical judgment,” which incorporates physical findings and consideration of the patient’s overall status, yields wound healing rates of 80% in BKAs and 90% in AKAs. Wagner and colleagues found that objective data may supplement clinical judgment but not replace it; more distal amputations were achieved with clinical judgment than with sole reliance on objective examinations.18 Experience is important; therefore, the determination of amputation level should not be relegated to junior sur­ geons or trainees. Skin Temperature Measurements The subjective interpretation of skin temperature as a guide for amputation is not reliable. However, several investigators have demonstrated that objective, direct skin temperature measurement may predict amputation healing with an accuracy of 80% to 90%.1,18,19 In a study comparing several non­ invasive techniques, direct skin temperature measurement at the level of amputation with a threshold of 90°F demonstrated the best accuracy.18 Special attention must be paid to room temperature, and comparison with a normal contralateral extremity can be helpful. Ankle and Toe Pressure Measurements The use of noninvasive hemodynamic tests has been extensively evaluated. Frequently employed tests are segmental arterial pressures, Doppler waveforms, and toe pressures. Absolute ankle pressures greater than 60 mm Hg can predict the healing of BKAs with an accuracy of 50% to 90%. Calf pressures and thigh pressures have shown similar reliability.1 However, Wagner and colleagues found that Doppler-derived pressures at the thigh, popliteal, calf, and ankle levels are less reliable than clinical judgment in predicting the healing of BKAs.18 This inaccuracy may be due in part to the high prevalence of diabetes in this population, making measured pressures less reliable because of medial calcinosis. The ankle-brachial index (ABI) should always be obtained, regardless of the presence of a palpable pulse. Marston and colleagues reported on the role of ABI in predicting the need for amputation in a cohort of high-risk patients with critical limb ischemia treated with meticulous wound care but without revascularization. In patients with an ABI less than 0.5, 28% and 34% of limbs required amputation at 6 and 12 months, respectively, versus 10% and 15% of limbs in patients with an ABI greater than 0.5 (P = .01).20

CHAPTER 75-1  ■  Lower Extremity Amputation: General Considerations  

The use of toe pressures has been advocated as being predictive of forefoot amputation healing. Vitti and associates demonstrated universal failure of minor amputations in patients with diabetes and toe pressures less than 38 mm Hg.21 However, there was no similar threshold value in patients without diabetes, limiting the usefulness of this parameter.

Arteriography Invasive testing with arteriography has been investigated as a means of determining amputation level, but the correlation between arteriographic findings and healing potential has been poor. Dwars and coworkers found that angiographic scores did not correlate with amputation healing.17 In fact, in their report, angiographic patency tended to be greater in limbs with failed or delayed healing than in limbs with successful healing. Radioisotope Scans, Scintigraphy, and Skin Perfusion Pressure All physiologic tests attempt to predict wound healing based on tissue perfusion or oxygen delivery at the proposed level of amputation. One technique of measuring skin blood flow involves injecting an intradermal isotope (xenon 133 or iodine 125) and then calculating blood flow by measuring the isotope washout rate using nuclear medicine scanning devices.2,22 Malone and associates initially reported excellent results with xenon 133 clearance, with an accuracy of 92% to 97%.23 However, in a follow-up report, this same group found that the overlap in values between patients with healed and failed amputations made this test too unreliable.22 Sarikaya and coworkers used technetium (Tc) 99m sestamibi scintigraphy to predict the healing of extremity amputation based on deep tissue perfusion.24 Perfusion to the ischemic limb was evaluated preoperatively based on the Tc 99m sestamibi uptake pattern. Nonviable tissue in the extremity was suggested by a clear-cut edge of perfused muscle. The most distal level of amputation was determined above the nonviable tissue. In their 25 patients, the proposed level of amputation based on physical examination and Doppler study was changed to a lower level after Tc 99m sestamibi scintigraphy in 65% of cases, and all amputation wounds healed. Skin perfusion pressure is another physiologic test to determine amputation level. This test involves a scintigraphic technique in which intracutaneous iodine 123 is injected at different amputation levels. External pressure is applied, and by measuring the washout of isotope, the skin perfusion pressure is determined. A level less than 20 mm Hg was predictive of wound failure in 89% of amputations, and a reading greater than 20 mm Hg predicted healing in 99%.17 Skin perfusion pressure can also be measured by laser Doppler velocimetry, thus avoiding the need for isotopes and markedly simplifying the test. Skin fluorescence employs a Wood’s or ultraviolet light following the intravenous injection of fluo­ rescein dye. A qualitative determination of regional blood flow is used to determine the level of amputation. Success rates in predicting healing have ranged from 86% to 100%.2 Owing to the wide availability of these tools and no requirement for radioisotope, this technique is more accessible than scintigraphic techniques. However, the fluorescein technique may be more affected by the presence of inflammation and cellulitis than are scintigraphic techniques. Transcutaneous Oxygen Measurements Transcutaneous oxygen measurement is a completely noninvasive method that is widely used to select an amputation level. A small sensor is placed on the skin in the area of interest. By heating the sensor and skin to 44°C, local skin hyperemia results in decreased flow resistance and arteriolarization of capillary blood. The partial pressure of oxygen measured transcutaneously (tcPo2) approximates the true arterial oxygen pressure in the area of interest.25 The sensors can be placed anywhere on the body, and readings are given in millimeters of mercury (mm Hg). Absolute readings can be recorded, or readings in areas of interest can be indexed to a reference site (often the chest). The probes are small and atraumatic, and multiple sites can be tested simultaneously, depending on the machine. Readings in the supine position are more predictive than measurements in the dependent position or during supplemental oxygen breathing.26 The values recorded are reliable and show an acceptable day-to-day variability in repeat measurements.27 Transcutaneous oxygen levels have an accuracy of 87% to 100% in predicting wound healing.1,16,22,28 Malone and coworkers reported no amputation failures in patients with tcPo2 greater than 20 mm Hg and universal failure when tcPo2 was less than 20 mm Hg.22 Unfortunately, other investigators have

e1417

e1418   SECTION 17  ■  VASCULAR - ARTERIAL DISEASE not confirmed any consistent absolute tcPo2 threshold.1,16,18 Some authors have reported a useful tcPo2 threshold of 30 mm Hg, while others have reported 16 mm Hg as a cutoff value. In general, tcPo2 readings greater than 40 mm Hg are associated with healing and readings less than 20 mm Hg are associated with failure.25,29 The lack of a consistent minimal level is likely due to the fact that nutrient blood flow may be present even in the setting of tcPo2 readings of 0 mm Hg. The tcPo2 may be arti­ ficially low in the setting of infection, inflammation, or edema, and repeat measurements are advised once such processes have resolved. Incidentally, in addition to modest utility in predicting wound healing, tcPo2 has accuracy in predicting outcome following revascularization. Increases in tcPo2 of greater than 30 mm Hg following revascularization are predictive of a successful clinical outcome.28 In direct comparisons with segmental pressures and skin blood flow, tcPo2 has been the most accu­ rate predictor of wound healing.22 This applies not only to major amputation but also to forefoot amputation.2,30 Transcutaneous oxygen measurements are also more accurate than fluorescein dye injections.5,10 In addition, tcPo2 has several advantages over other tests in terms of ease of measurement, reproducibility, and simple instrumentation that can be readily introduced into any vascular laboratory. Yamada and colleagues studied 211 patients with 403 ischemic limbs using skin perfusion pressure, toe pressure, ankle pressure, and tcPo2.31 The correlations between these methods demonstrate that the combination of skin perfusion pressure and toe pressure can accurately predict wound healing. However, the combination of skin perfusion pressure and tcPo2 did not result in a more accurate prediction. A meaningful approach to the accurate determination of amputation level must therefore include a combination of physical findings, clinical judgment, and objective testing.16–18,22,25

Technique Selection Whether skin temperature, skin blood flow or perfusion, or transcutaneous oxygen measurements are used depends on local experience and availability. Measurement of tcPo2 is easily incorporated into a noninvasive vascular laboratory, requires minimal equipment and training, and is reliable and reproduc­ ible. For these reasons, our preferred objective test is tcPo2, with a threshold value of 30 mm Hg. However, strict utilization of a single objective method, rather than taking all available clinical data into account, leads to unnecessarily proximal amputations and denies patients the best opportunity for successful rehabilitation. The various methods of predicting wound healing and their accuracies are shown in Table 75-1-1.31

REHABILITATION CONSIDERATIONS In 2005, 1.6 million people in the United States were living with the loss of a limb. Thirty-eight percent of them had amputations secondary to vascular disease. It is projected that the number of people living with the loss of a limb will more than double by the year 2050 to 3.6 million.32 Reha­ bilitation is crucial for maximizing the functional outcome of these patients. The significant physical and psychological changes following major amputation make rehabilitation a complex process. Inte­ grated rehabilitation requires an interdisciplinary team that incorporates members from surgery, inter­ nal and family medicine, psychiatry, physical therapy, occupational therapy, prosthetics, social services, nursing, nutrition, and recreational therapy.

TABLE 75-1-1  Prediction of Wound Healing by Noninvasive Vascular Studies31 Wound Healing (%) Study

Threshold (mm Hg)

Below Threshold

Above Threshold

Sensitivity (%)

Specificity (%)

SPP tcPo2 TBP ABP

40 30 30 80

10 14 12 11

69 63 67 45

72 60 63 74

88 87 90 70

ABP, ankle blood pressure; SPP, skin perfusion pressure; TBP, toe blood pressure; tcPO2, transcutaneous oxygen pressure.

CHAPTER 75-1  ■  Lower Extremity Amputation: General Considerations  

References 1. DeFrang RD, Taylor LM, Porter JM: Basic data related to amputations. Ann Vasc Surg 1991;5:202. 2. Malone JM: Lower extremity amputations. Moore WS Vascular Surgery: A Comprehensive Review. 2002 WB Saunders Philadelphia 875 3. Abou-Zamzam AM Jr, Teruya TH, Killeen JD, Ballard JL: Major lower extremity amputation in an academic vascular center. Ann Vasc Surg 2003;17:86. 4. Nehler MR, Coll JR, Hiatt WR, et al.: Functional outcome in a contemporary series of major lower extremity amputations. J Vasc Surg 2003;38:7. 5. Edwards JM, Taylor LM Jr, Porter JM: Limb salvage in end-stage renal disease (ESRD). Comparison of modern results in patients with and without ESRD. Arch Surg 1998;123:1164. 6. Krupski WC, Nehler MR, Whitehill TA, et al.: Negative impact of cardiac evaluation before vascular surgery. Vasc Med 2000;5:3. 7. Fleisher LA, Beckman JA, Brown KA, et al.: ACC/AHA 2007 Guidelines on Perioperative Cardiovascular Evaluation and Care for Noncardiac Surgery: Executive Summary: A Report of the American College of Cardiology/American Heart Association Task Force on Practice Guidelines (Writing Committee to Revise the 2002 Guidelines on Perioperative Car­ diovascular Evaluation for Noncardiac Surgery): Developed in Collaboration with the American Society of Echocardio­ graphy, American Society of Nuclear Cardiology, Heart Rhythm Society, Society of Cardiovascular Anesthesiologists, Society for Cardiovascular Angiography and Interventions, Society for Vascular Medicine and Biology, and Society for Vascular Surgery. Circulation 2007;116:1971. 8. Barone JE, Tucker JB, Rassias D, Corvo PR: Routine perioperative pulmonary artery catheterization has no effect on rate of complications in vascular surgery: a meta-analysis. Am Surg 2001;67:674. 9. de Virgilio C, Toosie K, Elbassir M, et al.: Dipyridamole-thallium/sestamibi before vascular surgery: a prospective blinded study in moderate-risk patients. J Vasc Surg 2000;32:77. 10. de Virgilio C, Wall DB, Ephraim L, et al.: An abnormal dipyridamole thallium/sestamibi fails to predict long-term cardiac events in vascular surgery patients. Ann Vasc Surg 2001;15:267. 11. McFalls EO, Ward HB, Moritz TE, et al.: Coronary-artery revascularization before elective major vascular surgery. N Engl J Med 2004;351:2795. 12. Malone JM, Moore W, Leal JM, Childers SJ: Rehabilitation for lower extremity amputation. Arch Surg 1981;116:93. 13. McWhinnie DL, Gordon AC, Collin J, et al.: Rehabilitation outcome 5 years after 100 lower-limb amputations. Br J Surg 1994;81:1596. 14. Toursarkissian B, Shireman PK, Harrison A, et al.: Major lower-extremity amputation: contemporary experience in a single Veterans Affairs institution. Am Surg 2002;68:606. 15. Keagy BA, Schwartz JA, Kotb M, et al.: Lower extremity amputation: the control series. J Vasc Surg 1986;4:321. 16. Ballard JL, Eke CC, Bunt TJ, Killeen JD: A prospective evaluation of transcutaneous oxygen measurements in the manage­ ment of diabetic foot problems. J Vasc Surg 1995;22:485. 17. Dwars BJ, van den Broek TA, Rauwerda JA, Bakker FC: Criteria for reliable selection of the lowest level of amputation in peripheral vascular disease. J Vasc Surg 1992;15:536. 18. Wagner WH, Keagy BA, Kotb MM, et al.: Noninvasive determination of healing of major lower extremity amputation: the continued role of clinical judgment. J Vasc Surg 1988;8:703. 19. Spence VA, Walker WF, Troup IM, Murdoch G: Amputation of the ischemic limb: selection of the optimum site by thermography. Angiology 1981;32:155. 20. Marston WA, Davies SW, Armstrong B, et al.: Natural history of limbs with arterial insufficiency and chronic ulceration treated without revascularization. J Vasc Surg 2006;44:108. 21. Vitti MJ, Robinson DV, Hauer-Jensen M, et al.: Wound healing in forefoot amputations: the predictive value of toe pres­ sure. Ann Vasc Surg 1994;8:99. 22. Malone JM, Anderson GG, Lalka SG, et al.: Prospective comparison of noninvasive techniques for amputation level selec­ tion. Am J Surg 1987;154:179. 23. Malone JM, Leal JM, Moore WS, et al.: The “gold standard” for amputation level selection xenon-133 clearance. J Surg Res 1981;30:449. 24. Sarikaya A, Top H, Aygit AC, et al.: Predictive value of (99m)Tc-sestamibi scintigraphy for healing of extremity amputa­ tion. Eur J Nucl Med Mol Imaging 2006;33:1500. 25. Ballard JL, Bianchi C: Transcutaneous oxygen tension: principles and application. Abu-Rhama AF Bergan JJ Noninvasive Vascular Diagnosis. 2000 Springer New York 403. 26. Scheffler A, Rieger H: A comparative analysis of transcutaneous oximetry (tcPO2) during oxygen inhalation and leg dependency in severe peripheral arterial occlusive disease. J Vasc Surg 1992;16:218. 27. Jorneskog G, Djavani K, Brismar K: Day-to-day variability of transcutaneous oxygen tension in patients with diabetes mellitus and peripheral arterial occlusive disease. J Vasc Surg 2001; 34:277. 28. Bunt TJ, Holloway GA: TcPO2 as an accurate predictor of therapy in limb salvage. Ann Vasc Surg 1996;10:224. 29. Bacharach JM, Rooke TW, Osmundson PJ, Gloviczki P: Predictive value of transcutaneous oxygen pressure and amputation success by use of supine and elevation measurements. J Vasc Surg 1992;15:558. 30. Gibbons GW: Lower extremity bypass in patients with diabetic foot ulcers. Surg Clin North Am 2003;83:659. 31. Yamada T, Ohta T, Ishibashi H, et al.: Clinical reliability and utility of skin perfusion pressure measurement in ischemic limbs – comparison with other noninvasive diagnostic methods. J Vasc Surg 2008;47:318. 32. Ziegler-Graham K, MacKenzie EJ, Ephraim PL, et al.: Estimating the prevalence of limb loss in the United States: 2005 to 2050. Arch Phys Med Rehabil 2008;89:422.

e1419

75-2 

UPPER AND LOWER EXTREMITY AMPUTATION Jeffry L. Kashuk  /  Kagan Ozer  /  Meryl Singer Livermore  /  Walter L. Biffl From Cioffi W, et al: Atlas of Trauma/Emergency Surgical Techniques, 1st edition (Saunders 2013)

FIGURE 75-2-1  A-E, Below knee amputation.

e1420

CHAPTER 75-2  ■  Upper and Lower Extremity Amputation  

FIGURE 75-2-2  A-D, Knee disarticulation preserving patellar tendon.

FIGURE 75-2-3  A-D, Above knee amputation.

e1421

e1422   SECTION 17  ■  VASCULAR - ARTERIAL DISEASE

FIGURE 75-2-4  A-D, Operative steps for hip disarticulation.

Further Reading Aulivola B, Hile CN, Hamdan AD, Sheahan MG, et al: Major lower extremity amputation: outcome of a modern series. Arch Surg 2004; 139: 395–399.

SELF ASSESSMENT Ference P. Nagy From Velasco J: Rush University Medical Center Review of Surgery, 5th edition (Saunders 2011)

75-3 

1. The principles of postoperative management after amputation include which of the following? A. Splinting of the stump dressing to avoid shift in position B. Compression dressing over the stump to avoid postoperative edema and hematoma formation C. Exercise and positioning to avoid contracture D. Early evaluation and care by a qualified physical therapist and prosthetist E. All of the above Ref.: 1, 2 COMMENTS: Conventional postoperative care begins with the application of a light compression dressing in the operating room, followed by repeated application of elastic dressings to avoid stump edema. Damage to the skin can result from excessively compressive dressings applied over bony promi­ nences (e.g., the anterior tibial area in a below-knee stump). Stump exercises and stretching prevent contracture after primary wound healing has taken place. Progressive training after suture removal allows the eventual application of a permanent prosthesis. Alternatively, application of a rigid dressing in the operating room allows the immediate use of a prosthetic device, but a rigid dressing may also be used without immediate prosthetic fitting. Rigid dressings offer the advantage of immediate use of the extremity. Wound healing may be enhanced by maximum control of edema and hematoma and by better tissue immobilization. When treatment is successful, resumption of full activity can be expected to occur within 4 to 6 weeks. All of the listed choices are important for proper amputation management.

ANSWER: E 2. Useful preoperative methods of evaluating the adequacy of blood flow in patients with peripheral vascular disease undergoing amputation include which of the following? A. Clinical assessment of cutaneous blood flow and assessment of the status of peripheral pulses B. Determination of transcutaneous Po2 and Pco2 C. Segmental Doppler systolic blood pressure determinations D. Laser Doppler velocimetric studies E. All of the above Ref.: 1–3 COMMENTS: The healing ability of an amputation stump is determined by the adequacy of nutri­ tional blood flow to the skin. Clinical assessment is successful in determining the level of amputation in approximately 80% of below-knee amputations and 90% of above-knee amputations. For amputations below the ankle, clinical judgment alone has been shown to be less effective, with a healing rate e1423

e1424   SECTION 17  ■  VASCULAR - ARTERIAL DISEASE of only 40%. Physical examination findings such as the extent of tissue necrosis, skin temperature, capillary refill, and pulse evaluation help determine the clinical assessment. The presence of pulses immediately above the proposed amputation site is a good prognostic indicator, but the absence of such a pulse does not necessarily preclude adequate wound healing. Several other methods have been used to preoperatively select amputation levels, including Doppler segmental blood pressure measurements, transcutaneous oxygen and carbon dioxide measurements, fluorescein dye measurements, laser Doppler velocimetric studies, isotope measurement of skin per­ fusion, conventional or magnetic resonance angiography, and others. Segmental Doppler blood pres­ sure measurement is probably the most commonly used first test to assist in determining the level of amputation. An absolute pressure of at least 50 to 70 mm Hg at the calf and 80 mm Hg at the thigh is highly predictive of successful healing of a below-knee amputation. However, a major caveat of this test modality is the falsely elevated pressures that result from calcification of the arterial wall, particularly in the tibial vessels of diabetic patients. Transcutaneous Po2 levels can also be measured and may help determine whether a particular level of amputation will heal. A transcutaneous Po2 value greater than 40 mm Hg is associated with successful healing, and a value below 20 mm Hg is associ­ ated with failure.

ANSWER: E 3. Which of the following statements regarding proper selection of the level of amputation is true? A. The extent of resection for malignant tumors must be compromised for functional considerations. B. The use of skin grafts and flaps to conserve bone length is appropriate in healthy, stable trauma patients. C. Unless 4 inches or more of tibia can be preserved, the knee joint should be sacrificed. D. The presence of contracture should not influence the level of amputation. E. None of the above. Ref.: 1–3 COMMENTS: As a general principle, the longer the amputation stump, the more functional the limb. However, when performing amputations for malignancy, adequate tumor excision, not preserva­ tion of stump length, is the primary concern. The irregular damage to skin caused by trauma can be treated by skin grafts and flaps to preserve bone length. Full-thickness skin should be maintained for weight-bearing surfaces. Amputations in patients with peripheral vascular disease succeed best when performed at levels that have adequate nutritional blood flow to the skin. Below-knee stumps as short as 2 inches can be successfully fitted with prostheses, but function is much better if at least 4 inches of stump is maintained. Preservation of the knee joint allows a bent-knee, end-weight-bearing pros­ thesis to be used and is generally preferable to a long above-knee stump. Amputations above the knee should remove at least 4 inches of femur to facilitate fitting a prosthetic knee joint. Relative contrain­ dications to below-knee amputation include the presence of a hip or knee contracture, which negates its functional advantage.

ANSWER: B 4. Indications for an above-knee amputation do not include which of the following? A. Absent popliteal pulses B. Gangrene at the tibial tuberosity C. Calf muscle rigor D. Knee or hip contractures E. Patient with minimal potential for rehabilitation and ambulation Ref.: 1, 2

CHAPTER 75-3  ■  Self Assessment  

COMMENTS: Assessment of vascular status at the level of amputation is important, but the absence of popliteal pulses or the presence of diabetes is not in itself an absolute indication for an above-knee amputation. The rigor of the calf muscles and the presence of gangrene of the skin at the level where the flaps would be constructed for a below-knee amputation are sufficient indications for an aboveknee amputation. Because knee and hip contractures make rehabilitation after below-knee amputation unlikely and because an above-knee amputation has the highest healing rate and the lowest reamputa­ tion rate in patients with severe peripheral vascular disease, patients so afflicted do best with an aboveknee amputation.

ANSWER: A 5. Which of the following statements regarding amputation of upper extremity digits is true? A. A shorter volar flap and a longer dorsal flap are desired. B. The root of the nail should always be preserved. C. During removal of the distal phalanx, the distal middle phalangeal cartilage should be preserved. D. Amputation at the metacarpophalangeal joint is preferable to amputation through the proximal phalanx. E. Even the smallest stump of the thumb is preferable to complete amputation with a prosthesis. Ref.: 1, 2 COMMENTS: Closure of the stump following amputation of an upper extremity digit is best accomplished with a longer volar flap so that the scar can be positioned away from pressure-bearing surfaces. However, bone and viable tissue should never be sacrificed to achieve ideal scar placement. Unless more than one half of the nail bed can be preserved, the nail root should be removed. If the distal phalanx must be removed, the exposed middle phalangeal cartilage should be resected. Given a choice, resection through the proximal phalanx is preferred over a metacarpophalangeal amputation. As is the case with the thumb, any stump, no matter how short, has function. When a digit must be removed in its entirety, preservation of function of the hand as a unit is the goal and may require a variety of secondary procedures.

ANSWER: E

References 1. Frymoyer JW, editor: Orthopaedic basic science, Rosemont, IL, 1993, American Academy of Orthopaedic Surgeons. 2. Kasser JR, editor: Orthopaedic knowledge update 5, Rosemont, IL, 1996, American Academy of Orthopaedic Surgeons. 3. Nehler MR: Extremity amputation for vascular disease. In Rutherford RB, editor: Vascular surgery, ed 6, Philadelphia, 2005, WB Saunders.

e1425

76 

Carotid Endarterectomy

GOALS/OBJECTIVES • • • •

ANATOMY INDICATIONS TECHNIQUE COMPLICATIONS

CEREBROVASCULAR DISEASE: GENERAL CONSIDERATIONS William C. Mackey From Cronenwett JL, Johnston KW: Rutherford’s Vascular Surgery, 7th edition (Saunders 2010)

76-1 

PATHOLOGY, PATHOGENESIS, AND PATHOPHYSIOLOGY Atherosclerotic Carotid Plaque Carotid plaque places asymptomatic patients at risk for TIA and stroke. The degree of stenosis, plaque density, and plaque progression are correlates of the degree of risk related to carotid artery atherosclerotic lesions. Patients at highest risk for TIA or stroke are those with greater than 80% stenosis secondary to soft, echolucent plaque or those whose plaque progresses from less than 80% to greater than 80% stenosis during follow-up. These clinical findings are consistent with our current understanding of plaque evolution and degeneration. Benign fatty streaks progress to fibrous plaque. Continued infiltration of lipid into the arterial wall leads to macrophage infiltration, elaboration of growth factors, and chronic inflammation with a slow increase in plaque mass. Macrophage lysis with release of proteolytic enzymes, coupled with further lipid infiltration, results in complex plaque with areas of lipid accumulation, necrotic debris, ongoing chronic inflammation, and calcification.1 Neovascularity within the arterial wall and overlying plaque results from this cycle of ongoing inflammation and healing. As illustrated in Figure 76-1-1, areas of intraplaque hemorrhage may “heal” and renew the cycle of macrophage infiltration, calcification, fibrosis, and ongoing plaque evolution. Alternatively, intraplaque hemorrhage can result in sudden plaque expansion with arterial stenosis or occlusion or in rupture of the fibrous cap with resultant embolization.1,2 Furthermore, once plaque rupture occurs, the resulting ulcer will act as a nidus for the accumulation of platelet aggregates and other thrombotic debris and thereby put the patient at risk for further episodes of embolization (Figure 76-1-2). Figure 76-1-3 is a color-flow duplex scan showing deeply ulcerated plaque. It is easy to see how this plaque is a potential source of cerebral embolization. Figure 76-1-4 is a photomicrograph of a typical deeply ulcerated carotid plaque containing loosely adherent thrombotic debris prone to embolization. Given our current understanding of the pathogenesis of cerebrovascular events based on this scenario, it is understandable that low-density plaque (more lipid pool or intraplaque hemorrhage), plaque causing greater stenosis, or plaque showing progression and therefore instability would be associated with a greater risk for cerebrovascular events. The clinical relevance of this pathologic description of plaque evolution lies in its description of “stable” plaque unlikely to result in TIA or stroke and “unstable plaque” associated with these clinical events. The “Holy Grail” in this field remains imaging findings or biochemical markers that will reliably distinguish between plaque destined to remain stable and that destined to become unstable. The ability to reliably distinguish between stable and unstable plaque will greatly improve patient selection for surgery or stenting, especially in cases of asymptomatic disease. Some current studies using MRI technology suggest that it can reliably measure fibrous cap thickness and integrity and other potentially important features.3 In a recent prospective study of 154 asymptomatic patients with 50% to 79% stenosis monitored for a mean of 38.2 months, Takaya and colleagues noted 12 cerebrovascular events. MRI findings at study entry of a thin or ruptured fibrous cap (odds ratio [OR], 17.2; P < .001), intraplaque hemorrhage (OR, 5.2; P = .005), larger area of intraplaque hemorrhage (OR for 10 mm2, 2.6; P = .006), larger lipid-rich or necrotic core (OR for a 10% increase, 1.6; P = .004), and greater maximal wall thickness (OR for a 1-mm increase, 1.6; P = .008) correlated well with clinical behavior.3 The ability to differentiate stable from unstable plaque with imaging studies looms on the horizon, and it is also possible that biochemical markers will provide even earlier prediction of plaque behavior. e1427

e1428   SECTION 17  ■  VASCULAR - ARTERIAL DISEASE

FIGURE 76-1-1  The possible events after an initial intraplaque hemorrhage. (Redrawn from Bergan JJ, Yao JST, eds. Cerebrovascular Insufficiency. New York, NY: Grune & Stratton; 1983:51.)

FIGURE 76-1-2  After fibrous cap rupture and plaque ulceration, the irregular, thrombogenic surface serves as a nidus for the deposition of platelet aggregates and other thrombotic debris, which can become dislodged and embolize to the brain.

FIGURE 76-1-3  Color-flow duplex ultrasonogram of deeply ulcerated carotid plaque in a patient with multiple episodes of transient monocular blindness. CCA, common carotid artery; ECA, external carotid artery; ICA, internal carotid artery.

CHAPTER 76-1  ■  Cerebrovascular Disease: General Considerations  

FIGURE 76-1-4  Photomicrograph of ulcerated plaque showing organizing thrombus loosely adherent to the wall of the ulcerated plaque. The potential for embolization is apparent.

Alvarez and coworkers studied matrix metalloproteinase-2 (MMP-2) and MMP-9 and found that serum levels of both were statistically significantly higher in symptomatic than in asymptomatic patients.4 Furthermore, MMP-9 levels correlated with the presence of unstable plaque as determined by infiltration of plaque by lymphocytes and macrophages.4 By logistic regression analysis, these investigators determined that the best predictors of unstable plaque were a previous neurologic event and an MMP level of greater than 607 ng/mL (sensitivity, 96%; specificity, 92%; negative predictive value, 94.7%; positive predictive value, 93%).4 Similarly, C-reactive protein (CRP) levels have been shown to correlate with the presence of unstable plaque. The same team of investigators that evaluated MMP levels found that mean CRP levels were 27.1 mg/L (258 nmol/L) in patients with unstable plaque but only 4.1 mg/L (39 nmol/L) in those with stable plaque.5 More recently, another team of investigators has shown that CRP levels correlate with carotid plaque progression.6 In the study by Arthurs and associates, patients with CRP levels in the highest quartile were statistically significantly more likely to suffer plaque progression as measured by duplex ultrasound than were those with lower CRP levels (OR, 1.8; 95% CI, 1.03 to 2.99; P < .05).6 The reliability and clinical applicability of these findings have yet to be determined. If reliable imaging or biochemical markers of plaque instability are determined, asymptomatic patients could be selected for surgery with much greater precision.

References 1. Ross R: The pathogenesis of atherosclerosis: an update. N Engl J Med 1986;314:488–500. 2. Ross R: Atherosclerosis – an inflammatory disease. N Engl J Med 1999;340:115–126. 3. Takaya N, Yuan C, Chu B, et al.: Association between carotid plaque characteristics and subsequent cerebrovascular events. Stroke 2006;37:818–823. 4. Alvarez B, Ruiz C, Chacon P, et al.: Serum values of metalloproteinase-2 and metalloproteinase-9 as related to unstable plaque and inflammatory cells in patients with greater than 70% carotid artery stenosis. J Vasc Surg 2004;40:469–475. 5. Alvarez Garcia B, Ruiz C, Chacon P, et al.: High sensitivity C-reactive protein in high grade carotid stenosis: risk marker for unstable plaque. J Vasc Surg 2003;38:1018–1024. 6. Arthurs ZM, Andersen C, Starnes BW, et al.: A prospective evaluation of C-reactive protein in the progression of carotid artery stenosis. J Vasc Surg 2008;47:744–751.

e1429

76-2 

CAROTID ARTERY DISEASE: ENDARTERECTOMY Glen S. Roseborough  /  Bruce A. Perler From Cronenwett JL, Johnston KW: Rutherford’s Vascular Surgery, 7th edition (Saunders 2010)

OPERATIVE TECHNIQUE Anesthesia A fundamental consideration in the conduct of CEA is selection of the anesthetic method. CEA may be performed under general anesthesia (GA), under regional anesthesia (RA) with deep or superficial cervical block, and even under pure local anesthesia (LA). Although early reports suggested a reduced length of hospital stay associated with CEA performed under RA, comparable lengths of stay are routinely documented today in patients who undergo surgery under GA. The majority of studies comparing the two techniques have reported improved perioperative cardiac stability with RA, but this does not necessarily result in a reduced incidence of myocardial infarction.1,2 Disadvantages of RA include patient discomfort or anxiety, risk of seizure or allergic reaction, anxiety for the operating surgeon, and compromise of technique in a teaching setting. A meta-analysis by Tangkanakul and associates,3 as well as one by Rerkasem and colleagues,2 showed no clear benefit for CEA performed under LA. The GALA trial, a large prospective European multinational study of more than 3500 patients, was designed to definitively determine whether one technique is superior to the other. It demonstrated no difference in outcomes between GA and LA.4 The main benefit of LA is that it facilitates efficient selective shunting if that is the surgeon’s preference. However, the benefit of selective shunting is unclear.

Patient Positioning Careful positioning of the patient is important to ensure patient comfort and adequate operative exposure (Figure 76-2-1). Positioning begins with placing a roll behind the scapulae to achieve some hyperextension of the neck. A padded ring is placed under the head to prevent neck injury from extreme hyperextension. If GA is used, the endotracheal tube should be taped to the corner of the mouth opposite the surgical field. If LA or RA is used, a Mayo stand is placed over the patient’s head to suspend the surgical drapes away from the patient’s face to prevent sensations of claustrophobia. It is our practice to tape the fan for a “Bair Hugger” (3M Company, St Paul, Minn.) warmer on the underside of the Mayo stand because blowing air on the patient’s face relieves discomfort. If GA is used, it is induced before placement of additional lines. A radial artery catheter is inserted for continuous blood pressure monitoring, and a Foley catheter is placed. The patient is placed in the flexed position with the table rotated to expose the side of the neck to be operated on.

Skin Incision One of two skin incisions may be used (Figure 76-2-2). The standard incision is a longitudinal incision parallel to the medial border of the sternocleidomastoid muscle. The upper portion of the incision is angled posterior to the earlobe if cephalic exposure above the angle of the jaw is required. An alternative method is to place the incision in an appropriately located skin crease, usually 1 to 2 cm inferior to the angle of the jaw. This incision provides excellent cosmesis postoperatively; frequently, the resulting scar is all but invisible. However, if the incision is made in a suboptimal location, it is difficult to obtain more cephalic and caudal exposure in the wound. Therefore, if the surgeon is not experienced with a skin crease incision, it is best to make a skin crease incision based on the location of the carotid bifurcation, known either from preoperative DSA, CTA, or MRA or from intraoperative Duplex e1430

CHAPTER 76-2  ■  Carotid Artery Disease: Endarterectomy  

FIGURE 76-2-1  Patient positioning for carotid endarterectomy. A, The table is placed in the reversed Trendelenburg position with the patient’s legs elevated. B, The patient’s head is turned to the contralateral side with a roll placed under the shoulders to extend the neck.

FIGURE 76-2-2  Longitudinal and transverse skin incisions for carotid endarterectomy.

examination, or to use a longitudinal incision. With experience the surgeon will become more comfortable making the incision based on the location of the carotid pulse, although this can be misleading at times. If the incision is made too low, more cephalic exposure can be obtained by extending the skin crease incision posteriorly. If the incision is made too high, more caudal exposure can be obtained by extending the incision more anteriorly.

Conventional versus Eversion Endarterectomy There are two basic surgical techniques for CEA: conventional and eversion. Regardless of which method is used, meticulous surgical technique is paramount for a successful operation. Manipulation

e1431

e1432   SECTION 17  ■  VASCULAR - ARTERIAL DISEASE of the carotid artery should be minimized because intraoperative embolization can result from careless handling. Any blood in the field should be carefully aspirated. The dissection is begun by dividing the platysma and mobilizing the medial border of the sternocleidomastoid muscle. The external jugular vein lies deep to the platysma and should be sought in this plane to avoid injury or to retrieve it if it is needed for patching. It is more commonly encountered with an oblique skin crease incision than with a longitudinal incision. The other structure located at this level is the greater auricular nerve; injury to this nerve leads to numbness of the earlobe, which is bothersome to patients, especially those who wear earrings (see later under “Cutaneous Sensory Nerves”). The medial border of the platysma is mobilized and retracted laterally with self-retaining retractors. The carotid sheath is entered and the medial border of the jugular vein dissected. The facial vein is identified crossing medially in the base of the wound and divided; sometimes it has an early bifurcation or trifurcation, and multiple branches need to be ligated. The jugular vein is then retracted laterally. The vagus nerve is identified at this point in the carotid sheath, usually located posteriorly between the jugular vein and carotid artery, although in a minority of patients it may lie anteriorly. Dissection is continued down onto the distal CCA and is controlled circumferentially with an umbilical tape and a Rumel tourniquet. At this point the ansa cervicalis nerve should be identified; it usually lies medial to the distal CCA. Identifying this nerve facilitates safe dissection of the carotid bifurcation and avoids injury to the hypoglossal nerve, which crosses medially from a superior to an inferior location. As the surgeon dissects superiorly along the ansa cervicalis, the dissection should be continued along the posterior edge of this nerve. As one follows the nerve cephalad and encounters its junction with the hypoglossal nerve, the surgeon will continue the dissection safely along the posterior border of the hypoglossal nerve with minimal risk of injury; if one dissects along the anterior border of the ansa, it is possible to inadvertently transect the hypoglossal nerve in the crotch of the junction of these two nerves before the hypoglossal nerve is identified (Figure 76-2-3). At this point the carotid bifurcation is carefully exposed. The superior thyroid artery is identified coming off the medial border of the carotid bifurcation or proximal ECA and controlled with a tie or plastic vessel loop. Dissection continues cephalad on the medial edge of the bifurcation until the origin of the ECA is identified. It is exposed and controlled circumferentially with a vessel loop. Finally, the ICA is exposed coming off the lateral side of the bifurcation. Extreme care must be taken during this part of the dissection. The artery should be controlled in a location that is above the plaque and completely free of disease; in this location the artery has a typical bluish appearance because of translucency of the vessel. During dissection of the carotid bifurcation and its branches, one should avoid dissecting in the crotch of the carotid bifurcation to avoid injuring the carotid body because such dissection can result in hemodynamic instability and troublesome bleeding. If hemodynamic instability results, the carotid body can be gently injected with 1% lidocaine.

FIGURE 76-2-3  Operative field. Note the internal jugular vein mobilized posteriorly after ligation of the anterior facial branch and the hypoglossal nerve crossing the vessels superior to the bifurcation.

CHAPTER 76-2  ■  Carotid Artery Disease: Endarterectomy  

Before clamping, the patient is administered 70 to 100 U/kg of heparin, which is allowed to circulate for 3 minutes. The ICA is clamped first to prevent the embolization that can result when the CCA or ECA is clamped. Care should be taken to make sure that the ICA is clamped on a normal portion of the artery distal to the plaque. There is typically a bluish discoloration of the artery in this location. If LA or intraoperative electroencephalography is used for selective shunting, a test clamp on the distal ICA should be applied for at least 3 minutes to check for changes in the neurologic examination or electroencephalographic (EEG) pattern. If such changes occur, the artery should be unclamped to allow reperfusion before reclamping and opening the carotid bifurcation; opening the bifurcation and placing a shunt may take 2 to 3 minutes and should not be performed while the brain is already ischemic. However, unclamping the ICA introduces the potential for embolization from disrupted plaque. If carotid stump pressure is to be measured, clamps are placed on the CCA and ECA, and a needle connected to a pressure line is placed into the distal CCA below the carotid bifurcation. Both clamping the CCA and placing the needle into the artery introduce the potential for embolization.

Conventional Endarterectomy The conventional technique for CEA consists of a vertical arteriotomy and closure by patch angio­ plasty. In this case a vertical arteriotomy is begun on the CCA and continued through the carotid bifurcation into the ICA. One should avoid making the incision too close to the flow divider at the ECA origin because this can distort the anatomy and make the closure more difficult. If a shunt is used, it is placed in the distal ICA and backbled before the proximal end is placed into the CCA. Two commonly used shunts are the Pruitt-Inahara and Javid shunts. A third shunt that we prefer is a simple vinyl tube, originally described by Collins and associates (Figure 76-2-4).5 Because this shunt lies entirely within the artery, it allows the surgeon to almost completely finish closing the arteriotomy before the shunt is removed. Its small diameter permits atraumatic placement in even small ICAs, whereas its short length offers less resistance to blood flow such that physiologic flow in the ICA is

FIGURE 76-2-4  Operative field after the arteriotomy has been made and the shunt placed (arrow). Note how the shunt lies within the vessel.

e1433

e1434   SECTION 17  ■  VASCULAR - ARTERIAL DISEASE maintained. Wilkinson and colleagues compared the Pruitt-Inahara and Javid shunts in a randomized trial.6 Using TCD, they found that the Pruitt shunt was less likely to maintain physiologic flow in the MCA, whereas the Javid shunt was associated with a higher incidence of cerebral embolism at declamping. The endarterectomy is begun in the CCA in the plane between the media and the adventitia. The proximal endpoint in the distal CCA is established and the plaque is trimmed in that location in a beveled manner. The endarterectomy is continued into the orifice of the ECA, first with a Freer elevator and then with a fine clamp that is passed up into the ECA in the plane of the endarterectomy. The clamp is spread apart to further mobilize the plaque away from the adventitia in the 6-, 9-, and 12-o’clock positions; it is usually hard to pass the clamp in this plane at the 3-o’clock position next to the flow divider. The vessel loop on the ECA is released transiently while the plaque is everted from within the ECA. The endpoint of the plaque is inspected; an ideal endpoint is gradually tapering and feathered (Figure 76-2-5). In our practice, all loose bits of intima and media in the orifice of the ECA are removed to perform complete endarterectomy of the ECA. However, Ascher and coworkers have found in a large series that endarterectomy of the ECA may be neglected without compromising results.7 The endarterectomy is continued up into the ICA. A technically perfect endpoint in the ICA is critical to avoid perioperative stroke and recurrent stenosis. In our experience, it is virtually always possible to achieve a satisfactory endpoint in the ICA, although special maneuvers may be required to expose the distal ICA and make an extended arteriotomy in this vessel to facilitate extraction of a long endarterectomy specimen, as seen in Figure 76-2-5. Tacking sutures at the distal endpoint should be avoided unless absolutely necessary; such suturing is problematic and associated with an increased perioperative stroke rate.8 The endarterectomy should be terminated in normal ICA with a gradual, tapered transition to normal intima; this is best accomplished by pulling the plaque transversely away from the artery with lateral traction. One should avoid pulling out or down on the plaque, which is more likely to result in a stepoff that can be difficult to correct without traumatizing the artery. We believe that repairing the arteriotomy with a patch angioplasty represents the standard of care in contemporary practice (Figure 76-2-6). The patch is sewn in with running nonabsorbable suture. A variety of patch materials are available for use, including autologous vein, polytetrafluoroethylene (PTFE), woven polyester (Dacron), and bovine pericardium. Some studies have suggested that autologous vein may be superior to synthetic patches, but of the prosthetic patches, no material appears to be clearly superior to another. Options for autologous vein include the external jugular and saphenous veins. The external jugular vein can be harvested through the same surgical incision and is generally used as a double-layer patch after inverting an intact tubular segment of vein without filleting open the vein. Care must be taken to keep the inverted tube flattened out as a rectangular patch while it is being sewn onto the artery. If this is not done, the edges can sometimes roll over or under and lead to a tapered, asymmetric, or severely deformed patch. It is our practice to always start the suture line at the superior end of the arteriotomy in the ICA, which is typically the most difficult – and critical – part of an anastomosis. As the patch material is sewn to one side of the artery, the artery must be stretched out with gentle tension so that an appropriate length of patch is used before it is trimmed; otherwise, there may not be enough patch material available to sew to the other side of the artery in the arteriotomy.

FIGURE 76-2-5  Carotid endarterectomy specimen. Note the smooth endpoints of the distal plaque from the internal and external carotid arteries (shorter extent of plaque).

CHAPTER 76-2  ■  Carotid Artery Disease: Endarterectomy  

FIGURE 76-2-6  Patch closure of the arteriotomy for carotid endarterectomy. Dacron was used in this case. Note the ansa cervicalis coursing along the vessel, the vagus nerve posteriorly, and the hypoglossal nerve at the apex of the incision.

When the suture line is nearly completed, the CCA and ICA are reclamped and the shunt is removed. Both clamps are briefly released to flush air or debris (or both) out of the arteries. The clamps should be placed proximal and distal to the patch or endarterectomized surface of the artery because these surfaces can be thrombogenic. The carotid bifurcation is flushed vigorously with heparinized saline and inspected again for debris or intimal flaps before the arteriotomy is finally closed. Once again the clamp on the ICA is briefly released to fill the bifurcation with blood. It is then replaced while the clamps on the CCA and ECA are released so that any remaining air or debris will be flushed up the territory of the ECA rather than the ICA. At this point the ICA clamp is removed. Any bleeding from the suture line is addressed at this time. However, a final important technical point in this step again relates to the thrombogenicity of stagnant blood in contact with the patch material or endarterectomized surface of the carotid bifurcation. One should avoid reclamping unless absolutely necessary to control bleeding at this stage and avoid the risk for formation of thrombus or a fibrin-platelet aggregate on the patch or endarterectomized vessel. This phenomenon should not be underestimated; in 2002, AbuRahma and colleagues noted a 5% carotid thrombosis rate with Dacron patches in a prospective trial with a resultant 7% perioperative stroke rate.9 As a result of that trial, the makers of the Dacron patch re-engineered the patch to make it less thrombogenic.

Eversion Endarterectomy Eversion endarterectomy is an excellent alternative technique that is practiced successfully in many centers throughout the world. Two different versions of eversion endarterectomy are performed. DeBakey originally described eversion endarterectomy with partial transection of the anterior portion of the carotid bifurcation.10 Etheredge improved on DeBakey’s technique with complete transection of the bifurcation,11 which allowed the origins of both the ICA and ECA to be everted for a longer distance. The endarterectomy is performed by mobilizing the entire circumference of the carotid adventitia off the plaque (described as a “circumcision” by Etheredge) and then everting the adventitia and mobilizing it upward while gentle caudad traction is applied to the plaque. This maneuver is performed distally into the orifices of the ICA and ECA and then proximally into the CCA. Once

e1435

e1436   SECTION 17  ■  VASCULAR - ARTERIAL DISEASE the endarterectomy is complete, the divided bifurcation is reunited with a simple end-to-end anastomosis. Advantages of this technique are that the anastomosis can be performed rapidly and it is not prone to restenosis, and therefore patching is not required. Disadvantages of this technique are that more extensive dissection is sometimes necessary to mobilize the vessels during the eversion, the procedure does not lend itself readily to shunting (although shunting is not precluded by this technique), and it can be difficult to visualize the endpoint in the ICA after the plaque has been removed; the artery tends to retract as soon as the plaque pulls away from the adventitia, and it can be difficult to expose and reinspect this area of the artery again. Therefore, in our opinion, a completion study should be performed with this technique. Kieney and coworkers introduced a modification of eversion endarterectomy in 1985 in which the origin of the ICA is excised obliquely off the carotid bifurcation, the ICA is inverted on its own, and endarterectomy of the CCA and ECA is performed through an arteriotomy in the side of the carotid bifurcation.12 With this technique it is easier to manipulate the ICA by itself, so eversion of the ICA is less cumbersome. The ICA is reanastomosed to the carotid bifurcation primarily. This technique allows rapid plaque extraction, the anastomosis is not prone to restenosis, and no prosthetic material is required. This technique is particularly effective for dealing with redundant, coiled, or kinked ICAs. The ICA can be pulled down and straightened and the redundant portion excised. The remaining portion of the ICA is spatulated and reattached to the arteriotomy on the carotid bifurcation. Although this procedure typically is not performed with a shunt, the technique does not preclude shunt use. Less dissection is required than with transection of the bifurcation. However, exposure for thorough endarterectomy of the CCA and ECA may be suboptimal with this approach.

Comparison of Conventional and Eversion Carotid Endarterectomy Numerous studies have compared standard CEA plus patching with eversion CEA. Perhaps the most important is the EVEREST (EVERsion carotid Endarterectomy versus Standard Trial study), a randomized prospective multicenter study performed in Italy that was published in 1997.13 More than 1400 patients were randomized to eversion or standard CEA, with shunting and patching done at the discretion of the operating surgeon. There were no statistically significant differences in outcomes between the two techniques, although a slightly higher incidence of perioperative complications was noted with eversion CEA and a slightly higher incidence of restenosis with standard CEA. In a subsequent publication in 2000, however, longer term follow-up in the EVEREST trial demonstrated that patients who underwent eversion CEA had a lower incidence of restenosis than did those who underwent standard CEA (patch and primary closure), but standard CEA with patch angioplasty had the lowest incidence of neurologic complications and the lowest rate of restenosis – 1.5% – versus 2.8% for eversion CEA and 7.9% for standard CEA with primary closure.14 Other studies have shown better outcomes with eversion CEA,15,16 and yet others have shown no difference between the two techniques.17,18 Thus, there is no clear consensus that one technique is superior to the other.

COMPLICATIONS Cardiac Myocardial infarction is responsible for 25% to 50% of all perioperative deaths after CEA.19–21 Furthermore, more late deaths are due to myocardial infarction than to stroke or other causes.19,20,22 These observations reflect the systemic nature of arteriosclerosis in general and, specifically, the prevalence of coronary artery disease (CAD) in patients with significant carotid lesions. At least 40% to 50% of patients who undergo CEA have symptomatic CAD.23–28 In a prospective angiographic study, severe surgically correctable CAD was identified in 20% of patients about to undergo treatment of carotid disease.29 However, despite the prevalence of CAD in patients undergoing CEA, as noted earlier, operative mortality has declined significantly over the past 2 decades. In large measure this decreased mortality has resulted from a significant reduction in the incidence of major cardiac complications. For example, in a recent large institutional series that included 2236 CEAs, the incidence of cardiac complications was 0.5%.30 At the Johns Hopkins Hospital, in a series of 1440 patients undergoing isolated CEA

CHAPTER 76-2  ■  Carotid Artery Disease: Endarterectomy  

TABLE 76-2-1  Incidence of Cranial Nerve Dysfunction after Carotid Endarterectomy Nerve Hypoglossal Recurrent laryngeal Superior laryngeal Marginal mandibular Glossopharyngeal Spinal accessory

Reported Incidence of Dysfunction (%) 4.4–17.5 1.5–15 1.8–4.5 1.1–3.1 0.2–1.5 25%) across the carotid stenosis and that it is related to temporary loss of cerebral autoregulation as evidenced by a transient increase in mean arterial velocity of the ipsilateral MCA after CEA.61–63 A study by Ascher and colleagues confirmed this latter finding and also found that CEA performed less than 3 months after contralateral CEA was associated with a higher risk for hyperperfusion syndrome.64

Other Complications Infections Unlike most peripheral vascular reconstructive procedures, in which chronic ischemia compromises wound healing and predisposes to infection, the vascular supply to the neck is rich, even in patients with severe atherosclerotic disease. Therefore, CEA is rarely associated with infectious complications. Difficulty in primary healing is occasionally seen when this operation is performed in a previously irradiated field but is almost nonexistent in the absence of this complicating factor.33 The reported incidence of wound infection, generally cellulitis, ranges from 0.09% to 0.15%.65,66 The increasing use of synthetic patches to repair the arteriotomy after CEA raises potential concern about an increased incidence of infectious complications. However, although the true incidence of patch infection has not been established, it must be extremely rare and essentially a case report type of complication. In several studies not a single case of patch infection was documented.66–68 In another comprehensive review of the literature, 57 carotid pseudoaneurysms were identified, and 40 were associated with carotid patches. However, only four (10%) were infectious in etiology.68

Bleeding Postoperative hemorrhage is a relatively uncommon yet important complication of CEA. The reported incidence ranges from 0.7% to 3.0%.26,49,69–71 Most cases result from diffuse capillary oozing secondary to administration of heparin during the procedure and the concomitant administration of antiplatelet drugs. Though not systematically studied, at least anecdotally this degree of oozing appears to be greater in patients who are taking the antiplatelet agent clopidogrel at the time of surgery. Nevertheless, if there are indications for the administration of this agent, such as a recent coronary stent procedure or symptomatic carotid disease, we would not stop clopidogrel before surgery. In general, unless bleeding appears excessive at surgery, we do not favor reversal of the heparin effect with protamine sulfate at completion of the procedure, but this practice varies widely. If the neck incision has been closed over a suction drain, the acute onset of drainage of bright red blood postoperatively is suggestive of suture line disruption and is an indication for urgent re-exploration. Similarly, the development of a hematoma in the neck, often caused by inadequate function of the drain, may compress the trachea and is an indication for re-exploration of the wound. More gradual development of a neck hematoma usually results from inadequate ligature of facial veins or other muscle arterioles or venules.33

Recurrent Carotid Stenosis One of the more important arterial complications of CEA is the development of recurrent carotid stenosis. The incidence of this complication depends on the definition of recurrent stenosis, method of diagnosis, and duration of follow-up. It has been estimated to occur in 5% to 22% of patients in several published institutional series, although only approximately 3% of these lesions were symptomatic.72–76 In a meta-analysis of 55 reports, the overall incidence of recurrent carotid stenosis

e1441

e1442   SECTION 17  ■  VASCULAR - ARTERIAL DISEASE after CEA was 6% to 14%, which reflected an annual incidence of restenosis or occlusion of 1.5% to 4.5%.77 In another analysis of the MEDLINE database, the rate of restenosis was 10% within the first year, 3% in the second, and 2% in the third year after CEA, thus suggesting that the rate of restenosis is clearly not a linear process biologically.78 Within the first 36 months after CEA, recurrent stenosis usually results from intimal hyperplasia. Evidence from serial duplex evaluation suggests that at least some of these lesions regress with time, and in part this regression may be responsible for some variability in the reported rates of recurrent stenosis. An occasional “recurrent” stenosis in fact represents residual arteriosclerotic disease after the endarterectomy. Lesions that develop more than a few years after CEA generally result from progressive or new arteriosclerotic disease. Recurrent stenoses develop more frequently in women, in patients who continue to smoke, and in hypercholesterolemic, diabetic, and hypertensive individuals.74 It has also been suggested that intraoperative injury secondary to arterial clamping, insertion of an intraluminal shunt, or placement of tacking sutures within the vessel may also predispose to early myointimal hyperplastic lesions.73 As noted earlier, there is compelling evidence that closure of the arteriotomy with a patch will reduce the incidence of recurrent stenosis, although the optimal patch material remains to be identified. Long-term follow-up of 950 CEAs performed at the Massachusetts General Hospital demonstrated that reintervention was required in 3.8% of patients with a cumulative follow-up of 4.5 years. There was no difference in restenosis rates between patients who underwent patch closure and eversion endarterectomy.18 In another series that included 1150 patients, 98.8% were free of occlusion or re­ stenosis greater than 70% at a mean follow-up of 74 months after CEA.79 In the EVEREST trial, the only large randomized trial to compare eversion and standard endarterectomy, life-table estimates of the cumulative risk for restenosis at 4 years after eversion, patch closure, and primary closure were 3.5%, 1.7%, and 12.6%, respectively.14 The difference in restenosis rates between eversion and patch closure was not statistically significant.

Repeat Carotid Endarterectomy Repeat carotid endarterectomy presents additional challenges with dissection and reconstruction that can increase the risk over that associated with a primary procedure, but with careful planning and technique, excellent results can be achieved in this situation as well. Early recurrent stenosis usually develops within 2 years of CEA and typically results from intimal hyperplasia, an inflammatory response that produces a firm, rubbery plaque rich in fibroblasts and smooth muscle cells surrounded by dense accumulation of collagen and acid mucopolysaccharide, and it typically develops within the endarterectomy bed. It is much less prone to ulceration or thromboembolic complications. Regression of these lesions is not uncommon and is observed in as many as a third or more of cases.80,81 Later restenoses typically have features of atheromatous plaque and are more widely distributed along the carotid artery. There are no prospective randomized trials to support repeat CEA, but most available evidence supports treatment of symptomatic and very high grade asymptomatic recurrent stenosis.73,82 Scarring typically makes the dissection more technically difficult such that a higher incidence of cranial nerve injury and hematoma has been reported.83 In addition, the more extensive disease within the carotid artery may necessitate carotid artery replacement with an interposition graft. This is technically more difficult, may preclude shunting, and could be associated with longer periods of cerebral ischemia, possibly leading to higher perioperative stroke rates.83 However, repeat endarterectomy is often possible, even after eversion endarterectomy.84

Carotid Artery Stenting versus Repeat Carotid Endarterectomy The increased technical difficulty and higher complication rate anticipated with repeat CEA has led some to advocate CAS as an alternative to CEA for recurrent stenosis. However, AbuRahma and coauthors reported lower perioperative stroke rates with repeat CEA than with CAS for recurrent stenosis, as well as significantly lower rates of recurrent stenosis after CEA; at 48 months, 100% of patients were free of restenosis after CEA versus 52% after CAS.85 Similarly, Bowser and associates reported lower stroke and death rates in 27 repeat CEAs than in 52 CAS procedures performed for recurrent stenosis (3.7% versus 5.7%), although this difference was not statistically significant.86 In fact, during the last decade several centers have reported excellent results with repeat CEA. Stoner and colleagues recently reported their experience with 153 repeat CEAs over a 14-year period: no

CHAPTER 76-2  ■  Carotid Artery Disease: Endarterectomy  

mortality or myocardial infarction, a 1.9% incidence of stroke, a 1.3% incidence of cranial nerve injury, and a 3.2% incidence of hematoma.87 Two thirds of patients who underwent reoperation had their carotid artery closed primarily during their first CEA. Similarly excellent results have been reported by Archie88 and by Cho and coworkers89 in smaller series of patients. In summary, repeat CEA is a viable therapeutic option that should be considered for symptomatic and asymptomatic high-grade recurrent stenosis. Although one should anticipate increased technical difficulty with dissection and reconstruction, including possibly the need for an interposition graft, good results can be expected.

References 1. Guay J: Regional or general anesthesia for carotid endarterectomy? Evidence from published prospective and retrospective studies. J Cardiothorac Vasc Anesth 2007;21:127–132. 2. Rerkasem K, Rothwell PM: Local versus general anaesthesia for carotid endarterectomy. Cochrane Database Syst Rev 2008;4:CD000126. 3. Tangkanakul C, Counsell C, Warlow C: Local versus general anaesthesia for carotid endarterectomy. Cochrane Database Syst Rev 2000;2:CD000126. 4. General anaesthesia versus local anaesthesia for carotid surgery (GALA): a multicentre, randomised controlled trial. Lancet 2008;372:2132–2142. 5. Collins GJ, Lough F, Rich NM, et al.: An expedient shunt for the small internal carotid artery. Am J Surg 1980;139:880–882. 6. Wilkinson JM, Rochester JR, Sivaguru A, et al.: Middle cerebral artery blood velocity, embolisation, and neurological outcome during carotid endarterectomy: a prospective comparison of the Javid and the Pruitt-Inahara shunts. Eur J Vasc Endovasc Surg 1997;14:399–402. 7. Ascher E, Hingorani A, Markevich N, et al.: Carotid surgery without external carotid endarterectomy: a 6-year clinical experience with 1027 cases. Eur J Vasc Endovasc Surg 2003;25:458–461. 8. Moore WS, Kempczinski RF, Nelson JJ, Toole JF: Recurrent carotid stenosis: results of the Asymptomatic Carotid Atherosclerosis Study. Stroke 1998;29:2018–2025. 9. AbuRahma AF, Hannay RS, Khan JH, et al.: Prospective randomized study of carotid endarterectomy with polytetrafluoro­ ethylene versus collagen-impregnated Dacron (Hemashield) patching: perioperative (30-day) results. J Vasc Surg 2002;35:125–130. 10. DeBakey ME, Crawford ES, Cooley DA, Morris GC: Surgical considerations of occlusive disease of innominate, carotid, subclavian, and vertebral arteries. Ann Surg 1959;149:690–710. 11. Etheredge SN: A simple technic for carotid endarterectomy. Am J Surg 1970;120:275–278. 12. Kieny R, Hirsch D, Seiller C, et al.: Does carotid eversion endarterectomy and reimplantation reduce the risk of restenosis? Ann Vasc Surg 1993;7:407–413. 13. Cao P, Giordano G, De Rango P, et al.: Eversion versus conventional carotid endarterectomy: a prospective study. J Vasc Surg 1997;14:96–104. 14. Cao P, Giordano G, De Rango P, et al.: Eversion versus conventional carotid endarterectomy: late results of a prospective multicenter randomized trial. J Vasc Surg 2000;31:19–30. 15. Entz L, Jaranyi Z, Nemes A: Comparison of perioperative results obtained with carotid eversion endarterectomy and with conventional patch plasty. Cardiovasc Surg 1997;5:16–20. 16. Ballotta E, Renon L, Da Giau G, et al.: A prospective randomized study on bilateral carotid endarterectomy: patching versus eversion. Ann Surg 2000;232:119–125. 17. Littooy FN, Gagovic V, Sandu C, et al.: Comparison of standard carotid endarterectomy with Dacron patch angioplasty versus eversion carotid endarterectomy during a 4-year period. Am Surg 2004;70:181–185. 18. Crawford RS, Chung TK, Hodgman T, et al.: Restenosis after eversion vs patch closure carotid endarterectomy. J Vasc Surg 2007;46:41–48. 19. Kazmers A, Cerqueira MD, Zierler RE: The role of preoperative radionuclide left ventricular ejection fraction for risk assessment in carotid surgery. Arch Surg 1988;123:416–419. 20. Hertzer NR, Lees CD: Fatal myocardial infarction following carotid endarterectomy: three hundred thirty-five patients followed 6–11 years after operation. Ann Surg 1981;194:212–218. 21. O’Donnell TF Jr, Callow AD, Willet C, et al.: The impact of coronary artery disease on carotid endarterectomy. Ann Surg 1983;198:705–712. 22. Hertzer NR, Arison R: Cumulative stroke and survival ten years after carotid endarterectomy. J Vasc Surg 1985;2:661–668. 23. Collaborative overview of randomized trials of antiplatelet therapy. 1. Prevention of death, myocardial-infarction, and stroke by prolonged antiplatelet therapy in various categories of patients. BMJ 1994;308:81–106. Erratum in: BMJ 1994;308:1540. 24. Baigent C, Sudlow C, Collins R, Peto R: Collaborative meta-analysis of randomised trials of antiplatelet therapy for prevention of death, myocardial infarction, and stroke in high risk patients. BMJ 2002;324:71–86. 25. Lindblad B, Persson NH, Takolander R, Bergqvist D: Does low-dose acetylsalicylic acid prevent stroke after carotid surgery? A double-blind, placebo-controlled randomized trial. Stroke 1993;24:1125–1128. 26. Nunn DB: Carotid endarterectomy: an analysis of 234 operative cases. Ann Surg 1975;182:733–738. 27. Deweese JA, Rob CG, Satran R, et al.: Results of carotid endarterectomies for transient ischemic attacks – five years later. Ann Surg 1973;178:258–264.

e1443

e1444   SECTION 17  ■  VASCULAR - ARTERIAL DISEASE 28. Hertzer NR, Loop FD, Taylor PC, Beven EG: Staged and combined surgical approach to simultaneous carotid and coronary vascular disease. Surgery 1978;84:803–811. 29. Hertzer NR, Beven EG, Young JR, et al.: Coronary artery disease in peripheral vascular patients. A classification of 1000 coronary angiograms and results of surgical management. Ann Surg 1984;199:223–233. 30. Matsen SL, Chang DC, Perler BA, et al.: Trends in the in-hospital stroke rate following carotid endarterectomy in California and Maryland. J Vasc Surg 2006;44:488–495. 31. Mcgirt MJ, Perler BA, Brooke BS, et al.: 3-Hydroxy-3-methylglutaryl coenzyme A reductase inhibitors reduce the risk of perioperative stroke and mortality after carotid endarterectomy. J Vasc Surg 2005;42:829–835. 32. Deweese JA, Rob CG, Satran R, et al.: Results of carotid endarterectomies for transient ischemic attacks – five years later. Ann Surg 1973;178:258–264. 33. Gelabert HA, Moore WS: Carotid endarterectomy: current status. Curr Probl Surg 1991;28:181–262. 34. Forssell C, Takolander R, Bergqvist D, et al.: Cranial nerve injuries associated with carotid endarterectomy. A prospective study. Acta Chir Scand 1985;151:595–598. 35. Schauber MD, Fontenelle LJ, Solomon JW, Hanson TL: Cranial/cervical nerve dysfunction after carotid endarterectomy. J Vasc Surg 1997;25:481–487. 36. Evans WE, Mendelowitz DS, Liapis C, et al.: Motor speech deficit following carotid endarterectomy. Ann Surg 1982;196:461–464. 37. Cunningham EJ, Bond R, Mayberg MR, et al.: Risk of persistent cranial nerve injury after carotid endarterectomy. J Neurosurg 2004;101:445–448. 38. Rosenbloom M, Friedman SG, Lamparello PJ, et al.: Glossopharyngeal nerve injury complicating carotid endarterectomy. J Vasc Surg 1987;5:469–471. 39. Tucker JA, Gee W, Nicholas GG, et al.: Accessory nerve injury during carotid endarterectomy. J Vasc Surg 1987;5:440–444. 40. Bageant TE, Tondini D, Lysons D: Bilateral hypoglossal-nerve palsy following a second carotid endarterectomy. Anesthesiology 1975;43:595–596. 41. Hertzer NR: Postoperative management and complications following extracranial carotid reconstruction. In Rutherford RB, editor: Vascular surgery, ed 2, Philadelphia, PA, 1984, WB Saunders, 1300. 42. Verta MJ Jr, Applebaum EL, McClusky DA, et al.: Cranial nerve injuring during carotid endarterectomy. Ann Surg 1977;185:192–195. 43. Matsumoto GH, Cossman D, Callow AD: Hazards and safeguards during carotid endarterectomy – technical considerations. Am J Surg 1977;133:458–462. 44. Dehn TC, Taylor GW: Cranial and cervical nerve damage associated with carotid endarterectomy. Br J Surg 1983;70:365–368. 45. Pine R, Avellone JC, Hoffman M, et al.: Control of postcarotid endarterectomy hypotension with baroreceptor blockade. Am J Surg 1984;147:763–765. 46. Lehv MS, Salzman EW, Silen W: Hypertension complicating carotid endarterectomy. Stroke 1970;1:307–313. 47. Angell-James JE, Lumley JS: The effects of carotid endarterectomy on the mechanical properties of the carotid sinus and carotid sinus nerve activity in atherosclerotic patients. Br J Surg 1974;61:805–810. 48. Bove EL, Fry WJ, Gross WS, Stanley JC: Hypotension and hypertension as consequences of baroreceptor dysfunction following carotid endarterectomy. Surgery 1979;85:633–637. 49. Ranson JH, Imparato AM, Clauss RH, et al.: Factors in the mortality and morbidity associated with surgical treatment of cerebrovascular insufficiency. Circulation. 1969;39 (5 Suppl 1):I269-I274. 50. Thompson JE: Complications of carotid endarterectomy and their prevention. World J Surg 1979;3:155–165. 51. Towne JB, Bernhard VM: The relationship of postoperative hypertension to complications following carotid endarterectomy. Surgery 1980;88:575–580. 52. Hertzer NR, Loop FD, Beven EG, et al.: Surgical staging for simultaneous coronary and carotid disease: a study including prospective randomization. J Vasc Surg 1989;9:455–463. 53. Takach TJ, Reul GJ, Cooley DA, et al.: Is an integrated approach warranted for concomitant carotid and coronary artery disease? Ann Thorac Surg 1997;64:16–22. 54. Thompson JE: Don’t throw out the baby with the bath water – a perspective on carotid endarterectomy. J Vasc Surg 1986;4:543–545. 55. Ascher E, Markevich N, Schutzer RW, et al.: Cerebral hyperperfusion syndrome after carotid endarterectomy: predictive factors and hemodynamic changes. J Vasc Surg 2003;37:769–777. 56. Karapanayiotides T, Meuli R, Devuyst G, et al.: Postcarotid endarterectomy hyperperfusion or reperfusion syndrome. Stroke 2005;36:21–26. 57. Abou-Chebl A, Yadav JS, Reginelli JP, et al.: Intracranial hemorrhage and hyperperfusion syndrome following carotid artery stenting – risk factors, prevention, and treatment. J Am Coll Cardiol 2004;43:1596–1601. 58. Kaku Y, Yoshimura S, Kokuzawa J: Factors predictive of cerebral hyperperfusion after carotid angioplasty and stent placement. AJNR Am J Neuroradiol 2004;25:1403–1408. 59. Perler BA, Williams GM: Post–carotid-endarterectomy intracerebral hemorrhage: a continuing challenge – Case reports. Vasc Surg 1996;30:71–75. 60. Hafner DH, Smith RB, King OW, et al.: Massive intracerebral hemorrhage following carotid endarterectomy. Arch Surg 1987;122:305–307. 61. Nielsen TG, Sillesen H, Schroeder TV: Seizures following carotid endarterectomy in patients with severely compromised cerebral circulation. Eur J Vasc Endovasc Surg 1995;9:53–57. 62. Jorgensen LG, Schroeder TV: Defective cerebrovascular autoregulation after carotid endarterectomy. Eur J Vasc Surg 1993;7:370–379.

CHAPTER 76-2  ■  Carotid Artery Disease: Endarterectomy   63. Schroeder T, Sillesen H, Sorensen O, Engell HC: Cerebral hyperperfusion following carotid endarterectomy. J Neurosurg 1987;66:824–829. 64. Ascher E, Markevich N, Schutzer RW, et al.: Cerebral hyperperfusion syndrome after carotid endarterectomy: predictive factors and hemodynamic changes. J Vasc Surg 2003;37:769–777. 65. Nouraei SA, Al Rawi PG, Sigaudo-Roussel D, et al.: Carotid endarterectomy impairs blood pressure homeostasis by reducing the physiologic baroreflex reserve. J Vasc Surg 2005;41:631–637. 66. Myers SI, Valentine RJ, Chervu A, et al.: Saphenous-vein patch versus primary closure for carotid endarterectomy – longterm assessment of a randomized prospective-study. J Vasc Surg 1994;19:15–22. 67. Perler BA, Ursin F, Shanks U, Williams GM: Carotid Dacron patch angioplasty: immediate and long-term results of a prospective series. Cardiovasc Surg 1995;3:631–636. 68. Rosenthal D, Archie JP Jr, Garcia-Rinaldi R, et al.: Carotid patch angioplasty: immediate and long-term results. J Vasc Surg 1990;12:326–333. 69. Rainer WG, Guillen J, Bloomquist CD, McCrory CB: Carotid artery surgery. Morbidity and mortality in 257 operations. Am J Surg. 1968;116:78–81. 70. Sundt TM, Sandok BA, Whisnant JP: Carotid endarterectomy. Complications and preoperative assessment of risk. Mayo Clin Proc 1975;50:301–306. 71. Kunkel JM, Gomez ER, Spebar MJ, et al.: Wound hematomas after carotid endarterectomy. Am J Surg 1984;148:844–847. 72. Lamuraglia GM, Stoner MC, Brewster DC, et al.: Determinants of carotid endarterectomy anatomic durability: effects of serum lipids and lipid-lowering drugs. J Vasc Surg 2005;41:762–768. 73. Sadideen H, Taylor PR, Padayachee TS: Restenosis after carotid endarterectomy. Int J Clin Pract 2006;60:1625–1630. 74. Callow AD: Recurrent stenosis after carotid endarterectomy. Arch Surg 1982;17:1082–1085. 75. Salvian A, Baker JD, Machleder HI, et al.: Cause and noninvasive detection of restenosis after carotid endarterectomy. Am J Surg 1983;146:29–34. 76. Zierler RE, Bandyk DF, Thiele BL, Strandness DE Jr: Carotid artery stenosis following endarterectomy. Arch Surg 1982;117:1408–1415. 77. Lattimer CR, Burnand KG: Recurrent carotid stenosis after carotid endarterectomy. Br J Surg 1997;84:1206–1219. 78. Frericks H, Kievit J, van Baalen JM, van Bockel JH: Carotid recurrent stenosis and risk of ipsilateral stroke: a systematic review of the literature. Stroke 1998;29:244–250. 79. Ballotta E, Da Giau G, Piccoli A, Baracchini C: Durability of carotid endarterectomy for treatment of symptomatic and asymptomatic stenoses. J Vasc Surg 2004;40:270–278. 80. Healy DA, Zierler E, Nicholls SC, et al.: Long-term follow-up and clinical outcome of carotid restenosis. J Vasc Surg 1989;10:662–669. 81. Zierler RE, Bandyk DF, Thiele BL, Strandness DE: Carotid-artery stenosis following endarterectomy. Arch Surg 1982; 117:1408–1415. 82. Reina-Gutierrez T, Serrano-Hernando FJ, Sanchez-Hervas L, et al.: Recurrent carotid artery stenosis following endarterectomy: natural history and risk factors. Eur J Vasc Endovasc Surg 2005;29:334–341. 83. AbuRahma AF, Jennings TG, Wulu JT, et al.: Redo carotid endarterectomy versus primary carotid endarterectomy. Stroke 2001;32:2787–2792. 84. Mehta M, Roddy SP, Darling RC, et al.: Safety and efficacy of eversion carotid endarterectomy for the treatment of recurrent stenosis: 20-year experience. Ann Vasc Surg 2005;19:492–498. 85. AbuRahma AF, Bates MC, Wulu JT, Stone PA: Early postsurgical carotid restenosis: redo surgery versus angioplasty/ stenting. J Endovasc Ther 2002;9:566–572. 86. Bowser AN, Bandyk DF, Evans A, et al.: Outcome of carotid stent-assisted angioplasty versus open surgical repair of recurrent carotid stenosis. J Vasc Surg 2003;38:432–438. 87. Stoner MC, Cambria RP, Brewster DC, et al.: Safety and efficacy of reoperative carotid endarterectomy: a 14-year experience. J Vasc Surg 2005;41:942–949. 88. Archie JP: Reoperations for carotid artery stenosis: role of primary and secondary reconstructions. J Vasc Surg 2001;33: 495–503. 89. Cho JS, Pandurangi K, Conrad MF, et al.: Safety and durability of redo carotid operation: an 11-year experience. J Vasc Surg 2004;39:155–161.

Further Reading Executive Committee of the Asymptomatic Carotid Atherosclerosis Study: Endarterectomy for asymptomatic carotid artery stenosis. JAMA 1995; 273:1421–1428. North American Symptomatic Carotid Endarterectomy Trial Collaborators: Beneficial effect of carotid endarterectomy in symptomatic patients with high-grade carotid stenosis. N Engl J Med 1991; 325:445–453.

e1445

76-3 

CEREBROVASCULAR DISEASE: CAROTID ENDARTERECTOMY – STANDARD APPROACH From Cronenwett JL, Johnston KW: Rutherford’s Vascular Surgery, 7th edition (Saunders 2010)

The video for this procedure can be accessed here

e1446

SELF ASSESSMENT Chad E. Jacobs  /  Walter J. McCarthy  /  Muhammad Asad Khan From Velasco J: Rush University Medical Center Review of Surgery, 5th edition (Saunders 2011)

76-4 

1. In which patients is carotid endarterectomy not indicated? A. Acute stroke, 70% carotid stenosis, rapid recovery, and negative findings on head CT B. Forty-five percent carotid stenosis with continued or worsening TIAs while the patient was treated with aspirin and clopidogrel (Plavix) C. Transient neurologic deficit (110  mm Hg) • Cardiopulmonary resuscitation within 10 days • Puncture of noncompresssible vessel • Intracranial tumor • Pregnancy • Diabetic hemorrhagic retinopathy • Recent eye surgery • Hepatic failure • Bacterial endocarditis

to minimize the risk of femoral hematoma; some authors recommend routine ultrasound-guided femoral artery puncture. Once access is obtained, a short 6 Fr sheath is introduced. After the initial abdominal aortogram, the target vessel or location is imaged. It is important to perform a complete evaluation of the runoff vessels before attempting to cross the occlusion. This assists in the recognition of embolic events and also provides a target vessel for outflow in the event of failed thrombolytic therapy. I routinely use a vertebral catheter (Cook Inc., Bloomington, IN) with an angled glide wire and liberal roadmapping techniques to cross the occlusion. Multiple oblique angiographic views to visualize the proximal stump of a thrombosed bypass graft may be required for access. Occasionally, if a graft cannot be crossed, ultrasound-guided direct puncture may be helpful. This is also used in patients with thrombosed axillofemoral grafts, with two separate counterpunctures and sheaths directed toward each other. Patients with prior aortobifemoral bypass grafts present a unique challenge in that it is often very difficult to get up and over these grafts. The thrombosed limb is usually best approached by an ultrasound-guided puncture of the ipsilateral proximal superficial femoral artery. Occasionally, I approach the thrombosed limb of an aortobifemoral graft by placing an up-and-over Simmons I catheter and then infusing the lytic agent via a Katzen wire placed through the Simmons catheter into the occlusion. Once the occlusion has been crossed, a distal angiogram is performed to confirm the catheter’s location in the true lumen distal to the occluded graft or native vessel. If using urokinase (ImaRx, ImaRx Therapeutics Inc., Tucson, AZ), an initial bolus dose of 240,000 units is given and then a drip at 60,000 units/hr. If using recombinant tissue plasminogen activator (rt-PA; Alteplase, Genentech, South San Francisco, CA), a 1-mg bolus and then a drip at 0.5 mg/hr are used. Heparin is administered through the proximal 6 Fr sheath at 500 units/hr to prevent perisheath thrombosis. I do not routinely place contralateral sheaths during the infusion. The most common infusion catheter is the Unifuse (Arrow International, Inc., Reading, PA), with the choice of catheter length depending on the length of the occlusion. When patients return for recheck angiograms, I routinely perform the angiogram via the infusion catheter.

Results Thrombolysis with agents such as urokinase, rt-PA, streptokinase (Streptase, Astra Pharmaceutical, Eatontown, NJ), and reteplase (Retevase, Centocor, Malvern, PA) has been investigated in uncontrolled trials as a therapeutic alternative to operation for acute peripheral arterial occlusion. In the 1990s, three multicenter randomized trials were published comparing thrombolysis with operation for arterial occlusion (Table 77-2-2).7–9 The first trial, the Rochester study, randomly assigned 114 patients with acute limb-threatening ischemia to thrombolysis with urokinase (57 patients) or to immediate operation (57 patients).7 At 1 year, the amputation-free survival rates were 75% and 52%, respectively, a statistically significant difference. A closer analysis revealed this finding to be the result of a higher mortality rate in the operative group caused by perioperative cardiopulmonary complications. It appeared that taking patients with severe limb ischemia directly to operation without the opportunity for preparation resulted in a high frequency of complications that culminated in death.

CHAPTER 77-2  ■  Acute Ischemia: Treatment  

TABLE 77-2-2  Outcome of Patients Treated with Initial Thrombolytic Therapy or Primary Operation for Acute Limb Ischemia Number of Patients

Series 7

Rochester STILE8 TOPAS-II9

114 393 544

THROMBOLYTIC THERAPY

PRIMARY OPERATION

Period (months)

Amputation (%)

Death (%)

Amputation (%)

12 6 12

18 12 15

16 6.5 20

18 11 13.1

Death (%) 42 8.5 17

STILE, Surgery versus Thrombolysis for Ischemia of the Lower Extremity; TOPAS, Thrombolysis or Peripheral Arterial Surgery.

The second large multicenter evaluation was the Surgery versus Thrombolysis for Ischemia of the Lower Extremity (STILE) trial.8 In this study, 393 patients were randomly assigned to surgery or to thrombolysis with either rt-PA or urokinase. Clinical outcomes for the rt-PA and urokinase groups were similar, so the data were combined for an overall comparison of thrombolysis and surgery. Post hoc stratification of patients into two subgroups on the basis of duration of symptoms before enrollment (greater or less than 14 days) showed that among patients with symptoms of longer duration, the surgical group had lower amputation rates than the thrombolysis group at 6 months (3% versus 12%). In contrast, among patients with symptoms of shorter duration, patients assigned to thrombolysis had lower amputation rates than did surgical patients (11% versus 30%). The third multicenter trial to evaluate thrombolytic therapy was the Thrombolysis or Peripheral Arterial Surgery (TOPAS) trial.9 Recombinant urokinase (r-UK) (4000 IU/min for 4 hours, followed by 2000 IU/min) was compared to primary operation in 544 patients with lower extremity native artery or bypass graft occlusions of 14 days’ duration or less. There was no significant difference in amputationfree survival rates or mortality rates at the time of discharge from the hospital. Likewise, the amputationfree survival rates 6 months after randomization were not significantly different: 71.8% in the r-UK group and 74.8% in the operative group. At the end of 6 months, 31.5% of the patients in the r-UK group had avoided amputation or death without the need for anything more than a percutaneous procedure. By contrast, the vast majority of the patients randomized to primary operation underwent open surgery (94.2%), a rate that was not unexpected owing to the design of the trial. The median length of hospitalization was 10 days in both treatment groups. Among the patients assigned to thrombolysis, those with occlusions in bypass grafts had better clinical outcomes, better rates of clot dissolution, and lower rates of major hemorrhagic complications than did those with native artery occlusions.

Complications Hemorrhagic complications are the primary cause of morbidity following CDT. In the TOPAS trial, major hemorrhagic complications occurred in 32 patients (12.5%) in the r-UK group, compared with 14 patients (5.5%) in the surgery group.9 Patient age, duration of infusion, and activated partial thromboplastin time at baseline were unrelated to the risk of bleeding. Intracranial hemorrhage occurred in four patients in the r-UK group (1.6%), one of whom died. There were no instances of intracranial hemorrhage in the surgery group. The risk of bleeding was significantly greater when therapeutic heparin was used. In 102 patients who received therapeutic heparin, bleeding occurred in 19 patients (19%). By contrast, in the 150 patients in whom therapeutic heparin was not used, bleeding occurred in only 13 patients (9%). Although not widely reported, distal embolization during CDT is not uncommon. Symptoms may worsen, or patients may lose a distal Doppler signal during the initial phase of lytic infusion. This is often easily managed by a transient (2 to 3 hours) increase in the dose of thrombolytic agent. More significant distal embolization that does not resolve in a few hours requires immediate re-imaging, with the occasional need to reposition the catheter to a more distal location.

SURGICAL REVASCULARIZATION Balloon catheter thrombectomy, first introduced by Fogarty and coworkers,10 became the cornerstone of therapy in the 1960s and 1970s.11 Interestingly, this marked the beginning of catheter-based

e1459

e1460   SECTION 17  ■  VASCULAR - ARTERIAL DISEASE endovascular options that introduced the concept of remote rather than direct open surgical intervention for the management of occlusive vascular disease.

Techniques Techniques for salvage of an ischemic limb include (1) balloon catheter thrombectomy or embolectomy (Figure 77-2-1), (2) bypass procedures to direct blood flow beyond the occlusion, (3) endarterectomy with or without patch angioplasty, and (4) intraoperative isolated limb thrombolysis.

Balloon Catheter Thrombectomy or Embolectomy These techniques are commonly used when dealing with an embolic event or graft thrombosis. In situ native vessel thrombosis superimposed on chronic occlusive disease is best treated surgically with a bypass graft. The technique involves exposure of the vessel (usually the common femoral artery) followed by proximal and distal control with vessel loops. Depending on the size of the vessel and associated atherosclerotic disease, a transverse or longitudinal arteriotomy is performed. Balloon embolectomy catheters are then passed proximally and distally until no visible thrombus is removed or a good pulse or backflow is established. Completion angiograms are helpful in evaluating the completeness of thrombus removal. Clinical examination alone may miss a culprit lesion that may result in early rethrombosis.

FIGURE 77-2-1  A, Acute embolic occlusion of the aortic bifurcation. B, Embolectomy, with occlusive material removed by a direct open surgical approach. C, Patch closure after embolectomy.

CHAPTER 77-2  ■  Acute Ischemia: Treatment  

Bypass Procedures Bypass procedures are more commonly performed in patients with known peripheral arterial disease or after failed open balloon thrombectomy. Proximal and distal targets can be evaluated by an on-table angiogram; however, if the clinical scenario permits, a preoperative angiogram from a contralateral femoral approach provides the best guide to open intervention. The ideal bypass conduit is an ipsilateral single-segment saphenous vein of adequate caliber (>3 mm). Otherwise, the contralateral saphenous vein, arm veins, or lesser saphenous vein should be used if below-knee revascularization is required. For above-knee revascularization, synthetic grafts can be used. Endarterectomy Endarterectomy is uncommonly used for limbs with ALI, but when it is, it is most often performed for in situ occlusion of the common femoral artery. After the diagnostic angiogram confirms occlusion of the common femoral artery, the target vessel is usually approached by a longitudinal groin incision. The common femoral, deep femoral, and proximal superficial femoral arteries should be adequately exposed and controlled. Atherosclerotic plaque often extends into the superficial femoral and profunda femoris orifices. Distal endpoints that extend into the superficial femoral artery or profunda femoris may require tacking sutures. Some type of patch material, either autogenous or synthetic, is required in most cases to prevent narrowing of the common femoral artery. Intraoperative Isolated Limb Thrombolysis This technique is seldom used, primarily because of the risk of bleeding at the operative site. The technique and lytic dose are variable, with the most common being a bolus infusion of urokinase (250,000 units in 10 mL of normal saline) or 1 mg of rt-PA injected through a cannula into the distal vessel via the arteriotomy. Some authors close the arteriotomy and then use a needle to infuse the lytic agent into the distal target. In my opinion, this is a desperate measure in a patient with few remaining options. In general, the infusion of lytic agents proximal to the thrombus burden fails because the lytic agent bypasses the thrombus via patent collaterals. This is why wire and catheter traversal of the occluding thrombus is necessary for CDT to be effective. Measures such as a proximal tourniquet followed by cannulation of the artery and vein for lytic infusion via an extracorporeal pump have also been described, but with limited success.

Results Although improvements in open surgical technique have diminished the rate of limb loss associated with ALI, the mortality rate remains unacceptably high (Table 77-2-3).2 In fact, patient survival has not changed dramatically since the report of Blaisdell and colleagues almost 40 years ago. The discordance of limb salvage and patient survival is explained by the specific factors controlling the two events. Mortality occurs as a result of medical co-morbidities and the fragile baseline medical state of patients presenting with ALI, whereas limb loss is related to an unsuccessful revascularization procedure. As such, the rate of amputation has diminished over the decades, presumably because of improvements in surgical technique. Unfortunately, the ability to rapidly restore arterial flow to the extremity with an operative procedure represents a significant insult to medically compromised individuals – one that all too frequently culminates in the patient’s death.

TABLE 77-2-3  Amputation, Mortality, and Long-term Limb Salvage for Open Surgery for Acute Limb Ischemia RESULTS Series 4

Campbell et al. Nypaver et al.5 Pemberton et al.6 NR, not reported.

Year

Number of Patients

Amputation (%)

Mortality (%)

1998 1998 1999

474 71 107

16 7 12

22 10 25

Limb Salvage NR 62% at 1 yr 75% at 2 yr

e1461

e1462   SECTION 17  ■  VASCULAR - ARTERIAL DISEASE

SPECIAL CONSIDERATIONS Myoglobinuria Myoglobinuria is not uncommon after the treatment of ALI. It is rarely a significant problem except in patients with pre-existing renal failure, when associated with the use of greater than 150 mL of ionic contrast agent, or when combined with hemoglobinuria. Hemoglobinuria is not uncommon after the use of certain PMT devices, especially with run times greater than 5 minutes. Patients with any of these risk factors are monitored for myoglobinuria, and a urine output of greater than 100 mL/hour is maintained. Alkalization of the urine is achieved by adding sodium bicarbonate to intravenous fluids. Acute renal failure due to myoglobinuria may require temporary dialysis until the kidney function improves.

Fasciotomy Compartment syndrome is most commonly seen in patients who have undergone open surgical revascularization or in those treated with PMT devices, owing to the speed of revascularization. Fasciotomy for compartment syndrome is less frequently required in patients undergoing CDT owing to the more gradual resolution of ALI. In addition, fasciotomies in patients receiving CDT is associated with significant bleeding. Any patient with early motor changes (Rutherford class IIb or III) should have fasciotomy after open surgical revascularization or successful PMT. It has been my practice to perform anterolateral and posterior release (a medial and lateral incision). Any patient not receiving a fasciotomy should be monitored on an hourly basis for any signs of compartment syndrome. It is also important to remember that a decrease in distal pulses and footdrop are late signs, and irreversible muscle and nerve damage may already have resulted at this stage. Any disparity in calf tension or tenderness compared with the normal contralateral leg requires an urgent fasciotomy. Compartment pressures can also be measured to confirm an elevation of pressure.

References 1. Rutherford RB, Baker DJ, Ernst C, et al.: Recommended standards for reports dealing with lower extremity ischemia: revised version. J Vasc Surg 1997;26:517–538. 2. Ouriel K, Shortell CK, DeWeese JA, et al.: A comparison of thrombolytic therapy with operative revascularization in the initial treatment of acute peripheral arterial ischemia. J Vasc Surg 1994;19:1021–1030. 3. Blaisdell FW, Steele M, Allen RE: Management of acute lower extremity arterial ischemia due to embolism and thrombosis. Surgery 1978;84:822–834. 4. Campbell WB, Rider BMF, Szymanska TH: Current management of acute leg ischemia: results of audit by the Vascular Surgical Society of Great Britain and Ireland. Br J Surg 1998;85:1498–1503. 5. Nypaver TJ, Whyte BR, Endean ED, et al.: Nontraumatic lower-extremity acute arterial ischemia. Am J Surg 1998;176:147–152. 6. Pemberton M, Varty K, Nydahl S, Bell PR: The surgical management of acute limb ischemia due to native vessel occlusion. Eur J Vasc Endovasc Surg 1999;17:72–76. 7. Ouriel K, Shortell CK, DeWeese JA, et al.: A comparison of a thrombolytic therapy with operative revascularization in the initial treatment of acute peripheral arterial ischemia. J Vasc Surg 1994;19:1021–1030. 8. The STILE Trial: results of a prospective randomized trial evaluating surgery versus thrombolysis for ischemia of the lower extremity. Ann Surg 1994;220:251–266. 9. Ouriel K, Veith FJ, Sasahara AA: A comparison of recombinant urokinase with vascular surgery as initial treatment for acute arterial occlusion of the legs for the Thrombolysis or Peripheral Arterial Surgery (TOPAS) investigators. N Engl J Med 1998;338:1105–1111. 10. Fogarty TJ, Cranley JJ, Krause RJ, et al.: A method for extraction of arterial emboli and thrombi. Surg Gynecol Obstet 1963;116:241–244. 11. Kasirajan K, Ouriel K: Management of acute lower extremity ischemia; treatment strategies and outcomes. Curr Intervent Cardiol Rep 2000;2:119–129.

SELF ASSESSMENT Ferenc P. Nagy From Velasco J: Rush University Medical Center Review of Surgery, 5th edition (Saunders 2011)

77-3 

1. Sudden pain and weakness in the left leg develop in a patient with a history of coronary artery disease and atrial fibrillation. Examination reveals a cool, pale extremity with an absence of pulses below the groin and a normal contralateral leg. Which of the following is the most likely diagnosis? A. Cerebrovascular accident B. Arterial thrombosis C. Arterial embolism D. Acute thrombophlebitis E. Aortic dissection Ref.: 1, 2 COMMENTS: See Question 3.

ANSWER: C 2. For the initial evaluation of the patient described in Question 1, which of the following tests is mandatory? A. Electrocardiography B. Venography C. Arteriography D. Abdominal ultrasound studies E. CT angiography of the affected extremity Ref.: 1, 2 COMMENTS: See Question 3.

ANSWER: A 3. If the patient described in Question 1 had a history of intermittent left calf claudication and if examination showed, in addition, diminished pulses in the contralateral leg and trophic skin changes bilaterally, which of the following would be true? A. Arteriographic findings are unlikely to help plan the appropriate surgical approach. B. Venography is mandatory for ruling out phlegmasia alba dolens. C. Indications for surgical intervention are unchanged. D. The anticipated surgical procedure is unchanged. E. Irreversible muscular necrosis may occur after 24 to 48 hours. Ref.: 1, 2 COMMENTS: The classic signs of acute arterial occlusion are pain, pallor, absence of pulse, paralysis, and paresthesia (the five P’s). The common causes of acute arterial occlusion are embolism, thrombosis, and trauma. In the patient described in Question 1, the history of atrial fibrillation, coupled with the classic findings of acute arterial occlusion, make arterial embolism the most likely diagnosis. e1463

e1464   SECTION 17  ■  VASCULAR - ARTERIAL DISEASE Clinical findings that suggest arterial thrombosis rather than embolism as the cause include an absence of cardiac disease commonly associated with embolization phenomena, symptoms of underlying occlusive atherosclerotic disease, and physical findings suggestive of chronic ischemia. It can be difficult, however, to differentiate embolism from thrombosis on clinical grounds alone. Embolism can certainly occur in patients with underlying peripheral vascular disease. Prompt operative intervention is indicated, regardless of cause, when there is acute limb-threatening ischemia. It is important, however, to distinguish arterial embolism from arterial thrombosis superimposed on atherosclerotic plaque because the extent of surgery may vary considerably. Although embolism may be treated successfully by simple embolectomy and extraction of the thrombus that forms distal to the embolism, effective treatment of arterial thrombosis can be much more difficult, with arterial reconstruction sometimes being required. Arteriography may be helpful for differentiating between embolic and thrombotic occlusions. A careful history and physical examination permit a diagnosis of embolic occlusion in most cases. Arteriography is not always necessary and should not be performed if it will delay operative re-establishment of blood flow. Patients with arterial embolism should undergo electrocardiography and radiography of the chest because of the high association with intrinsic cardiac disease and its potential for myocardial infarction. Acute arterial occlusion can be differentiated from acute venous thrombosis in that venous thrombosis is usually associated with edema and preservation of peripheral pulses. Severe venous obstruction produces phlegmasia cerulea dolens. When this is associated with arterial thrombosis and spasm, phlegmasia alba dolens may occur. Untreated, this process may progress to venous gangrene. In rare instances, an aortic dissection mimics acute embolism by producing loss of peripheral pulses, but the diagnosis may be suspected because of the presence of back or chest pain and hypertension. Acute arterial occlusion that rapidly produces paralysis and paresthesia may be mistaken for a stroke. However, the physical examination should direct attention toward the compromised extremity and eliminate stroke from the differential diagnosis. Prompt diagnosis of arterial occlusion is critical because irreversible muscular necrosis necessitating amputation may occur within 4 to 6 hours.

ANSWER: C 4. For an acute arterial embolus to the lower extremity with limb-threatening ischemia, appropriate initial treatment includes which of the following? A. Intravenous 5000-unit heparin bolus followed by continuous-drip administration B. Delay in heparinization until anesthesia is administered because heparinization precludes spinal anesthesia C. Routine preoperative trial of vasodilators D. Attempt at thrombolytic therapy with drugs such as tissue plasminogen activator E. Immediate angiography before an operation Ref.: 1, 2 COMMENTS: Treatment of arterial embolism must be initiated promptly to prevent irreversible ischemic damage. Intravenous heparin should be administered to prevent the formation and propagation of thrombosis distal to the embolus and is the most important first step. Heparinization should not be delayed, particularly because most embolectomies can be performed with the use of local anesthesia. Furthermore, the degree of distal thrombosis is an important determinant of surgical success and limb salvage. Although arterial spasm accompanies acute arterial occlusion, the routine use of vasodilators is not advocated. Fibrinolytic agents have an important role in the treatment of patients with acute thrombosis superimposed on chronic ischemia. Their routine use to treat acute arterial embolism with limb-threatening ischemia is not advocated, however, because timely intervention is of utmost importance. Because patients with arterial embolism often have associated cardiac disease and may be compromised further by the metabolic effects of ischemic tissue, preoperative attention must be given to careful physiologic monitoring and to the fluid balance, electrolyte balance, and arterial blood gas status of the patient.

ANSWER: A

CHAPTER 77-3  ■  Self Assessment  

5. With regard to the operative management of lower extremity arterial embolism, which of the following statements is true? A. Embolectomy can be performed in most cases via arteriotomy distal to the site of obstruction. B. Suspected aortoiliac emboli should be removed through an abdominal approach. C. Brisk backbleeding is a reliable indicator of successful complete distal embolectomy. D. Wide fasciotomy should be avoided in heparinized patients because of the risk for hemorrhage. E. Palpable pulses or audible Doppler signals are reliable indicators of complete embolectomy. Ref.: 1, 2 COMMENTS: In most cases, embolectomy can be performed with the use of balloon catheters introduced through arteriotomies proximal to the embolic lodging site. Aortoiliac emboli can be removed successfully via bilateral femoral arteriotomies. Backbleeding does not necessarily indicate adequate removal of the embolus distally because it may originate from an arterial branch proximal to the thrombus that remains. For this reason, restoration of distal pulses or Doppler signals and intraoperative arteriography, when necessary, constitute the gold standard used to assess the completeness of thromboembolectomy. Fasciotomy is an important concomitant procedure if the limb has been subjected to ischemia for 4 to 6 hours or longer. Fasciotomy should be performed, even in heparinized patients. Compartment syndrome can develop after reperfusion of an ischemic limb, and close postoperative attention is thus required.

ANSWER: E 6. After undergoing femoral embolectomy and fasciotomy, a patient becomes oliguric, and the urine is brownish red. Immediate treatment includes which of the following? A. Cessation of intravenous administration of heparin B. Restoration of the serum potassium level C. Intravenous administration of sodium bicarbonate and mannitol D. Renal arteriography E. Intra-arterial vasodilators Ref.: 1 COMMENTS: When an extremity has been subjected to ischemia and muscular necrosis occurs, reperfusion can result in metabolic acidosis and profound hyperkalemia. Rhabdomyolysis releases myoglobulin, which precipitates in acidic urine and produces brownish red urine that is free of red blood cells. Treatment of patients in this situation requires prompt reversal of hyperkalemia to prevent cardiac arrest (intravenous insulin and glucose), administration of sodium bicarbonate to alkalinize the urine and to treat the systemic metabolic acidosis, and osmotic diuresis with mannitol to prevent renal tubular obstruction. Fasciotomy is indicated if it has not already been performed. Continuation of anticoagulation therapy is critical because the patient remains at significant risk for recurrent embolism from the underlying cardiac disease. Less than 10% of arterial emboli involve the renal vessels, and renal arteriography is not indicated in this case.

ANSWER: C

References 1. Belkin M, Owens CD, Whittemore AD, et al: Peripheral arterial occlusive disease. In Townsend CM Jr, Beauchamp RD, Evers BM, et al, editors: Sabiston textbook of surgery: the biological basis of modern surgical practice, ed 18, Philadelphia, 2008, WB Saunders. 2. Lin PH, Kougias P, Bechara C, et al: Arterial disease. In Brunicardi FC, Andersen DK, Billiar TR, et al, editors: Schwartz’s principles of surgery, ed 9, New York, 2010, McGraw-Hill.

e1465

78 

Vena Cava Filter – Insertion

GOALS/OBJECTIVES • • •

INDICATIONS ACCESS COMPLICATIONS

VENA CAVA INTERRUPTION Marc A. Passman From Cronenwett JL, Johnston KW: Rutherford’s Vascular Surgery, 7th edition (Saunders 2010)

78-1 

CLINICAL INDICATIONS Recommended indications for the use of vena cava filters are shown in Box 78-1-1. Systemic anti­ coagulation is the therapy of choice for venous thromboembolism. However, anticoagulation may be contraindicated in some patients. Without anticoagulation, the risk of PE developing in patients with venous thromboembolism is high, and it may be fatal in as many as 25% of patients. Because of this significant risk, vena cava interruption with an implantable filtering device should be considered in patients with documented venous thromboembolism and contraindications to anticoagulation. Resumption of anticoagulation as soon as possible is recommended because although vena cava filters are effective in preventing PE, they are not effective for prevention of DVT. Evidence-based guidelines from the American College of Chest Physicians (ACCP)1 recommend vena cava filter placement in patients with documented venous thromboembolism and a contraindication to anticoagulation, complication of anticoagulation, or recurrent venous thromboembolism despite therapeutic anticoagulation. Contraindications to or complications of anticoagulation can include need for major surgery, intracranial hemorrhage, pelvic or retroperitoneal hematoma, ocular injury, solid intra-abdominal organ injury, uncorrected major coagulopathy, coagulation disorder, peptic ulcer disease, and other associated medical problems. Beyond these strong evidence-based guidelines, expanded relative indications based on inconclusive evidence have included poor compliance with anticoagulation; free-floating iliocaval thrombus; renal

Box 78-1-1  EVIDENCE-BASED GUIDELINES, RELATIVE EXPANDED INDICATIONS, AND CONTRAINDICATIONS TO VENA CAVA FILTER PLACEMENT Evidence-Based Guidelines

• Documented VTE with contraindication to anticoagulation • Documented VTE with complications of anticoagulation • Recurrent PE despite therapeutic anticoagulation • Documented VTE with inability to achieve therapeutic anticoagulation Relative Expanded Indications

• Poor compliance with anticoagulation • Free-floating iliocaval thrombus • Renal cell carcinoma with renal vein extension • Venous thrombolysis/thromboembolectomy • Documented VTE and limited cardiopulmonary reserve • Documented VTE with high risk for anticoagulation complications

• Recurrent PE complicated by pulmonary hypertension • Documented VTE – cancer patient • Documented VTE – burn patient • Documented VTE – pregnancy • VTE prophylaxis – high-risk surgical patients • VTE prophylaxis – trauma patients • VTE prophylaxis – high-risk medical condition Contraindications

• Chronically occluded vena cava • Vena cava anomalies • Inability to access the vena cava • Vena cava compression • No location in the vena cava available for placement

PE, pulmonary embolism; VTE, venous thromboembolism.

e1469

e1470   SECTION 18  ■  VASCULAR - VENOUS Box 78-1-2  RELATIVE RECOMMENDATIONS FOR VENA CAVA FILTER USE AS VENOUS THROMBOEMBOLISM PROPHYLAXIS, INCLUDING HIGH-RISK PATIENT FACTORS AND/OR HIGH-RISK SITUATION COMBINED WITH AN INCREASED BLEEDING RISK Prophylaxis in High-Risk Patients

• Critically ill • Previous DVT • Family history of DVT • Morbid obesity • Malignancy • Known hypercoagulable state • Prolonged immobility Prophylaxis in Trauma

• Multiple traumatic injuries • Spinal cord injury

• Closed head injury • Complex pelvic fractures • Multiple long-bone fractures Increased Bleeding Risk

• Major operation • Intracranial hemorrhage • Solid intra-abdominal organ injury • Pelvic or retroperitoneal hematoma • Ocular injury • Medical problems (cirrhosis, end-stage renal disease, peptic ulcer disease, medication, coagulation disorder)

DVT, deep venous thrombosis.

cell carcinoma with renal vein extension; placement in conjunction with venous thrombolysis or thromboembolectomy; presence of DVT and limited cardiopulmonary reserve or chronic obstructive pulmonary disease; recurrent PE complicated by pulmonary hypertension; proven DVT in an oncology, burn, or pregnant patient; and venous prophylaxis in high-risk surgical, medical, or trauma patients. The later category is the most controversial because strong level I randomized data are still lacking. A clinical decision algorithm for vena cava filter use for prophylaxis of venous thromboembolism is shown in Box 78-1-2. Recommendations should be based on a combination of high-risk patient factors or situations with a high risk of bleeding prohibiting use of anticoagulation for prophylaxis. Despite lack of support based on safety and efficacy for some of these indications, filter use has recently expanded significantly in the United States, especially for prophylaxis of venous thromboembolism. A retrospective study conducted through the National Hospital Discharge Survey database found that the number of filters in the United States increased almost 25-fold from an estimated 2000 filters placed in 1979 to more 49,000 placed in 1999.2 A multicenter prospective registry of 5451 patients with acute DVT showed a current preference for the use of vena cava filters, with pharmacologic options being used in 33% of patients.3

Permanent versus Optional Filter The only proven benefit of vena cava filters is prevention of PE. Unfortunately, evidence-based recommendations for vena cava filter use are derived predominantly from nonrandomized data, with substantial differences existing among studies in terms of study populations, immediate and longterm endpoints, and duration of follow-up. Because of difficulties comparing data on different filters, several guidelines outlining reporting standards for filter devices have been published.4–6 Overall reported complications for vena cava filters include PE (2% to 5%), fatal PE (0.7%), death linked to filter insertion (0.12%), venous access site thrombosis (2% to 28%), filter migration (3% to 69%), vena cava penetration (9% to 24%), vena cava obstruction (6% to 30%), venous insufficiency (5% to 59%), filter fracture (1%), and guide wire entrapment (1000 mL/min) can lead to a significant increase in cardiac output. The possibility of limb loss in the event of infectious complications and the increased risk of infection make this site less desirable.20 With the longer survival of chronic hemodialysis patients, the surgeon may be asked to evaluate a patient who requires vascular access but whose extremity access sites have all been expended. In this circumstance, a more central location, such as a bridge arteriovenous fistula placed between the axillary

CHAPTER 81-1  ■  Hemodialysis and Vascular Access  

artery on one side and the axillary vein on the other side, has been used successfully.21 The grafts are of fairly large diameter, so they are easy to cannulate; flow is reported to be excellent, and despite the location of the access site on the anterior chest wall, patients adapt promptly.22 The major drawback of central access sites is that when complications occur, they are serious and more difficult to manage.

Materials for Prosthetic Arteriovenous Bridge Grafts Both biological and prosthetic materials have been used in the creation of arteriovenous bridge fistulas for hemodialysis since this modality was introduced in 1969.23 Although saphenous vein, bovine heterografts, human umbilical vein, cryopreserved homografts, and Dacron velour grafts have all been tried during the last 4 decades, only expanded PTFE grafts have had an extended period of observation. Although one report on the use of autologous tissue-engineered vascular grafts in high-risk patients demonstrated primary patency in 78% of patients 1 month after implantation and 60% of patients 6 months after implantation, larger trials are needed to better determine the safety and efficacy of these grafts.24 Since its initial introduction as an alternative material for the creation of arteriovenous bridge fistulas in 1976,25 expanded PTFE has become the most commonly used material. Much of its popularity stems from the fact that it is easy to handle, requires no preclotting, is widely available, has a long shelf life, and has relatively high patency rates with secondary revisions. PTFE bridge fistulas are consistently reported to have 12-month secondary patency rates of more than 70%.26 In a large comparative clinical study comprising 187 graft placements, 36-month patency rates of PTFE grafts were significantly greater than those of bovine heterografts (62% vs. 24%).27 Forty-eight–month patency rates of 43% to 60% have been reported.28,29 However, multiple procedures for revision are usually required to maintain patency, with one study reporting an average of one operation for revision required every 1.1 years (range, 1 to 16 revisions per graft).30 Thrombosis of the conduit is a relatively common event in PTFE bridge fistulas, with figures ranging from 7% to 55%.28,29 Endovascular techniques such as thrombolysis, percutaneous mechanical thrombectomy, and angioplasty allow for re-establishment of graft flow and function. Surgical revision with Fogarty thrombectomy and patch angioplasty of the stenotic outflow vein is also effective.29 Infection of PTFE is not uncommon, with one report of 80 AV grafts monitored for 30 months showing an overall incidence of infection of 19%, with 67% of these infections occurring during the initial 4 months of use.31 Of the infected grafts, 73% required excision, and the remainder were treated successfully with antibiotics. The most common type of graft infection today occurs at needle puncture sites. Pseudoaneurysm at needle puncture sites develops in approximately 5% of fistulas.28 Recently, heparin-bonded ePTFE grafts have been introduced in reports of increased patency compared with conventional ePTFE grafts.32 The long-term advantages and complications of heparinbonded ePTFE grafts are not yet known.

References 1. Reilly DT, Wood RF, Bell PR: Prospective study of dialysis fistulas: problem patients and their treatment. Br J Surg 69(9):549–553, 1982. 2. Brescia MJ, Cimino JE, Appel K, et al: Chronic hemodialysis using venipuncture and a surgically created arteriovenous fistula. The New England Journal of Medicine 275(20):1089–1092, 1966. 3. Lampropoulos G, Papadoulas S, Katsimperis G, et al: Preoperative evaluation for vascular access creation. Vascular 17(2):74–82, 2009. 4. Mouquet C, Bitker MO, Bailliart O, et al: Anesthesia for creation of a forearm fistula in patients with endstage renal failure. Anesthesiology 70(6):909–914, 1989. 5. Johnson G: Local pathophysiology of an arteriovenous fistula. In Swam KG, editor: Venous surgery in the lower extremity, St. Louis, 1975, Warren H. Greene, pp 41–50. 6. Bennion RS, Williams RA. The radiocephalic fistula. Contemp Dial 3:12–16, 1982. 7. Anderson CB, Etheredge EE, Harter HR, et al: Local blood flow characteristics of arteriovenous fistulas in the forearm for dialysis. Surg Gynecol Obstet 144(4):531–533, 1977. 8. Johnson G Jr, Dart CH Jr, Peters RM, et al: The importance of venous circulation in arteriovenous fistula. Surg Gynecol Obstet 123(5):995–1000, 1966. 9. Mindich BP, Levowitz BS: Enhancement of flow through arteriovenous fistula. Arch Surg 111(2):195–196, 1976. 10. Rodriguez Moran M, Almazan Enriquez A, Ramos Boyero M, et al: Hand exercise effect in maturation and blood flow of dialysis arteriovenous fistulas ultrasound study. Angiology 35(10):641–644, 1984. 11. Clinical practice guidelines for vascular access. Am J Kidney Dis 48(Suppl 1):S248–S273, 2006. 12. Bender MH, Bruyninckx CM, Gerlag PG: The brachiocephalic elbow fistula: a useful alternative angioaccess for permanent hemodialysis. Journal of Vascular Surgery: Official Publication, the Society for Vascular Surgery [and] International Society for Cardiovascular Surgery, North American Chapter. 20(5):808–813, 1994.

e1551

e1552   SECTION 19  ■  VASCULAR - ACCESS 13. Nguyen TH, Bui TD, Gordon IL, et al: Functional patency of autogenous AV fistulas for hemodialysis. J Vasc Access 8(4):275–280, 2007. 14. Wilson SE, Stabile BE, Williams RA, et al: Current status of vascular access techniques. Surg Clin North Am 62(3):531– 551, 1982. 15. Owens ML, Stabile BE, Gahr JE, et al: Vascular grafts for hemodialysis: an evaluation of sites and materials. Dial Transplant 8:521–525, 1979. 16. Rohr MS, Browder W, Frentz GD, et al: Arteriovenous fistulas for long-term dialysis. Factors that influence fistula survival. Arch Surg 113(2):153–155, 1978. 17. Fee HJ Jr, Golding AL: Lower extremity ischemia after femoral arteriovenous bovine shunts. Ann Surg 183(1):42–45, 1976. 18. Humphries AL Jr, Nesbit RR Jr, Caruana RJ, et al: Thirty-six recommendations for vascular access operations: lessons learned from our first thousand operations. Am Surg 47(4):145–151, 1981. 19. Rizzuti RP, Hale JC, Burkart TE: Extended patency of expanded polytetrafluoroethylene grafts for vascular access using optimal configuration and revisions. Surg Gynecol Obstet 166(1):23–27, 1988. 20. Wilson SE, Hillman M, Owens ML: Hemodynamic effects of bovine femorosaphenous fistula. Dial Transplant 6:84–89, 1977. 21. Garcia-Rinaldi R, VonKoch L: The axillary artery to axillary vein bovine graft for circulatory access: surgical considerations. Am J Surg 135(2):265–268, 1978. 22. Giacchino JL, Geis WP, Buckingham JM, et al: Vascular access: long-term results, new techniques. Arch Surg 114(4):403– 409, 1979. 23. May J, Tiller D, Johnson J, et al: Saphenous-vein arteriovenous fistula in regular dialysis treatment. The New England Journal of Medicine 280(14):770, 1969. 24. McAllister TN, Maruszewski M, Garrido SA, et al: Effectiveness of haemodialysis access with an autologous tissueengineered vascular graft: a multicentre cohort study. Lancet 373(9673):1440–1446, 2009. 25. Haimov M, Burrows L, Schanzer H, et al: Experience with arterial substitutes in the construction of vascular access for hemodialysis. J Cardiovasc Surg (Torino) 21(2):149–154, 1980. 26. Baker LD Jr, Johnson JM, Goldfarb D: Expanded polytetrafluoroethylene (PTFE) subcutaneous arteriovenous conduit: an improved vascular access for chronic hemodialysis. Trans Am Soc Artif Intern Organs 22:382–387, 1976. 27. Sabanayagam P, Schwartz AB, Soricelli RR, et al: A comparative study of 402 bovine heterografts and 225 reinforced expanded PTFE grafts as AVF in the ESRD patient. Trans Am Soc Artif Intern Organs 26:88–92, 1980. 28. Munda R, First MR, Alexander JW, et al: Polytetrafluoroethylene graft survival in hemodialysis. JAMA 249(2):219–222, 1983. 29. Palder SB, Kirkman RL, Whittemore AD, et al: Vascular access for hemodialysis. Patency rates and results of revision. Ann Surg Aug 202(2):235–239, 1985. 30. Schuman ES, Gross GF, Hayes JF, et al: Long-term patency of polytetrafluoroethylene graft fistulas. Am J Surg 155(5):644– 646, 1988. 31. Bhat DJ, Tellis VA, Kohlberg WI, et al: Management of sepsis involving expanded polytetrafluoroethylene grafts for hemodialysis access. Surgery 87(4):445–450, 1980. 32. Davidson I, Hackerman C, Kapadia A, et al: Heparin bonded hemodialysis e-PTFE grafts result in 20% clot free survival benefit. The Journal of Vascular Access 10(3):153–156, 2009.

HEMODIALYSIS ACCESS: GENERAL CONSIDERATIONS Robyn A. Macsata  /  Anton N. Sidawy From Cronenwett JL, Johnston KW: Rutherford’s Vascular Surgery, 7th edition (Saunders 2010)

81-2 

PREOPERATIVE EVALUATION A thorough preoperative evaluation of the arterial and venous systems is imperative for the successful placement of permanent AV access.

History and Physical Examination A thorough patient history should document the patient’s dominant extremity; any recent history of peripheral intravenous lines; sites of indwelling or previous central lines, including pacemakers and defibrillators; all previous access procedures; any history of trauma or previous nonaccess surgery to the extremity; and all co-morbid conditions. On physical examination, the brachial, radial, and ulnar arteries should be evaluated for compressibility and equality bilaterally. An Allen test should be performed to evaluate palmar arch patency. The superficial venous system should be evaluated with and without a venous pressure tourniquet in place, examining for distensibility and interruptions. The arm should be examined for prominent venous collaterals and edema, which are signs of central venous stenosis.1

Arterial Assessment If any abnormality is noted on the clinical arterial examination, the patient should be further evaluated with segmental pressures and duplex ultrasound scanning or pulse volume recordings. For optimal outcomes, no pressure gradient should be noted between the bilateral upper extremities, the arterial diameter should be greater than or equal to 2 mm throughout the extremity, and a patent palmar arch should be present.2 Any abnormality noted on noninvasive testing should prompt the selection of an alternative site or further evaluation with an arteriogram, which allows the surgeon to identify and possibly treat an arterial inflow stenosis. In patients nearing the initiation of dialysis, the risk of contrast arteriography should be weighed against the need for the access to mature before dialysis begins. Renal protective agents should be used before the arteriogram; these include intravenous fluids, N-acetylcysteine, and sodium bicarbonate.3–6

Venous Assessment If superficial veins cannot be visualized with a venous pressure tourniquet in place, or if any abnormality is noted on the superficial venous examination, the patient should be further evaluated with superficial venous duplex ultrasound scanning. Using venous duplex imaging, superficial veins should be examined for diameter, distensibility, and continuity. The minimal acceptable diameter for use was reported to be 2 mm by Mendes and associates, who noted a successful early maturation rate of 76%.7 Other authors use criteria ranging from 2 to 3 mm. Using a minimal vein diameter of 2.5 mm, Silva and colleagues were able to achieve 63% autogenous access, with a 92% early maturation rate and an 83% 1-year patency rate.2 Using a minimal vein diameter of 3 mm, Huber and coworkers were able to achieve 90% autogenous access, with an 84% early maturation rate.8 Central venous stenosis should be suspected if there are any prominent venous collaterals or edema, a differential in extremity diameter, any history of previous central venous catheter placement, or multiple previous access sites in the extremity intended to be used. If any of these abnormalities are identified, the patient should be examined with deep venous duplex ultrasound imaging, followed e1553

e1554   SECTION 19  ■  VASCULAR - ACCESS by venography if necessary.1 Passman and associates imaged 60 upper extremities of preoperative access patients with both duplex ultrasound and venography.9 Five duplex ultrasounds (8%) were nondiagnostic owing to artifact from central venous catheters or incomplete visualization of the central venous system. Among the studies that were diagnostic, the investigators noted an 81% sensitivity and 97% specificity of duplex ultrasound imaging, with no statistical difference compared to venography. Venography should be performed in patients with either nondiagnostic or abnormal duplex ultrasound imaging for further evaluation and possible treatment. As with arteriography, the risk of contrast venography must be weighed against the need for access maturation before the start of dialysis. Before venography, patients should be treated with intravenous fluids, N-acetylcysteine, or sodium bicarbonate.3–6

ACCESS LOCATION SELECTION The goal of AV access procedures is to provide long-term dialysis access that will remain patent over time with a low risk of complications. To accomplish this goal, a few general principles are applied: 1. The AV access site is located as far distally in the extremity as possible, to preserve proximal sites for future access. 2. Given its superior patency rate and lower complication rate, autogenous AV access should always be attempted before prosthetic AV access. These autogenous access configurations include, in order of preference, direct AV anastomosis, venous transpositions, and venous translocations. 3. Owing to easier accessibility and a lower infection rate, upper extremity access sites are used first, with the nondominant arm preferred over the dominant arm.

Forearm Access Cephalic Vein For autogenous forearm access, the cephalic vein is preferred to the basilic vein because of its lateral location and the need for only minimal dissection. Possible sites of arterial inflow include the posterior branch of the radial artery, the trunk of the radial artery, the ulnar artery, and the brachial artery. The ulnar artery is usually not the first choice, owing to its distance from the cephalic vein. The access is placed as distally in the arm as possible where an adequate artery has been identified by preoperative evaluation; this preserves more proximal sites of inflow for future access. Therefore, in patients with an adequate posterior branch of the radial artery, an autogenous posterior radial branch–cephalic wrist direct access (snuffbox fistula) is performed. In patients with an inadequate posterior branch of the radial artery but an adequate radial artery, an autogenous radialcephalic wrist direct access (BresciaCimino-Appel fistula) is performed. In either case, if the cephalic vein is thought to be too deep (as might be the case in obese patients) or is not located near the radial artery in the wrist, an autogenous radial-cephalic forearm transposition is performed. If the radial artery is inadequate, the ulnar artery may provide a distal inflow site; alternatively, the entire trunk of either the radial or the ulnar artery may provide an arterial source. If the radial and ulnar arteries are inadequate but the brachial or proximal radial arteries are adequate, an autogenous brachial (or proximal radial)–cephalic forearm looped transposition is performed.

Basilic Vein When the cephalic vein is not adequate for autogenous AV access, the basilic vein is the preferred alternative. Secondary to its medial location in the forearm, a transposition is always required. Possible sites of arterial inflow include the distal radial artery, ulnar artery, proximal radial artery, and brachial artery. Use of the posterior branch of the radial artery is not possible because of its distance from the basilic vein. Similar to with the cephalic vein, AV access is placed as distally in the arm as possible where an adequate artery has been identified by preoperative evaluation to preserve more proximal sites of inflow for future access. Therefore, when the radial artery is adequate, an autogenous radialbasilic forearm transposition is performed. If the radial artery is inadequate but the ulnar artery is adequate, an autogenous ulnar–basilic forearm transposition is performed. If the distal radial and ulnar arteries are inadequate, a more proximal segment may be used (autogenous proximal radial–basilic forearm looped transposition). If the radial and ulnar arteries are inadequate throughout but the brachial artery is adequate, an autogenous brachial–basilic forearm looped transposition is performed.

CHAPTER 81-2  ■  Hemodialysis Access: General Considerations  

Alternative Veins When the cephalic and basilic forearm veins are not adequate for autogenous AV access, translocations of the femoral and saphenous veins are appropriate alternatives. Prosthetic Graft If no vein is available, a prosthetic AV forearm access is performed. Sources of arterial inflow include the distal or proximal radial and brachial arteries. Similar to with autogenous access, the AV access is placed as distally in the arm as possible. Therefore, when the distal radial artery is adequate, a prosthetic radial–antecubital forearm straight access is performed. If the radial artery is inadequate but the brachial or proximal radial artery is adequate, a prosthetic brachial (or proximal radial)–antecubital forearm looped access is performed. Patients should be told that this forearm prosthetic access is a “bridge” to autogenous access. The nephrologist should be asked to minimize the number of attempts to salvage the access by endovascular means to avoid ruining the venous outflow so that it can still be used for upper arm auto­ genous access. One of the interesting debates in this area is whether, after exhausting the forearm autogenous options, the surgeon should recommend a forearm prosthetic access before placing an upper arm autogenous access. The SVS practice guidelines recommend that the surgeon discuss these alternatives and their advantages and disadvantages with the patient to allow an informed decision.10

Upper Arm Access Cephalic Vein When use of the forearm has been exhausted, efforts to achieve access are directed to the upper arm. Similar to the forearm, the upper arm cephalic vein is preferred to the basilic vein, for the reasons stated earlier. For upper arm access, possible sites of arterial inflow include the proximal radial and brachial arteries. AV access is placed as distally in the arm as possible to lower the risk of arterial steal. Therefore, in patients with an adequate cephalic vein and an adequate proximal radial artery, an autogenous proximal radial–cephalic upper arm direct access is performed. If the proximal radial artery is inadequate and the brachial artery is adequate, an autogenous brachial-cephalic upper arm direct access is performed. If the cephalic vein is too deep or is located too far from the artery, an autogenous brachial (or proximal radial)–cephalic upper arm transposition is performed.

Basilic Vein When the cephalic vein is inadequate for autogenous AV access, the upper arm basilic vein is the preferred alternative. Because of its medial and deep location, transpositions are always required for access using the basilic vein. Similar to with upper arm cephalic vein AV access, possible sites of arterial inflow include the proximal radial and brachial arteries; the AV access is placed as distally in the arm as possible to lower the risk of arterial steal. Therefore, in patients with an adequate basilic vein and an adequate proximal radial artery, an autogenous proximal radial–basilic upper arm transposition is performed. If the proximal radial artery is inadequate and the brachial artery is adequate, an auto­ genous brachial-basilic upper arm transposition is performed. Alternative Veins When the cephalic or basilic veins are inadequate for upper arm autogenous access, brachial vein transpositions as well as femoral and saphenous vein translocations are appropriate alternatives. Prosthetic Graft If no vein is available, an upper arm prosthetic AV access is performed. Similar to with upper arm autogenous AV access, possible sites of arterial inflow include the proximal radial and brachial arteries; the AV access is placed as distally in the arm as possible to lower the risk of arterial steal. Therefore, when the proximal radial artery is adequate, a prosthetic proximal radial–axillary vein (or brachial vein) upper arm straight access is performed. If the proximal radial artery is inadequate and the brachial artery is adequate, a brachial–axillary vein (or brachial vein) upper arm straight access is performed.

References 1. http://www.kidney.org/professionals/KDOQI/guidelines.cfm. Accessed June 17, 2015. 2. Silva MB, Hobson RW, Pappas PJ, et al.: A strategy for increasing use of autogenous hemodialysis access procedures: impact of preoperative noninvasive evaluation. J Vasc Surg 1998;27:302–308.

e1555

e1556   SECTION 19  ■  VASCULAR - ACCESS 3. Kimmel M, Butscheid M, Brenner S, et al.: Improved estimation of glomerular filtration rate by serum cystatin C in preventing contrast induced nephropathy by N-acetylcysteine or zinc – preliminary results. Nephrol Dial Transplant 2008;23:1241–1245. 4. Stenstrom DA, Muldoon LL, Armijo-Medina H, et al.: N-Acetylcysteine use to prevent contrast medium-induced nephropathy: premature phase III trials. J Vasc Interv Radiol 2008;19:309–318. 5. Kelly AM, Kwamena B, Cronin P, et al.: Meta-analysis: effectiveness of drugs for preventing contrast-induced nephropathy. Ann Intern Med 2008;148:284–294. 6. Morcos SK: Prevention of contrast media-induced nephrotoxicity after angiographic procedures. J Vasc Interv Radiol 2005;16:13–23. 7. Mendes RR, Farber MA, Marston WA, et al.: Prediction of wrist arteriovenous fistula maturation with preoperative vein mapping with ultrasonography. J Vasc Surg 2002;36;460–463. 8. Huber T, Ozaki C, Flynn TC, et al.: Prospective validation of an algorithm to maximize native arteriovenous fistulae for chronic hemodialysis access. J Vasc Surg 2000;36:452–459. 9. Passman MA, Criado E, Farber MA, et al.: Efficacy of color flow duplex imaging for proximal upper extremity venous outflow obstruction in hemodialysis patients. J Vasc Surg 1998;28:869–875. 10. Sidawy AN, Spergel LM, Besarab A, et al.: Society for Vascular Surgery. The Society for Vascular Surgery: clinical practice guidelines for the surgical placement and maintenance of arteriovenous hemodialysis access. J Vasc Surg 2008;48(5 Suppl):2S–25S.

AUTOGENOUS RADIALCEPHALIC OR PROSTHETIC BRACHIAL-ANTECUBITAL FOREARM LOOP AVF IN PATIENTS WITH COMPROMISED VESSELS

81-3 

P. P. G. M. Rooijens  /  J. P. J. Burgmans  /  T. I. Yo  /  W. C. J. Hop  /  A. A. E. A. de Smet  /  M. A. van den Dorpel  /  W. M. Fritschy  /  H. G. W. de Groot  /  H. Burger  /  J. H. M Tordoir From Rooijens PPG, Burgmans JPJ, Yo TI, et al: Autogenous radial-cephalic or prosthetic brachialantecubital forearm loop AVF in patients with compromised vessels: A randomized, multicenter study of the patency of primary hemodialysis access. J Vasc Surg 2005;42:481–487

A well-functioning vascular access remains the lifeline of end-stage renal disease patients needing chronic intermittent hemodialysis. The Kidney Dialysis Outcomes Quality Initiative (K/DOQI) and European guidelines for vascular access propose the construction of an autogenous radial-cephalic direct wrist arteriovenous fistula (RCAVF) as the primary and best option.1,2 The usefulness of an RCAVF depends on an efficient dilatation and arterialization of the forearm veins used for the creation of arteriovenous anastomosis, which makes repeated successful cannulations possible. RCAVFs that mature without any early complications may function for many years. However, 10%–24% of RCAVFs thrombose directly after operation or do not function adequately due to failure of maturation.3–7 This results in delay of initiation of dialysis treatment with the need for placement of central venous catheters with their related morbidity and mortality. Usually, arteriovenous fistula (AVF) nonmaturation depends on the quality and size of the vessels used for the arteriovenous anastomosis and the ability of vessel adaptation induced by the augmented blood flow volumes. To predict successful maturation, duplex-derived criteria have shown beneficial effects of using well-sized radial arteries and cephalic veins.8–10 In cases of very small or diseased arteries and/or veins, the risk of access failure is probably higher and an alternative vascular access may be considered. However, there are only few data on the outcome of RCAVFs in patients with poor or questionable vessels, and no information on the performance of alternative access in these patients is available. An upper arm direct AVF, anastomosing the brachial artery with the cephalic or basilic vein may be a good second best option after failure of an RCAVF, but in K/DOQI guidelines, no consensus for either this option or the implantation of a prosthetic graft implant has been outlined. In addition, upper arm access has a considerably higher incidence of peripheral ischemia and cardiac failure due to high access flow. Therefore, to address this subject, we performed a randomized multicenter study comparing RCAVF vs forearm prosthetic graft (polytetrafluoroethylene [PTFE]) implantation in patients with poor (questionable) vessels.

METHODS Between January 1999 and April 2003, 383 consecutive new patients from six dialysis facilities with end-stage renal failure were screened for enrollment in the study. This study was approved by the Medical Ethics Committee of all participating hospitals. According to the defined vessel criteria from the preoperative duplex scanning, 140 patients were allocated to primary placement of an RCAVF and 61 patients to primary prosthetic graft implantation. The remaining 182 patients (97 men, 85 women; e1557

e1558   SECTION 19  ■  VASCULAR - ACCESS mean age, 59 years) were randomized to receive either an RCAVF (n = 92) or prosthetic graft implant (n = 90) (Figure 81-3-1). Patient characteristics of both groups are shown in Table 81-3-1. Preoperative assessment included a standard physical examination and blood pressure measurement on both arms according to the Riva-Rocci method with a proximal pressure cuff and auscultation of the brachial artery. All patients underwent preoperative duplex ultrasonography of the arteries and superficial veins of the upper extremity. Duplex scanning was performed according to a standard protocol by experienced vascular technicians. The angle of the emitted Doppler ultrasound wave from the probe was adjusted to 60 degrees to achieve the Doppler signal of the strongest intensity. The anteroposterior internal diameter of the vessel was measured using B-mode technique with a proximal tourniquet to engorge the veins. Vessels were diagnosed as obstructed when no Doppler signal could be obtained. From the literature, we defined certain vessel diameters as cutoff values for randomization. When the radial artery had a diameter 2 mm and the cephalic vein was 1.6 mm, an RCAVF was created. Patients with a radial artery between 1 and 2 mm and/or a cephalic vein ≤1.6 mm were randomized for the creation of either an RCAVF or prosthetic brachial-antecubital forearm loop. The number of complications and interventions were registered and primary, assisted primary, and secondary patencies were calculated by life table methods.

Surgical Procedure All procedures were performed under local/regional or general anesthesia with the use of antibiotic prophylaxis. RCAVFs were constructed by exposing the radial artery and cephalic vein through a longitudinal or transverse incision 4–5 cm proximal to the radial styloid process. After sufficient vein mobilization an end-to-side vein-to-artery anastomosis was performed with a running

FIGURE 81-3-1  Algorithm for the creation of vascular access in new patients.

TABLE 81-3-1  Patient Characteristics in Patients with RCAVF and Prosthetic AVF RCAVF No. Male Mean age (y) Diabetes Hypertension Ischemic cardiac disease Peripheral arterial obstructive disease Cerebrovascular disease

Prosthetic AVF

86 44 (51%) 57.6 27 (31%) 69 (80%) 11 (13%)

84 45 (54%) 62.5 26 (31%) 56 (67%) 17 (20%)

13 (15%)

18 (21%)

9 (10%)

12 (14%)

RCVAF, radial-cephalic direct wrist arteriovenous fistula; AVF, arteriovenous fistula.

CHAPTER 81-3  ■  Prosthetic Brachial-Antecubital Forearm Loop AVF in Patients with Compromised Vessels  

7-0 polypropylene monofilament suture (Prolene; Ethicon, Johnson & Johnson, Amersfoort, The Netherlands). The length of the arteriotomy was 10–15 mm and internal vessel diameters were measured with coronary probes. Thin-walled stretch PTFE grafts (Gore-Tex, WL Gore & Associates, Flagstaff, Ariz) with a wall thickness of 0.5 mm and an internal diameter of 6 mm, were positioned in a subcutaneous loop with the use of a tunneler device in the forearm between the brachial artery and a suitable elbow vein. Arterial and venous anastomoses were created with running 7-0 polypropylene sutures. AVF patency was confirmed perioperatively by palpation and Doppler examination. Coumarin (Sintrom [acenocoumarol]; Novartis Pharma GmbH, Nürnberg, Germany) was given after surgery to all patients in a dose that was sufficient for an adequate anticoagulation (international normalized ratio >2.5) for a period of 3 months. Postoperative evaluation was done by palpation and auscultation. Patients were regularly seen by the nephrologist, and the decision to start dialysis treatment was made based on the severity of deterioration of renal function. The first cannulation of the RCAVF was performed when the vessels had matured adequately, usually after 4–6 weeks. When cannulation was not possible due to nonmaturation, dialysis was started by means of a central vein catheter.

Follow-Up Clinical follow-up was performed during a 12-month period. Early and late complications and radiographic and surgical interventions were registered. Complications were treated according to the standard clinical practice of the hospital where the patient was being dialyzed. Patency was defined as functional patency with adequate dialysis. Clinical criteria were used for detection of AVF thrombosis and nonmaturation. Inability to cannulate the AVF or to obtain sufficient dialysis blood flow (≥250 mL/ min) within 6 weeks after fistula creation were indicators of a poorly functioning AVF. All patients with nonmaturing RCAVFs underwent angiography, visualizing the proximal arterial inflow by retrograde contrast filling initiated through a proximal occluding cuff. Venous outflow vessels were imaged by contrast injection after the release of the proximal cuff.

End Points End points were defined as AVF failure, death, successful kidney transplantation, or transfer to continuous ambulatory peritoneal dialysis (CAPD) treatment.

Statistical Analysis For statistical analysis, the statistical package of SPSS 11.0 for Windows (SPSS, Inc., Chicago, Ill) was used. Before the start of the study, a power calculation had been performed to determine the number of patients needed per group to demonstrate an improvement in 1-year primary patency rate of 18%. For a power of 80%, at α = 0.05, the resulting group size was 90 patients per study group. The incidence rate was defined as the number of complications or interventions per patient-year (py), the cumulative follow-up time of all patients and analyzed with the Mann-Whitney test. Patency rates were obtained by Kaplan-Meier life table analysis and compared with the log-rank test. Primary patency rate was defined as the percentage of AVFs that functioned well without any intervention after implantation. Assisted primary patency rate was defined as the percentage of failing but still patent AVFs undergoing elective intervention, and secondary patency was defined as the proportion of patent AVFs still in use for hemodialysis after successful intervention for thrombosis.11 Patients with a patent AVF who died, received a kidney transplant, or were withdrawn from hemodialysis alive were censored. Differences were considered statistically significant when P < 0.05.

RESULTS Of the patients randomized for an RCAVF, one patient was still waiting for an operation, one patient underwent a successful kidney transplantation while waiting for an operation, three patients died before undergoing the operation, and one patient was lost to follow-up. A total of 86 patients randomized for an RCAVF were subjected to further analysis. Six patients exhibited insufficient wrist vessels noticed during the operation and received PTFE graft AVFs. These six cases were considered primary failures. Fifty-nine percent of the RCAVFs were functional for dialysis treatment after 6 weeks, resulting in a primary failure rate of 41% due to early thrombosis (n = 8), failure to mature (n = 21), or insufficient vessels noticed during operation (n = 6). After 6 weeks, one patient developed steal (1.2%)

e1559

e1560   SECTION 19  ■  VASCULAR - ACCESS with the need for fistula ligation, one patient was lost to follow-up, one patient stopped dialysis treatment, and eight patients died in the first year after creation of the RCAVF. Of the patients randomized for a prosthetic AVF, two patients were still on the list for operation, and four patients died before undergoing the operation. A total of 84 patients randomized for prosthetic graft implant were subject for further analysis. In one patient, it was not possible to implant a prosthetic graft due to abnormal vascular anatomy at the elbow; this patient received an RCAVF. The mean brachial artery diameter in the prosthetic group was 3.8 mm (range, 2.0–6.0 mm) and the mean cephalic vein diameter at the elbow was 3.1 mm (range, 0.8–7.2 mm). In only one patient, thrombotic occlusion occurred within the first 6 weeks after operation. No attempt at revision was made, and this patient received a new vascular access. Thus, 98% of prosthetic graft AVFs were functional for dialysis treatment. One patient developed a steal syndrome (0.01/py), five patients were lost to follow-up, one patient stopped cooperating with the study, 17 patients died of complications of their renal failure with a patent graft, three patients underwent successful kidney transplantation, two patients switched to peritoneal dialysis, and in two patients, the graft was explanted because of infection (0.02/py). Patients with RCAVFs developed a total of 102 (1.19/py) vs 122 (1.45/py; P = .739) complications in the prosthetic AVFs (Table 81-3-2). The incidence of thrombosis was significantly higher in the prosthetic graft group (0.54/py vs 0.19/py; P = .049). Also infection was seen significantly more often in patients with prosthetic grafts (0.13/py vs 0.03/py; P = .009). Pseudoaneurysm formation was only seen in the prosthetic AVF group. A total of 43 (0.50/py) interventions in the RCAVF group and 79 (0.94/py) in the prosthetic graft group were needed for access salvage (P = .077) (Table 81-3-3). Significantly more surgical thrombectomies were done in the prosthetic graft group (0.45/py vs 0.10/ py; P = .008). However, percutaneous transluminal angioplasty (PTA) was almost equally performed in both groups (0.26/py in the RCAVF group vs 0.30/py in the prosthetic graft group; P = .437). In addition, surgical revisions (0.10/py in the RCAVF group vs 0.05/py in the prosthetic graft group; P = .086) were executed for access salvage. Of the 43 interventions in the RCAVF group, 13 interventions were performed in nine patients to improve maturation. Anastomotic stenosis in two patients was successfully treated by PTA. In two other patients, PTA failed, and in these two subjects, a prosthetic graft was implanted. One subject underwent repeated PTA for cephalic vein stenosis, without success. Therefore, a surgical revision was performed, also without success. Finally, after a last unsuccessful PTA, the patient received a new AVF in the contralateral arm. Another three patients underwent unsuccessful surgical revisions, including basilic vein transposition, venous interposition, and ligation of tributary veins. Last, one patient underwent two surgical interventions, with ligation of tributary veins, followed by a new proximal

TABLE 81-3-2  Number of Complications Per Patient-Year   in RCAVF Group and Prosthetic AVF Group No. Hematoma Seroma Infection Thrombosis Pseudoaneurysm Steal syndrome Stenosis Nonmaturation Inability to cannulate Bleeding Others Total no. of complications

RCAVF

Prosthetic AVF

P

86 0.20 0 0.03 0.19 0 0.01 0.29 0.24 0.14

84 0.13 0.02 0.13 0.54 0.10 0.01 0.30 — 0.05

— NS NS .009 .049 .006 NS NS — NS

0.02 0.06 1.19

0.04 0.14 1.45

NS NS NS

RCAVF, radial-cephalic direct wrist arteriovenous fistula; AVF, arteriovenous fistula; NS, not significant.

CHAPTER 81-3  ■  Prosthetic Brachial-Antecubital Forearm Loop AVF in Patients with Compromised Vessels  

TABLE 81-3-3  Number of Interventions Per Patient-Year   in RCAVF Group and Prosthetic AVF Group RCAVF No. PTA Surgical thrombectomy Surgical revision Other interventions Total no. of interventions

86 0.26 0.10 0.10 0.03 0.50

Prosthetic AVF 84 0.30 0.45 0.05 0.14 0.94

P — NS .008 NS .028 NS

RCAVF, radial-cephalic direct wrist arteriovenous fistula; AVF, arteriovenous fistula; NS, not significant; PTA, percutaneous transluminal angioplasty.

FIGURE 81-3-2  Primary patency rates. Patency rate is shown in percentages and time in months. Number of patients is presented in the graph. P values calculated with the log-rank test.

radiocephalic anastomosis. However, in these patients, surgical revisions failed and prosthetic grafts were implanted. At the end, two of nine patients with nonmatured RCAVFs received successful interventions. Primary and assisted primary 1-year patencies were 33% ± 5.3% vs 44% ± 6.2% (P = .03; Figure 81-3-2) and 48% ± 5.5% vs 63% ± 5.9% (P = .035; Figure 81-3-3) for RCAVF and prosthetic AVF, respectively. Secondary patencies were 52% (± 5.5%) vs 79% (± 5.1%) (P = .0001; Figure 81-3-4) for RCAVF and prosthetic AVF, respectively.

DISCUSSION For the past decades, the autogenous RCAVF has been accepted as the vascular access of first choice. Therefore, it seems logical to obtain autogenous fistulas in all new dialysis patients. However, data

e1561

e1562   SECTION 19  ■  VASCULAR - ACCESS

FIGURE 81-3-3  Assisted primary patency rates. Patency rate is shown in percentages and time in months. Number of patients is presented in the graph. P values calculated with the log-rank test.

FIGURE 81-3-4  Secondary patency rates. Patency rate is shown in percentages and time in months. Number of patients is presented in the graph. P values calculated with the log-rank test.

from a recent meta-analysis show a high primary failure rate and moderate patency rates at 1 year of follow-up.12 Similar outcomes could be confirmed by the current study in which a high primary failure rate of 41% and moderate 1-year patencies of 33% and 52% (primary and secondary, respectively) were found in the patients with RCAVFs. This is mainly due to early postoperative thrombosis and failure to mature. Nonfunctioning vascular access results in delay of initiation of dialysis treatment with additional morbidity. Access abandonment leads to the need for temporary central vein catheter placement with the risk of catheter thrombosis (24%–40%),13–15 infection and sepsis (2%–18%),14–16 and central vein obstruction (30%).17 Usually, primary failure of RCAVFs depends on the quality and size

CHAPTER 81-3  ■  Prosthetic Brachial-Antecubital Forearm Loop AVF in Patients with Compromised Vessels  

of the vessels used for the arteriovenous anastomosis and the ability of vessel adaptation (remodeling) induced by the increased blood flow volumes. An adequate preoperative vessel assessment with noninvasive Doppler ultrasonography can select well-sized arteries and veins for RCAVF creation with subsequent improvement in the outcome of the vascular access.8–10 Certain duplex-derived parameters may predict the risk of failure or dysmaturation. The internal radial artery diameter has been used in several studies to predict the outcome of RCAVFs or to plan strategies for vascular access. Wong et al18 observed primary failure of RCAVFs in patients with a radial artery and/or cephalic vein diameter 20 mL of 0.5 mg/mL of concentration infiltrates)

Local Care

Antidote Administration

Cold/ice pack for 15–20 min qid for 24–48 hr

Administer dexrazoxane (Totect) by IV in a vein away from the extravasation site. Infuse 1000 mg/m2 within 6 hr of extravasation on day 1, 1000 mg/m2 on day 2, and 500 mg/m2 on day 3. Maximum daily dose is 2000 mg. Dimethyl sulfoxide (DMSO) should not be applied, and topical cooling (e.g., ice packs) should be removed 15 min before, and every 15 min during, administration. Topical application of 99% DMSO

Cold/ice pack for 15–20 min qid for 24–48 hr For all vinca alkaloids, elevate and apply warm pack for 15–20 min at least qid for 24–48 hr

For vinca alkaloids extravasation, inject hyaluronidase subcutaneously, 1 to 6 mL of 150 units/mL solution into area of extravasation in clockwise manner, 1 mL of solution for 1 mL of extravasated drug.

Cold/ice pack 15–20 min qid for 24–48 hr Mix 1.2 mL of 25% sodium thiosulfate with 8.4 mL of sterile water for injection. Inject 2 mL of antidote for each 1 mg of drug extravasated. Use sodium thiosulfate as described for mechlorethamine, 2 mL for each 100 mg of cisplatin infiltrated.

From Schulmeister L: Extravasation management: Clinical update, Semin Oncol Nurs 27:82, 2011.

References 1. Maki R, Kluger DM, Crinich CJ: The risk of bloodstream infection in adults with different intravascular devices: A systematic review of 200 published prospective studies, Mayo Clinic Proceedings, Sept 2006, 1159. 2. Mermel LA, Allon M, Bouza E, et al: Clinical practice guidelines for the diagnosis and management of intravascular catheter-related infection: 2009 Update by the Infectious Diseases Society of America. Clin Infect Dis. 49:1, 2009. 3. Kuter D: Thrombotic complications of central venous catheters in cancer patients. Oncologist 9:207, 2004. 4. Saber W, Moua T, Williams EC, et al: Risk factors for catheter-related thrombosis (CRT) in cancer patients: A patient-level data (IPD) meta-analysis of clinical trials and prospective studies. J Thromb Haemost 9:312, 2011. 5. Palumbo A, Rajkumar SV, Dimopoulos MA, et al: Prevention of thalidomide- and lenalidomide-associated thrombosis in myeloma. Leukemia 22:414, 2008. 6. Monreal M, Raventos A, Lerma R, et al: Pulmonary embolism in patients with upper extremity DVT associated to venous central lines: A prospective study. Thromb Haemost 72:548, 1994. 7. Streiff MB, Bockenstedt PL, Cataland SR, et al: NCCN clinical practice guidelines in oncology for venous thromboembolic disease. J Natl Compr Canc Netw 9:714, 2011. 8. Schulmeister L: Extravasation management: Clinical update. Semin Oncol Nurs 27:82, 2011 Feb.

INDWELLING VASCULAR DEVICES: EMERGENCY ACCESS AND MANAGEMENT Scott H. Witt  /  Diann M. Krywko

83-2 

From Roberts JR: Roberts and Hedge’s Clinical Procedures in Emergency Medicine, 6th edition (Saunders 2013)

FIGURE 83-2-1  A, Broviac Pediatric 4.2-French Single-Lumen CV Catheter with SureCuff Tissue Ingrowth Cuff and VitaCuff Antimicrobial Cuff. B, The Groshong catheter has a valve to prevent backbleeding.

e1597

e1598   SECTION 19  ■  VASCULAR - ACCESS

FIGURE 83-2-2  A, Subclavian-placed catheter with a subcutaneous tunnel. B, Quinton dialysis catheter in the right internal jugular vein, often used for emergency dialysis or as a bridge until a dialysis fistula or graft is ready for use (i.e., when it matures).

CHAPTER 83-2  ■  Indwelling Vascular Devices: Emergency Access and Management  

FIGURE 83-2-3  A, Port-a-Cath double-lumen port (for chest placement). B, Port-A-Cath single-lumen port (for upper extremity placement). The Port-A-Cath system is accessed by inserting a Huber needle through the skin into the portal septum. C, The Infuse-A-Port is similar to the Port-A-Cath.

FIGURE 83-2-4  A, Porta-A-Cath system (Deltec, Inc., St. Paul, MN). This device is subcutaneous and accessed with a Huber needle introduced through the skin into the portal septum. B, The Huber needle is used to access the septum. Always use sterile technique.

e1599

e1600   SECTION 19  ■  VASCULAR - ACCESS

FIGURE 83-2-5  The double-lumen percutaneously inserted central catheter (5.0 Fr, 18 gauge) is placed in the arm with the tip of the catheter in the superior vena cava. Shorter 20-cm versions (not shown) look similar but terminate in the axillary vein and are termed midline peripheral catheters.

FIGURE 83-2-6  A, Percutaneously inserted central catheter (PICC) line placement in the upper extremity with the internal catheter tip at the superior vena cava (SVC). B, Most PICC lines are used for outpatient therapy, such as prolonged antibiotic therapy, so proper aseptic technique at the catheter site is essential.

CHAPTER 83-2  ■  Indwelling Vascular Devices: Emergency Access and Management  

FIGURE 83-2-7  Mahurkar 11.5-Fr. × 16-cm DoubleLumen Catheter for temporary hemodialysis, apheresis, and infusion.

FIGURE 83-2-8  MedComp Duo-Flow Internal Jugular Vascular Catheter (11.5 Fr × 15 cm). The angle of the catheter makes it more comfortable for the patient.

FIGURE 83-2-9  A, Various possible anastomotic configurations between the artery and vein for autogenous fistula formation. A thrill should be palpated if this fistula is functioning. B, Older dialysis fistula. Fistulas can develop multiple aneurysms (arrows) from multiple-time use. It may be difficult to distinguish a fistula from a graft by merely looking at the site. (A, Adapted from Ozeran RS. Construction and care of external arteriovenous shunts. In: Wilson SE, Owens ML, eds. Vascular Access Surgery. Chicago: Year Book Medical; 1980.)

e1601

83-3 

SELF ASSESSMENT Benjamin Lind  /  Crea Fusco  /  José M. Velasco From Velasco J: Rush University Medical Center Review of Surgery, 5th edition (Saunders 2011)

1. Which of the following factors is the least important independent risk factor associated with increased risk for DVT? A. Obesity B. Central venous catheters C. Hospitalization with recent surgery D. Trauma E. Previous DVT Ref.: 1 COMMENTS: Venostasis of the lower extremities is associated with prolonged bed rest, standing, or sitting. It is also caused by the immobilization and muscular paralysis associated with trauma and general and spinal anesthesia. The most significant risk factors include previous DVT and hospitalization with recent surgery. Additional risk factors include advanced age, diabetes mellitus, and the presence of malignancy. Obesity in conjunction with other risk factors leads to increased risk for DVT, but it may not be an independent risk factor. Patients with blood group type O are at lower risk for DVT, whereas patients with group A blood are at higher risk. In both instances the reason is unknown. Oral contraceptives and pregnancy are associated with increased levels of fibrinogen and factors VII, VIII, IX, and X, and both are associated with increased risk for DVT. The incidence of DVT in surgical patients is 20% to 50%. The incidence in patients with hip fractures or in those undergoing knee or hip replacement may exceed 50%.

ANSWER: A 2. Which of the following statements concerning radial artery cannulation is true? A. Aortic systolic pressure is higher than radial systolic pressure. B. The Allen test is an outdated mode of assessing collateral flow of the ulnar and radial arteries. C. The incidence of infection is higher with catheters placed by surgical cutdown. D. The catheter should be replaced every 3 days. E. Intermittent flushing to keep the catheter free of clots is desirable. Ref.: 2 COMMENTS: The incidence of complications after arterial catheterization seems to be operator independent, unlike the case with pulmonary artery (PA) catheterization. Known risk factors include intermittent punctures, age younger than 10 years, prolonged catheterization (>4 days), anticoagulant therapy, and use of a catheter larger than 20 gauge or made of polypropylene rather than Teflon. The radial artery is the site most frequently used for catheterization, provided that the ulnar artery and palmar arterial arch are patent. Therefore, the Allen test should be performed before attempting radial artery catheterization. A normal test result consists of a palmar blush within 7 seconds after the ulnar artery is released. Most patients with arterial thrombosis remain asymptomatic. Symptoms can be e1602

CHAPTER 83-3  ■  Self Assessment  

minimized by placing lines in arteries with good collateral circulation. Most thrombi (43%) are present at the time of catheter removal, and another 30% develop within 24 hours. A higher incidence of thrombosis occurs within the first 24 hours when surgical cutdown is performed (48% versus 23% with percutaneous placement), but the incidence of thrombosis at 1 week is the same for both methods of placement. Brachial artery cannulation has a high incidence of embolic occlusion of the distal arteries (5% to 41%) and should therefore be avoided. Infection remains the most common complication. Predisposing factors are prolonged catheterization, surgical cutdown, local inflammation, pre-existing bacteremia, and failure to change the saline flush fluid, transducer, and flush tubing every 48 hours. The need for intermittent arterial catheter replacement is not established and indeed is controversial. The aortic mean arterial pressure (MAP) and diastolic arterial pressure are slightly higher than the radial MAP and diastolic arterial pressure. However, systolic pressure is consistently higher in the radial artery than in the aorta. This discrepancy increases with distal progression, smaller arterial caliber, and age and is explained by the reflection of pressure waves from capillary beds, which results in augmentation of the systolic and reduction of the diastolic values measured.

ANSWER: C

3. A 70-kg, 72-year-old man known to suffer from congestive heart failure (CHF), arthritis, diabetes mellitus, and a first-degree heart block is intubated in the ICU on postoperative day 2 after exploratory laparotomy for perforated sigmoid diverticulitis. His urine output has dropped to 10 mL/h for the last shift, and he is hypotensive despite several fluid boluses. A PA catheter is placed through the right internal jugular vein with some difficulty. As the line is advanced to 50 cm, the patient has a 14-beat run of ventricular tachycardia, which resolves when the catheter is pulled back. It is finally advanced to 62 cm and the balloon is inflated with 3 cc of air by the resident. As the line is being secured, a large amount of blood is noted in the endotracheal tube and the patient becomes hypotensive. Select the best intervention for this patient: A. Place external pacing wires and administer lidocaine to treat the ventricular tachycardia. B. Place a double-lumen endotracheal tube and occlude the appropriate bronchus with a Fogarty catheter. C. Pull the PA catheter back 2 cm with the balloon inflated. D. Suction the endotracheal tube while deflating the balloon by 2 cc of air. E. Obtain a chest radiograph to confirm correct placement of the line. Ref.: 3, 4 COMMENTS: The indications for pulmonary artery catheters and their value in patients with sepsis or hemodynamic instability are uncertain, but they may be useful in the management of patients unresponsive to the use of fluids and vasoactive agents. Dysrhythmias occur in 12% to 67% of patients undergoing catheterization but are usually self-limited, premature ventricular contractions. Complete heart block can develop in patients with pre-existing left bundle branch block. A prophylactic pacing wire should be used in these patients. Prophylactic lidocaine and full inflation of the balloon may prevent ventricular ectopy. Hemoptysis in patients with a PA catheter suggests the diagnosis of perforation or rupture. Mechanisms involved in PA rupture include (1) overinflation of the balloon, (2) incomplete balloon inflation (